Chuyên đề bất đẳng thức – Võ Quốc Bá Cẩn

TR NG THPT CHUYÊN LÝ T TR NG TOÁN - TIN H C Chuyên www.toanmath.com B B T T N N G G T T H H C C Thc hi n: Võ Qu c Bá C n c sinh chuyên Toán, niên khóa 2004 - 2006 TPCT - 2006 T ải thêm tài li ệu môn Toán THPT t ại: + Trang web: www.toanmath.com + Fanpage: www.face book.com/toanmath + Groups: https://www.facebook.com/groups/toanmath Tìm tài liệu Toán ? Chuyện nhỏ - www.toanmath.com1 i nói u ----oOo---- t ng th c là m t trong nh ng v n hay và khó nh t c a ch ng trình toán ph thông b i nó có mt trên hu khp các lnh v c ca toán hc và nó òi hi chúng ta ph i có mt vn ki n th c t ng i v ng vàng trên t t c các l nh v c. i ng i chúng ta, c bit là các bn yêu toán, dù ít dù nhiu thì cng ã t ng au u tr c m t b t ng th c khó và c ng ã t ng có c m t c m giác t hào khi mà mình ch ng minh c bt ng th c ó. Nhm “kích hot” ni m say mê t ng th c trong các b n, tôi xin gi i thi u v i v i các b n cu n sách “chuyên t ng th c”. Sách g m các ph ng pháp ch ng minh bt ng th c m i mà hin nay ch a c ph bin cho l m. Ngoài ra, trong sách g m m t s l ng l n bt ng th c do tôi sáng tác, còn l i là do tôi l y toán trên internet nh ng ch a có l i gi i ho c có i gi i nh ng là li gi i hay, l , p m t. Ph n ln các bài t p trong sách u do tôi gii nên không th nào tránh khi nh ng ng nhn, sai lm, mong các bn thông m. Hy v ng r ng cu n sách s giúp cho các b n m t cái nhìn khác v b t ng th c và mong rng qua vic gii các bài toán trong sách s giúp các bn có th tìm ra ph ng pháp c a riêng mình, nâng cao c t duy sáng to. Tôi không bit các n ngh sao nh ng theo quan m ca bn thân tôi thì nu ta hc tt v bt ng th c thì c ng có th h c t t các l nh v c khác c a toán h c vì nh ã nói trên b t ng th c òi hi chúng ta phi có mt kin th c tng h p t ng i v ng vàng. Tôi không nói suông âu, chc hn bn c ng bit n anh Ph m Kim Hùng, sinh viên h CNTN khoa toán, tr ng HKHTN, HQG Hà N i, ng i ã c tham hai k thi IMO và u t kt qu cao nht trong i tuyn VN. Bn bit không? Trong th i hc ph thông, anh y ch chuyên tâm rèn luy n bt ng th c thôi. (Các bn l u ý là tôi không khuyn khích bn làm nh tôi và anh y âu nhé!) Tìm tài liệu Toán ? Chuyện nh ỏ - www.toanmath.com2 c dù ã c g ng biên so n m t cách th t c n th n, nh ng do trình có h n nên không th tránh khi nh ng sai sót, mong các bn thông c m và góp ý cho tôi cu n sách ngày càng c hoàn thin h n. Chân thành c m n. i óng góp xin g i v mt trong các a ch sau: + Võ Qu c Bá Cn, C65 khu dân c Phú An, ph ng Phú Th , qun Cái R ng, thành ph C n Th . (071.916044 + Email.babylearnmath@yahoo.com Kính tng các thy ng Bo Hòa, Phan i Nh n, Trn Diu Minh, Hu nh B u Tính, cô T Thanh Thy Tiên và toàn th các thy cô giáo trong t Toán Tin, thân ng các bn cùng l p. Tìm tài liệu Toán ? Chuyện nh ỏ - www.toanmath.com3 T S B T NG THC THÔNG D NG 1. Bt ng th c AM-GM. u 12 , ,..., n aaa là các s th c không âm thì 12 1 1 . ... n n in i a aaa n = ≥ ∑ ng th c xy ra khi và ch khi 12 ... n aaa = == . 2. B t ng th c AM-HM. u 12 , ,..., n aaa là các s th c d ng thì 1 1 11 . 11 . n i n i i i a n na = = ≥ ∑ ∑ ng th c xy ra khi và ch khi 12 ... n aaa = == . 3. Bt ng th c Bunhiacopxki. Cho 2n s th c 12 , ,..., n aaa và 12 , ,..., n bbb . Khi ó, ta có 2222222 1 2 1 2 1 1 22 ( ... )( ... ) ( ... ) n n nn a a a b b b ab ab ab + ++ + ++ ≥ + ++ ng th c xy ra khi và ch khi 12 12 .... n n a aa bbb = == 4. Bt ng th c Minkowski. Cho 2n s th c d ng 12 , ,..., n aaa và 12 , ,..., n bbb . Khi ó v i m i 1, r ≥ ta có 1 11 1 11 () n nn r rr r rr ii ii i ii ab ab = == ⎛ ⎞ ⎛ ⎞ ⎛⎞ + ≤+ ⎜ ⎟ ⎜ ⎟ ⎜⎟ ⎝ ⎠ ⎝ ⎠ ⎝⎠ ∑ ∑∑ 5. B t ng th c AM-GM m r ng. u 12 , ,..., n aaa là các s th c không âm và 12 , ,..., n βββ là các s th c không âm có tng bng 1 thì 12 11 22 12 ... .. n nnn a a a aaa β ββ β β β + ++≥ 6. B t ng th c Chebyshev. Cho 2n s th c 12 ... n aaa ≤ ≤≤ và 12 , ,..., n bbb . Khi ó a) Nu 12 ... n bbb ≤≤≤ thì 1 11 . n nn ii ii i ii nab ab = == ⎛ ⎞⎛⎞ ≥ ⎜ ⎟⎜⎟ ⎝ ⎠⎝⎠ ∑ ∑∑ a) Nu 12 ... n bbb ≥≥≥ thì 1 11 . n nn ii ii i ii nab ab = == ⎛ ⎞⎛⎞ ≤ ⎜ ⎟⎜⎟ ⎝ ⎠⎝⎠ ∑ ∑∑ Tìm tài liệu Toán ? Chuyện nh ỏ - www.toanmath.com4 ng th c xy ra khi và ch khi 12 12 ... ... n n aaa bbb = == ⎡ ⎢ = == ⎣ 7. Bt ng th c Holder. Cho 2n s th c không âm 12 , ,..., n aaa và 12 , ,..., n bbb . Khi ó v i m i ,1 pq > th a 11 1, pq += ta có 11 1 11 n nn pq pq ii ii i ii ab ab = == ⎛ ⎞⎛⎞ ≤ ⎜ ⎟⎜⎟ ⎝ ⎠⎝⎠ ∑ ∑∑ 8. B t ng th c Schur. i mi b ba s không âm ,, abc và 0, r ≥ ta luôn có b t ng th c ( )( ) ( )( ) ( )( ) 0 r rr a ab ac b bc ba c c a cb -- + - - +- - ≥ ng th c xy ra khi và ch khiabc == ho c ,0 a bc == và các hoán v. 9. B t ng th c Jensen. Gi s () fx là m t hàm li trên [ ,] ab . Khi ó, v i m i 12 , ,..., [ , ] n x x x ab ∈ và 12 , ,..., 0 n α α α≥ th a 12 ...1 n α α α + + += ta có b t ng th c 11 () nn ii ii ii f x fx αα == ⎛⎞ ≥ ⎜⎟ ⎝⎠ ∑∑ 10. B t ng th c s p x p l i. Cho 2 dãy n u cùng tng 12 ... n aaa ≤ ≤≤ và 12 ... n bbb ≤ ≤≤ . Khi ó, v i 12 , ,..., n iii là mt hoán v bt kì ca 1,2,...,n ta có 1 1 22 1122 1 211 ... ... ... nn nniiii ii n nn ab ab ab ab a b a b ab ab ab - + + + ≥ + ++ ≥ + ++ 11. Bt ng th c Bernulli. i 1 x >- , ta có +u 10 rr ≥ ∨≤ thì(1 )1 r x rx + ≥+ +u 10 r >> thì(1 )1 r x rx + ≤+ Tìm tài liệu Toán ? Chuyện nh ỏ - www.toanmath.com5 T NG TH C THU N NH T 1. M u. u ht các b t ng th c c n (AM-GM, Bunhiacopxki, Holder, Minkowsky, Chebyshev ...) u là các b t ng th c thu n nh t. u này hoàn toàn không ng u nhiên. V logíc, có th nói rng, ch có các i l ng cùng bc m i có th so sánh i nhau m t cách toàn c c c. Chính vì th , b t ng th c thu n nh t chi m m t t l r t cao trong các bài toán b t ng th c, c bit là bt ng th c i s (khi các hàm s là hàm i s, có bc u h n). i v i các hàm gi i tích (m , l ng giác, logarith), các b t ng th c ng c coi là thu n nh t vì các hàm s có b c ∞ (theo công th c Taylor). Trong bài này, chúng ta s cp t i các ph ng pháp c bn ch ng minh bt ng th c thu n nh t, c ng nh cách chuy n t m t b t ng th c không thu n nh t m t b t ng th c thu n nh t. N m v ng và v n d ng nhu n nhuy n các ph ng pháp này, chúng ta có th ch ng minh c h u h t các b t ng th c s c p. 2. B t ng th c thu n nh t. Hàm s 12 ( , ,..., ) n fxxx c a các bi n s th c 12 , ,..., n xxx c là hàm thu n nh t b c α nu v i m i s th c t ta có 1 2 12 (, ,..., ) ( ,,...,) nn f tx tx tx t f x x x α = t ng th c d ng 12 ( ,,..., )0 n fxxx ≥ i f là m t hàm thu n nh t c g i là b t ng th c thu n nh t (b c α ). Ví d các bt ng th c AM-GM, bt ng th c Bunhiacopxki, bt ng th c Chebyshev là các bt ng th c thun nht. Bt ng th c Bernoulli, bt ng th c sinxx < v i 0 x > là các bt ng th c không thun nht. Tìm tài liệu Toán ? Chuyện nh ỏ - www.toanmath.com6 3. Ch ng minh b t ng th c thu n nh t. 3.1. Ph ng pháp d n bi n. c m c a nhiu bt ng th c, c bit là các bt ng th c i s là du bng y ra khi t t c ho c m t vài bi n s b ng nhau (xu t phát t b t ng th c c b n 2 0 x ≥ !). Ph ng pháp dn bin d a vào c m này làm gi m s bin s ca t ng th c, a b t ng th c v d ng n gi n h n có th ch ng minh tr c ti p ng cách kh o sát hàm m t bi n ho c ch ng minh b ng quy n p. ch ng minh b t ng th c 12 ( , ,..., ) 0 (1) n fxxx ≥ Ta có th th ch ng minh 12 12 12 ( , ,..., ) , ,..., (2) 22 nn xx xx fxxxfx ++ ⎛⎞ ≥ ⎜⎟ ⎝⎠ hoc ( ) 1 2 1 2 12 ( , ,..., ) , ,..., (3) nn f x x x f xx xx x ≥ Sau ó chuy n vi c ch ng minh (1) v vi c ch ng minh b t ng th c 1 13 13 ( , , ,..., ) ( , ,..., ) 0 (4) nn f xxx x gxx x =≥ c là mt bt ng th c có s bin ít h n. D nhiên, các bt ng th c (2), (3) có th không úng hoc ch úng trong mt s u kin nào ó. Vì ta ch thay i 2 bin s nên thông th ng thì tính úng n ca bt ng th c này có th ki m tra c d dàng. Ví d 1. Cho ,,0 abc > . Ch ng minh b t ng th c 3 3 3 2 2 2 222 3 a b c abc a b b c c a ab bc ca + + + ≥ + + + ++ Ch ng minh. Xét hàm s 3 3 3 2 2 2 222 (,,)3() f a b c a b c abc a b b c c a ab bc ca = + + + - + + + ++ Ta có 2 5 (,,) , , () 224 bcbc a f abc f a b c b c ++ ⎛ ⎞⎛⎞ - =+-- ⎜ ⎟⎜⎟ ⎝ ⎠⎝⎠ Tìm tài liệu Toán ? Chuyện nh ỏ - www.toanmath.com7 Do ó, n u min{ , , } a abc = ( u này luôn có th gi s ) thì ta có (,,) ,, 22 b cbc f abc f a ++ ⎛⎞ ≥ ⎜⎟ ⎝⎠ Nh vy, ch ng minh bt ng th c u bài, ta ch cn ch ng minh (,,)0 f abb ≥ Nh ng b t ng th c này t ng ng v i 3 3 2 22 2 3 23 322 2 2 3 ( )0 20 ( )0 a b ab ab ab ba b ba b a ab ab aab + + - +++ ++ ≥ ⇔ + -≥ ⇔ -≥ Ví d 2. (Vietnam TST 1996) Cho ,, abc là các s th c bt k. Ch ng minh rng 4 4 4 444 4 (,,)( ) ( ) ( ) .( )0 7 Fabc ab bc c a abc = + + + + + - + +≥ i gi i. Ta có 4 4 4 444 44 44 4 4 4 4 44 3 3 3 2 22 (, ,) , , 22 4 ( )( )( ) .( ) 7 4 2 ( ) .2 2 72 4() ( )( )2 . 2 78 (4 4 ( ) ) 3 (2 2 ( bcbc Fabc Fa ab bc c a abc bc bc a bca bc bc a b c a a bc a b c bc a b c bc ++ ⎛⎞ -= ⎜⎟ ⎝⎠ = + + + + + - + +- ⎛⎞ ++ ⎛ ⎞ ⎛⎞ - + -+ ++⎜⎟ ⎜ ⎟ ⎜⎟ ⎜⎟ ⎝ ⎠ ⎝⎠ ⎝⎠ ⎛⎞ ++ ⎛⎞ =+ + + - + + -- ⎜⎟ ⎜⎟ ⎝⎠ ⎝⎠ = + -+ + + -+ 4 2 44 2 2 2 2 22 2 2 22 3 () )) 78 3 3( )() 3() ()(77 10) 56 3 3( )() ()(77 10) 56 bc bc abcbc a bc b c b c bc aabcb c b c b c bc ⎛⎞ + + +- ⎜⎟ ⎝⎠ = + - + - + - ++ = ++ - + - ++ Tìm tài liệu Toán ? Chuyện nh ỏ - www.toanmath.com8 h ng 2 22 3 ( )(7 7 10) 56 b c b c bc - ++ luôn không âm. N u ,, abc cùng du thì bt ng th c cn ch ng minh là hin nhiên. Nu ,, abc không cùng du thì phi có ít nht 1 trong ba s ,, abc cùng d u v iabc ++ . Không m t tính t ng quát, gi s ó là a . ng th c trên suy ra (, ,) , , 22 bcbc Fabc Fa ++ ⎛⎞ ≥ ⎜⎟ ⎝⎠ . Nh vy ta ch còn cn ch ng minh 4 4 44 (,,) 0, 4 2( ) (2 ) .( 2 ) 0 , 7 F abb ab a b b a b ab ≥ ∀∈ ⇔ + + - + ≥ ∀∈ R R u 0 b = thì b t ng th c là hi n nhiên. N u 0 b ≠ , chia hai v c a b t ng th c cho 4 b r i t a x b = thì ta c bt ng th c t ng ng 44 4 2( 1) 16 .( 2) 0 7 xx + + - +≥ t ng th c cui cùng có th ch ng minh nh sau Xét 44 4 ( ) 2( 1) 16 .( 2) 7 fxxx = + +-+ Ta có / 33 / 3 16 ()8( 1). 7 2 ( ) 0 1 . 2.9294 7 ( 2.9294) 0.4924 0 min fxxx fx x xx ff = +- = ⇔ + = ⇔ =- =- => (Các phn tính toán cui c tính v i chính xác t i 4 ch s sau du phy. Do min f tính c là 0.4924 nên nu tính c sai s tuyt i thì giá tr chính xác c a min f v n là m t s d ng. Vì ây là m t b t ng th c r t ch t nên không th tránh Tìm tài liệu Toán ? Chuyện nh ỏ - www.toanmath.com9 c các tính toán v i s l trên ây. Ch ng h n n u thay 4 7 b ng 16 27 3 min x =- thì * min f có giá tr âm! ây * 44 4 ( ) 2( 1) 16 .( 2) 7 fxxx = + +-+ .) 3.2. Ph ng pháp chu n hóa. ng th ng gp ca bt ng th c thun nht là 12 12 ( , ,..., ) ( , ,..., ) nn fxxx gxxx ≥ trong ó f và g là hai hàm thu n nh t cùng b c. Do tính cht ca hàm thun nht, ta có th chuyn vic ch ng minh bt ng th c trên v vi c ch ng minh b t ng th c 12 ( , ,..., ) n fxxxA ≥ v i m i 12 , ,..., n xxx th a mãn u ki n 12 ( , ,..., ) n gxxxA = . Chu n hóa m t cách thích h p, ta có th làm n gin các biu th c ca bt ng th c cn ch ng minh, tn dng c mt s tính ch t c bi t c a các h ng s . Ví d 3. (B t ng th c v trung bình l y th a) Cho b n s th c d ng 12 ()( , ,...,) n x xxx = . V i m i s th c r ta t 1 12 ... () rrr r n r xxx Mx n ⎛⎞ + ++ = ⎜⎟ ⎝⎠ Ch ng minh r ng v i m i 0 rs >> ta có () (). rs M x Mx ≥ i gi i. Vì ( ) () rr M tx tM x = v i m i 0 t > nên ta ch c n ch ng minh b t ng th c úng cho các s th c d ng 12 , ,..., n xxx tho mãn u ki n ()1 s Mx = , t c là cn ch ng minh ()1 r Mx ≥ v i m i 12 , ,..., n xxx tho mãn u ki n ()1 s Mx = . u này có th vit n gin li là Ch ng minh 12 ... rrr n x x xn + + +≥ v i 12 ... sss n x x xn + + += . ch ng minh bt ng th c cui cùng, ta áp dng bt ng th c Bernoulli ( ) (1 ( 1)) 1 .( 1) 1 , rr rs ss ss ii ii r x x x x in s = = + - ≥ + - ∀= ng các bt ng th c trên li, ta c u phi ch ng minh. Tìm tài liệu Toán ? Chuyện nh ỏ - www.toanmath.com10 Ví d 4. (VMO 2002) Ch ng minh r ng v i ,, xyz là các s th c b t k ta có b t ng th c 3 22 2 22 2 2 6( )( ) 27 10( ) x y zx yz xyz x yz + + + + ≤ + ++ i gi i. t ng th c này rt cng knh. Nu th c hin phép bi n i tr c tip s rt khó kh n (ví d th bình ph ng kh c n). Ta th c hi n phép chu n hóa n gi n hóa b t ng th c ã cho. N u 2 22 0 x yz + += , thì 0 xyz = == , b t ng th c tr thành ng th c. N u 2 22 0 x yz + +> , do b t ng th c ã cho là thu n nh t, ta có th gi s 2 22 9 x yz + += . Ta c n ch ng minh 2( ) 10 x y z xyz ++ ≤+ v i u ki n 2 22 9 x yz + += . ch ng minh u này, ta ch cn ch ng minh 2 [2( ) ] 100 x y z xyz ++-≤ Không mt tính tng quát, có th gi s x yz ≤≤ . Áp d ng b t ng th c Bunhiacopxky, ta có ( ) 22 222 22 2 2 33 2 [2 ] [2( ) (2 )] [( ) ][4 (2 ) ] (9 2 )(8 4 ) 72 20 2 100 ( 2) (2 7) x y z xyz x y z xy x y z xy xy xy x y xy x y xy xy xy ++ - = + +- ≤ + + +- = + -+ = - ++ =++- 2 22 3 2 6, x y z z xy xy ≤ ≤ ⇒≥ ⇒ ≤+ ≤ t c là 2 ( 2) (2 7) 0 xy xy + -≤ . Tây, t h p v i ánh giá trên ây ta c u cn ch ng minh. u b ng x y ra khi và ch khi 22 20 xyz xy xy + ⎧ = ⎪ - ⎨ ⎪ += ⎩ . ây gi i ra c 1, 2,2 x yz =- == . thut chun hóa cho phép chúng ta bin mt bt ng th c ph c tp thành mt t ng th c có dng n gin h n. u này giúp ta có th áp dng các bi n i i s m t cách d dàng h n, thay vì ph i làm vi c v i các bi u th c c ng k nh ban Tìm tài liệu Toán ? Chuyện nh ỏ - www.toanmath.com11 u. c bi t, sau khi chu n hóa xong, ta v n có th áp d ng ph ng pháp d n bi n gi i. Ta a ra l i gi i th hai cho bài toán trên t (,,)2() f x y z x y z xyz = + +- . Ta c n ch ng minh ( , , ) 10 f xyz ≤ v i 2 22 9 x yz + += . Xét ( ) 22 22 2 22 2 22 () , , (,,)22() 222 2 () 2 2() yz yz xy z f x fxyz y z yz x yz y z yz ⎛⎞ ++- ⎜ ⎟- = + - -- ⎜⎟ ⎝⎠ ⎛⎞ ⎜⎟ =-- ⎜⎟ + ++ ⎝⎠ + Nu ,,0 xyz > , ta xét hai trng h p * 1 xyz ≤ ≤≤ . Khi ó 2 22 2( ) 2 3( ) 1 6 3 1 10 x y z xyz x y z + + - ≤ + + - = -< *01 x <≤ . Khi ó 222 2( ) 2 2 2( ) 2 2 2(9 ) ( ) x y z xyz x y z x x gx ++ - ≤ + += + -= Ta có ( ) 2 / 2 292 ()0 9 xx gx x -- => - , suy ra ( ) (1) 10 gxg ≤= . + N u trong 3 s ,, xyz có m t s âm, không m t tính t ng quát, ta có th gi s là 0 x < . Khi ó 2 2 22 , , ( , ,) 22 yz yz f x f xyz ⎛⎞ ++ ⎜⎟ ≥ ⎜⎟ ⎝⎠ , nên ta ch c n ch ng minh 22 22 2 2 32 , , 10 22 (9) 2 2 2(9 ) 10 2 ( ) 5 4 2(9 ) 20 yz yz fx xx xx hxxxx ⎛⎞ ++ ⎜⎟ ≤ ⎜⎟ ⎝⎠ - ⇔+ --≤ ⇔ = - + -≤ Ta có /2 2 42 ()35 9 x hxx x = -- - . Tìm tài liệu Toán ? Chuyện nh ỏ - www.toanmath.com12 Gii ph ng trình / ()0 hx = (v i 0 x < ), ta c 1 x=- . ây là m c c i ca h, do ó ( ) ( 1) 20 hxh ≤ -= . ng cách chun hóa, ta có th a mt bài toán bt ng th c v bài toán tìm giá tr l n nht hay nh nht ca mt hàm s trên mt min (chng hn trên hình cu 2 22 9 x yz + += nh ví d 4). u này cho phép chúng ta v n d ng c m t s thut tìm giá tr l n nht, giá tr nh nht (ví d nh bt ng th c Jensen, hàm i,...). Ví d 5. Cho ,, abc là các s th c d ng. Ch ng minh r ng 222 2 22 222 () () () 3 5 ( ) ( ) () bca c a b ab c a bc b c a c ab + - + - +- + +≥ + + + + ++ i gi i. Ta ch cn ch ng minh bt ng th c cho các s d ng ,, abc tho 1 abc ++= . Khi ó b t ng th c có th vi t l i thành 2 22 2 22 2 22 (1 2 ) (12) (12) 3 5 2 2 12 2 12 2 1 1 1 1 27 5 2 2 12 2 12 2 1 27 ( ) ( ) ( ) (5.1) 5 a bc a ab bc c aa b b cc f a f b fc - -- + +≥ - + - + -+ ⇔ + +≤ - + - + -+ ⇔ ++≤ Trong ó 2 1 () 2 21 fx xx = -+ ý r ng 271 3 53 f ⎛⎞ = ⎜⎟ ⎝⎠ , ta th y (5.1) có d ng b t ng th c Jensen. Tuy nhiên, tính o hàm c p hai c a () fx , ta có 2 // 23 4(6 6 1) () (2 2 1) xx fx xx -+ = -+ Tìm tài liệu Toán ? Chuyện nh ỏ - www.toanmath.com13 hàm ch li trên khong 3 333 , 66 ⎛⎞ -+ ⎜⎟ ⎝⎠ nên không th áp dng b t ng th c Jensen mt cách tr c tip. Ta ch ng minh 27 ()()() 5 fa fb fc ++≤ b ng các nhn xét b sung sau 1 2 2 max ff ⎛⎞ == ⎜⎟ ⎝⎠ () fx tng trên 1 0, 2 ⎛⎞ ⎜⎟ ⎝⎠ và gi m trên 1 ,1 2 ⎛⎞ ⎜⎟ ⎝⎠ 3 3 3 3 12 6 67 ff ⎛ ⎞ ⎛⎞ -+ == ⎜ ⎟ ⎜⎟ ⎝ ⎠ ⎝⎠ u có ít nh t 2 trong 3 s ,, abc n m trong kho ng 3 333 , 66 ⎛⎞ -+ ⎜⎟ ⎝⎠ , ch ng h n là a, b thì áp d ng bt ng th c Jensen ta có 2 14 ()()22 22 1 abc fafbff c +- ⎛ ⎞ ⎛⎞ +≤ == ⎜ ⎟ ⎜⎟ + ⎝ ⎠ ⎝⎠ Nh v y trong tr ng h p này, ta ch c n ch ng minh 22 1 4 27 5 2 211 c cc +≤ -++ Quy ng mu s và rút gn ta c bt ng th c t ng ng 4 32 22 27 27 18 7 1 0 (3 1) (3 1) 0 (ñuùng) c c cc c cc - + - +≥ ⇔ - - +≥ Nh vy, ta ch còn cn xét tr ng h p có ít nht hai s nm ngoài khong 3 333 , 66 ⎛⎞ -+ ⎜⎟ ⎝⎠ . N u ch ng h n 33 6 a + ≥ thì rõ ràng 33 , 6 bc - ≤ và nh v y, do nh n xét trên 36 27 ()()() 75 fa f b fc ++≤ < . Ta ch còn duy nh t m t trng h p c n xét là có hai s , ch ng h n 33 , 6 ab - ≤ . Tìm tài liệu Toán ? Chuyện nh ỏ - www.toanmath.com14 Lúc này, do 3 1 3 ab + ≤- nên 31 32 c≥> . Theo các nh n xét trên, ta có 3 3 3 24 15 6 3 27 ()()() 2 . 6 3 7 135 fa f b fc f f ⎛ ⎞ ⎛⎞ -+ ++≤ + = + < ⎜ ⎟ ⎜⎟ ⎝ ⎠ ⎝⎠ Ghi chú. Bài toán trên có mt cách gii ngn gn và c áo h n nh sau t ng th c có th vit li thành 2 22 222 ( ) ( ) ( )6 5 ( ) ( ) () ab c bc a ca b a bc b c a c ab + ++ + +≤ + + + + ++ Không mt tính tng quát, có th gi s 1 abc ++= . Khi ó, bt ng th c vit li thành 2 22 (1 ) (1 ) (1 ) 6 5 2 2 1 2 2 1 2 21 aabbcc a ab bc c - -- + +≤ - + - + -+ Ta có 2 ( 1) 2(1) 4 a aa + -≤ . Do ó 2 2 (1) (1)(3 ) 12 21 44 a aa aa + -+ -+ ≥-= . T ó 2 (1) (1)4 (1 )(3 ) 3 2 21 4 aa aaa aa a aa -- ≤= -+ + -+ ng t 2 2 (1)4 3 2 21 (1)4 . 3 2 21 bbb b bb ccc c cc - ≤ + -+ - ≤ + -+ Và ch ng minh bt ng th c u bài, ta ch cn ch ng minh 4 4 46 3 3 35 abc abc ++≤ + ++ t ng th c cui cùng này t ng ng v i 1 1 19 3 3 3 10 abc ++≥ + ++ là hi n nhiên (Áp dng BT AM-GM). Tìm tài liệu Toán ? Chuyện nh ỏ - www.toanmath.com15 Chun hóa là mt k thut c bn. Tuy nhiên, k thut ó cng òi hi nh ng kinh nghi m và tinh t nht nh. Trong ví d trên, ti sao ta li chun hóa 2 22 9 x yz + += mà không phi là 2 22 1 x yz + += (t nhiên h n)? Và ta có t c nh ng hi u qu mong mun không n u nh chu n hóa 1 xyz + += ? ó là nh ng vn mà chúng ta phi suy ngh tr c khi th c hin b c chun hóa. 3.3. Ph ng pháp tr ng s . t ng th c AM-GM và b t ng th c Bunhiacopxki là nh ng b t ng th c thun nht. Vì th, chúng rt h u hiu trong vic ch ng minh các bt ng th c thun nht. Tuy nhiên, do u kin xy ra d u bng ca các bt ng th c này r t nghiêm ngt nên vic áp dng mt cách tr c tip và máy móc ôi khi khó em l i t qu. áp dng tt các bt ng th c này, chúng ta phi nghiên c u k u ki n x y ra d u b ng và áp dng ph ng pháp trng s. Ví d 6. Ch ng minh r ng n u ,, xyz là các s th c không âm thì 3 2 22 2 22 2 6( )( ) 27 10( ) x y zx yz xyz x yz -+ + ++ + ≤ ++ i gi i. d ng nguyên lý c b n « u b ng x y ra khi m t c p bi n s nào ó b ng nhau » , ta có th tìm ta c du bng ca bt ng th c trên xy ra khi 2 yzx == . u này cho phép chúng ta mnh dn ánh giá nh sau 3 2 22 2 22 2 1 2 22 2 22 2 11 2 2 2 2 22 222 22 2 22 2 22 10( ) 6( )( ) ( ) 10( ) 6( ) 10 ().()(122)6( ) 3 10 ( ).(22)6() 3 ( )(28 2 2 ) (6 3 x yz x y zx yz x yz x yz x y z x yz x yz x y z x y z x y z x yz x y z xyz ++ - -+ + ++ = ⎛⎞ = ++ ++ - -+ + ⎜⎟ ⎜⎟ ⎝⎠ ⎛⎞ = + + + + + + - - ++ ⎜⎟ ⎜⎟ ⎝⎠ ⎛⎞ ≥ + + + + - - ++ ⎜⎟ ⎝⎠ + + ++ = .1) Tìm tài liệu Toán ? Chuyện nh ỏ - www.toanmath.com16 Áp dng bt ng th c AM-GM, ta có 44 2 2 2 2 2 88 22222 9 9 8 7 87 99 4499 4 4 44 4 28 2 2 7.4 2 2 9 (4 ) (2 )(2 ) 9 4 y z y z x yz xyzxx xyz xyz x y z xyz ⎛ ⎞ ⎛⎞ ⎛ ⎞⎛⎞ + +=+ + ≥ = ⎜ ⎟ ⎜⎟ ⎜ ⎟⎜⎟ ⎝ ⎠ ⎝⎠ ⎝ ⎠⎝⎠ ++ = ++ ≥ = Nhân hai b t ng th c trên v theo v , ta c 2 88 2 2 2 87 9 9 8 ( )(28 2 2 ) 9 .9 4 81 (6.2) 4 x yz x y z x y z x yz xyz ++++≥= (6.1) và (6.2) ta suy ra bt ng th c cn ch ng minh. Trong ví d trên, chúng ta ã s dng c bt ng th c Bunhiacopxki và bt ng th c AM-GM có tr ng s . L i gii rt hiu qu và n t ng. Tuy nhiên, s thành công c a l i gi i trên n m hai dòng ng n ng i u. Không có c « oán » ó, khó có th thu c kt qu mong mu n. D i ây ta s xét m t ví d v vic chn các trng s thích h p bng ph ng pháp h s bt nh các u kin xy ra du bng c tho mãn. Ví d 7. Ch ng minh r ng n u 0 xy ≤≤ thì ta có bt ng th c 11 22 222 22 13 ( ) 9 ( ) 16 xyx xyx y - + +≤ i gi i. Ta s áp d ng b t ng th c AM-GM cho các tích v trái. Tuy nhiên, n u áp d ng t cách tr c ti p thì ta c 2 2 2 2 22 22 13( ) 9( ) 9 11 (7.1) 22 x yx x yx VT xy +- ++ ≤ + =+ ây không phi là u mà ta cn (Tây ch có th suy ra 2 20 VTy ≤ ). S d ta không thu c ánh giá cn thit là vì du bng không th ng th i xy ra hai n áp d ng b t ng th c AM-GM. u ch nh, ta a vào các h s d ng , ab nh sau Tìm tài liệu Toán ? Chuyện nh ỏ - www.toanmath.com17 11 2 2 22 22 222 2 222 2 13( )( ) 9( )( ) 13( ) 9( ) (7.2) 22 ax y x by y x VT ab ax y x bxyx ab -+ =+ + - ++ ≤+ ánh giá trên úng v i m i ,0 ab > (ch ng h n v i 1 ab == ta c (7.1)) và ta s ph i ch n , ab sao cho a) V ph i không ph thuc vào x b) Du bng có th ng th i xy ra hai bt ng th c Yêu cu này t ng ng v i h 22 22 22 22 22 13( 1) 9( 1) 0 22 ,: ab ab ax yx xy bx yx ⎧ -+ += ⎪ ⎪ ⎨ ⎧ =- ⎪ ⎪ ∃ ⎨ ⎪ =+ ⎪ ⎩ ⎩ c là có h 22 22 13( 1) 9( 1) 0 22 11 ab ab ab ⎧ -+ += ⎪ ⎨ ⎪ +=- ⎩ . Gi i h ra, ta c 1 2 3 2 a b ⎧ = ⎪ ⎪ ⎨ ⎪ = ⎪ ⎩ . Thay hai giá tr này vào (7.2) ta c 22 22 222 9 13 3 16 44 xx VT yx yxy ⎛ ⎞⎛⎞ ≤ +- + ++ = ⎜ ⎟⎜⎟ ⎝ ⎠⎝⎠ Ghi chú. Trong ví d trên, th c cht ta ã c nh y và tìm giá tr l n nht ca v trái khi x thay i trong n[0,] y . 4. B t ng th c thu n nh t i x ng. Khi gp các bt ng th c dng a th c thun nht i x ng, ngoài các ph ng pháp trên, ta còn có th s d ng ph ng pháp khai tri n tr c ti p và d ng nh lý v nhóm các s h ng. Ph ng pháp này c ng k nh, không th t p nh ng ôi lúc t ra Tìm tài liệu Toán ? Chuyện nh ỏ - www.toanmath.com18 khá hiu qu. Khi s d ng bng ph ng pháp này, chúng ta th ng dùng các ký hi u quy c sau n gi n hóa cách vi t 1 2 (1) (2) ( ) ( , ,..., ) ( , ,..., ) nn sym Qxx x Qx xx σ σσ σ = ∑∑ trong ó, σ ch y qua t t c các hoán v c a { 1,2,..., } n . Ví d v i 3 n = và ba bi n s ,, xyz thì 3 3 33 2 22 sym x x yz = ++ ∑ 2 22 2 2 22 6 sym sym x y xy yz zx xz z y yx xyz xyz = ++ + ++ = ∑ ∑ i v i các biu th c không hoàn toàn i x ng, ta có th s dng ký hiu hoán v vòng quanh nh sau 2 2 22 cyc xy xy yz zx = ++ ∑ Ph ng pháp này c xây d ng d a trên tính so sánh c ca mt s tng i ng cùng bc - nh lý v nhóm các s hng (h qu ca bt ng th c Karamata) mà chúng ta s phát bi u và ch ng minh d i ây. Trong tr ng h p 3 bi n, ta còn có ng th c Schur. u 12 ( , ,..., ) n s sss = và 12 ( , ,..., ) n t ttt = là hai dãy s không tng. Ta nói rng s là tr i c a t n u 1 2 12 1 2 12 ... ... ... ... 1, nn ii ss s tt t s s s t t t in + + + = + ++ ⎧ ⎪ ⎨ + + + ≥ + + + ∀= ⎪ ⎩ . nh lý Muirhead. ( « Nhóm » ) u s và t là các dãy s th c không âm sao cho s là tr i c a t thì 1 2 12 1 2 12 ... ... nn st s s tt nn sym sym xx x xxx ≥ ∑∑ Ch ng minh. Tìm tài liệu Toán ? Chuyện nh ỏ - www.toanmath.com19 u tiên ta ch ng minh r ng n u s là tri c a t thì tn ti các hng s không âm k σ , v i σ chy qua t p h p tt c các hoán v ca {1,2,..., } n , có t ng b ng 1 sao cho (1) (2) ( ) 1 2 ( , ,..., ) ( , ,..., ) nn ks s s ttt σσ σσ σ = ∑ Sau ó, áp dng bt ng th c AM-GM nh sau (1) (2) () ((1)) ((2)) ( ()) (1) (2) () 1 2 1 2 12 , ... ... ... n nn s s s s s s t tt n nn x x x kx x x xxx σ σ σ στ στ στ σ σ σ τ σ στσ =≥ ∑ ∑∑ Ví d , v i (5,2,1) s = và (3,3,2) t = , ta có 3311 (3,3,2) .(5,2,1) . .(2,1,5) .(1, 2,5) 8888 = +++ Và ta có ánh giá 5 2 2 5 2 5 25 3 32 33 8 xyz x yz x yz xyz xyz + ++ ≥ ng bt ng th c trên và các bt ng th c t ng t , ta thu c bt ng th c 5 2 3 32 sym sym xyz xyz ≥ ∑∑ Ví d 8. Ch ng minh r ng v i m i s th c d ng ,, abc ta có 3 3 3 3 33 111 1 abc a b abc b c abc c a abc + +≤ + + + + ++ i gi i. Quy ng mu s và nhân hai v cho 2, ta có 3 3 33 3 3 3 3 33 7 44 52 2 3 3 3 33 3 63 4 4 5 22 7 6 3 5 22 ( )() 2( )( )( ) (34) ( 2 3 2) (2 2 )0 sym sym sym sym a b abc b c abc abc a b abc b c abc c a abc abc abc abc abc abc ab abc bc abc ab abc ++ ++≤ ≤ + + + + ++ ⇔ + + +≤ ≤ + +++ ⇔ -≥ ∑ ∑ ∑ ∑ t ng th c này úng theo nh lý nhóm. Tìm tài liệu Toán ? Chuyện nh ỏ - www.toanmath.com20 Trong ví d trên, chúng ta ã g p may vì sau khi th c hi n các phép bi n i i s , ta thu c mt bt ng th c t ng i n gin, có th áp dng tr c tip nh lý nhóm. Tuy nhiên, không ph i tr ng h p nào nh lý này c ng gi i quy t v n . Trong tr ng h p 3 bin s , ta có m t kt qu rt p khác là nh lý Schur. nh lý. (Schur) Cho ,, xyz là các s th c không âm. Khi ó v i mi 0 r > ( )( ) ( )( ) ( )( ) 0 r rr x x y x z y yz yx z zx z y - - + - - + - -≥ u b ng x y ra khi và ch khix yz == hay khi hai trong ba s ,, xyz bng nhau còn s th ba b ng 0. Ch ng minh. Vì bt ng th c hoàn toàn i x ng i v i ba bin s, không mt tính tng quát, ta có th gi s xyz ≥≥ . Khi ó b t ng th c có th vi t l i d i d ng ( )( ( ) ( )) ( )( ) 0 r rr x y xx z y y z z x zy z - - - - + - -≥ và m i m t th a s v trái u hi n nhiên không âm. Tr ng h p hay c s dng nht ca bt ng th c Schur là khi 1 r = . B t ng th c này có th vi t l i di d ng 22 ( 2 )0 sym x x y xyz - +≥ ∑ ây chính là b t ng th c ví d 1. Ví d 9. Cho ,, abc là các s d ng. Ch ng minh r ng 2 22 1 1 19 () 4 ( ) ( ) () ab bc ca ab bc ca ⎛⎞ ++ + +≥ ⎜⎟ + ++ ⎝⎠ i gi i. Quy ng mu s, khai trin và rút gn, ta c 5 42 3 3 4 3 2 22 2 (4 3 2 ) 0 (9.1) sym ab ab ab abc abc abc - - + - +≥ ∑ Dùng b t ng th c Schur ( )( ) ( )( ) ( )( ) 0 xxyx z yy z y x zz xzy - - + - - + - -≥ Tìm tài liệu Toán ? Chuyện nh ỏ - www.toanmath.com21 Nhân hai v v i 2xyz r i c ng l i, ta c 4 3 2 2 22 ( 2 ) 0 (9.2) sym abc abc abc - +≥ ∑ Ngoài ra, áp dng nh lý nhóm (hay nói cách khác - b t ng th c AM-GM có tr ng s ) ta có 54233 (4 3 ) 0 (9.3) sym ab ab ab - -≥ ∑ (9.2), (9.3) suy ra (9.1) và ó chính là u phi ch ng minh. Nói n bt ng th c thun nht i x ng, không th không nói n các hàm s i x ng c bn. ó là các biu th c 1 2 12 11 , ,..., ... n i ijnn i i jn S xS xx S xxx = ≤<≤ = == ∑∑ . i các b t ng th c liên quan n các hàm i x ng này, có m t th thu t r t h u hiu c gi là « th thut gi m bin s bng nh lý Rolle » . Chúng ta trình bày ý ng c a th thu t này thông qua ví d sau Ví d 10. Cho , ,, abcd là các s th c d ng. Ch ng minh r ng 11 23 64 ab ac ad bc bd cd abc abd acd bcd + + + + + + ++ ⎛ ⎞⎛⎞ ≥ ⎜ ⎟⎜⎟ ⎝ ⎠⎝⎠ i gi i. t 23 , S ab ac ad bc bd cd S abc abd acd bcd = + + + + + = + ++ . Xét a th c 4 32 23 ( ) ( )( )( )( ) ( ) P x x a x b x c x d x a b c d x S x S x abcd = - - - - = - +++ + -+ () Px có 4 nghi m th c , ,, abcd (nu có các nghi m trùng nhau thì ó là nghim i). Theo nh lý Rolle, / () Px cng có 3 nghi m (u d ng) ,, uvw. Do / () Px có h s cao nh t b ng 4 nên / 32 ( ) 4( )( )( ) 4 4( ) 4( ) 4 P x x u xv x w x u v wx uvvwwux uvw = - - - = - ++ + + +- t khác /32 23 ()43() P x x ab cdx SxS = - +++ +- Tìm tài liệu Toán ? Chuyện nh ỏ - www.toanmath.com22 suy ra 23 2( ), 4 S uv vw wu S uvw = ++= và bt ng th c cn ch ng minh u bài có th vit li theo ngôn ng ,, uvw là 1 1 2 3 () 3 uv vw wu uvw ++ ⎛⎞ ≥ ⎜⎟ ⎝⎠ t ng th c này hin nhiên úng theo bt ng th c AM-GM. 5. Thu n nh t hóa b t ng th c không thu n nh t. Trong các ph n trên, chúng ta ã trình bày các ph ng pháp c b n ch ng minh t b t ng th c thu n nh t. ó không ph i là t t c các ph ng pháp (và d nhiên không bao gi có th tìm c t t c !), tuy v y có th giúp chúng ta nh h ng t t khi g p các bt ng th c thun nht. Nh ng nu gp bt ng th c không thun nht thì sao nh? Có th bng cách nào ó a các bt ng th c không thu n nht v các bt ng th c thun nht và áp dng các ph ng pháp nói trên c không? Câu tr l i là có. Trong hu ht các tr ng h p, các bt ng th c không thun nht có th a v bt ng th c thun nht bng mt quá trình mà ta gi là thun nht hóa. Chúng ta không th “ch ng minh” mt “ nh lý” c phát bi u kiu nh th, nh ng có hai lý do tin vào nó: th nht, th c ra ch có các i ng cùng bc m i có th so sánh c, còn các i l ng khác bc ch so sánh c trong các ràng bu c nào ó. Th hai, nhi u b t ng th c không thu n nh t ã c “t o ra” b ng cách chu n hóa ho c thay các bi n s b ng các h ng s . Ch c n chúng ta i ng c li quá trình trên là s tìm c nguyên dng ban u. t ví d rt n gin cho lý lun nêu trên là t bt ng th c thun nht 3 3 3 2 22 x y z xy yz zx + + ≥ ++ , b ng cách cho 1 z = , ta c bt ng th c không thu n nh t 3 3 22 1 x y xy yx + +≥ ++ Ví d 11. (England 1999) Cho ,, pqr là các s th c d ng tho u kin 1 pqr ++= . Ch ng minh 7( ) 29 p q r pqr + + ≤+ Tìm tài liệu Toán ? Chuyện nh ỏ - www.toanmath.com23 Ví d 12. (IMO 2000) Cho ,, abc là các s th c d ng tho mãn u kin 1 abc = . Ch ng minh 111 1 1 11 a bc bca ⎛ ⎞⎛ ⎞⎛ ⎞ - + -+ -+ ≤ ⎜ ⎟⎜ ⎟⎜ ⎟ ⎝ ⎠⎝ ⎠⎝ ⎠ ng dn. t ,, x yz abc y zx = == ! Ví d 13. (IMO, 1983) Ch ng minh r ng n u ,, abc là ba cnh ca mt tam giác thì 2 22 ( ) ( ) ( )0 aba b bcb c cac a - + - + -≥ ng dn. t ,, a yzb zxc xy =+=+=+ ! Tìm tài liệu Toán ? Chuyện nh ỏ - www.toanmath.com24 Bài tp Bài 1. Cho,,0 xyz > . Ch ng minh r ng 3 3 333 3 2 22 3 33 3 33 2 22 xy z x z y xyz yz zx xy yz zx xy y zx zy x xyz ++ + ++ ≥ + + ++ + Bài 2. Ch ng minh b t ng th c sau v i mi s th c d ng ,, xyz 92 4( ) ( )( ) ( )( ) ( )( ) x yz xy z xy x z y z yx zxz y xyz ≥++ ≥ + + + + + + + + ++ Bài 3. Cho ,, xyz là các s th c d ng tho mãn u ki n2 4 72 x y z xyz + += . Tìm giá tr nh nh t c a bi u th c P xyz =++ Bài 4. Cho ,, abc là các s th c d ng tho 222 4 a bcabc + + += . Ch ng minh r ng 3 abc + +≤ Bài 5. (IMO 1984) Cho ,, xyz là các s th c không âm tho mãn u ki n 1 xyz + += . Ch ng minh ng 7 02 27 xy yz zx xyz ≤ + + -≤ Bài 6. (Iran, 1996) Cho ,,0 abc > . Ch ng minh r ng 2 22 1 1 19 () 4 ( ) ( ) () ab bc ca ab bc ca ⎛⎞ ++ + +≥ ⎜⎟ + ++ ⎝⎠ Tìm tài liệu Toán ? Chuyện nh ỏ - www.toanmath.com25 Bài 7. (VMO 1996) Cho , ,, abcd là các s th c không âm tho mãn u ki n 2( ) 16 ab ac ad bc bd cd abc abd acd bcd + + + + + + + + += Ch ng minh r ng 3()2() a b c d ab ac ad bc bd cd + ++ ≥ + + + ++ Bài 8. (Poland 1996) Cho ,, abc là các s th c tho mãn u kin 1 abc ++= . Ch ng minh r ng 2 22 9 10 1 11 abc abc ++≤ +++ Bài 9. (Poland 1991) Cho ,, xyz là các s th c tho mãn u kin 2 22 2 x yz + += . Ch ng minh r ng 2 x y z xyz + +≤+ Bài 10. (IMO 2001) Cho ,,0 abc > . Ch ng minh r ng 222 1 8 88 a bc a bc b ca c ab + +≥ + ++ Tìm tài liệu Toán ? Chuyện nh ỏ - www.toanmath.com26 PH NG PHÁP D N BI N I. M u. c m chung c a nhi u b t ng th c, c bi t là các b t ng th c i s là d u ng x y ra khi tt c hoc mt vài bi n s bng nhau. Có mt ph ng pháp ánh giá trung gian cho phép ta gi m bi n s c a b t ng th c c n ch ng minh. Ph ng pháp d n bi n d a vào c m này làm gi m s bi n s c a b t ng th c, a t ng th c v d ng n gi n h n có th ch ng minh tr c ti p b ng cách kh o sát hàm m t bi n. ch ng minh b t ng th c d ng 12 ( , ,..., ) 0, n fxxx ≥ ta ch ng minh 12 ( , ,..., ) ( , ,..., ) nn fxx x fttx ≥ Trong ó t là l ng trung bình ca 12 , ,... xx ch ng h n nh trung bình nhân ho c trung bình cng. Nu c nh vy thì tip tc sang b c th hai ca phép ch ng minh là ch ra r ng (,,...,)0 n fttx ≥ t nhiên, b t ng th c này ã gi m s bi n s i m t và th ng là d ch ng minh n bt ng th c ban u. Vic l a chn l ng trung bình nào dn bin tùy thu c vào c thù c a bài toán, và ôi khi l ng t khá c bit. Th ng thì, b c th nht trong 2 b c chính trên là khó h n c vì th c cht ta n phi làm vic v i các c l ng có ít nht là ba bin s . Sau ây là m t vài ng d n bi n th ng g p. II. Ph ng pháp d n bi n trong i s . 1. D n bi n ba bi n s . ây là ph n n gi n nh t c a ph ng pháp d n bi n. Và ng c l i c ng có th nói ph ng pháp d n bi n hi u qu nh t trong tr ng h p này. Tìm tài liệu Toán ? Chuyện nh ỏ - www.toanmath.com27 Ví d 1.1. Cho ,,0 abc ≥ tha mãn 222 3 abc ++= . Ch ng minh r ng 2 2 2 2 22 a b c ab bc ca ++≥ ++ i gi i. t 2 2 2 2 22 ( ,,) f abc a bcab bc ca =++- -- Gi s min{ , , } a abc = thì d th y 22 1, 22 a b c bc ≤ + ≥ ⇒ +≥ Xét hi u 22 22 2 2 22 2 ()1 (,,) , , () 224 2() 21 ()0 4 22 b c b c bc f abc f a b c b c bc bc ⎛⎞ ⎛⎞ +++ ⎜⎟ ⎜⎟ - =-- ⎜⎟⎜⎟ +++ ⎝⎠⎝⎠ ⎛⎞ ≥- -≥ ⎜⎟ + ⎝⎠ Do ó 22 22 2 22 2 2 2 22 22 2 22 2 2 2 2 (,,) ,, 22 () 2() () 4 (3) 2(3 ) (3 ) 4 3(1)3 ( 1) 4 2(3 ) 3 33 (1)0 44 (, ,) 0 b c bc f abc f a bc a b c abc a a a aa a a aa a f abc ⎛⎞ ++ ⎜⎟ ≥ ⎜⎟ ⎝⎠ + =+ + - +- - =+ - - -- ⎛⎞ + ⎜⎟ =-- ⎜⎟ - +- ⎝⎠ ⎛⎞ ≥ - -= ⎜⎟ ⎝⎠ ⇒≥ ng th c xy ra khi và ch khi 1. abc === Ví d 1.2. Cho ,,0 abc ≥ tha mãn 3 abc + += . Ch ng minh r ng 2 22 ( , , ) ( 1)( 1)( 1) 27 fabc a ab bc c = + + + + + +≤ i gi i. Gi s , 1, 2. a bc a bc ≤ ⇒ ≤ +≥ Xét hi u Tìm tài liệu Toán ? Chuyện nh ỏ - www.toanmath.com28 2 22 (,,) ,, 22 ( 1)( )(4() ()4) 0 16 bc bc f abc f a a a b c bc bc bc ++ ⎛⎞ -= ⎜⎟ ⎝⎠ + + - -+ -+- =≤ 2 2 2 22 (, ,) , , 22 (1)1 22 ( 1) ( ( 1)( 12 48) 37 71) 27 16 27 (, ,) 27 bc bc f abc f a b c bc aa a aaa aa f abc ++ ⎛⎞ ⇒≤ ⎜⎟ ⎝⎠ ⎛⎞ ++ ⎛⎞ = ++ ++ ⎜⎟ ⎜⎟ ⎜⎟ ⎝⎠ ⎝⎠ - - - + -- =+ ≤ ⇒≤ ng th c xy ra khi và ch khi 1. abc === Ví d 1.3. Cho ,, abc ∈ R . Ch ng minh r ng 2 22 (,,)0 f a b c a b c ab bc ca = + + - - -≥ i gi i. Xét hi u 2 22 2 3 (,,) , , .( )0 2 24 (, ,) , , ( ) 0 22 22 (, ,) 0 bc bc f abc f a b c bc bc bc bc f abc f a a a b c a f abc ++ ⎛⎞ - = -≥ ⎜⎟ ⎝⎠ ++ ++ ⎛ ⎞ ⎛ ⎞⎛⎞ ⇒ ≥ =-++ =-≥ ⎜ ⎟ ⎜ ⎟⎜⎟ ⎝ ⎠ ⎝ ⎠⎝⎠ ⇒≥ Nh n xét. Ch c ai c ng c m th y ây là m t b t ng th c quá d , quá c b n và tôi ngh ch c ng có ng i không hiu ni ti sao tôi li a ví d này vào. Nh ng hãy chú ý ng nh ng cái hay trong nh ng bài toán n gin không phi là không có và bây gi tôi s trình bày ý t ng mà tôi c m thy thích thú nht trong bài này mà mình phát hin c (có th không ch mình tôi). Vì ( ,,) f abc là hàm i x ng v i các bin ,, abc nên theo trên, ta có Tìm tài liệu Toán ? Chuyện nh ỏ - www.toanmath.com29 ( ) ,, ,, 22 ,, 22 22 ,, 2 44 ... ... b c bc f abc f a b c bc fa bc abc a bc f ++ ⎛⎞ ≥ ⎜⎟ ⎝⎠ ++ ⎛⎞ = ⎜⎟ ⎝⎠ + + + ++ ⎛⎞ ≥ ⎜⎟ ⎝⎠ =≥ Và ý t ng dãy s b t u xu t hi n. Xét các dãy s ( ),( ),( ) nnn abc c xác nh b i 0 00 22 2 1 2 21 21 21 21 22 2 1 2 2 22 ,, ,, 2 ,, 2 nn n n nn nn nn nn a ab bc c bc aabcn ac abbcn + ++ ++ ++ ++ = == + = = = ∀∈ + = = = ∀∈ N N th y lim lim lim 3 nnn n nn abc a b ct →+∞ →+∞ →+∞ ++ = = == Và ( , , ) ( , , ), nnn fabc fa b c n ≥ ∀∈ N Do hàm ( ,,) f abc liên t c nên ( , , ) ( lim , lim , lim ) ( , , ) 0 (, ,) 0 nnn n nn f a b c f a b c f ttt f abc →+∞ →+∞ →+∞ ≥ == ⇒≥ ng th c xy ra khi và ch khi . abc == Cách là trên là mt ý t ng có th nói là khá c áo và là c s hình thành nên cách th c d n bin b n bin s mà chúng ta s xét ngay bây gi . 2. D n bi n b n bi n s . Khác v i ba bin s dn bin bn bin s khó khn và ph c t p h n nhiu. Trong tr ng h p này ki u d n bi n thông th ng mà chúng ta v n làm v i ba bi n vô tác ng. Và ví d 1.3 chính là tin xây d ng nên ng li tng quát gii quy t các bài b t ng th c có th gi i b ng d n bi n k t h p dãy s . Tìm tài liệu Toán ? Chuyện nh ỏ - www.toanmath.com30 Ví d 2.1. (D tuy n IMO 1993) Cho,,,0 a b cd ≥ tha mãn 1 a bcd + + += . Ch ng minh r ng 1 176 . 27 27 abc abd acd bcd abcd + ++≤ + i gi i. t 176 (, ,, ) . 27 176 (). 27 176 (). 27 f a b c d abc abd acd bcd abcd bc a d ad b c bc ad b c bc a d ad = + ++- ⎛⎞ = + + +- ⎜⎟ ⎝⎠ ⎛⎞ = + + +- ⎜⎟ ⎝⎠ V i m i b b n s ( , , ,) abcd tha mãn 1 a bcd + + += , nu t n ti hai s trong n s này, ch ng h n , bc tha mãn 176 .0 27 b c bc +-≤ thì 3 176 (, ,, ) ( ) . 27 () 3 1 27 f a b c d bc a d ad b c bc bcad b cad ⎛⎞ = + + +- ⎜⎟ ⎝⎠ ≤+ + ++ ⎛⎞ ≤ ⎜⎟ ⎝⎠ = Do ó, không mt tính tng quát có th gi s v i mi b bn s ( , , ,) abcd tha mãn 1 a bcd + + += thì hai s b t k trong b b n s này, ch ng h n ,, ad u th a mãn 176 .0 27 a d ad +-≥ Khi ó, ta có 2 176 (, ,, ) ( ) . 27 176 (). 2 27 f a b c d ad b c bc a d ad bc ad b c a d ad ⎛⎞ = + + +- ⎜⎟ ⎝⎠ +⎛⎞⎛⎞ ≤ + + +- ⎜⎟⎜⎟ ⎝⎠⎝⎠ Tìm tài liệu Toán ? Chuyện nh ỏ - www.toanmath.com31 ,,, 22 b c bc fad ++ ⎛⎞ = ⎜⎟ ⎝⎠ Xét các dãy( ),( ),() n nn b cd c xác nh b i 0 00 22 21 2 21 21 21 21 2 2 2 1 22 22 ,, ,, 2 ,, 2 nn nnnn nn n n nn b bc cdd bc bdcdn bc bccdn + ++ ++ + + ++ = == + = = = ∀∈ + = = = ∀∈ N N Khi ó, d th y 1 1 lim lim lim 3 n nn n nn nnn abcdn a b cd →+∞ →+∞ →+∞ + + + = ∀∈ ⎧ ⎪ ⎨ - = == ⎪ ⎩ N cách t, ta có (, ,, ) (, , , ), n nn f abc d f ab c d n ≤ ∀∈ N Do f liên t c nên 2 33 2 ( , , , ) ( , lim , lim , lim ) 111 ,,, 333 1 1 1761 3. 3 3 273 (4 1) (11 14) 1 729 27 1 27 n nn nnn fabcd fa b c d aaa fa a aa aa aaa →+∞ →+∞ →+∞ ≤ --- ⎛⎞ = ⎜⎟ ⎝⎠ - -- ⎛ ⎞ ⎛⎞ ⎛⎞ = +- ⎜ ⎟ ⎜⎟ ⎜⎟ ⎝ ⎠ ⎝⎠ ⎝⎠ -- =+ ≤ ⇒ pcm. ng th c xy ra khi và ch khi 1 1 1 1 111 ( , , , ) , , , , ,,,0 4 4 4 4 333 abcd ⎛ ⎞⎛⎞ = ⎜ ⎟⎜⎟ ⎝ ⎠⎝⎠ . Ngoài cách trên ta có th làm n gi n nh sau Ta có th gi s (, , , ) , ,, 22 a d ad f abc d f bc ++ ⎛⎞ ≤ ⎜⎟ ⎝⎠ v i m i,,,0 abcd ≥ th a mãn u ki n 1 a bcd + + += (vì trong tr ng h p ng c li bài toán c gii quyt). Vì tính i x ng c a hàm (, ,, ) f abcd ta có Tìm tài liệu Toán ? Chuyện nh ỏ - www.toanmath.com32 (, ,, ) , ,, , , , 2 2 2 2 22 11 , ,, 2 2 44 11 11 1 ,,, 4 444 27 ad ad ad bcbc a d f abc d f bc f a d bc f f + + + + ++ ⎛ ⎞⎛⎞ ≤≤ ⎜ ⎟⎜⎟ ⎝ ⎠⎝⎠ ++ ⎛⎞ ≤ ⎜⎟ ⎝⎠ ⎛⎞ ≤= ⎜⎟ ⎝⎠ Cách làm trên khá hay nh ng ch có th áp dng c v i mt s ít bài toán d ng này. Ví d 2.2. Cho,,,0 a b cd ≥ tha mãn 1 a bcd + + += . Ch ng minh r ng 4 4 44 148 1 ( , , ,) 27 27 f abc d a b c d abcd =++++≥ i gi i. Xét hi u 22 7 37 (, , , ) , ,, ( ) .( ) 3 . 2 2 8 27 a b ab D f abcd f cd ab ab ab cd ++⎛ ⎞⎛⎞ = - = - - +- ⎜ ⎟⎜⎟ ⎝ ⎠⎝⎠ ó, n u có 0 (, , , ) , ,, 22 ab ab ab cd D f ab cd f cd ++ ⎛⎞ ≥⇒≥⇒≥ ⎜⎟ ⎝⎠ Gi s abc d ≥≥≥ . Xét các dãy s ( ),( ),( ) nnn abc c xác nh b i 0 00 * 2 1 21 2 2122 22 2121 212 ,, , 2 , 2 nn nn nn nn nn nn a ab bcc ac abbcn ab ab cc n -- - +++ = == + = = = ∀∈ + = = = ∀∈ N N th y 1 1 lim lim lim 33 nnn nnn nnn n nn abcdn abcdn abcd a bc →+∞ →+∞ →+∞ ⎧ ⎪ + + + = ∀∈ ⎪ ≥ ∀∈ ⎨ ⎪ ++- ⎪ === = ⎩ N N Và Tìm tài liệu Toán ? Chuyện nh ỏ - www.toanmath.com33 (,,, ) ( ,,, ) nnn fabcd fa b c d n ≥ ∀∈N Do f liên t c nên ( , , , ) ( lim , lim , lim , ) 111 ,,, 333 nnn n nn f abc d f a b c d ddd fd →+∞ →+∞ →+∞ ≤ --- ⎛⎞ = ⎜⎟ ⎝⎠ 43 4 2 1 148 1 3 3 273 (4 1) (19 20) 1 729 27 1 27 dd dd ddd -- ⎛ ⎞ ⎛⎞ = ++ ⎜ ⎟ ⎜⎟ ⎝ ⎠ ⎝⎠ -+ =+ ≥ ⇒ pcm. ng th c xy ra khi và ch khi 1 1 1 1 111 ( , , , ) , , , , ,,,0 4 4 4 4 333 abcd ⎛ ⎞⎛⎞ = ⎜ ⎟⎜⎟ ⎝ ⎠⎝⎠ . Ví d 2.3. Cho,,,0 a b cd ≥ tha mãn 4 a bcd + + += . Ch ng minh r ng 1623() abcd ab ac ad bc bd cd + ≥ + + + ++ i gi i. Ta có 2 2 22 1623() 3( ) 4 16 abcd ab ac ad bc bd cd a b c d abcd + ≥ + + + ++ ⇔ +++ +≥ t 2 2 22 (,,, ) 3( )4 f a b c d a b c d abcd = + + ++ Xét hi u 2 3 ( ,, , ) ,, , ( ) 222 cd cd D f abc d f ab c d ab ++ ⎛ ⎞ ⎛⎞ = - =-- ⎜ ⎟ ⎜⎟ ⎝ ⎠ ⎝⎠ ó nh n th y n u32 0 (,,,) ,,, 22 cd cd ab D f abc d f ab ++ ⎛⎞ ≥⇒≥⇒≥ ⎜⎟ ⎝⎠ n ây có th s d ng dãy s nh bài trc ho c có th làm nh sau Gi s 1 abc d ab ≤ ≤ ≤ ⇒≤ Tìm tài liệu Toán ? Chuyện nh ỏ - www.toanmath.com34 22 22 2 22 ( ,, , ) ,, , 22 3 3( ) .() () 2 3 ((4 ) 6) 3( ) (4 ) 2 cd cd f abc d f ab a b cd abcd ab ab ab ab ++ ⎛⎞ ⇒≥ ⎜⎟ ⎝⎠ =+ + + ++ = - - - + + + -- 22 9 ( 8 10) . 12 24 2 ( ,) x x y xx g xy = - + + -+ = Trong ó , xa byab =+= . Ta có 2 2. yx ≤≤ Xét các tr ng h p + Nu 2 22 9 94 8 10 0 ( , ) . 12 24 . 16 16 2 23 xx gxyxxx ⎛⎞ - +≥ ⇒ ≥ - + = - +≥ ⎜⎟ ⎝⎠ + Nu 2 8 100 xx - +< 2 22 22 9 ( 2) ( 4 8) ( , ) ( 8 10). . 12 24 16 16 424 x x xx gxy x x xx - -+ ⇒ ≥ - + + - + = +≥ ⇒ pcm. ng th c xy ra khi và ch khi 444 ( , , , ) (1,1,1,1), , , , 0 333 abcd ⎛⎞ = ⎜⎟ ⎝⎠ . Ví du 2.4. (Vasile Cirtoaje) Cho,,,0 a b cd ≥ tha mãn 2 2 22 1 a bcd + + += . Ch ng minh r ng (1 )(1 )(1 )(1 ) a b c d abcd - - - -≥ i gi i. Ta có B sau . (China TST 2004) Cho,,,0 a b cd ≥ tha mãn 1 abcd = . Khi ó, ta có 2 2 22 1 1 11 (, ,, ) 1 (1 )(1 )(1 )(1 ) f abcd abcd = + + +≥ + +++ Ch ng minh. th y, n u,0 xy > tha mãn 1 xy ≥ thì Tìm tài liệu Toán ? Chuyện nh ỏ - www.toanmath.com35 ( ) 2 22 1 12 (1 ) (1) 1 xy xy +≥ ++ + ó ta có n u 1 ab ≥ thì ( ) (, ,, ) , ,, f a b cd f ab ab cd ≥ Gi s abc d ≥≥≥ và xét các dãy s ( ),( ),( ) nnn abc c xác nh b i 0 00 2 1 21 2 2 21 2 2222 2121 2221 ,, ,, ,, nn nn nn n n n n nn a ab bc c a b ab c cn ab ac cb n +++ ++ + + ++ = == = = = ∀∈ = = = ∀∈ N N th y 3 3 1 1 1 lim lim lim nnn nn nnn n nn abcdn abn a b c abc d →+∞ →+∞ →+∞ ⎧ ⎪ = ∀∈ ⎪ ⎪ ≥ ∀∈ ⎨ ⎪ ⎪ === = ⎪ ⎩ N N ó ( ) ( ) ( ) ( ) 333 3 2 22 3 2 3 33 2 42 33 2 2 3 (,,, ) (, ,, ), ( , , , ) ( lim , lim , lim , ) 111 ,,, 31 (1) 1 12 2 43 1 1(1) 1 (, ,, )1 nnn nnn n nn fabcdfabcdn f abc d f a b c d fd ddd d d d d d d d dd dd f abcd →+∞ →+∞ →+∞ ≥ ∀∈ ⇒≥ ⎛⎞ = ⎜⎟ ⎝⎠ =+ + + - + + ++ =+ ++ ≥ ⇒≥ N y B c ch ng minh. Tr l i bài toán, ta có 2 2 22 1 , , , [0,1] a b c d abc d +++=⇒∈ u 0 abcd = thì (1 )(1 )(1 )(1 ) a b c d abcd - - - -≥ . u 0 abcd > . Tìm tài liệu Toán ? Chuyện nh ỏ - www.toanmath.com36 t 1 1 11 , , , ,,,0 a b cd x y z t x y zt a b cd - - -- == = = ⇒> Gi thi t 2 2 22 2 2 22 111 1 11 ( 1 ) (1 ) (1 ) (1 ) abcd x y zt +++=⇔ + + += + + ++ Và b t ng th c c n ch ng minh t ng ng v i 1 xyzt ≥ Gi s ng c li 1 xyzt < . Khi ó, t / 1 t xyz = thì / 1 xyzt = và / tt < . Áp d ng B , ta c 2 2 2 /2 2 2 22 1111 1 (1 ) (1 ) (1 ) (1 ) 111 1 1 (1 ) (1 ) (1 ) (1 ) x y zt x y zt ≤ + ++ + + ++ < + + += + + ++ y u gi s sai. 1 xyzt ⇒≥ ⇒ pcm. ng th c xy ra khi và ch khi 1 2 abcd = = == . Nh n xét. ây là mt bài toán hay và l i gii v a ri ã s dng hai công c là i bin và n bi n (v i các bi n m i). Ngoài ra có th d n bi n tr c ti p v i các bi n ban u (dành cho mi ng i). 3. D n bi n v i nhi u bi n s h n. Ví d 3.1. Cho ,,,,0 ab c de ≥ tha mãn 5 a b c de ++ + += . Ch ng minh r ng 2 2 2 22 ( , , , , ) 4( ) 5 25 f a b c d e a b c d e abcde = +++++≥ i gi i. Xét hi u 2 5 (, , , , ) , ,, , ( ) 2 . 224 d ede D f abc d e f a bc d e abc ++ ⎛ ⎞ ⎛⎞ = - =-- ⎜ ⎟ ⎜⎟ ⎝ ⎠ ⎝⎠ Tìm tài liệu Toán ? Chuyện nh ỏ - www.toanmath.com37 ó, ta có n u 8 0 (, ,, ,) , ,, , 5 22 dede abc D f abc d e f abc ++ ⎛⎞ ≤⇒≥⇒≥ ⎜⎟ ⎝⎠ . Gi s abc d e ≤ ≤ ≤≤ và xét các dãy s ( ),( ),( ) n nn cde c xác nh b i 000 * 22 22 21 2 2 21 21 * 21 21 2 21 22 ,, ,, 2 ,, 2 nn n n nn nn n n nn ccd dee de ccden ce cdden -- - - -- -- - === + = = = ∀∈ + = = = ∀∈ N N th y 1 8 min{ , ,} 5 n nn nnnn abcden ab cde nabcn + + + + = ∀∈ ≤ ≤ ∀∈ ⇒ ≤ ∀∈ Và 5 lim lim lim 33 n nn n nn c d e ab c de →+∞ →+∞ →+∞ + + -- = = == ó, ta có ( , ,, ,) (, , , , ) n nn f abc d e f abc d e n ≥ ∀∈N Suy ra 3 222 2 ( , , , , ) ( , , lim , lim , lim ) 555 ,,,, 333 4 5(5) 4( ) .(5 ) 3 27 4 4( ) 8 .(5 3 n nn n nn f abc d e f a b c d e ab a b ab f ab ab ab a b ab ab aba →+∞ →+∞ →+∞ ≥ -- - - -- ⎛⎞ = ⎜⎟ ⎝⎠ -- = ++ --+ = + - + -- 3 2 2 32 5(5) ) 27 5 (5 ) 16 40 100 8 273 () ab ab b y x xx y gy -- + - -+ = -+ = Trong ó , xa byab =+= . Ta có 3 / 10 (5 ) ()8 27 yx gy - =- Tìm tài liệu Toán ? Chuyện nh ỏ - www.toanmath.com38 + Nu 3 10 (5 ) 80 27 yx - -≥ thì 2 2 16 40 100 16 5 ( ) (0) . 25 25 3 34 xx gygx -+ ⎛⎞ ≥ = = - +≥ ⎜⎟ ⎝⎠ + Nu 3 10 (5 ) 80 27 yx - -< thì 2 2 3 22 2 32 () 4 5 (5) 4 16 40 100 8. 27 43 ( 2) ( 5 55 135 225) 25 108 x gyg x x x xx x x xx ⎛⎞ ≥ ⎜⎟ ⎝⎠ ⎛⎞ ⎛⎞ - ⎜⎟ ⎜⎟ -+ ⎝⎠⎜⎟ = -+ ⎜⎟ ⎜⎟ ⎜⎟ ⎝⎠ - - + -+ =+ dàng ch ng minh 32 5 55 135 225 0 [0,2] xx xx - + - + ≥ ∀∈ Do ó ( ) 25 gy ≥ ⇒ pcm. ng th c xy ra khi và ch khi 5555 ( , , , , ) (1,1,1,1,1), , , , , 0 . 4444 abc d e ⎛⎞ = ⎜⎟ ⎝⎠ Ví d 3.2. Cho 12 , ,..., 0 n xxx ≥ tha mãn 12 ...1 n xxx + + += . Tìm giá tr l n nh t c a bi u th c 12 1 ( , ,..., ) ( ) n ij ij i jn fxx x xx xx ≤<≤ =+ ∑ i gi i. Ta có 2 222 12 1 1 11 ( , ,..., ) . .(1 ) ij nn n j i i j ii i jn i j n i ji i fxx x xx xx x x xx ≤<≤ ≤<≤ = ≠ = ⎛⎞ = + = =- ⎜⎟ ⎜⎟ ⎝⎠ ∑ ∑ ∑ ∑∑ Xét hi u Tìm tài liệu Toán ? Chuyện nh ỏ - www.toanmath.com39 11 ( ,..., ,...,0, ) ( ,..., ,..., ,..., ) 2 (2 3( )) ij n i j n ij ij fxx x x fxx x x xx x x + - = -+ Do ó, n u3( )2 ij xx +≤ , thì 11 ( ,..., ,..., ,..., ) ( ,..., ,...,0, ) i j n i jn fxx x x fxx x x ≤+ . Xét t t c các b s 12 ( , ,..., ) n xxx sao cho 12 ( , ,..., ) n fxxxt max f . Trong ó, ch n ra b s 12 ( , ,..., ) n aaa sao cho s ph n t d ng trong b s ó là ít nh t (luôn có th ch n c vì s s d ng là h u h n). Gi s 1 2 12 ... 0 ... . k k kn aa a a aa ++ ≥ ≥ ≥ > = = == u 3 k ≥ thì ta có 23 1 2 23 23 23 3 1 ... .()3()2 22 n aa a a a aa aa aa + = ++ + ≥ ++ = + ⇒ + ≤ Do ó 12 12 3 12 3 ( , ,..., ) ( , ,0,..., ) ( , ,0,..., ) max n nn faa a faaaa faaaa f ≤ + ⇒ += u này vô lý do b s 1 23 ( , ,0,..., ) n aaaa + có s s d ng ít h n b s 12 ( , ,..., ) n aaa . y 2 k ≤ . Do ó 12 121 2 1 1 1 ( , ,..., ) ( ) (1 ) 4 n faa aaaa aa a = + = -≤ Do ó 12 1 ( , ,..., ) 4 n fxxx ≤ ng th c xy ra chng hn khi 1 2 34 1 , ... 2 n xx xxx == == = . 4. Các ki u d n bi n khác. Trong môt s tr ng h p, các kiu d n bin thông th ng ( ã nói phn m u) vô tác dng (th ng do du bng không phi xy ra khi tt c các bin bng nhau). Vì v y, xu t hi n m t s ki u d n bi n khác. Ví d 4.1. Cho,,0 xyz ≥ tha mãn 1 xy yz zx ++= . Tìm min ca 1 11 ( , ,) f x yz x y y z zx = ++ + ++ i gi i. Tìm tài liệu Toán ? Chuyện nh ỏ - www.toanmath.com40 Khác v i nh ng ví d tr c, ví d này có hai u khi n vi c d n bi n khó kh n n là c c trt c không phi khi c ba bin bng nhau và biu th c u kin a bin ht s c khó ch u. Sau ây là m t trong nh ng l i gii cho bài này. Gi s , x yz ≥ và t a yz =+ thì 1 ax ≤ và 2xa ≥ . Xét hi u 2 22 22 2 1(1)(2) (,,)0,,0 (1)(1) 1 ( 1)(2 2) 55 ( , , ) 0,, 22 2(1) ax x a ax fxyz fa a xa a aa fxyz fa a aa - -+ ⎛⎞ -=≥ ⎜⎟ ++ ⎝⎠ - -+ ⎛⎞ ⇒ ≥ = +≥ ⎜⎟ + ⎝⎠ ng th c xy ra khi và ch khi ( , , ) (1,1, 0). x yz = y 5 min ( , ,) 2 f xyz = Ví d 4.2. Cho ,,0 abc ≥ tha mãn 1 abc ++= . Tìm giá tr l n nh t c a bi u th c 3 33 ( , , ) ( 7)( 7)( 7) fabc a ab bc c = + + + + ++ i gi i. ng tính toán tr c ti p (ho c gi s cóbc = ), ta doán c max 441 f =t c ch ng h n khi 1, 0. a bc = == T ó, dn n l i gii nh sau Gi s 2 , 3 a bc bc ≤ ⇒ +≥ . t khác, do 22 0 ,, 1 1,1 a b c b c b c bc ≤ ≤⇒ + ≤+≤≤ . Xét hi u 3 22 22 3 ( , , ) ( , ,0) ( 7) ( 7( ) 1 21( )) 2 ( 7) 1 7 1 21. 3 0 f abc f ab c a a bcb c b c b c a a bc - + = ++ + + +-+ ⎛⎞ ≤ + + + +- ⎜⎟ ⎝⎠ ≤ 33 23 (, ,) ( , ,0) 7( 7)((1 ) 1 7) 7 ( 1)((1 )(2 ) 19) 441 441 f abc f ab c aa aa aa a aa ⇒ ≤+ = ++ - + -+ = - - - + + +≤ Tìm tài liệu Toán ? Chuyện nh ỏ - www.toanmath.com41 ng th c xy ra khi và ch khi ( , , ) (1,0,0). abc = y max 441 f = . III. D n bi n trong tam giác. 1. D n bi n l ng giác trong tam giác. Trong tam giác ph ng pháp dn bin a bt ng th c ã cho tr ng h p tam giác th ng v tr ng h p tam giác cân. Ví d 5.1. Cho tam giác ABC không tù. Cg ng minh r ng sin .sin sin .sin sin .sin 5 ( , ,) sin sin sin 2 BC C A AB f ABC A BC = ++≥ i gi i. Gi s , 23 ABCA ππ ≥ ⇒ ≥≥ . Xét hi u 2 22 2 2 2 sin 4sin .sin 22 (,,),, .1 2 2 sin sin .sin sin 2 . 4sin 1 sin 2 0 sin 1 2 ( , , ) , , 2sin 2sin 2 2 sin 22 cotg BCA A BC BC f ABC fA A BC BC A A BC BC BC A fABCfA AA A -⎛⎞ ⎜⎟ ++ ⎛⎞ - =- ⎜⎟ ⎜⎟ ⎝⎠ ⎜⎟ ⎝⎠ - ⎛⎞ ≥- ⎜⎟ ⎝⎠ ≥ + ++ ⎛⎞ ⇒ ≥ = + =+ ⎜⎟ ⎝⎠ t 1 2 cotg A tt = ⇒≥ . Và 2 22 1 4 1 ( 1)( 4 5) 55 2sin . 2 2 2 22 1 2( 1) .cotg A t t tt At tt - -+ + = + = +≥ ++ 5 ( ,,) 2 f A BC ⇒≥ Tìm tài liệu Toán ? Chuyện nh ỏ - www.toanmath.com42 ⇒ pcm. ng th c xy ra khi và ch khi , 24 A BC ππ = == và các hoán v t ng ng. Nh n xét. ây là d ng l ng giác c a ví d 4.1. D th y r ng d n bi n bài này d ch u và d ngh h n bài kia rt nhiu. Ví d 5.2. (VMO 1993) Cho tam giác ABC . Tìm min ca 222 ( , , ) (1 cos )(1 cos )(1 cos ) fABC ABC =+++ i gi i. + Cách 1. Gi s 1 , cos 32 ABCAA π ≤ ⇒≤⇒≥ Xét hi u ( ) 22 22 2 22 2 2 2 (,,) ,, 22 6cos cos( ) 1 (1 cos ).sin . 22 3 11 (1 cos ).sin . 22 0 (,,) ,, 22 (1 cos ) 1 cos 2 (1 cos ) 3 cos 4 (2cos 1) (4(1 cos )(4 cos ) 3) 64 B C BC f ABC fA BC A BC A BC A B C BC f ABC f A BC A AA A AA ++ ⎛⎞ -= ⎜⎟ ⎝⎠ - - -- =+ - -- ≥+ ≥ ++ ⎛⎞ ⇒≥ ⎜⎟ ⎝⎠ + ⎛⎞ =++ ⎜⎟ ⎝⎠ +- = - - -+ = 125 64 125 64 + ≥ Tìm tài liệu Toán ? Chuyện nh ỏ - www.toanmath.com43 ng th c xy ra khi và ch khi 3 ABC π = == . y 125 min(,,). 64 f ABC = + Cách 2. Gi s 3 cos 3 22 C A B CC π ≥≥⇒ ≤ ⇒ ≥ Ta có 2 2 22 2 22 22 2 (1 cos )(1 cos ) (cos cos ) (1 cos cos ) 4sin .cos cos 1 cos 2 2 22 cos 2 A B A B AB C AB AB C AB f + + = + +- --⎛⎞ = + -- ⎜⎟ ⎝⎠ - ⎛⎞ = ⎜⎟ ⎝⎠ Ta có /2 2 22 22 cos 4sin 2 cos 1 cos 2 2 22 2 cos 1 3cos 22 0 AB C ABC f ABC -- ⎛⎞ ⎛⎞ = + -- ⎜⎟ ⎜⎟ ⎝⎠ ⎝⎠ -⎛⎞ = +- ⎜⎟ ⎝⎠ ≤ Do ó 2 2 cos (1) 1 sin 22 (,,) ,, 22 ABC ff AB AB fABCfC 2 - ⎛ ⎞ ⎛⎞ ≥ =+ ⎜ ⎟ ⎜⎟ ⎝ ⎠ ⎝⎠ ++ ⎛⎞ ⇒≥ ⎜⎟ ⎝⎠ n ây, l p lu n hoàn toàn tng t nh cách 1, ta có 125 min(,,). 64 f ABC = Ví d 5.3. Cho tam giác ABC . Ch ng minh r ng 2 cos cos cos .(sin sin sin ) 2 22 3 AB BC C A ABC - -- + + ≥ ++ i gi i. Tìm tài liệu Toán ? Chuyện nh ỏ - www.toanmath.com44 Gi s , 3 A BCA π ≤ ⇒≤ . Bt ng th c cn ch ng minh t ng ng 324 cos 2cos .cos .sin .cos .cos 0 2 4 4 22 33 BC BC A A BC A π ---- + --≥ 2 4 32 1 .cos . 2cos 1 2cos .cos .sin 0 2 4 44 33 A BC BCA A π - -- ⎛⎞⎛⎞ ⇔- -+ -≥ ⎜⎟ ⎜⎟ ⎝⎠ ⎝⎠ Xét hàm s 2 4 32 ( ) 1 .cos .(2 1) 2 .cos sin 24 33 AA fx xxA π- ⎛⎞ =- -+- ⎜⎟ ⎝⎠ i 2 cos ,1 42 BC xx ⎛⎤ - = ⇒∈ ⎜ ⎥ ⎝⎦ Ta có / 43 ( ) 4 1 .cos 2cos 24 3 43 4 1 .cos 2cos 64 3 3 4 2cos 4 23 4. 2cos 24 0 4 32 ( ) (1) 1 .cos 2cos .sin ( ) 24 33 AA fxx A x A x A AA f x f A gA π ππ π π π - ⎛⎞ =-+ ⎜⎟ ⎝⎠ - ⎛⎞ ≤-+ ⎜⎟ ⎝⎠ - =-+ - <-+ < - ⇒ ≥ =- + -= Ta có / 2 2 33 ( ) .cos .sin .sin 224 33 2 32 sin cos 2sin 1 . sin cos 44 2 44 4 3 0(0) 3 ()0 3 do AA gAA AA A AA A gAg π π π - =- ++ ⎛⎞ ⎛⎞⎛ ⎞⎛⎞ = + - +- ⎜⎟ ⎜⎟⎜ ⎟⎜⎟ ⎝⎠⎝ ⎠⎝⎠ ⎝⎠ ≤ <≤ ⎛⎞ ⇒ ≥= ⎜⎟ ⎝⎠ (1) 0 f ⇒≥ Tìm tài liệu Toán ? Chuyện nh ỏ - www.toanmath.com45 ()0 ñpcm. fx ⇒≥ ⇒ Nh n xét. Vi c s d ng công c o hàm trong ph ng pháp d n bi n r t có l i khi vic bin i t ng ng ph c tp. 2. D n bi n theo các c nh. Ví d . Cho tam giác ABC tha mãn , a bc ≥ . Ch ng minh r ng 3 .() 2 abc l m m abc + + ≤ ++ i gi i. Ta coù ( ) 2 2 2 2 2 2 22 1 .() .22 22 2 (,,) abc bc lm m b c a a bc ab c bc f abc + + = + - + + - + -+ + = Tröôùc heát, ta chöùng minh 2 2 22 22 22 2 2 2 2 2 2 (1) 2 bc a bc ab ca + ⎛⎞ + -+ -+ ≤+ ⎜⎟ ⎝⎠ Thaät vaäy 2 ( )( )( )0 (1) (hieån nhieân ñuùng) bc ab c bc a ⇔ - + + + -≥ Maët khaùc, ta laïi coù 22 22 1 .() .() (2) 2 bc bca bca bc +- ≤ +- + Töø (1) vaø (2), ta coù 2 2 22 1 (,,).()2 22 , , (3) 22 bc f abc b c a a bc bc fa + ⎛⎞ ≤ + - ++ ⎜⎟ ⎝⎠ ++ ⎛⎞ = ⎜⎟ ⎝⎠ Ta seõ chöùng minh Tìm tài liệu Toán ? Chuyện nh ỏ - www.toanmath.com46 3 , , .( ) (4) 2 22 bc bc f a abc ++ ⎛⎞ ≤ ++ ⎜⎟ ⎝⎠ Thaät vaäy 2 2 22 13 (4) . ( ) 2 .( ) 2 22 bc b c a a abc + ⎛⎞ ⇔ + - + + ≤ ++ ⎜⎟ ⎝⎠ 22 22 3 1 8 31 1 8 3(1 ) ( (1,2]) ( 2) ( 2) 0 trong ñoù (hieån nhieân ñuùng) bc b c bc a aa bc x x x xx a xx + ++ ⎛ ⎞ ⎛ ⎞ ⎛⎞ ⇔ -+ + ≤+ ⎜ ⎟ ⎜ ⎟ ⎜⎟ ⎝ ⎠ ⎝ ⎠ ⎝⎠ + ⇔ - + + ≤ + = ⇒∈ ⇔ - +≤ . Keát hôïp (3) vaø (4), ta suy ra ñpcm. Ñaúng thöùc xaûy ra khi vaø chæ khiabc == Tuy ã rt c gng nh ng bài vit này cng không th vét ht các kiu và dng bài p dn bin cng nh nói v t duy và cách th c hình thành ph ng pháp. Nh ng tôi ngh nó c ng ã các b n hình thành nên ph ng pháp này trong u, t ó các bn s t cm nhn c cái hay ca ph ng pháp này cng nh các ki u dn bin khác mà bài vit này ch a cp n. Chú ý rng các l i gii trên là phù p v i bài vit này nên c ng có th có nh ng cách khác hay h n. VI. Bài t p. Bài 1. (Vietnam TST 1996) ,, Cho .Tìm giaù trò nhoû nhaát cuûa bieåu thöùc abc ∈ R 4 4 4 444 4 ()()().() 7 P a b b c c a abc = + + + + + - ++ Bài 2. (China TST 2004) ,,,0 Cho thoûa maõn 1. Chöùng minh raèng a b c d abcd >= 2 2 22 1 1 11 1 (1 )(1 )(1 )(1 ) P abcd = + + +≥ + +++ Tìm tài liệu Toán ? Chuyện nh ỏ - www.toanmath.com47 Bài 3. ,,01 Cho thoûa maõn . Chöùng minh raèng abc abc ≥ + += 3 33 151 . 44 a b c abc +++≥ Bài 4. ,,,04 Cho thoûa maõn . Chöùng minh raèng abc d a b c d ≥ + + += 2 2 2 2 22 2 22 2 22 8 abc abd acd bcd a b c a b d a c d b c d + + + + + ++≤ Bài 5. (Ph m Kim Hùng) ,,03 Cho thoûa maõn . Chöùng minh raèng abc abc ≥ ++= 2 22 ( )( )( ) 13 abbcc a abc + + +≤+ Bài 6. ,,,04 Cho thoûa maõn . abc d a b c d ≥ + + += a) Ch ng minh r ng ( ) 24 a b c d abc abd acd bcd + + + ≥ + +++ b) Tìm min c a ( ) 7 P a b c d abc abd acd bcd = + + + - + ++ Bài 7. Cho tam giác nh n ABC . Ch ng minh r ng 2 22 sin .sin sin .sin sin .sin 9 sin sin sin 4 BCCAAB A BC ⎛ ⎞⎛ ⎞⎛ ⎞ ++≥ ⎜ ⎟⎜ ⎟⎜ ⎟ ⎝ ⎠⎝ ⎠⎝ ⎠ Bài 8. Cho tam giác ABC . Ch ng minh r ng ( ) 2 3 34 pRr ≤ +- Bài 9. Cho ,,,,0 ab c de ≥ tha mãn 1 a b c de + + + += . Ch ng minh r ng 5 5 5 55 1845 1 . 256 256 a b c d e abcde + ++ ++ ≥ Tìm tài liệu Toán ? Chuyện nh ỏ - www.toanmath.com48 Bài 10. ,,03 Cho thoûa maõn . abc a b c ≥ + += Tìm giá tr l n nht và giá tr nh nht ca biu th c ( )( )( ) 2 22 3 33 P a a b b cc = + + + + ++ Bài 11. (Ph m Kim Hùng) Cho ,,0 abc > tha mãn 1 abc = . Ch ng minh r ng 2 22 3 33 3 ( 1) ( 1) ( 1) abc abc + ++ ++≥ +++ Bài 12. Cho,,0 xyz ≥ tha mãn 1 xy yz zx ++= . Ch ng minh r ng 1 1 11 2 2 y z z x xy + + ≥+ + ++ Bài 13. ,,01 Cho thoûa maõn . abc a b c ≥ ++= . Ch ng minh rng 1 1 12 1 111 3 abc abc - -- ++≤+ +++ Bài 14. Cho,,,0 a b cd ≥ . Ch ng minh r ng 4 44 4 33 3 3 3( )4( )() a b c d abcd a b c d a b c d + ++ + ≥ + + + ++ + Bài 15. (Ph m Kim Hùng) ,,03 Cho thoûa maõn . abc a b c ≥ + += Ch ng minh r ng 33 3333 333 36( ) ( )( ) ab bc ca a b b c c a a b c + + ≥ + + ++ Bài 16. (Võ Qu c Bá C n) ,,01 Cho thoûa maõn . abc a b c ≥ ++= Tìm min 44 4444 4 44 ( )() ab bc ca P abbcca abc ++ = + + ++ Tìm tài liệu Toán ? Chuyện nh ỏ - www.toanmath.com49 N BI N KHÔNG XÁC NH I. D n bi n không xác nh. Cái tên nghe có v l nh ? tìm hi u ph ng pháp mi m này chúng ta hãy cùng bàn n hai bài toán quen thu c sau Bài toán 1. Cho n là s nguyên d ng và 12 , ..., n xxx là các s th c thuc n[,] pq v i , pq là hai s th c cho tr c. Tìm giá tr l n nh t c a bi u th c 12 ( , ..., ) n fxxx Bài toán 2. Cho n là s nguyên d ng và là 12 , ..., n xxx các s th c không âm có tng b ng n. Tìm giá tr nh nht ca biu th c 12 ( , ..., ) n fxxx c hai bài trên thì 12 ( , ..., ) n fxxx u là các bi u th c i x ng ca 12 , ..., n xxx ) Thông th ng i v i các Bài toán 1 chúng ta th ng sp th t các bin và dn giá tr ca bin v hai biên so sánh tr c tip chúng. Chng hn so sánh 12 ( , ..., ) n fxxx v i 2 ( , ..., ) n fpxx v i m c ích là a bài toán v tr ng h p n gin v i s l ng bin ít h n. Còn v i Bài toán 2 chc chn các bn s ngh ngay n ánh giá 12 12 12 ( , ..., ) , ,..., 22 nn xx xx fxxxfx ++ ⎛⎞ ≥ ⎜⎟ ⎝⎠ ho c hi h u l m thì chúng ta có ánh giá 1 2 12 ( , ..., ) (0, ,..., ) nn fxx x f x xx ≥+ . Có th thy nh ng suy ngh nh trên là vô cùng t nhiên nh ng nói chung là khó th c hi n vì nh ng bài có th gi i tr c tip là t ng i n gin. Vì v y chúng ta cn m t b c phát trin h n cho ph ng pháp này ó là dn bin không xác nh. Vy dn bin không xác nh là gì? Tôi có th gi i thi u luôn t t ng chính c a ph ng pháp này là “D n các bi n do v m t trong nh ng m c bi t mà ta ch a th xác nh rõ s d n c th v m c bit nào”. Có v h i khó hiu phi không? Chúng ta s cùng quay tr li i 2 bài toán trên Tìm tài liệu Toán ? Chuyện nh ỏ - www.toanmath.com50 (i) V i Bài toán 1, thay vì ch ng minh 122 ( , ..., ) ( , ,..., ) nn fxx x fpxx ≤ chúng ta s ch ng minh 12 22 ( , ..., ) { ( , ,..., ), ( , ,..., )} max n nn f x x x f px x f qx x ≤ (ii) V i Bài toán 2, thay vì ánh giá ã nói trên chúng ta s ch ra c 12 12 1 2 12 ( , ..., ) min , ,..., , (0, ,..., ) 22 n nn x x xx fxx x f x f x xx ++⎧⎫ ⎛⎞ ≥+ ⎨⎬ ⎜⎟ ⎝⎠ ⎩⎭ c n ây bn ng vi c i vì nó ch tin b h n ph ng pháp ban u m t chút khi u ki n d n bi n c n i l ng mà trông l i có v ph c t p v i max, min ng nh ng! B n hãy xem th s c m nh ca t t ng này thông qua ví d quen thu c sau ây nh ng trc h t chúng ta hãy n v i B c b n 1. Cho ,, abc là các s th c th a mãnbc ≥ . Khi ó ít nh t m t trong hai b t ng th c sau úng (i)ac ≥ (ii)ab ≤ Ch ng minh. Gi s c hai b t ng th c trên u sai ta suy rac abc > >≥ (Mâu thu n). qu 1. Cho , ab là các s th c. Khi ó ít nht m t trong hai bt ng th c sau úng (i)ab ≥ (ii)ab ≤ Các bn ng nên xem th ng B 1, tuy ây là mt B n gin theo úng ngh a c a nó nh ng l i là m t B c c k hi u qu y. Sau ây là m t ví d cho th y u ó Ví d 1. Cho , pq là hai s th c d ng, n là s nguyên d ng và 12 , ,..., n xxx là các s th c thuc n [,] pq v i , pq là hai s th c d ng cho tr c. Tìm giá tr l n nht c a biu th c Tìm tài liệu Toán ? Chuyện nh ỏ - www.toanmath.com51 1 2 12 12 111 ( , ,..., ) ( ... ) ... nn n fxx x xxx xxx ⎛⎞ = + ++ + ++ ⎜⎟ ⎝⎠ i gi i. t 23 23 111 ... , ... n n S x x xT xxx = + + + = + ++ 122 1 1 1 1 1 ( , ,..., ) ( , ,..., ) 11 ( ) () ()0 (1) nn fxxx fpxx xS T pST xp S x pT px S T px ≤ ⎛⎞ ⎛⎞ ⇔ + + ≤++ ⎜⎟ ⎜⎟ ⎝⎠ ⎝⎠ ⎛⎞ ⇔ - -≤ ⎜⎟ ⎝⎠ ⇔≤ 122 1 1 1 1 1 ( , ,..., ) ( , ,..., ) 11 ( ) () ()0 (2) nn fxxx fqxx xS T qST xq S x qT qx S T qx ≤ ⎛⎞ ⎛⎞ ⇔ + + ≤++ ⎜⎟ ⎜⎟ ⎝⎠ ⎝⎠ ⎛⎞ ⇔ - -≤ ⎜⎟ ⎝⎠ ⇔≥ Do 11 SS px qx ≥ nên theo B 1 s có ít nht mt trong hai bt ng th c (1), (2) úng. Suy ra 12 22 ( , ..., ) { ( , ,..., ), ( , ,..., )} max n nn f x x x f px x f qx x ≤ Hoàn toàn t ng t ta nh n c k t qu sau n t i 12 , ,..., { , } n y y y pq ∈ sao cho 1 2 12 ( , ,..., ) ( , ,..., ) nn fxx x fyyy ≤ Bài toán a v tìm giá tr l n nht c a 12 ( , ,..., ) n fyyy v i 12 , ,..., { , } n y y y pq ∈ . Không quá khó kh n chúng ta tìm c Tìm tài liệu Toán ? Chuyện nh ỏ - www.toanmath.com52 22 12 () max ( , ,..., ) 4 n npq fyyy pq + += v i n chn khi trong tp 12 { , ,..., } n yyy có 2 n sng p và 2 n s còn l i b ng q . 22 12 ( 1)() max ( , ,..., ) 1 4 n n pq fyyy pq -+ + =+ v i n l khi trong tp 12 { , ,..., } n yyy có 1 2 n - s b ng p và 1 2 n + s còn l i b ng q ho c ng c l i. ây chúng ta i n k t lu n cho bài toán. Chc hn các bn ã t ng gi i quyt bài toán này bng cách s dng ph ng pháp hàm li cng rt nhanh gn nh ng có l chúng ta phi công nhn v i nhau rng cách gii bng t t ng dn bin không xác nh trên rt p và phù h p v i trình c a c các bn Trung h c c s . Bng phép ch ng minh t ng t , chúng ta có th gi i c bài toán sau Ví d 2. Cho , pq là hai s th c d ng, n là s nguyên d ng và 12 , ,..., n xxx là các s th c thu c n[,] pq . Tìm giá tr l n nh t c a bi u th c 12 12 12 ... ( , ,..., ) ... nnn n n n xxx fxxx xxx + ++ = hai ví d trên u ã có trên t p chí Toán H c Và Tu i Tr cùng v i tr ng h p 3,1,2 n pq = == tuy nhiên cách ch ng minh theo tôi c bi t r t thi u t nhiên và khó có kh n ng gi i t ng quát. Nh v y là i v i các bài toán bt ng th c có biên rõ ràng nh Bài toán 1 thì chúng ta ã có m t l i gii h p lý còn v i Bài toán 2 thì sao? Dù các bin không m trong mt gi i hn rõ ràng nh ng chúng ta có th t m hiu c rng v i hai bin 12 , xx thì chúng luôn nm trong 12 [0,] xx + và có nh ng cp m c bit cn chú ý là 12 (0, ) xx + và 12 12 , 22 xx xx ++ ⎛⎞ ⎜⎟ ⎝⎠ . gi i quy t tri t Bài toán 2 chúng ta c th hóa t t ng d n bi n không xác nh b ng nh lý sau Tìm tài liệu Toán ? Chuyện nh ỏ - www.toanmath.com53 II. nh lý d n bi n không xác nh U.M.V (Undefined Mixing Variables). nh lý U.M.V. Cho 12 , ,..., n xxx là các s th c không âm có tng là m t hng s ng cho tr c. 12 ( , ,..., ) n fxxx là mt hàm liên tc, i x ng ca 12 ( , ,..., ) n xxx th a mãn u ki n 12 12 1 2 12 ( , ,..., ) min , ,..., , (0, ..., ) 22 n nn xx xx fxx x f x f x xx ++⎧⎫ ⎛⎞ ≥+ ⎨⎬ ⎜⎟ ⎝⎠ ⎩⎭ i mi 12 ( , ,..., ) n xxx th a mãn u kin ã cho. Khi ó, giá tr nh nht c a 12 ( , ,..., ) n fxxx là giá tr nh nh t ca ( 0,1,2,..., 1) t Ctn =- trong ó ( 0,1,2,..., 1) t Ctn =- là giá tr ca 12 ( , ,..., ) n fxxx khi trong 12 ( , ,..., ) n xxx có t s bng 0 và nt - s còn l i b ng nhau. Ch ng minh. Tr c h t, ta ch ng minh B sau 2. Cho mt b s th c không âm 12 ( , ,..., ) ( 2) n xx xn ≥ th c hi n phép bi n i Δ nh sau Chn 12 ( , ,..., ) max in x xxx = và 12 ( , ,..., ) min jn x xxx = . Gán , ij xx b i 2 ij xx + nh ng v n gi nguyên v trí c a chúng trong 12 ( , ,..., ) n xxx . Khi ó sau vô h n l n th c hi n ta c 12 12 ... ... n n xxx xxx n + ++ == == . Ch ng minh. Ký hi u dãy ban u là 111 12 ( , ,..., ) n xxx . Ta ch ng minh b ng quy n p. i 2 n = thì B hi n nhiên úng. Gi s b úng v i:1 nn =- ta ch ng minh nó úng v i : nn = . Tht vy, gi s ln th k nào ó th c hi n phép bi n i Δ ta s nhn c b 12 ( , ,..., ) kkk n xxx . i 12 12 min{ , ,..., }, { , ,..., } max kkk kkk k nkn m xxx M xxx == . Tìm tài liệu Toán ? Chuyện nh ỏ - www.toanmath.com54 th y{} k m là dãy không gi m b ch n trên b i 1 M nên lim k k mm →∞ ∃= , còn{} k M là dãy không tng b chn d i b i 1 m nên lim k k MM →∞ ∃= . u b c th k nào ó th c hi n phép bi n i Δ mà 1 k k xm = ho c 1 k k xM = thì 1 x c gi là có tham gia vào phép bin i Δ b c th k . i 12 ... s uuu < << là t t c nh ng l n 1 x tham gia phép bin i d i vai trò s nh nh t, còn 12 ... t vvv < << là t t c nh ng l n 1 x tham gia phép bi n i d i vai trò s l n nh t. *) Nu st + <∞ , t 0 { ,} max k st = suy ra t b c 0 k tr i thì 1 x s không tham gia vào phép bin i Δ n a. Nh th ta ch áp dng phép bin i này cho b 0 00 23 ( , ,..., ) kkk n xxx . Áp dng gi thit quy np, ta nhn c b 0 00 0 00 23 23 ... ... 1 k kk k kk n n xxx xxx n + ++ == == - . Do 1 x không tham gia vào phép bi n i Δ nào n a nên 0 00 0 00 23 12 ... ... 1 k kk k kk n n xxx xxx n + ++ == == - ây ta có pcm. **) N ust + =∞ . Không gi m t ng quát, gi s s =∞ suy ra 1 lim k u k xm →∞ = . + Tr ng h p 1. t <∞ Do lim ,lim kk kk m m MM →∞ →∞ == nên theo nh ngh a gi i h n thì v i m i 0 > ε nh thì 1 n ∃ sao cho v i m i 1 Nn > thì N mm - <ε 2 n ∃ sao cho v i m i 2 Nn > thì N MM - <ε Chn 3 12 { , ,} max t n vnn = , suy ra v i mi 3 1 i un -> thì 11 , ii uu m m MM -- -< -< εε Tìm tài liệu Toán ? Chuyện nh ỏ - www.toanmath.com55 mà 11 1 2 ii i uu u mM x -- + = nên 1 2 i u Mm x + -< ε 1 1 lim 2 lim 2 i u i k k Mm x Mm x →∞ →∞ + ⇒= + ⇒= + Tr ng h p 2. t =∞ . Hoàn toàn tng t ta suy ra 1 1 lim 2 lim 2 i i u i v i Mm x Mm x →∞ →∞ + = + = Vì v y 1 lim 2 k k Mm x →∞ + = Do ó trong m i tr ng h p ta u có 1 lim 2 k k Mm x →∞ + = Hoàn toàn t ng t ta nhn c kt qu sau lim 2 k i k Mm x →∞ + = v i m i 1,2,..., in = nên ta có pcm. Ch ng minh nh lý. Th c hi n thu t toán t β v i {0,1,2,..., 1} tn ∈- cho tr ng hp t p 12 ( , ,..., ) n xxxã có t s 12 ...0 t xxx = = == nh sau cho gn ta quy c 12 ( , ,..., ) ( , ) n ij fxx x fxx = trong ó 12 12 {, ,..., }, min{, ,..., } max i njn x xxxx xxx == tha mãn 0 j x > . Ti n hành so sánh ( ,) ij fxx v i , 22 i j ij xx xx f ++ ⎛⎞ ⎜⎟ ⎝⎠ và (0, ) ij f xx + . *) Nu (,) (0,) i j ij fxx f xx <+ thì(,), 22 ij ij ij xx xx fxxf ++ ⎛⎞ ≥ ⎜⎟ ⎝⎠ . Khi ó áp d ng thut toán Δ cho 12 { , ,..., } ttn xxx ++ . Nu trong m t b c nào ó li có Tìm tài liệu Toán ? Chuyện nh ỏ - www.toanmath.com56 (,) (0,) i j ij fxx f xx ≥+ thì chuyn sang thut toán 1 t+ β . Nu không có thì phép bi n i Δ s c th c hi n vô h n l n nên 12 ... t tn xxx ∞ ∞∞ ++ = == . **) N u(,) (0,) i j ij fxx f xx ≥+ ta chuyn tr c tip sang thut toán 1 t+ β . Rõ ràng thut toán 1 n- β ã là thu t toán h ng và ó là k t qu c nh. Vì vy nh lý ã c ch ng minh hoàn ch nh. Trong nh lý U.M.V ta có th thay th u kin tng các bin bng các u kin khác nh t ng bình ph ng, t ng l p ph ng...và có cách d n bi n t ng ng thì nh lý v n úng và cách ch ng minh không có gì khác. qu. Cho 12 , ,..., n xxx là các s th c không âm có t ng là mt hng s d ng cho tr c. 12 ( , ,..., ) n fxxx là m t hàm liên t c, i x ng c a 12 ( , ,..., ) n xxx th a mãn u ki n 12 12 1 2 12 23 12 1 2 12 ( , ..., ) min , ,..., , (0, ..., ) 22 (0, , ,..., ) 0 ... ... ... , ,..., 0 n nn n n nn x x xx fxxx f x f x xx f xxx x xxx xx x xx f n nn ⎧ ++⎧⎫ ⎛⎞ ≥+ ⎨⎬ ⎪ ⎜⎟ ⎝⎠ ⎩⎭ ⎪ ⎪ ≥ ⎨ ⎪ ++ ++ ++ ⎛⎞ ⎪ ≥ ⎜⎟ ⎪ ⎝⎠ ⎩ v i m i 12 ( , ,..., ) n xxx th a mãn u ki n ã cho thì 12 ( ,,..., )0 n fxxx ≥ . III. M t s ng d ng c a ph ng pháp d n bi n không xác nh. s dng ph ng pháp dn bin không xác nh rõ ràng ta phi th c hin theo trình t hai bc c 1. Xác l p u ki n d n bi n. c 2. Gi i quy t bài toán v i u ki n ã xác l p bên trên. n nhiên B c 2 chính là n i dung c a nh lý U.M.V và ã c gi i quy t m t cách hoàn toàn trit . Do ó, phn quan tr ng nht ca chúng ta cn phi làm ó là th c hi n c B c 1. M t u kì l là b c này th ng c x lý r t g n nh ng cách s dng B 1, mt b g n nh hi n nhiên d a trên quan h th ta các s trên trc s th c. Chúng ta hãy tìm hiu rõ h n qua các ví dc tr ng sau Tìm tài liệu Toán ? Chuyện nh ỏ - www.toanmath.com57 Ví d 3. (Phát tri n t m t bài IMO) Cho n là s nguyên d ng và 12 , ,..., n xxx là các s th c không âm có t ng b ng n . Tìm s th c d ng n k t t nh t b t ng th c sau luôn úng 1 2 12 (1 )(1 )...(1 ) 2 .( ... 1) n n nn x x x k xxx ++ +≤+- i gi i. t 1 2 1 2 12 ( , ,..., ) (1 )(1 )...(1 ) 2 .( ... 1) n n n nn fxx x x x x k xxx =++ +--- 34 34 (1 )(1 )...(1 ) ... n n S x xx T xxx =+ ++ = 1 2 12 12 22 1 2 12 1 2 12 2 12 ( , ,..., ) , ,..., (3.1) 22 (1)(1)10 22 ( )0 2 (3.2) nn nn n n x x xx fxxxfx x x xx xxSkxxT SkT xx kTS kTS ++ ⎛⎞ ≤ ⎜⎟ ⎝⎠ ++ ⎛ ⎞ ⎛⎞ ⇔++- -+ +≤ ⎜ ⎟ ⎜⎟ ⎝ ⎠ ⎝⎠ - ⎛⎞ ⇔ -≤ ⎜⎟ ⎝⎠ ⇔≤ ( )( ) 1 2 12 1 2 12 12 ( , ,..., ) (0, ,..., ) (3.3) (1 )(1 ) (1 ) 0 (3.4) nn n n f xxx f x xx x xS kxxT x xS kTS ≤+ ⇔+ + - -++≤ ⇔≥ (3.2), (3.4) ta có ngay ít nh t m t trong hai b t ng th c (3.1), (3.3) úng suy ra 12 12 1 2 12 ( , ..., ) , ,..., , (0, ..., ) 22 max n nn xx xx fxx x f xf x xx ++⎧⎫ ⎛⎞ ≤+ ⎨⎬ ⎜⎟ ⎝⎠ ⎩⎭ Theo nh lý U.M.V ta có 12 max ( , ,..., ) max ( 0,1,..., 1) nt fxxx Ctn = =- 01 max{ , } CC = 1 21 max 0, 2 1 n n n n k n - ⎧⎫ -⎪⎪ ⎛⎞ = -+ ⎨⎬ ⎜⎟ - ⎝⎠ ⎪⎪ ⎩⎭ Vì v y b t ng th c bài th a mãn thì Tìm tài liệu Toán ? Chuyện nh ỏ - www.toanmath.com58 1 1 21 20 1 21 2 1 n n n n n n n k n n k n - - - ⎛⎞ -+≤ ⎜⎟ - ⎝⎠ - ⎛⎞ ⇔≤- ⎜⎟ - ⎝⎠ Do ó giá tr t t nh t c a n k th a mãn bài là 1 21 2 1 n n n n k n - - ⎛⎞ =- ⎜⎟ - ⎝⎠ Ví d 3 th c s là mt bài toán rt khó ã t ng có mt dng này hay dng khác trong các thi vô ch. Chc chn các bn ã t ng c m nhn c biu th c t giá tr t t nh t ngoài tr ng h p n bi n b ng nhau thì còn m t tr ng h p m t bi n ng 0 nh ng vn vô cùng t c t i vì không có cách nào ép nó v c 0. Giây U.M.V ã cho bn m t h ng i khá sáng s a. Ví d 4. ( inh Ng c An) Chopn ≤ là các s nguyên d ng và 12 , ,..., n xxx là các s th c không âm có t ng ng n. Tìm giá tr l n nh t c a bi u th c 12 12 12 1 ... ( , ,..., ) ( ... ) p p k n iii i i in fxx x xxx ≤ ≤ ≤ ≤≤ = ∑ Trong ó k là s th c không nh h n 2. i gi i. t 122 122 121 121 12 12 3 ... 3 ... 3 ... ( ...) ( ...) ( ...) p p p p p p k iii ii in k iii ii in k iii i i in A xxx B xxx C xxx - - - - ≤≤≤≤≤ ≤≤≤≤≤ ≤ ≤ ≤ ≤≤ = = = ∑ ∑ ∑ Ta s ch ng minh 12 12 1 2 12 ( , ..., ) , ,..., , (0, ..., ) 22 max n nn xx xx fxx x f xf x xx ++⎧⎫ ⎛⎞ ≤+ ⎨⎬ ⎜⎟ ⎝⎠ ⎩⎭ Th t v y Tìm tài liệu Toán ? Chuyện nh ỏ - www.toanmath.com59 1 2 12 12 2 12 12 12 12 ( , ..., ) , ,..., (4.1) 22 () 20 22 nn kk kk kk xx xx fxxxfx xx xx xxAxxB AB ++ ⎛⎞ ≤ ⎜⎟ ⎝⎠ ++ ⎛⎞ ⎛⎞ ⇔++- -≤ ⎜⎟ ⎜⎟ ⎝⎠ ⎝⎠ 2 12 12 12 12 2 2 2 k kk k kk xx xx B A xx xx + ⎛⎞ - ⎜⎟ ⎝⎠ ⇔≥ + ⎛⎞ +- ⎜⎟ ⎝⎠ 1 2 12 12 1 2 12 12 1 2 12 ( , ..., ) (0, ..., ) (4.2) ( ) ( )0 () nn kkkkk kk kkk fxxx f x xx xxA x x B x xB xx B A x x xx ≤+ ⇔ + + -+≤ ⇔≥ + -- ít nh t m t trong hai b t ng th c (4.1), (4.2) ch c ch n úng thì 2 12 12 12 1 2 12 12 12 2 12 12 12 12 12 1 2 12 2 1 2 12 12 12 1 2 12 1 2 () 2 2 2 22 () ( )2 22 () ( k kk kk k kkk kk kk kk kk kk kkk kk k kk kkk xx xx xx x x xx xx xx x x xx xx xx xx x x xx x x xx xx xx x x xx xx + ⎛⎞ - ⎜⎟ ⎝⎠ ≥ + -- + ⎛⎞ +- ⎜⎟ ⎝⎠ ++ ⎛⎞ ⎛⎞ - +- ⎜⎟ ⎜⎟ ⎝⎠ ⎝⎠ ⇔≥ + -- ++ ⎛⎞ ⎛⎞ +- ⎜⎟ ⎜⎟ ⎝⎠ ⎝⎠ ⇔≥ + -- + ⇔ 21 2 12 21 12 1 2 12 21 1 2 1 2 1 2 12 ) (2 1)() 2 2(( )) ( )(( ) ) (2 2) k kk kkk k kkk k kkk k k kk xx xx x x xx x x x x x x xx - - + -+ ≥ + -- ⇔ + + - - ≥- u này hi n nhiên do 2 1 2 12 ( ) 2() k kk x x xx +≥ (theo b t AM-GM) 2 1 2 1 2 12 ( ) (2 2)( ) k kkk k x x x x xx + - - ≥- v i 2 k ≥ Tìm tài liệu Toán ? Chuyện nh ỏ - www.toanmath.com60 y ta có 12 12 1 2 12 ( , ..., ) , ,..., , (0, ..., ) 22 max n nn xx xx fxx x f xf x xx ++⎧⎫ ⎛⎞ ≤+ ⎨⎬ ⎜⎟ ⎝⎠ ⎩⎭ Vì th theo nh lý U.M.V ta có 12 max ( , ,..., ) max ( 0,1,..., 1) max . ( 0,1,..., 1) nt kp p nt fxxx Ctn n C tn nt - = =- ⎛⎞ = =- ⎜⎟ - ⎝⎠ 1 max ,. 1 kp pp nn n CC n - ⎧⎫ ⎪⎪ ⎛⎞ = ⎨⎬ ⎜⎟ - ⎝⎠ ⎪⎪ ⎩⎭ i 3 n = ta có bài toán quen thu c Cho,,,0 a bck ≥ tha mãn 3 abc + += . Ch ng minh r ng 2 ( )( )( ) 3 ax 3, 2 k kkk ab bc ca m ⎧⎫ ⎪⎪ ⎛⎞ + +≤ ⎨⎬ ⎜⎟ ⎝⎠ ⎪⎪ ⎩⎭ n th y có u gì kì l không? Hình nh U.M.V này ch ng thèm quan tâm n s bin 3 n = hay n bt kì thì cng th. Ví d 5. (t ng quát t b t Turkervici) Cho n là s nguyên d ng và 122 , ,..., n xxx là các s th c không âm. Ch ng minh ng 2 22 1 2 2 122 12 ( 1)( ... ) ... n n n nn n n ij ijn n x x x nxx x xx ≤<≤ - ++++≥ ∑ i gi i. t ng th c ã cho t ng ng v i 2 2 2 22 1 2 2 122 1 (2 1)( ... ) 2 ... n nnnn n ni i n xx x nxxxx = ⎛⎞ - ++++≥ ⎜⎟ ⎝⎠ ∑ t 2 2 2 22 1 2 2 1 2 2 122 1 ( , ,..., ) (2 1)( ... ) 2 ... n nnnn n n ni i fxx x n xx x nxxxx = ⎛⎞ =- ++++- ⎜⎟ ⎝⎠ ∑ Tìm tài liệu Toán ? Chuyện nh ỏ - www.toanmath.com61 12 s xx = 12 12 2 nn n xx t xxs + = ≥= Ta có 12 12 12 2 32 2 22 1 2 2 122 2 2 22 12 12 3 2 32 ( , ,..., ) , , ,..., (5.1) 22 (2 1)( ... ) 2 ... (2 1) 2 ... 2 ... 0 22 n n nn nn nn n nn nn nn nn nn n nn x x xx fxx x f xx n x x x nxxx xx xx n x x n xx ⎛⎞ ++ ⎜⎟ ≥ ⎜⎟ ⎝⎠ ⇔- ++++- ⎛⎞⎛⎞ ⎛⎞ ++ ⎜⎟ ⎜⎟ -- +++-≥ ⎜⎟ ⎜⎟ ⎜⎟ ⎝⎠ ⎝⎠ ⎝⎠ 2 32 12 121 ... () (21)0 2 ... nn n nnn nxx xx n t tss --- -⎛⎞ ⇔ --≥ ⎜⎟ + ++ ⎝⎠ 121 32 21 .( ... ) ... nnn n n t t s s xx n --- - ⇔ +++≥ ( ) 12 2 1 2 32 2 2 2 22 12 2 122 122 11 12 32 12 11 3 2 12 ( , ,..., ) 0, , ,..., (5.2) (2 1)( ... ) 2 ... (2 1)(( ) ... ) 0 21 ... .0 21 .... nn n nn nn n nnn nnn nn n nn n fxx x f x xxx nxx x nxxx n xxx n xx x x xx n n x x xx n -- -- ≥+ ⇔ - + ++ + - - + ++ ≥ - ⎛⎞ ⇔ -≥ ⎜⎟ ⎝⎠ - ⇔≥ Vì 1 2 1 1 11 12 21 21 21 .( ... ) . n n n n nn n nn t t s s s xx n nn - - - - -- - -- +++≥ = nên theo B 1 thì có ít nh t m t trong hai b t ng th c (5.1), (5.2) úng. y ( ) 12 12 12 2 32 1 2 32 ( , ,..., ) min , , ,..., , 0, , ,..., 22 n n nn nn n nn n nn x x xx fxx x f xx f x xxx ⎧⎫ ⎛⎞ ++⎪⎪ ⎜⎟ ≥+ ⎨⎬ ⎜⎟ ⎪⎪ ⎝⎠ ⎩⎭ Theo Bt ng th c Bunhiacopxki thì 2222 2 3 2 232 (2 1)( ... ) ( ... ) n n n nnn nn n x x x xxx - + ++ ≥ + ++ nên 232 (0, , ,..., ) 0 n f xxx ≥ . Tìm tài liệu Toán ? Chuyện nh ỏ - www.toanmath.com62 t khác 2 (,,...,) 0 soá nt fttt = 123 nên theo H qu c a nh lý U.M.V, ta có u ph i ch ng minh. ng th c x y ra khi và ch khi 122 ... n xxx = == hoc 1 232 0, ... n x xxx = = == và các hoán v. Ví d 5 là bài toán t ng quát c a B t ng th c Turkervici ( 4) n = . Trên th c t v i tr ng h p riêng này, bài toán ã rt khó và v i tr ng h p t ng quát nó ã th hin c g n nh toàn b v p c a ph ng pháp này... B n th y không? Nó c ng “d th ng” y ch ? Tìm tài liệu Toán ? Chuyện nh ỏ - www.toanmath.com63 Bài t p ng dng Bài 1. (inh Ngc An) Cho n là s nguyên d ng và 12 , ,..., n xxx là các s th c thu c [1, 2] . Tìm giá tr l n nh t c a bi u th c 2 1 2 12 12 111 ( , ,..., ) ( ... ) ... nn n fxx x xxx xxx ⎛⎞ = + ++ + ++ ⎜⎟ ⎝⎠ Bài 2. (inh Ngc An) Cho ,, abc là các s th c không âm có tng b ng 3, , km là các s th c tha mãn 1,0 km ≥≥ . Tìm giá tr l n nh t c a bi u th c 222 (,,) [( )( )( )] k k k kkk f a b c a b c m ab bc ca = + + + ++ Bài 3. (inh Ngc An) Chopn ≤ là các s nguyên d ng và 12 , ,..., n xxx là các s th c không âm có t ng ng n. Tìm giá tr l n nh t c a bi u th c 12 12 12 1 ... ( , ,..., ) ( ... ) p p k n iii i i in fxx x xxx ≤ ≤ ≤ ≤≤ = ∑ Trong ó k là s th c bt kì. Bài 5. (Ph m Kim Hùng) Cho , ,, a b cd là các s th c không âm tha mãn 4 a bcd + + += . Tìm giá tr l n nh t c a bi u th c 2 2 22 ( , , , ) (2 )(2 )(2 )(2 ) f abc d a b c d =+ + ++ Bài 6. (inh Ngc An) Cho n là s nguyên d ng và 12 , ,..., n xxx là các s th c không âm có t ng b ng n . Tìm s th c m tt nht sao cho bt ng th c sau úng v i mi b 12 ( , ,..., ) n xxx th a mãn bài 1 2 12 ... ... 1 mmm nn xx xxx x n + + + + ≥+ Tìm tài liệu Toán ? Chuyện nh ỏ - www.toanmath.com64 Bài 7. (IMO Shortlist 1993) Cho,,,0 a b cd ≥ tha mãn 1 a bcd + + += . Ch ng minh r ng 1 176 . 27 27 abc abd acd bcd abcd + + + ≤+ Bài 8. (Crux mathematicorum) Cho ,,0 abc ≥ . Ch ng minh r ng 48 48 48 1 1 1 15 a bc bc c a ab + ++ ++≥ + ++ Bài 9. (inh Ngc An) Tìm th c k tt nht bt ng th c sau úng v i mi ,,0 abc ≥ 3 3 3 222 () 2( ) 13() k ab bc ca a b c abc abc ++ + + + +≥ ++ ++ Bài 10. (Ph m Kim Hùng) Cho n là s nguyên d ng và 12 , ,..., n xxx là các s th c không âm có t ng b ng n . Tìm giá tr nh nht ca biu th c 222 1 2 1 2 12 12 111 ( , ,..., ) ... ... ... n nn n fxx x xx xxx x xxx ⎛⎞ = + ++ + + ++ ⎜⎟ ⎝⎠ Bài 11. (V ình Quý) Cho n là s nguyên d ng và 12 , ,..., n xxx là các s th c không âm có t ng b ng n . Tìm giá tr tt nht ca s th c k sao cho b t ng th c sau luôn úng 12 12 1 11 ... ... 1 1 11 n n kxxxk nx nx nx ++++ ≤+ - + -+ -+ Tìm tài liệu Toán ? Chuyện nh ỏ - www.toanmath.com65 PHÖÔNG PHAÙP THAM SOÁ HOÙA 1. t v n . i v i ph n l n các b t ng th c i s không i x ng v i các bi n thì d u b ng trong các b t ng th c này x y ra khi các giá tr các bi n không b ng nhau. Trong ch g trình ph thông thì các bt ng th c cn nh Cauchy, Bunhiacopski li c phát biu d i dng i x ng, du ng th c xy ra khi các bin bng nhau hoc t l. Vic áp dng các bt ng th c cn trên gii các bài toán c c tr không i x ng c n c quan tâm m t cách thích áng. Qua bài vi t này, tôi mu n nêu m t ph ng pháp gi i bài toán c c tr không i x ng b ng cách s d ng các t ng th c c n thông dng gi là ph ng pháp tham s hóa. i dung ch yu ca ph ng pháp này nh sau: t vic phân tích tính không i ng c a các bin có trong bài toán c c tr , th ng c cho d i các dng: ng 1. H s các bin trong biu th c cn tìm c c tr là không bng nhau. ng 2. Các bi n thu c các mi n khác nhau c a t p s th c. ng 3. u ki n ràng buc ca các bin trong gi thit bài toán là không i x ng v i các bi n. Ta a thêm bào các tham s ph cn thit th ng là các h s hoc ly th a ca các bi n có trong các ánh giá trung gian, sau ó ch n các tham s ph t t c các u ng th c xy ra, tó nhn c 1 h ph ng trình mà n là các bin và các tham s ph, tham s ph c chn h p lí ch khi h ph ng trình t ng ng có nghi m. Trong bài vit này tôi nêu mt l p bài toán c c tr không i x ng th ng p, tác gi ngh rng nh ng mô hình c th này tht có ý ngh a vì v i kt qu ca các bài toán này s cho ta mt l p bài toán c c tr không i x ng c th min là xây d ng c b bin tha mãn u kin ràng buc t ng ng. 2. M t s bài toán n hình. Bài toán 1. Cho ,, xyz là các s th c d ng thay i tha mãn u ki n 1 xy yz zx ++= và cho a là s th c d ng không i. Tìm giá tr nh nht c a biu th c Tìm tài liệu Toán ? Chuyện nh ỏ - www.toanmath.com66 2 22 ( ). P ax yz = ++ i gi i. Phân tích. u kin ràng buc i x ng v i ,, xyz . Bi u th c P i xng v i ,, xy vai trò ca z trong bi u th c P là không i x ng i , xy . Do vy, ta có th ngh rng m c c tr s t c khixy = , và 2 22 2 z xy αα == . phân tích trên, ta có th trình bày l i gi i c a bài toán nh sau i 0 α> (ch n sau), áp d ng b t ng th c AM-GM cho 2 s dng, ta có ( ) 2 2 2 2 22 2 22 2 22 2 22 z x xz z y yz x y xy α α α α αα +≥ +≥ +≥ ng v các bt ng th c trên ta nhn c 2 22 ( ) 2 ( )2 2 22 x y z xy yz zx α αα α ⎛⎞ + + +≥ ++ = ⎜⎟ ⎝⎠ Chn α sao cho . 2 a α α+= hay 1 18 24 a α -++ = u ng th c x y ra khi và ch khi 2 22 2 1 z xy xy yz zx αα ⎧ == ⎪ ⎨ ⎪ ++= ⎩ hay 4 4 1 18 1 18 2 18 xy a a z a ⎧ == ⎪ + ⎪ ⎨ -++ ⎪ = ⎪ + ⎩ t lu n 1 18 min. 2 a P -++ = Tìm tài liệu Toán ? Chuyện nh ỏ - www.toanmath.com67 Bài toán 2. Cho ,, abc là các s th c d ng tha mãn u kin 1 ab bc ca + += và , uv là các d ng c nh. Tìm giá tr nh nh t c a bi u th c 2 22 . P ua vbc = ++ i gi i. t cách t nhiên t l i gi i c a Bài toán 1, ta phân tích , ,1 u x yv zt mn = + = + =+ trong ó , , ,,, x y zt mn là các s d ng s ch n sau. Áp dng bt ng AM-GM cho 2 s d ng, ta có 22 22 22 2, 2, 2. xa tb xtab ya nc ynca zb mc zmbc +≥ +≥ +≥ ng v các bt ng th c trên, ta nhn c 2 2 2. P xtab ynca zmbc ≥ ++ u ng th c x y ra khi và ch khi 22 22 22 xa tb ya nc zb mc ⎧ = ⎪ ⎪ = ⎨ ⎪ = ⎪ ⎩ hay 2 2 2 2 2 2 . (1) xb t a na xzn ytm y c zc m b ⎧ = ⎪ ⎪ ⎪ = ⇒= ⎨ ⎪ ⎪ ⎪ = ⎩ Chn , , ,,, x y zt mn sao cho 2 xt yn zm k = == th a mãn (1). Ta có (1) ( )( )( ) x y z t m n uv ⇔+ + += Tìm tài liệu Toán ? Chuyện nh ỏ - www.toanmath.com68 ( ) 2 2 ( )() ( )2 12 xz xt yz yt m n uv xzm xtm yzm ytm xzn xtn yzn ytn uv x y m n z t k xzn uv u v k xzn uv ⇔ + + + += ⇔ + + + ++ + += ⇔ +++++ += ⇔ ++ += Mà 6 ( )() xzn utm k = nên 3 . xzn k = Do ó 32 2 ( 1) 0 (2) k u v k uv + + + -= Rõ ràng (2) có nghi m d ng duy nh t 0 k . y 0 min2 Pk = v i 0 k là nghim d ng duy nht ca ph ng trình (2). Nh n xét. Bài toán 1 và Bài toán 2 th c s có ý ngha khi ta chn ,, xyz hoc ,, abc là các bin c bit, min là u kin ràng buc ca các bin c tha mãn. Chng hn, khi ta chn mô hình là tam giác ABC . u t tg , tg , tg , 2 22 A BC x yz = == ta s có 1 xy yz zx ++= , áp d ng vào mô hình Bài toán 1 hoc Bài toán 2 ta s thu c m t l p các bài toán c c tr dng không i x ng trong tam giác. Hoc là cotg , cotg , cotg , x AyBzC = == ta cng s có ràng buc 1 xy yz zx ++= , ng t ta c ng s có m t l p các bài toán c c tr không i x ng khác i v i tam giác. Nói chung, t t ng chính ca Bài toán 1 và Bài toán 2 là mun xây d ng mt l p các bài toán m i ta ch cn xây d ng m t l p các bin i s , hoc l ng giác th a mãn u kin ràng buc t ng ng. Thit ngh rng t t t ng này có th xây ng c r t nhi u l p bài toán nh th . Bài toán 3. Cho 12 , ,..., n xxx là các s th c tha mãn u ki n 12 ...0 n xxx + + += và 12 ...1 n xxx + ++= . Tìm giá tr l n nh t c a bi u th c 1 ij i jn P xx ≤<≤ =- ∏ . Tìm tài liệu Toán ? Chuyện nh ỏ - www.toanmath.com69 i gi i. + Tr ng h p 1. 2 n = là tr ng h p tm th ng vì lúc này P = 1 không i, + Tr ng h p 2. 3 n = , không m t tính t ng quát ta gi s 1 23 x xx ≤≤ . Áp d ng b t ngth c AM-GM cho 3 s không âm, ta có 31 2 1 32 3 31 2 1 32 3 31 ( ) () 22 ( ) () 2 3 2 1 8 xx P x x xx xx x x xx xx - ⎛⎞ =-- ⎜⎟ ⎝⎠ - ⎛⎞ ⎛⎞ - + +- ⎜⎟ ⎜⎟ ⎝⎠ ⎜⎟ ≤ ⎜⎟ ⎜⎟ ⎝⎠ - ⎛⎞ = ⎜⎟ ⎝⎠ ≤ Do ó 1 4 P ≤ ng th c xy ra khi và ch khi 1 123 3 11 23 2 31 3 2 1 32 1 0 2 10 1 0 2 2 x x xx xxxxxx xx x x x xx ⎧ ⎧ =- ⎪ ⎪ ++= ⎪ ⎪ - = + + = ⇔= ⎨⎨ ⎪⎪ - ⎪⎪ = - = = -≥ ⎩ ⎩ . y 1 max 4 P = . + Tr ng h p 3. 4 n = , m t cách t nhiên ta d oán r ng max P t c khi 14 23 xx xx =- ⎧ ⎨ =- ⎩ i gi thit 1 234 x x xx ≤ ≤≤ . Nh v y thì 2 1 43 xxxx - =- . u xem hiu 2 1 43 xxxx - =- là n v và t 32 x xa -= , thì ta s có b bin mà biu th c P t max cn tha mãn u kin Tìm tài liệu Toán ? Chuyện nh ỏ - www.toanmath.com70 3 1 32 4 2 41 2 1 43 1 12 xx xx x x xx xxxx a a aa -- -- -=-= = == + ++ cách phân tích trên, l i gii c a bài toán trong tr ng h p 4 n = s nh sau i gi thit 1 234 x x xx ≤ ≤≤ , ta có 2 1 3 1 4 1 3 24 24 3 ( )( )( )( )( )( ) P x xx xx xx x x x x x = -- -- - - Do ó 2 3 1 32 4 1 42 2 1 43 6 3 1 32 4 1 42 2 1 43 4 1 32 ( 2)( 1) ( ) () ( ) () ( ). . . . .() 121 ( ) () ( ) () ( ) () 121 6 1 1 11 ( )1 1 () 121 6 P aaa xx xx x x xx x x xx a a aa xx xx x x xx xx xx a a aa x x xx a a aa = ++ -- -- =-- +++ - - -- ⎛⎞ - + + + + +- ⎜⎟ +++ ≤ ⎜⎟ ⎜⎟ ⎝⎠ ⎛⎞ ⎛ ⎞⎛⎞ - + + +-++- ⎜ ⎟⎜⎟ ⎜ +++ ⎝ ⎠⎝⎠ ⎜ = ⎜ ⎜ ⎝⎠ 6 ⎟ ⎟ ⎟ ⎟ Ta ch n 0 a > sao cho 1 1 11 11 121 a a aa + + =-++ +++ hay 2 1. a=- Khi ó, 1 1 1 1 32 11 1 2 12 a a aa + + =-++= +++ và ta thu c ( )( )( ) 6 1 234 29 32 .() 1 2 6 2 2 1 2 12 x x xx P ⎛⎞ - -++ ⎜⎟ ≤⎜ ⎟≤ ⎜⎟ -+ ⎜⎟ ⎝⎠ hay 8 1 2 P ≤ ng th c xy ra khi và ch khi Tìm tài liệu Toán ? Chuyện nh ỏ - www.toanmath.com71 1234 1 2 3 4 1 234 3 2 3 1 4 2 41 2 1 43 0 1 0 2 1 2 2 21 x x xx x x x x x xxx x xx x x xx x xxxx ⎧ ⎪ + + += ⎪ ⎪ - - + + = + + += ⎨ ⎪ - - -- ⎪ -=-= = = =≥ ⎪ -+ ⎩ Gi i h phng trình này ta nh n c 41 32 22 4 2 4 xx xx ⎧ - =-= ⎪ ⎪ ⎨ ⎪ =-= ⎪ ⎩ t lu n 8 1 2 maxP = . + Tr ng h p 4. 5 n = . Phân tích. i gi thi t 1 2345 x x xxx ≤ ≤≤≤ , t l i gi i c a các trng h p 2 và 3, m t cách nhiên, ta ngh ngay rng b s Pt max là 5 1 4 23 , ,0 x x x xx =- =- = . Do v y 54 2 132 43 ,, x x x xx x xx -=- -=- tó ta có th oán nhn rng nu xem hiu 21 xx - b ng n v và 32 xx - bng a thì b s Pt max cn phi tha u ki n 43 32 3 1 53 21 52 5 1 54 4 2 41 1 11 2 2 1 2 1 2 21 x x xx xx xx xx a a aa xx xx xx xx xx a a aa - - -- - === == ++ - -- -- = = = == + ++ cách phân tích trên, l i gii c a bài toán trong tr ng h p 5 n = s nh sau Không m t tính tng quát, ta gi s 1 2345 x x xxx ≤ ≤≤≤ , t ó suy ra 2 13 14 1 5 13 2 42 52 43 53 54 ( )( )( )( )( )x x( )( )( )( )( ) P x xx xx xx xx x x x xx x x x x xx = -- --- - - --- Xét bi u th c 2 32 4 ( 1) (2 1) P Q aaa = ++ Tìm tài liệu Toán ? Chuyện nh ỏ - www.toanmath.com72 Vit Q d i d ng 31 51 3 2 21 41 52 43 53 5 4 42 () ()( ) () () . ... x 1 1 2 1 22 ( )( )( )( ) () x . ... 2 21 11 xx xx xx xx xx Q a a aa x x x x xx xx xx a a aa - -- -- = + ++ - --- - ++ Áp d ng b t ngth c AM-GM cho 10 s không âm, ta có 31 51 3 2 2 1 41 10 10 52 43 5 3 54 42 10 5 1 42 10 () ()( ) 1( ) () 1 1 2 1 22 10 ( )( )( )( ) () 2 21 11 1 1 3 13 ( )1 ( )1 2 12( 1) 2 12 10 xx x x xx x x xx Q a a aa xx x x x x xx xx a a aa x x xx a a aa - -- -- ⎛ ≤ ++ ++ + ⎜ + ++ ⎝ - --- - ⎞ +++ + + ⎟ ++ ⎠ ⎛⎞ ⎛⎞ ⎛⎞ = - ++ + - -++ ⎜⎟ ⎜⎟ ⎜⎟ +++ ⎝⎠ ⎝⎠ ⎝⎠ Chn 0 a > sao cho 1 3 13 11 2 12( 1) 2 12 a a aa ++ =-++ + ++ hay 1 2 a = . Khi ó, 1 3 1 35 11 2 12( 1) 2 12 2 4 27 a a aa P Q ⎧ ++ =-++ = ⎪ + ++ ⎪ ⎨ ⎪ = ⎪ ⎩ ây, ta thu c 10 1 2 45 10 20 151 ..() 2102 Q x xxx ⎛⎞ ≤ --++≤ ⎜⎟ ⎝⎠ Do ó 22 27 2 P ≤ ng th c xy ra khi và ch khi Tìm tài liệu Toán ? Chuyện nh ỏ - www.toanmath.com73 1 2345 1 2 4 5 1 2345 31 51 41 2 1 32 5 2 53 42 4 3 54 0 1 2() 2() 3 23 2() 2()0 23 x x x xx x x x x x xx xx xx xx xx x x xx xx xx x x x x xx + + + += ⎧ ⎪ - -++ = + + + + = ⎪ ⎪ -- - ⎨ - = = = = -= ⎪ ⎪ -- = - = = - = =-≥ ⎪ ⎩ Gi i h phng trình này ta nh n c 15 24 3 3 8 1 8 0 xx xx x ⎧ =- =- ⎪ ⎪ ⎪ =- =- ⎨ ⎪ = ⎪ ⎪ ⎩ . t lu n 22 27 max 2 P = . Nh n xét. ng ph ng pháp t ng t s tìm c l i gii c a bài toán v i n 6. Bài toán 4. (Võ Qu c Bá C n) Cho ,, mnp là dài ba cnh ca mt tam giác cho tr c và tam giác ABC nhn. Tìm giá tr nh nht ca biu th c tg .tg .tg . m np P A BC = i gi i. Xét bi u th c 1 cotg .cotg .cotg m np Q A BC P == . Bài toán ã cho t ng ng v i tìm max ca Q . Khi nhìn th y bi u th c Q , ít nhiu ta c ng ngh n ng th c quen thu c cotg .cotg cotg .cotg cotg .cotg 1 A B B C CA ++= Và t ây, ta ngh ngay r ng bài này có th dùng b t ng AM-GM suy r ng, do ó ta a vào các tham s d ng ,, xyz (ch n sau) sao cho Tìm tài liệu Toán ? Chuyện nh ỏ - www.toanmath.com74 (cotg .cotg ) .(cotg .cotg ) .(cotg .cotg ) (cotg ) .(cotg ) .(cotg ) . x yz xz x y yz Q A B B C CA A BC + ++ = = Ta ph i ch n ,, xyz sao cho 1 .() 2 1 .() 2 1 .() 2 x m np x zm xy n y m np y zp z m np ⎧ = +- ⎪ += ⎧ ⎪ ⎪⎪ + = ⇔=-++ ⎨⎨ ⎪⎪ += ⎩ ⎪ = -+ ⎪ ⎩ . ây, ta có cotg .cotg cotg .cotg cotg .cotg .. y xz x yz Q A B B C CA x yz xyz ⎛⎞ ⎛ ⎞ ⎛⎞ = ⎜ ⎟ ⎜⎟ ⎜⎟ ⎝ ⎠ ⎝⎠ ⎝⎠ Áp dng bt ng th c Cauchy suy rng, ta có 1 cotg .cotg cotg .cotg cotg .cotg . () 1 () x yz xyz xyz x yz Q xyz A B B C CA x yz x yz x yz xyz ++ ++ ++ ≤ ⎛⎞ ⎛⎞ ⎛ ⎞ ⎛⎞ ≤ ++ ⎜⎟ ⎜ ⎟ ⎜⎟ ⎜⎟ ++ ⎝ ⎠ ⎝⎠ ⎝⎠ ⎝⎠ = ++ Do ó () x yz xyz xyz Q x yz ++ ≤ ++ Suy ra ( ) () ( ) () () xyz m n p x y z mn p m n p mn p x y z m np P xyz mn p mn p mn p + + ++ - ++ - + +- + + ++ ≥= - ++ - + +- ng th c xy ra khi và ch khi cotg .cotg cotg .cotg cotg .cotg A B B C CA x yz == . Hay Tìm tài liệu Toán ? Chuyện nh ỏ - www.toanmath.com75 ()() cotg ( )( )() ( )() cotg ( ) ( )() ( )() cotg ( ) ( )() xz m n p m np A yx y z m n p m np xy m n p m np B zx y z mn p mn p yz m np m np C xx y z m np m np ⎧ - + +- == ⎪ + + - ++ ++ ⎪ ⎪ - + + +- ⎪ == ⎨ + + -+ ++ ⎪ ⎪ - + - ++ ⎪ == + + +- ++ ⎪ ⎩ t lu n () min ( ) () () m np mn p m n p mn p m np P mn p m n p mn p ++ - + + -+ +- ++ = - ++ - + +- . Bài toán 5. (Vietnam TST 2001) Cho ,,0 abc > và 21 2 8 12 ab bc ca ++≤ . Tìm giá tr nh nh t c a bi u th c 1 23 . P abc = ++ i gi i. Phân tích. n gi n, ta s t 1 23 ,, x yz a bc = == thì ta nh n c mt bài toán ng ng nh sau “,,0 xyz > và 6 12 21 6 x y z xyz ++≤ . Tìm giá tr nh nh t c a bi u th c . P xyz =++ ” Nhn thy t gi thit 6 12 21 6 x y z xyz ++≤ , ta có th suy ra c ( , , 0) mnp x y z k mnp ≥> Do ó ta ngh ngay rng bài này có th s dng bt ng AM-GM suy rng c. Th t v y 1 .. ( ) .. p mn m np x yz Pm np m np x yz m np mnp ++ = ++ ⎛⎞ ⎛⎞ ⎛ ⎞ ⎛⎞ ⎜⎟ ≥ ++ ⎜ ⎟ ⎜⎟ ⎜⎟ ⎜⎟ ⎝ ⎠ ⎝⎠ ⎝⎠ ⎝⎠ Tìm tài liệu Toán ? Chuyện nh ỏ - www.toanmath.com76 1 () m np m np k m np mnp ++ ⎛⎞ ≥ ++ ⎜⎟ ⎝⎠ Nh v y, nhi m v c a ta bây gi ch là ph i tìm ,, mnp n a thôi. Rõ ràng, ta ch c n xét 1 m np + += là . Khi ó, ta có 1 21 6 12 6 12 21 6 6 12 21 6 . 12 . 21 . (6 12 21 ) . . p mn m np xyz xyz x yz mnp m np x yz m np m np ++ ≥ ++ =++ ⎛⎞ ⎛⎞ ⎛ ⎞ ⎛⎞ ⎜⎟ ≥ ++ ⎜ ⎟ ⎜⎟ ⎜⎟ ⎜⎟ ⎝ ⎠ ⎝⎠ ⎝⎠ ⎝⎠ tìm ,, mnp ta c n ph i gi i h sau 6 1 6 12 21 12 1 6 12 21 21 1 6 12 21 1 m km m np n kn m np p kp m np m np ⎧ -= ⎪ ++ ⎪ ⎪ -= ⎪ ++ ⎨ ⎪ ⎪-= ++ ⎪ ⎪ + += ⎩ Hay 2 2 6 12 6 12 21 1 12 12 6 12 21 4 10 6 52 21 5 2 31 12 6 12 21 1 m m m np m np n n m np n np np p p np np p m np m np ⎧ -= ⎪ ++ ⎪ = -- ⎧ ⎪ -= ⎪ ⎪ ++ ⇔ + + -= ⎨⎨ ⎪⎪ + -+= ⎩ ⎪-= ++ ⎪ ⎪ + += ⎩ Xét h 2 2 4 10 6 52 (*) 5 2 31 n np np p np np ⎧ + + -= ⎪ ⎨ + -+= ⎪ ⎩ t ( 0) n tpt => , h (*) tr thành 2 22 (2 5) (3 ) 1 (1) (4 10 ) (6 5) 2 (2) t p tp t tp tp ⎧ + + -= ⎪ ⎨ + + -= ⎪ ⎩ y (2) 2 (1) x - , ta c Tìm tài liệu Toán ? Chuyện nh ỏ - www.toanmath.com77 2 ((4 6 10) 8 11) 0 p t t pt + - + -= u 1 t = thì h (*) vô nghi m, do ó 1 t ≠ . 2 118 (3) 4 6 10 t p tt - ⇒= +- Do 0,0 pt >> nên 11 1 8 t << . Thay (3) vào (1) và thu g n, ta c 4 32 2 16 12 146 30 175 0 (4 5)(2 5)(2 4 7) 0 5 11 (do 1 ) 48 t t tt ttt t tt - - + += ⇔ - + - -= ⇔ = << ó, ta có 2 5 1 3 4 15 m n p ⎧ = ⎪ ⎪ ⎪ = ⎨ ⎪ ⎪ = ⎪ ⎩ . Th l i, ta th y th a. ng th c trên xy ra khi và ch khi 6 12 21 5 5 15 3 24 6 12 21 5 3 5 2 2 1 3 4 5 3 2 x yz m np x y z xyz xz y xyz xyz x y z a b c ⎧ == ⎪ ⎨ ⎪ ++= ⎩ ⎧ == ⎪ ⇔ ⎨ ⎪ ++= ⎩ = ⎧ ⎪ ⎪ ⇔= ⎨ ⎪ = ⎪ ⎩ ⎧ = ⎪ ⎪ ⎪ ⇒= ⎨ ⎪ ⎪ = ⎪ ⎩ Tìm tài liệu Toán ? Chuyện nh ỏ - www.toanmath.com78 các phân tích và ch n tham s trên, ta i n mt l i gii c c k n gin nh sau t 1 43 ,, 3 52 a bc xyz = == , bài toán chuyn v “,,0 xyz > và 3 5 7 15 x y z xyz + +≤ . Tìm giá tr nh nh t c a bi u th c 1 .(6 5 4 ). 2 P x yz = ++ ” Áp dng bt ng AM-GM cho 15 s d ng, ta có 3 57 15 12 10 8 15 6 54 15 15 3 5 7 15 1 1 xyz x y z x y z xyz x yz ≥ ++≥ ⇒≥ ⇔≥ i áp dng bt ng th c AM-GM cho 15 s d ng, ta có 6 54 15 1 15 15 .(6 5 4). 2 22 P x y z x yz =++≥≥ ng th c xy ra khi và ch khi 1 3 4 1. 15 3 57 5 3 2 a x yz xyzb xyz x yz c ⎧ = ⎪ ⎪ == ⎧ ⎪ ⇔ = = =⇔= ⎨⎨ = ++ ⎩ ⎪ ⎪ = ⎪ ⎩ t lu n 15 min. 2 P = Bài toán 6. Cho,,0 xyz ≥ và 3. xyz + += Tìm giá tr nh nh t c a bi u th c 4 44 23 P x yz = ++ i gi i. i m i s dng ,, abc , theo b t ng th c Holder, ta có 4 4433 3 34 ( 2 3 ) ( 2 3) P a b c ax b y cz + + ≥ ++ Tìm tài liệu Toán ? Chuyện nh ỏ - www.toanmath.com79 Chn ,, abc sao cho 3 3 33 2 3, a b ck = == khi ó, ta có 12 4 34 4 4 43 4 4 43 ( ) (3) ( 2 3 ) ( 2 3) kxyzk P a bc a bc ++ ≥= + + ++ ng th c x y ra thì ta ph i có 1 x y z x yz a b c abc ++ ==== ++ Do v y, ta có 3 3 33 3 3 33 3 23 2 3 3 1 23 abc a b ck ak bk ck k ++= ⎧ ⎨ = == ⎩ = ⎧ ⎪ = ⎪ ⎪ ⇔ ⎨ = ⎪ ⎪ = ⎪ ++ ⎩ ây, ta d dàng suy ra k t qu c a bài toán. Bài toán 7. Ch ng minh r ng v i m i s d ng 12 , ,..., n aaa ta luôn có 1 12 12 1 2 1 2 111 ... 4 ... ... nn n a aa aa a a a a ⎛⎞ + + + < + ++ ⎜⎟ + + ++ ⎝⎠ Ch ng minh. Áp dng bt ng th c Bunhiacopxki, ta có 2 22 2 12 1 2 12 12 2 22 12 2 1 2 12 12 ( ... ) ... ( ... ) . ... ... ( ...) k kk k k kk k x xx aa a xxx aaa x k k xx aa a aaa xxx ⎛⎞ + ++ + ++ ≥ + ++ ⎜⎟ ⎝⎠ ⎛⎞ ⇒ ≤ + ++ ⎜⎟ + ++ + ++ ⎝⎠ nh các s 12 , ,..., n xxx và cho k ch y t 1 n n, r i l y t ng, ta c 12 1 1 2 1 2 12 12 ... ... ... n nn c n cc a aa aa a a a a + + + ≤ + ++ + + ++ Trong ó Tìm tài liệu Toán ? Chuyện nh ỏ - www.toanmath.com80 2 22 2 22 12 12 1 12 ( 1) ... ( ... ) ( ... ) ( ... ) k kk k k kn kx k x nx c xxx xxx xxx + + = + ++ + ++ + ++ + + + Ta có th ch n 1,, k x k kn = ∀= khi ó 2 2 22 2 2 2 2 2 22 2 2 22 2 1 ... (1 2 ... ) (1 2 ... ( 1)) (1 2 ... ) 1 4 ... (1) (1)( 2) (1) 1 11 4 ... ( 1) ( 1)( 2) ( 1) 1 1 1 111 4 . .. 1 1 2 12 k k kn ck k kn k kn k k k k k nn k k k k k nn k k kk k kk ⎛⎞ + = + ++ ⎜⎟ + + + + ++ + + ++ ⎝⎠ ⎛⎞ + = + ++ ⎜⎟ + +++ ⎝⎠ ⎛⎞ = + ++ ⎜⎟ + +++ ⎝⎠ ⎛ ⎞ ⎛⎞ = - + -+ ⎜ ⎟ ⎜⎟ + + + ++ ⎝ ⎠ ⎝⎠ 2 2 22 2 2 2 1 11 ... 11 11 1 11 4 ... 1 ( 1) ( 2) ( 1) 11 1 11 4 ... 1 ( 1)( 2) ( 2)( 3) ( 1)( 2) 1 11 4 11 11 4 1 4 1 4 n nn k knk kn k k n k k k k nn k kkn k kk k k ⎛⎞ ⎛⎞ +- ⎜⎟ ⎜⎟ ++ ⎝⎠ ⎝⎠ ⎛⎞ = - - - -- ⎜⎟ ++ ++ ⎝⎠ ⎛⎞ < - - - -- ⎜⎟ + ++ + + ++ ⎝⎠ ⎛⎞ = -- ⎜⎟ ++ ⎝⎠ ⎛⎞ <- ⎜⎟ + ⎝⎠ < + < t ng th c c ch ng minh hoàn toàn. Bài toán 8. Ch ng minh b t ng th c sau v i m i s th c 12 , ,..., n xxx 2 2 2 222 12 12 1 12 ... ... 4( ... ) 2 n n xxx xx x xxx n + ++ + ⎛⎞ ⎛⎞ + ++ ≤ + ++ ⎜⎟⎜⎟ ⎝⎠⎝⎠ Ch ng minh. i m i s dng 12 , ,..., n ccc tùy ý, ta có Tìm tài liệu Toán ? Chuyện nh ỏ - www.toanmath.com81 2 22 2 12 1 2 12 12 ... ( ... ) ( ... ) k kk k x xx cc c xxx ccc ⎛⎞ + + + + ++ ≥ + ++ ⎜⎟ ⎝⎠ Do ó 2 22 1 2 12 12 12 22 1 2 12 2 ... ... ... .. ... .... k kk k k k k xx x cc c ccc xx k kc kc ccc x kc + + + + ++ + ++ ⎛⎞ ≤ ++ ⎜⎟ ⎝⎠ + ++ ++ Cho k ch y t 1 n n , r i l y t ng, ta c 2 2 2 2 22 12 12 1 1 1 22 ... ... ... 2 n nn xxx xx x x xx n α αα + ++ + ⎛⎞ ⎛⎞ + + + ≤ + ++ ⎜⎟⎜⎟ ⎝⎠⎝⎠ Trong ó 12 1 2 1 12 2 22 ... ... ... ... 1, ( 1) k kn k k kk ccccc c ccc kn k c k c nc α + + ++ + ++ + ++ = + + + ∀= + Ta ch n 12 1 ... kk ck k c c ck = - - ⇒ + + += 32 32 32 1 111 . ( 1) k k ck kn α ⎛⎞ ⇒=++ ⎜⎟ + ⎝⎠ Chú ý r ng 32 32 32 32 11 11 22 11 11 22 22 1 1 1 11 2 2 22 1 2 1 1 1 1 11 ... 2 2(1)2 1 11 2 22 kk kk kk k k kk k kkn k kn + -- -= ⎛ ⎞⎛⎞ -+ -+ ⎜ ⎟⎜⎟ ⎝ ⎠⎝⎠ = ⎛⎞ ⎛ ⎞⎛⎞ ++ - -+ ⎜⎟⎜ ⎟⎜⎟ ⎝ ⎠⎝⎠ ⎝⎠ ≥ ⇒ ≥ - ≥ + ++ + - -+ Tìm tài liệu Toán ? Chuyện nh ỏ - www.toanmath.com82 ( ) 21 2 4 11 22 k k kk ckk α +- ⇒≤ =≤ -- t ng th c c ch ng minh hoàn toàn. 3. Bài t p ngh. Bài 1. (Vietnam TST 1994) Cho , ,, a bcd là các s th c th a mãn u ki n 2 2 22 1 1 2 a b cd ≤ + + +≤ . Tìm giá tr nh nh t và giá tr l n nh t c a bi u th c 2 2 22 ( 2 )( 2 )( 2)(2) P ab c bc d ba cd = - + +- + +- +- Bài 2. Cho,0 xy > và 4 xy +≥ . Tìm giá tr nh nh t c a bi u th c 6 10 23 P xy xy = + ++ Bài 3. Cho ,,0 abc > và 2 3 20 a bc + +≥ . Tìm giá tr nh nh t c a bi u th c 394 2 P abc a bc = +++ ++ Bài 4. Cho ,,0 abc > và 3 abc + += . Tìm giá tr l n nh t c a bi u th c 2 43 P ab bc ca = ++ Bài 5. (Toán H c Tu i Tr 2005) a) Cho tam giác ABC . Tìm giá tr l n nh t c a bi u th c 23 sin .sin .sin P A BC = b) Cho tam giác ABC, ,, mnp là các s th c d ng cho tr c. Tìm giá tr n nh t c a bi u th c sin .sin .sin m np P A BC = Tìm tài liệu Toán ? Chuyện nh ỏ - www.toanmath.com83 Bài 6. (VMEO 2004) Cho tam giác nh n ABC. Tìm giá tr nh nh t c a bi u th c 25 P tgA tgB tgC = ++ Bài 7. (VMEO 2005) Cho ,, abc là các s th c d ng cho tr c và ,, xyz là các s th c d ng th a mãn ax by cz xyz + += . Tìm giá tr nh nh t c a bi u th c P xyz =++ Bài 8. Cho 12 , ,..., n aaa là n s th c d ng cho tr c và 12 , ,..., n xxx là n s th c d ng th a mãn 1 1 . n n iii i i axx == = ∑ ∏ Tìm giá tr nh nh t c a bi u th c 1 n i i Px = = ∑ Bài 9. ch n i tuyn HSP Hà N i 2005) Cho ,, xyz là các s th c d ng th a mãn 7 xy yz zx xyz ++= . Tìm giá tr nh nh t a bi u th c 4 56 2 22 8 1 108 1 16 1 x yz P x yz + ++ = ++ Bài 10. (Toán H c Tu i Tr 2005) Cho , , [0,1] xyz ∈ . Tìm giá tr l n nh t c a bi u th c ( )( )( )( ) P x y yz zx x yz = - - - ++ Bài 11. Ch ng minh r ng v i m i dãy s d ng 12 , ,..., n aaa ta có 1 12 12 1 2 1 1 1 111 ... 2 ... ... nn a aa aa a a a a ⎛⎞ + + + < + ++ ⎜⎟ + + ++ ⎝⎠ Tìm tài liệu Toán ? Chuyện nh ỏ - www.toanmath.com84 PHÖÔNG PHAÙP HEÄ SOÁ BAÁT ÑÒNH Trong thôøi caáp 2, khi ñoïc lôøi giaûi cuûa khaù nhieàu baøi toaùn ñaëc bieät laø baát ñaúng thöùc, toâi khoâng theå hieåu noåi taïi sao ngöôøi ta laïi nghó ra ñöôïc lôøi giaûi ñoù vaø toâi caûm thaáy ñoù laø moät lôøi giaûi thieáu töï nhieân nhöng toâi cuõng caûm thaáy voâ cuøng thaùn phuïc ngöôøi ñaõ nghó ra lôøi giaûi ñoù. Nhöng baây giôø khi ñaõ ñöôïc laøm quen vôùi taát caû caùc kieán thöùc toaùn sô caáp, toâi môùi hieåu ñöôïc ñaáy khoâng phaûi laø moät caùi gì môùi laï caû maø noù ñaõ coù moät phöông phaùp haún hoi. Trong baøi naøy, toâi xin giôùi thieäu vôùi caùc baïn moät trong nhöõng phöông phaùp ñoù: “Phöông phaùp heä soá baát ñònh”. Phöông phaùp naøy tuy coù moät soá haïn cheá nhöng noù vaãn laø moät phöông phaùp hay vaø khaù maïnh. Caùc baïn neân chuù yù ñeán noù vì ngoaøi vieäc giuùp ta chöùng minh moät baát ñaúng thöùc khoù thì noù coøn laø 1 “lieàu thuoác boå “ cho moät phöông phaùp chöùng minh baát ñaúng thöùc cöïc maïnh: “Phöông phaùp phaân tích bình phöông S.O.S” vì noù giuùp ta ñöa moät baát ñaúng thöùc veà daïng S.O.S nhanh choùng hôn caùc kieåu bieán ñoåi thoâng thöôøng. Sau ñaây laø moät soá ví duï Ví duï 1. (USAMO 2003) Cho,,0. Chöùng minh raèng abc > 222 2 2 2 2 22 (2 )(2 )(2 ) 8 2 ( )2 ( )2 ( ) a b c bc a c ab abc bca c ab + + + + ++ + +≤ + + + + ++ Nhaùp. Nhaän xeùt raèng daáu baèng xaûy ra khi vaø chæ khi . abc == Do caû hai veá cuûa baát ñaúng thöùc ñaõ cho ñoàng baäc neân ta coù theå chuaån hoùa cho 3 abc + += . Khi ñoù, baát ñaúng thöùc caàn chöùng minh trôû thaønh Tìm tài liệu Toán ? Chuyện nh ỏ - www.toanmath.com85 2 22 2 2 2 2 22 ( 3) ( 3) ( 3) 8 2 (3 ) 2 (3 ) 2 (3 ) a bc a a b b cc + ++ + +≤ +- +- +- Ta seõ tìm soá thöïc α sao cho baát ñaúng thöùc cho moïi (0,3) a ∈ 2 22 32 (3)8 ( 1) 2(3)3 ( ) 3 (7 9 ) (15 22) 15 9 0 a a aa faa aa α α α αα + ≤ -+ +- ⇔ = + - + - + -≥ Ta caàn tìm α sao cho () 0 (0,3) () 0 1. vaø fa a faa ≥ ∀∈ = ⇔= Ñeå coù ñöôïc ñieàu naøy, ta caàn coù / 4 (1) 0 9 2(7 9 ) 15 22 0 3 f α ααα =⇔ + - + - =⇔ = Vaäy nhieäm vuï cuûa ta baây giôø laø xeùt xem baát ñaúng thöùc sau coù ñuùng hay khoâng 2 22 ( 3) 44 . 2 (3 ) 33 a a aa + ≤+ +- Vôùi nhöõng laäp luaän nhö treân, ta ñi ñeán moät lôøi giaûi khoâng maáy töï nhieân nhö sau Lôøi giaûi. Khoâng maát tính toång quaùt, coù theå giaû söû 3 abc + += . Khi ñoù, baát ñaúng thöùc caàn chöùng minh trôû thaønh 2 22 2 2 2 2 22 ( 3) ( 3) ( 3) 8 2 (3 ) 2 (3 ) 2 (3 ) a bc a a b b cc + ++ + +≤ +- +- +- Ta seõ chöùng minh 2 22 ( 3) 44 . (*) 2(3 ) 33 a a aa + ≤+ +- Thaät vaäy 2 (*) ( 1)(4 3)0 (ñuùng) aa ⇔ - +≥ Vaäy (*) ñuùng. Töông töï, ta coù Tìm tài liệu Toán ? Chuyện nh ỏ - www.toanmath.com86 2 22 2 22 ( 3) 44 . 2(3 ) 33 ( 3) 44 . 2 (3 ) 33 b b bb c c cc + ≤+ +- + ≤+ +- Do ñoù 2 22 2 2 2 2 22 ( 3) ( 3) ( 3) 4 .( ) 48 2 (3 ) 2 (3 ) 2 (3 ) 3 ñpcm. a bc abc a a b b cc + ++ + + ≤ + + += +- + - +- ⇒ Rieâng ñoái vôùi baøi toaùn treân coøn coù moät caùch tìm α cöïc nhanh laø 2 2 22 ( 3) 1 86 1 86 4 4 . 2(3 ) 33( 1) 63 6 33 a aa a a aa + ++ =+ ≤+ = + + - -+ Nhöng vôùi ñöôøng loái naøy thì ta khoù maø laøm maïnh baát ñaúng thöùc hôn ñöôïc. Thaät vaäy, toâi ñaõ coá gaéng raát nhieàu ñeå duøng ñöôøng loái naøy ñeå chöùng minh baát ñaúng thöùc sau nhöng vaãn baát löïc 2 22 2 2 2 2 22 ( 3) ( 3) ( 3) 6 4 (3 ) 4 (3 ) 4 (3 ) a bc a a b b cc + ++ + +≤ +- +- +- Vôùi,,0 abc > thoûa 3. abc ++= Coù theå thaáy caùch tìm α ban ñaàu laø caùch tìm hay nhaát, nhöng noù ñoøi hoûi khaù nhieàu tính toaùn raát baát lôïi cho nhöõng baïn tính toaùn khoâng ñöôïc toát cho laém, vaø ñoâi khi bieåu thöùc ñeà baøi cho quaù phöùc taïp (chaúng haïn nhö quaù nhieàu caên thöùc). Vì nhöõng lí do ñoù, toâi xin ñöôïc giôùi thieäu vôùi caùc baïn moät caùch tìm α khaù hieäu quaû döïa treân baát ñaúng thöùc AM-GM, cuï theå laø ñoái vôùi baøi toaùn treân Ta seõ tìm , αβ sao cho baát ñaúng thöùc sau ñuùng cho moïi soá döông ,, abc 2 22 (2) 2() abc a b c a b c abc α ββ ++ ++ ≤ + + ++ AÙp duïng baát ñaúng thöùc AM-GM (xin ñöôïc löu yù vôùi caùc baïn laø trong caùch tìm naøy, ta khoâng caàn ñeå yù ñeán chieàu baát ñaúng thöùc, toâi xin ñöôïc kyù hieäu → ñeå thay Tìm tài liệu Toán ? Chuyện nh ỏ - www.toanmath.com87 cho daáu baát ñaúng thöùc vaø ta cuõng khoâng caàn ñeå yù ñeán , αβ aâm hay döông vì ñaây chæ laø nhaùp thoâi), ta coù 11 2 36 22 11 23 23 (2 )8 ..() 2() 3 2 . .() 3 abc a bc a bc abc a bc abc αβ αβ αβ α β β αβ - -- ++ ++ → ++ +++ → ++ Ta choïn , αβ sao cho 28 11 2 33 11 2 36 b αβ α αβ αβ ⎧ ⎪ += ⎪ ⎪ -= ⎨ + ⎪ ⎪ - =- ⎪ + ⎩ . Giaûi heä naøy, ta ñöôïc 16 3 . 4 3 α β ⎧ = ⎪ ⎪ ⎨ ⎪ = ⎪ ⎩ Vaäy nhieäm vuï cuûa chuùng ta baây giôø laø xeùt tính ñuùng ñaén cuûa baát ñaúng thöùc 2 22 (2 ) 16 44 2() 3( ) abc a b c a bc abc + + ++ ≤ + + ++ Ta coù theå chuaån hoùa cho 3 abc + += roài chöùng minh töông töï nhö treân, hoaëc bieán ñoåi töông ñöông. Ví duï 2. Cho , , 0. abc > Chöùng minh raèng 333 3 3 3 3 33 1 ( ) ( ) () abc a bc b c a c ab + +≥ + + + + ++ Nhaùp. Ñaây laø moät baøi toaùn hay, töông ñoái khoù. Ta coù theå giaûi baèng caùch laøm töông töï nhö treân, xin daønh cho caùc baïn. ÔÛ ñaây, toâi xin giôùi thieäu moät caùch giaûi khaùc nhö sau Nhaän xeùt raèng daáu baèng xaûy ra khi vaø chæ khiabc == . Ta seõ tìm p sao cho baát ñaúng thöùc sau ñuùng Tìm tài liệu Toán ? Chuyện nh ỏ - www.toanmath.com88 3 33 () p p pp aa a bc abc ≥ + + ++ Chuùng ta coù 2 caùch choïn p söû duïng ñaïo haøm hoaëc döïa vaøo baát ñaúng thöùc AM- GM, veà phía toâi, toâi raát ngaïi tính toaùn neân chæ xin ñöôïc trình baøy caùch döïa vaøo baát ñaúng thöùc AM-GM, mong caùc baïn thoâng caûm. AÙp duïng baát ñaúng thöùc AM-GM, ta coù 42 3 33 33 2 33 1 ..() ( )3 1 . .() 3 pp p ppp a a bc a bc a a bc abc - - → ++ → ++ Töø ñaây, baèng caùch ñoàng nhaát heä soá, ta suy ra ñöôïc 2. p = Vaäy nhieäm vuï cuûa chuùng ta baây giôø laø kieåm tra tính ñuùng ñaén cuûa baát ñaúng thöùc 32 3 3 222 () aa a b c abc ≥ + + ++ Vôùi nhöõng laäp luaän nhö treân, ta ñi ñeán lôøi giaûi nhö sau Lôøi giaûi. Ta seõ chöùng minh 32 3 3 2 22 (*) () aa a b c abc ≥ + + ++ Thaät vaäy: 222 33 2222 33 2 22 222 3 22 2 2 22 3 22 2 2 22 3 22222 1 (*) () ( ) ( ( )) 2( )( ) () ( )(( ) ( )) ( ) ( )(( ) ( ) ) 2( ) ( ) ( ) ( )(( ) ( ) ) ( )( ) 0 (ñuù a abc a bc abc aa b c abc bc ab c bc ab a c ab c b c a b ac b c abc abc b c ab a c abcbc ⇔≥ ++ ++ ⇔ + + ≥ ++ ⇔ + + + ≥+ ⇔ + + + + ≥+ ⇔ + - + - + + + ≥+ ⇔ + -+ - + + -≥ ng) Tìm tài liệu Toán ? Chuyện nh ỏ - www.toanmath.com89 Vaäy (*) ñuùng. Töông töï, ta coù 32 3 3 222 32 3 3 222 () () bb b c a abc cc c ab abc ≥ + + ++ ≥ + + ++ Do ñoù 333 222 3 3 3 3 3 3 2 22 1 ( ) ( ) () ñpcm. a b c abc a b c b c a c a b abc ++ + + ≥= + + + + + + ++ ⇒ Ñaúng thöùc xaûy ra khi vaø chæ khi . abc == * Nhaän xeùt 1. Caû hai ví duï treân ñeàu söû duïng ñaúng thöùc 1 ()() 1. p pp p pp a b c abc abc abc abc αβ αβ + + ++ + ++ == + + + ++ Moät caâu hoûi ñaët ra cho ta laø khi naøo thì ta phaûi tìm p vaø khi naøo thì ta phaûi tìm , αβ ? Coù leõ caùc baïn seõ hôi luùng tuùng ôû choã naøy nhöng thaät ra thì ta chæ caàn nhìn bieåu thöùc ôû ñeà baøi laø bieát ngay thoâi, chaúng haïn nhö ôû ví duï 1, xeùt baát ñaúng thöùc 2 22 (2 )8 2() p p pp abc a a bc abc ++ ≤ + + ++ Khi cho 0,1 a bc → == thì ta coù 1,0 VT VP →→ neân baát ñaúng thöùc naøy khoâng theå ñuùng vôùi moïi soá döông , ,. abc Ví duï 3. Cho , , 0. abc > Chöùng minh raèng 333 2 2 2 2 22 2 a b c abc ab bc c a ++ + +≥ + ++ Nhaùp. Tìm tài liệu Toán ? Chuyện nh ỏ - www.toanmath.com90 Nhaän xeùt raèng daáu baèng xaûy ra khi vaø chæ khi . abc == Ta seõ tìm α sao cho baát ñaúng thöùc sau ñuùng 3 22 2 (1) a ab ab αα ≥ +- + AÙp duïng baát ñaúng thöùc AM-GM, ta coù 3 21 22 1 2 (1) a ab ab a b ab αα αα - - → + +-→ Töø ñaây, baèng caùch ñoàng nhaát heä soá, ta coù 2 α= . Vaäy nhieäm vuï cuûa ta baây giôø laø kieåm chöùng tính ñuùng ñaén cuûa baát ñaúng thöùc 3 22 2 2 a ab ab ≥- + Ta ñi ñeán lôøi giaûi nhö sau Lôøi giaûi. Ta coù 3 22 2 2 (*) a ab ab ≥- + Thaät vaäy 2 (*) ( )0 (ñuùng) bab ⇔ -≥ Vaäy (*) ñuùng. Töông töï, ta coù 33 2 2 22 22 2,2 bc b c ca b c ca ≥ - ≥- ++ Do ñoù 333 2 2 2 2 22 222 (ñpcm) abc abc ab bc ca + + ≥ ++ + ++ Ñaúng thöùc xaûy ra khi vaø chæ khi . abc == * Nhaän xeùt 2. Tìm tài liệu Toán ? Chuyện nh ỏ - www.toanmath.com91 Baèng kinh nghieäm baûn thaân, toâi cho raèng ñieàu kieän caàn ñeå söû duïng phöông phaùp naøy vôùi caùc baát ñaúng thöùc thuaàn nhaát laø 1) Daáu ñaúng thöùc xaûy ra khi vaø chæ khi caùc bieán soá baèng caùc giaù trò trong moät taäp höõu haïn naøo ñoù (thöôøng taäp naøy chæ goàm coù 1 giaù trò, toái ña laø 2 giaù trò). 2) Baát ñaúng thöùc ñeà baøi cho laø toång cuûa moät daõy caùc bieåu thöùc ñoãi xöùng nhau vaø toàn taïi moät caùch chuaån hoùa ñeå moãi bieåu thöùc chæ coøn phuï thuoäc vaøo moät bieán soá hoaëc caùc bieåu thöùc laø hoaùn vò lieân tieáp cuûa nhau. Baây giôø ta seõ xeùt moät soá ví duï veà baát ñaúng thöùc coù ñieàu kieän Ví duï 4. Cho,,0 abc > thoûa 2 22 3. abc + += Chöùng minh raèng 1 1 14 .( )7 3 abc abc + + + + +≥ Nhaùp. Nhaän xeùt raèng ñaúng thöùc xaûy ra khi vaø chæ khi 1. abc === Ta coù ( ) 2 22 3 , , 0, 3. a b c abc ++=⇒∈ Ta seõ tìm α sao cho baát ñaúng thöùc sau ñuùng vôùi moïi ( ) 0,3 a ∈ 2 147 . ( 1) (*) 33 aa a α + ≥ -+ Ta coù 32 (*) () 3 4 (7 3) 30 faaaa αα ⇔=- +- -≤ Ta caàn tìm α sao cho ( ) ( ) 0 0, 3 ( ) 0 1. vaø fa a faa ≤ ∀∈ = ⇔= Ñeå coù ñöôïc ñieàu naøy ta caàn coù / 1 (1) 0 9 873 0 6 f α αα = ⇔ - + - = ⇔= Baây giôø ta chæ coøn phaûi xeùt tính ñuùng ñaén cuûa baát ñaúng thöùc Tìm tài liệu Toán ? Chuyện nh ỏ - www.toanmath.com92 2 1417 . .( 1) 363 aa a + ≥ -+ Ta ñi ñeán lôøi giaûi nhö sau Lôøi giaûi. Ta coù ( ) 2 22 3 , , 0, 3. a b c abc ++=⇒∈ Ta seõ chöùng minh 2 1417 . .( 1) (**) 363 aa a + ≥ -+ Thaät vaäy 2 (**) ( 1) (6 ) 0 3 0) (ñuùng do aaa ⇔ - - ≥ >> Vaäy (**) ñuùng. Töông töï, ta coù 2 2 1417 . .( 1) 363 1417 . .( 1) 363 bb b cc c + ≥ -+ + ≥ -+ Do ñoù 2 22 11141 .( ) .( 3) 7 36 ñpcm. abc a b c abc + + + + + ≥ + + - += ⇒ Ñaúng thöùc xaûy ra khi vaø chæ khi 1. abc === Xin ñöôïc löu yù vôùi caùc baïn raèng khoâng phaûi luùc naøo ta cuõng löïa choïn haøm laø nhöõng haøm tuyeán tính hoaëc haøm luõy thöøa khoâng thoâi, maø ñoâi luùc ta caàn phaûi löïa choïn haøm phaân thöùc, haøm caên, ... Ví duï sau seõ cho chuùng ta thaáy roõ ñieàu ñoù Ví duï 5.(APMO 2005) Cho , , 0 8. thoûa Chöùng minh raèng a b c abc >= 222 3 3 3 3 33 4 3 (1 )(1 ) (1 )(1 ) (1 )(1 ) abc a b b c ca + +≥ + + + + ++ Tìm tài liệu Toán ? Chuyện nh ỏ - www.toanmath.com93 Lôøi giaûi. Ta coù 2 3 12 0 (*) 2 1 x x x ≥ ∀> + + Thaät vaäy, ta coù 223 22 (*) (2 ) 4(1 ) ( 2)0 (ñuùng) xx xx ⇔ + ≥+ ⇔ -≥ Vaäy (*) ñuùng. Do ñoù 22 22 33 4 2(,,)2 36 (2 )(2 ) 36 ( , , ) (1 )(1 ) 1 ( ,,) cyc cyc a a S abc a b S abc ab S abc ≥ == + ++ ++ + ∑∑ trong ñoù 2 22 22 2222 (,,)2() Sabc abc ab bcca = + + + ++ Theo baát ñaúng thöùc AM-GM, ta coù 2222 3 2222224 3 3 ( ) 12 3 ( ) 48 (, ,) 72 a b c abc ab bc ca abc S abc ++≥= ++≥= ⇒≥ Do ñoù 2 33 2 24 . 36 36 3 (1 )(1 ) 11 (, ,) 72 ñpcm cyc a ab S abc ≥ ≥ =⇒ ++ ++ ∑ Ñaúng thöùc xaûy ra khi vaø chæ khi 2. abc = == Tìm tài liệu Toán ? Chuyện nh ỏ - www.toanmath.com94 BAØI TAÄP. Baøi 1. (IMO 2001) Cho , , 0. abc > Chöùng minh raèng 222 222 1 888 a bc a bc b ca c ab + +≥ +++ Baøi 2. Cho , , 0. abc > Chöùng minh raèng 3 33 2 22 222 3 a b c abc a ab b b bc c c ca a ++ + +≥ + + ++ ++ Baøi 3. Cho , , , 0. abcd > Chöùng minh raèng 4 4 44 2 2 2 2 2 2 22 ( )( )( )( )( )( )( )( ) 4 a b c d a b cd ab ab b c bc cd c d d a d a + ++ + + +≥ + + + + + + ++ Baøi 4. Cho , , , 0 1. thoûa abcd abcd > + + += Chöùng minh raèng 33 3 3 2 22 2 1 6() 8 a b cd abc d ++ + ≥ + ++ + Baøi 5. (Voõ Quoác Baù Caån) Cho,,0. Chöùng minh raèng abc > 2 2 2 222 2 22 22 22 ( ) () () 3( ) 2() 2 ( ) 2 ( ) 2( ) bc a c ab abc a b c a bc b c a c ab abc + - + - + - ++ ++≥ + + + + + + ++ Baøi 6. Cho,,02 thoûa . Tìm giaù trò lôùn nhaát cuûa bieåu thöùc abc abc > ++= 3 3 3 3 33 2 22 11 11 11 4 44 ba c b ac P ab b bc c ca a --- = ++ +++ Tìm tài liệu Toán ? Chuyện nh ỏ - www.toanmath.com95 Baøi 7. Cho , , , 0. Chöùng minh raèng abcd > 3 3 33 3 3 33 1 63 63 63 63 abcc a bcd b cda c dab c abc + ++≥ + + ++ Baøi 8. Cho 3 33 , , 0 3. thoûa Chöùng minh raèng abc abc > ++= 2 22 1 1 1 5 27 .() 44 abc abc + + + + +≥ Baøi 9. Cho , , 0. abc > Chöùng minh raèng 222 2 2 2 2 22 ( 3)( 3)( 3) 1 ( )2 ( )2 ( )2 2 a bc bc a c ab ab c bc a cab + - + - +- + +≥ + + + + ++ Baøi 10. Cho , , 0. abc > Chöùng minh raèng 333 3 3 3 3 33 (3 ) (3 ) (3 ) 375 ( )3 ( )3 ( )3 11 ab c bc a c ab bc a cab abc + + + + ++ + +≤ + + + + ++ Baøi 11. Cho ,, abc laø ñoä daøi ba caïnh cuûa moät tam giaùc. Chöùng minh raèng 111 9 1 11 4 a b c ab c ab bc c a ⎛⎞ +++ ≥ ++ ⎜⎟ ++ + ++ ⎝⎠ Baøi 12. Cho,,01 thoûa . Tìm giaù trò lôùn nhaát cuûa bieåu thöùc xyz x yz > ++= 2 22 1 11 xyz P xyz =++ + ++ Tìm tài liệu Toán ? Chuyện nh ỏ - www.toanmath.com96 Baøi 13. Cho , , 0. abc > Chöùng minh raèng 3 33 3 8 a bc bc c a ab ⎛ ⎞ ⎛ ⎞⎛⎞ + +≥ ⎜ ⎟ ⎜ ⎟⎜⎟ + ++ ⎝ ⎠ ⎝ ⎠⎝⎠ Baøi 14. (Moldova 2005) Cho 4 44 , , 0 3. thoûa Chöùng minh raèng abc abc > + += 1 11 1 4 44 ab bc ca + +≤ - -- Baøi 15. Cho , , 0. abc > Chöùng minh raèng 2 2 2 2 22 22 22 222 ( )( )( ) 9( ) ( )3 ( )3 ( )3 7( ) ab c bca c a b a b c ab c bc a c a b abc + - + - + - ++ + +≥ + + + + + + ++ Baøi 16. Cho , , 0. abc > Chöùng minh raèng 222222 22 22 222 () () () 7( ) 17 7( ) 17 7( ) 17 5( ) abc b c a c ab a b c ab c bc a c a b abc + - + - + - ++ + +≥ + + + + + + ++ Baøi 17. Cho,,01 thoûa . Chöùng minh raèng a b c abc >= 222 1 11 2 22 abc abc + ++ ++≤++ Baøi 18. Cho,,03 thoûa . Chöùng minh raèng abc abc > + += 3 33 7 77 1 1 13 2 7 77 abc ++≥ +++ Tìm tài liệu Toán ? Chuyện nh ỏ - www.toanmath.com97 Baøi 19. (Vasile Cirtoaje) Cho,,,04 thoûa . Chöùng minh raèng abcd abcd ≥ + + += 2 2 22 1 1 11 2 1 1 11 abc d + + +≥ + + ++ Baøi 20. Cho,,,04 thoûa . Chöùng minh raèng abcd abcd ≥ + + += 2 2 22 1 53 53 53 53 2 a b cd a bcd + + +≤ + + ++ Baøi 21. (Olympic 30 - 4 - 2006) Cho , , 0. Chöùng minh raèng abc > 4 44 4 6 6 3 32 4 6 6 3 32 4 6 6 3 32 3 33 1 ( )( ) ( )( ) ( )( ) a bc a a b ac b bc ba c c a c b ++≤ + + + + + + + ++ Baøi 22. (Japan 1997) Cho , , 0. Chöùng minh raèng abc > 222 2 22 222 3 ( ) ( ) ( )5 a bc abc bca c ab + +≥ + + + + ++ Baøi 23. (Phaïm Vaên Thuaän) Cho , , 0. Chöùng minh raèng abc > 333 3 33 333 1 ( ) ( ) ( )3 abc a bc b c a c ab + +≥ + + + + ++ Baøi 22. Cho , , 0. Chöùng minh raèng abc > 2 2 2 2 22 1 2( ) 2( ) 2( ) a bc a b c b c a c ab + +≥ + + + + ++ Baøi 23. (Phaïm Kim Huøng, V oõ Quoác Baù Caån) Cho , , , 0 4 2. thoûa vaø Chöùng minh raèng abcd abc d k ≥ +++=≥ 1 1 11 ( 1)( 1)( 1)( 1) ( 1)( 1)( 1)( 1) k k k k k k kk a b c d a bcd + + ++ + + + +≥ + + ++ Tìm tài liệu Toán ? Chuyện nh ỏ - www.toanmath.com98 Baøi 24. Cho,,01 thoûa . Chöùng minh raèng abc abc > + += 1 1 1 36 (2 ) (2 ) (2 ) 5 a bc c b a c ba + ++ + +≥ - -- Baøi 25. (Romania 2005) Cho,,03 thoûa . Chöùng minh raèng abc abc > + += 2 22 2 22 1 11 abc abc ++≥ + + Baøi 26. (Phaïm Vaên Thuaän) Cho,,03 thoûa . Chöùng minh raèng abc abc ≥ + +≥ 222 111 1 a b c b c a c ab + +≤ + + + + ++ Tìm tài liệu Toán ? Chuyện nh ỏ - www.toanmath.com99 PH NG PHÁP PHÂN TÍCH BÌNH PH NG S.O.S A. N I DUNG PH NG PHÁP. I. Bài toán m u và nh lý. Thông th ng, khi ng tr c m t bài toán quen bi t, cách chúng ta th ng b t u gii quyt không phi là th mò m m các bt ng th c ã bit, không phi là tìm ngay mt cách dn bin nào ó mà thông th ng nht là a v các dng bình ph ng. u này d a trên tính cht c bn nht ca s th c “ 2 0, xx ≥ ∀∈R ”. Có t nhiu bài toán, cho dù bn ch ng hay vô tình, u ã s dng ph ng pháp này trong ch ng minh. Tuy nhiên, rt có th nh ng u bn sp c c trong c này s làm bn th c s ngc nhiên… Chúng ta s m u v i bt ng th c AM-GM, ây có th coi là bt ng th c cn nht trong nh ng bt ng th c c bn. Nh ng chúng ta ch tìm hiu bt ng th c này trong tr ng h p n r t nh . V i 2 n = chng hn, ta có bt ng th c Ví d 1. i mi ,0 ab ≥ , ta có bt ng th c 22 2. a b ab +≥ không có nhi u u c n ph i bàn ti b t ng th c trên, ngay khi các b n h c s th c thì vi c ch ng minh b t ng th c ó quá d . B t ng th c t ng ng i 2 ( )0 ab -≥ m t u quá hi n nhiên. Bây gi , chúng ta xét ti p khi 3 n = và bt ng th c sau ây Ví d 2. i mi ,,0 abc ≥ , ta có bt ng th c 3 33 3. a b c abc ++≥ Khi h i v m t cách ch ng minh th t c th cho b t ng th c này, chúng ta s c m th y có m t chút b i r i! T t nhiên, b t ng th c trên không khó, l i gi i ch trong duy nh t m t dòng… 2 22 1 .( )(( ) ( ) ( ) ) 2 VT VP ab c ab bc ca - = + + - + - +- Và chc chn ây là cách làm thông minh nht, vì chúng ta không phi qua mt c trung gian nào c. C hai ví d trên u c ch ng minh bng ph ng pháp Tìm tài liệu Toán ? Chuyện nh ỏ - www.toanmath.com100 phân tích bình ph ng nh ng theo m t ngh a t ng i h p. Thu n l i r t l n trong i gii bài toán bng cách này là vic s dng rt ít kin th c “cao cp”, thm chí n không cn bit bt k mt nh lý nào v bt ng th c c. Ngoài ra, nó còn là t ph ng pháp rt t nhiên theo suy ngh c a chúng ta. u c k các bài toán ch ng tr c, các b n ã g p không ít nh ng bài toán s ng ph ng pháp này trong ch ng minh. Còn bây gi , chúng ta s khái quát hóa cách s dng và i tìm bn cht ca mt ph ng pháp c c k hiu qu. Bài toán quan trng mà chúng ta phi xét n trong mc này là m t bt ng th c i ting ã c gi i thiu ch ng tr c, bt ng th c Iran 96. Bài toán 1. (Iran 96) i m i s th c ,, abc không âm, ta có 2 22 1 1 19 () 4 ( ) ( ) () ab bc ca ab bc ca ⎛⎞ ++ + +≥ ⎜⎟ + ++ ⎝⎠ ây c ng là bài toán có hình th c phát biu rt n gin và p mt. Ngoài ra, nó còn là m t b t ng th c r t khó khi b n ch a c ti p c n tr c ó. Nh ng tr c tiên, chúng ta hãy xem li bt ng th c trong k thi IMO 2005 và tìm mt ch ng minh th t t nhiên cho nó. Ví d 3. (IMO 2005) ,, xyz là các s th c dng th a 1 xyz ≥ . Ch ng minh 5 2 5 2 52 5 2 2 5 22 52 2 0 xxyyzz x y z yz x zx y --- + +≥ + + + + ++ Ch ng minh. Không mt tính tng quát ta ch cn xét tr ng h p 1 xyz = là (các bn hãy t tìm hi u lý do t i sao nhé!). Khi ó, ta có ( ) 52 52 4 2 42 22 5 22 4 22 4 222 5 22 . 2 () ( ) 2 () x x x x xyz x x yz x x y z x yz x yz yz x yz x y z xyz - - - -+ = =≥ + + ++ ++ ++ t 2 22 ,, ax b y cz = == . Khi ó, ta ch cn ch ng minh Tìm tài liệu Toán ? Chuyện nh ỏ - www.toanmath.com101 2 22 2 () 0 2() cyc a abc a bc -+ ≥ ++ ∑ 2 2 22 2 22 2 2 222 ()0 2 ( ) 2 () ().0 (2 ( ))(2 ( )) (ñuùng) cyc cyc ab ab abc bca c ac bc a b ab ab abc bca ⎛⎞ ⇔- -≥ ⎜⎟ + + ++ ⎝⎠ ++ ++ - ⇔-≥ + + ++ ∑ ∑ ⇒ pcm. ng th c xy ra khi và ch khi 1 abc x y z == ⇔ = == . Ch ng minh trên không phi là cách duy nht, có th còn nhiu ch ng minh c áo h n. Nh ng n u xem xét khách quan thì ch ng minh trên hoàn toàn r t t nhiên và c b n. Nói khái quát, khi ng tr c m t b t ng th c b t k ba bi n ,, abc ta tìm cách a chúng v d ng tng các bình ph ng ký hi u 222 ( )( )( ) 0 a bc S b c Sc a Sa b - + -+ -≥ Phn a v dng chính tc trên là b c u tiên trong cách s dng ph ng pháp S.O.S. Nu bn ã khá quen v i bt ng th c thì vic lp công th c trên là t ng i n gi n, ch c n bi t qua m t s phép bi n i và h ng ng th c, còn n u b n ch a quen, thì các thc mc s c gii quyt trong m c “Biu din c s c a ph ng pháp S.O.S và m t s k thu t phân tích”. t nhiên, n u trong bi u di n c s ó, các h s ,, abc S SS u không âm thì bài toán c ch ng minh. T tr c t i nay, ây vn là cách bn th ng làm nh ng ây ch là tr ng h p n gin nht trong k thut ch ng minh ca ph ng pháp S.O.S. u quan tr ng h n, S.O.S giúp chúng ta gi i quy t các tr ng h p mà theo quan nim c là không th áp dng c “có mt h s trong ,, abc S SS không ng”. Thông th ng, trong các bài toán i x ng ta có th gi s abc ≥≥ . V i các bài toán hoán v thì ph i xét thêm tr ng h pabc ≤≤ . Trong tr ng h pabc ≥≥ , ta có các nh n xét sau 1. N u 0 b S ≥ , do 2 22 ( ) ( ) () a c ab bc -≥ -+ - nên Tìm tài liệu Toán ? Chuyện nh ỏ - www.toanmath.com102 2 2 2 22 ( ) ( ) ( ) ( )( ) ( )( ) a b c a b cb S b c S c a S ab S S bc S S ab - + - + - ≥ + - + +- và phn còn li ca bài toán là ch ng minh 0, 0. ab bc S S SS + ≥ +≥ Nh ng hai b t ng th c này luôn có th ch ng minh khá n gi n, vì chúng không còn ph i nhân thêm v i các bình ph ng 2 22 ( ),( ),() ab bc ca - -- . 2. N u 0 b S ≤ , do 2 22 ( ) 2( ) 2( ) a c ab bc - ≤ - +- nên 2 2 2 22 ( ) ( ) ( ) (2)( ) ( 2)() a b c a b cb S bc S c a S ab S S bc S S ab -+ - + - ≥ + -+ + - ng v y, vi c ch ng minh còn l i 20,20. ab cb S S SS +≥ +≥ s n gi n h n r t nhiu. Trong nhi u tr ng h p, ta c n thêm m t s c l ng m nh h n, ch ng h n c ng hay dùng n là .()() a ac bc a b c b - ≥ - ≥≥ Ch ng h n khi ta có ,0 bc SS ≥ thì 22 2 2 2 2 22 22 () () ( ) ( ) . .() .( ) ab ab b a a bc S b c S c a S bc S bc aS bS bb - -+-≥-+ -=+ và nh v y bài toán s c ch ng minh n u 22 0 ba aS bS +≥ . Ta có th tóm t t các k t qu trên thành nh lý nh sau nh lý S.O.S. Xét bi u th c 222 (,,) ( ) ( ) () a bc S f abc S bc S c a S ab = = - + - +- trong ó ,, abc S SS là các hàm s theo ,, abc . 1. Nu,,0 abc SSS≥ thì 0 S ≥ . 2. Nuabc ≥≥ và, 0,0 b a b bc SS S SS + ≥ +≥ thì 0 S ≥ . 3. Nuabc ≥≥ và ,, 20,20 acab cb SSS S SS +≥ +≥ thì 0 S ≥ . 4. Nuabc ≥≥ và 22 ,,0 bc ba S S a S bS +≥ thì 0 S ≥ . 5. Nu 0 abc SSS ++≥ và 0 ab bc ca SSSSSS + +≥ thì 0 S ≥ . Ngoài ra, 0 S ≥ v i m i ,, abc thì ta ph i có Tìm tài liệu Toán ? Chuyện nh ỏ - www.toanmath.com103 0, 0, 0. ab bc ca ab bc ca S S S S SS === + ≥ + ≥ +≥ Trong ó, ab ab SS = + có ngha là ta xét bi u th c ab SS + khiab = . V i các bài toán i x ng, ta có ngay ab SS = khiab = . Nh n xét này r t quan tr ng trong các bài toán tìm hng s t t nht. ng nh nh lý này còn có v quá n gi n và nêú nói r ng nó có ng d ng vi u ht các bt ng th c 3 bi n thì tht khó mà t ng t ng c. Nh ng th c t S.O.S ã làm c u này và ây là m t u rt ngc nhiên. t câu hi n a t ra là v i nh ng biu th c nào thì ta có th chuyn v dng chính tc S.O.S nh v y? Câu tr l i là mi hàm s i x ng ( ,,) f abc th a mãn u ki n ( ,,) 0 f aaa = và f có th ch a cn th c, phân th c ca ,, abc luôn luôn có biu din y. Ch ng minh u này bn xem trong phn tip theo. Bây gi là mt s ví d c th ch ng minh tính hiu qu ca ph ng pháp này, và nu có th thì tr c tiên bn hãy th ch ng minh chúng theo cách khác. Ví d 4. Ch ng minh r ng v i m i s th c không âm ,, abc ta luôn có 22 8 2 ( )( )( ) 2 a b c abc ab bc ca a b b c c a ++ +≥ + + + ++ Ch ng minh. Ta chú ý n hai ng th c sau ây ( )( )( ) 222 2 22 222 1 .(( )( )( )) 2 8 ( ) ( ) () a b c ab bcca ab bc c a ab bc c a abc cab abc bca + + - - - = - + - +- + + + - =- +- + - Nh th sau khi thêm b t 1 mi s hng v trái, ta có bt ng th c t ng ng 2 2 2 222 ( ) ( )( ) 2( )2( )2( ) ( )( )() ab bc ca cab abc bca ab bcca ab bc c a - +- +- - + - +- ≥ + + + ++ Ta tìm c ( )( )( ) 2 a a b b c c a abc S a bca ab bc ca ab bc ca + ++ = - = + -- + + ++ Tìm tài liệu Toán ? Chuyện nh ỏ - www.toanmath.com104 ( )( )() 2 ( )( )() 2 b c a b b c c a abc S b c ab ab bc ca ab bc ca a b b c c a abc S c abc ab bc ca ab bc ca + ++ = - = + -- + + ++ + ++ = - = + -- + + ++ Do tính i x ng nên có th gi s abc ≥≥ , khi ó d th y ,0 bc SS ≥ . D a vào tiêu chun th nht, ta ch cn ch ng minh rng 0 ab SS +≥ là xong. Nh ng u này r t hi n nhiên vì 2 2 2() 20 ab abc c a b S Sc ab bc ca ab bc ca + +=- =≥ ++ ++ y b t ng th c ã c ch ng minh. ng th c xy ra khi và ch khi abc == hoc ,0 a bc == hoc các hoán v t ng ng. Chúng ta hãy tr li v i bt ng th c Iran 96. Ví d 5. (Iran TST 1996) i m i s th c ,, xyz không âm, ta có 2 22 1 1 19 () 4 ( )( )( ) xy yz zx x y y z zx ⎛⎞ ++ + +≥ ⎜⎟ + ++ ⎝⎠ Ch ng minh. t ,, a x yb yzc zx = + = + =+ . Ta ph i ch ng minh 222 222 1 1 19 (222) 4 ab bc ca a b c abc ⎛⎞ ++- - - + + ≥ ⎜⎟ ⎝⎠ ng bin i n gin, ta có th chuyn bt ng th c v dng 2 2 21 ()0 cyc ab ab c ⎛⎞ - -≥ ⎜⎟ ⎝⎠ ∑ 2 2 2 21 21 21 a b c S bc a S ca b S ab c =- =- =- Tìm tài liệu Toán ? Chuyện nh ỏ - www.toanmath.com105 Gi s r ngabc ≥≥ thì ,0 ab SS ≥ . S dng tiêu chun 4, ta ch cn ch ng minh 22 33 0 bc bS cS b c abc +≥ ⇔ +≥ nh ng b t ng th c này hi n nhiên úng vì 33 ( ). a b c b c bc b c abc ≤ + ⇒ + ≥ +≥ ng th c x y ra khi và ch khi a = b = c ho c a = b, c = 0 ho c các hoán v t ng ng. Có m t vài ch ng minh khác cho b t ng th c Iran 96, cách thông th ng chúng ta bi t là khai tri n và s dng b t ng th c Schur (hoc dùng nh lý Muirhead), ho c dùng a th c i x ng. Tuy nhiên, b n c s ng ý v i tôi r ng các ph ng pháp ó ch có ý ngha là ch ng minh bt ng th c úng v mt toán hc, ch không li nhiu n t ng. Vic bit s d ng ph ng pháp S.O.S ã làm cho bài toán tr nên n gi n h n rt nhiu, ây th c s là m t l i gii p và ngn g n, th a mãn c m quan toán h c c a nhi u ng i. Ph ng pháp phân tích bình ph ng ã t ng xu t hi n theo cách này hay cách khác trong mt s bt ng th c, vì nó là mt h ng suy ngh rt t nhiên i v i bt ng th c. Nh ng chc chn ây s là ln u tiên mà ph ng pháp này c h th ng và c coi là ph ng pháp chính th ng cho chúng ta. Nó em l i cho chúng ta m t cách nhìn ch ng và vô cùng hi u qu i v i các bài toán mà ch m t th i gian ngn tr c còn là nh ng bài toán vô cùng khó khn. Bt ng th c Iran 96 c coi là bài toán c bn ng dng ph ng pháp này (mc dù tác gi nghn S.O.S t m t b t ng th c c h n). S.O.S là tên l y t ch cái u tiên c a c m t Sum of Square. II. Bi u di n c s ph ng pháp S.O.S. 1. M u. Trong các bài toán c d n ra các m c tr c h n các b n ã nh n th y s l p i p l i ca bi u th c d ng 222 (, ,) ( ) ( ) ( ). a bc Fabc Sb c Sc a Sa b = - + - +- Các nh lý sau ây s cho th y s t n t i c a bi u di n ó. Chúng tôi t gi i h n mình trong các l p b t ng th c 3 bi n i x ng, tuy nhiên u ó s không làm h n ch t m Tìm tài liệu Toán ? Chuyện nh ỏ - www.toanmath.com106 ng dng ca ph ng pháp này. Các bn có th s dng các ví d ki m ch ng ng v i cùng tng d i ây, h u h t các b t ng th c hoán v ba bi n c ng có nh ng bi u di n ng t . Chúc các bn may mn! 2. Các khái ni m c b n. 2.1. Tp xác nh (TX). ây tri nu không có gì thay i, cho bài toán rõ ràng và tránh nh ng phi n ph c không áng có, TX c a t t c các hàm s và b t ng th c s gii h n trong tp s th c 3 + R , h n n a, ôi khi h p lý chúng ta s b i m (0,0,0). 2.2. nh ngh a 1: Hàm i x ng ba bi n. t hàm phân th c ba bi n (,,) F abc c gi là i x ng nu và ch nu ng nh t th c sau ( ,,) ( , ,) F abc F xyz = úng v i m i hoán v ( , ,) x yz c a( ,,) abc .H n a nu v i m i s th c d ng x mà (,,)0 F xxx = thì (,,) F abc c gi là hàm i x ng ba bin chun. 2.3. nh ngh a 2: Hàm n a i x ng ba bi n. t hàm phân th c ba bi n (,,) G abc c g i là n a i x ng n u và ch n u ng nh t th c sau (, ,) (, , ) G abc G acb =úng v i m i b ba s th c d ng( ,,) abc . H n a nu v i mi cp hai s th c d ng , xy mà (,, )0 G xyy = thì (,,) G abcc i là hàm n a i x ng ba bin chun. 3. Các nh lý c s. 3.1. nh lý 1: C s c a ph ng pháp S.O.S. Gi s (,,) F abc là m t a th c i x ng ba bi n chu n, thì t n t i m t a th c n a i x ng (,,) G abc sao cho ng nh t th c sau úng 222 (, ,) (, ,)( ) ( ,, )( ) ( , , )( ) F a bc G a bc b c Gbca c a G cab a b =-+ -+- Tr c khi a ra mt ch ng minh c a nh lý này d a trên mt s hi u bit n gi n v không gian vect chúng tôi mu n nh n m nh v i các b n r ng nh lý trên là áp d ng i v i t t c các hàm phân th c i x ng ba bi n. B i vì nh lý 1 n ch trong các l p a th c ba bin nên có th nói t i bc ca a th c. Trong a Tìm tài liệu Toán ? Chuyện nh ỏ - www.toanmath.com107 th c ba bi n ,, abc s ch a (và ch ch a!) các h ng t ,, m np m np t a bc trong ó ,, mnp là các s nguyên không âm. Ch ng minh nh lý 1. Ta ch ng minh nh lý 1 cho l p các a th c b c n . Ký hi u () SF là t p h p t t c các a th c ba bi n (,,) F abc i x ng chu n b c n , () SQ là t p h p t t c các a th c (,,) G abc i x ng ba bi n chu n b c n d ng 222 (, ,) (, ,)( ) ( ,, )( ) ( , , )( ) G abc G abc b c Gbc a c a G cab a b =-+ -+- ây (,,) G abc là a th c n a i x ng ba bi n b c 2 n - (ta xét 2 n ≥ vì v i 1 n = thì nh lý hi n nhiên úng). Rõ ràng () SQ là không gian vect con c a không gian vect (,,) F abc . Và do ó, chi u c a () SQ không v t quá s chi u ca () SF. (*) i các s nguyên không âm ,, α βγ xét các a th c c bi t sau ây (i) ,, (,,) sym F abc abc αβγ αβγ = ∑ (ii) ( ) ,, ,, G abcabcabc αβγαγβ αβγ =+ (iii) 2 ,, ,, (,,) (,,)( ) Q abcG abcbc αβγ αβγ = -+ 22 ,, ,, 1 (,,)() (,,)() G bcaca G cabab αβγ αβγ + -+- Ký hi u n f là t p h p t t c các b s ( , ,) α βγ th a mãn các u kin ,. n α β γ α βγ + + = ≥≥ Rõ ràng t p h p t t c các a th c ,, ( ,,) F abc α βγ v i( , ,) n f α βγ∈ là h sinh c l p tuy n tính c a () SF do ó s chi u c a () SF b ng s ph n t c a n f .(1) Ký hi u n q là t p h p t t c các b s ( , ,) α βγ th a mãn các u kin 2,2 n α β γ α βγ + + = - + ≥≥ . Rõ ràng tp h p tt c các a th c ,, ( ,,) G abc α βγ v i( , ,) n q αβγ∈ là h vect c p tuyn tính ca () SQ do ó s chiu ca () SQ không nh h n s phn t c a n q. (2) Tìm tài liệu Toán ? Chuyện nh ỏ - www.toanmath.com108 các kt qu (1), (2) v i chú ý là n f và n q có cùng s phn t ta suy ra s chiu a () SQ không nh h n s chi u c a () SF. (**) y t các k t qu ( * ), ( ** ) suy ra s chiu ca hai không gian () SQ , () SF là bng nhau, tó suy ra mi phn t ca không gian () SFu có th biu din qua các ph n t ca không gian () SQ . ây là kt qu cn phi ch ng minh. nh lý này có th nhn thy mt thut toán tìm biu din c s , ó là tìm ma tr n chuy n gi a hai không gian vect () SQ và () SF . D i ây là m t thut toán c p h n. 3.2 nh lý 2: Thu t toán tìm bi u di n c s . Gi s ( ,,), ( ,,) M abc N abc là hai a th c n a i x ng ba bi n, h n n a v i m i s th c d ng x thì phân s ( , ,) ( , ,) M xxx N xxx là mt hng s t . Khi ó tn ti hàm s n a i x ng ba bin (,,) G abc sao cho ng nh t th c sau úng 222 ( , , ) ( , ,) ( ,, ) (, ,) 3 ( ,, ) ( , , ) ( , ,) (, ,)( ) ( ,, )( ) ( , , )( ) M abc M b c a M c ab Fabct N abc N bca N cab G ab c b c Gbca c a G cab a b = ++- =-+ -+- Ch ng minh nh lý 2. i v i hàm n a i x ng (,,) G abc chúng ta tin hành ghép cp các hng t n a i x ng . mn p m pn a bc a bc + Sau ó, nhóm tt c các hng t có cùng bc vào mt nhóm. B s 12 ( , ,..., ) k nnn v i 12 ... k nnn > >> g m tt c các giá tr b c ca a th c ó sp theo th t gi m dn gi là b ch th cho a th c ó. Khi ó, ta có th vi t ,, 1, (, ,) . ( ) i k m n p pn mnp i mn pn n p G abc g a b c b c = ++=≥ =+ ∑∑ Rõ ràng u ki n ( , ,) ( , ,) M xxx N xxx là m t h ng s vi m i s th c d ng x t ng ng i s ki n b ch th c a các a th c ( ,,), ( ,,) M abc N abc là gi ng nhau. Và do ó ta xét hi u Tìm tài liệu Toán ? Chuyện nh ỏ - www.toanmath.com109 ,, 1, .() ( ,,) ( ,,) ( ,,) i k m n p pn mnp i m n p n np a b c bc M abc t N abc N abc α = ++=≥ + -= ∑∑ trong ó, ,, ,, ,, mn p mnp mnp m tn α =- và do ó ,, , 0, 1,2,..., . i mnp mn pn n p in α ++=≥ = ∀= ∑ Bây gi i vi m i t ng bên trong t ng ng vi m i giá tr i n c a t s chúng ta ti n hành s p x p l i th t các h ng t trong t s c a phân s trên sau ó s dùng t bi n i nh làm xu t hi n các nhân t ,,. abbcc a - -- Tr c ht ta chia các nghi m nguyên không âm( , ,) mnp tha mãn np ≥ ca ph ng trình i m npn + += thành i n nhóm theo các giá tr m. Sp xp li th t các nhóm theo gi m d n c a m. Trong m i nhóm thì giá tr c a m là c nh, ta p x p l i các nghi m nguyên không âm c a ph ng trình i npn m + =- theo gi m dn ca n nu i nm - l và theo tng dn ca n nu i nm - ch n. Sau khi ã sp th t xong, chúng ta có mt th t m i ca các tp nghi m ban u, mà ta ký hiu là { } ( , , ) 1,2,..., , j jj mnpjl = ây l là mt hàm s ph thuc i n . n gin ta ký hiu ,, ( ), jj jjj j jj pp mnn j j m np a a bc b c ba = += Khi ó m u s có th vi t l i m t cách n gi n là 1 1 22 1 2 1 2 3 1 2 1 12 ... ( ) ( )( ) ... ( )( ... ). nn lll ab ab ab aabaabb a abbb - + ++= =- + - ++ + - + + + d ng u ki n 12 ...0 l bbb + + += và chia các hiu 1 22 31 , ,..., ll a aa aaa - - -- vào ba loi sau (i) 11112 ()(). .() npnp mnppn mnp pn mnp bc abcbc abc bc abc bc bc -- + + ++ - +-+=- - (ii) 1 11 ( ) ( ) [( )( )] mnnnn mnnnn mnn abcbc abcbc abcabca + ++ + - + = --- Xét bi u th c [( ) ( )] [( ) ( )] [( ) ( )] (, ,) ( , , ) ( , , ) m nn mn n m n n abc ab c a bca bc a b cab c b bc N a bc N b c a N c ab -- - - - - --- ++ Tìm tài liệu Toán ? Chuyện nh ỏ - www.toanmath.com110 Ti n hành ghép t ng ph n trong ba h ng t trong bi u th c này thành ba c p theo các nhân t ,,. abbcc a - -- M t trong ba hng t m i s là 2 () ().(,,) (, ,) ( ,, ) mn mn n nn ab a babc a b Gc a b N abc N b c a -- ⎡⎤ - - =- ⎢⎥ ⎣⎦ trong ó . ( ,, ) . (, , ) (,,). (, , ). ( ,, ) n n n mn mn c a b a N b c a b N abc G cab N abc N b c a a b -- - = - ây, ta ã s d ng ( ,, ) ( , ,). N bca N bac = Do c t s và mu s ca phân s trên u là nh ng a th c n a i x ng ba bi n ,, abc và i x ng hai bi n , ab nên (,,) G cab là hàm n a i x ng ba bin. (iii) 11 1 11 11 ( ) ( ) [( ) ( )] mnnnn mnn nn mnn abcbc abc bc abccabbca ++ + ++ ++ + - + = --- Xét bi u th c [() ( )] [( ) ()] (, , ) ( ,, ) [ ( ) ( )] ( , ,) m nn mn n m nn a b c c a b bc a b c a ab c ca b N abc N bca c a b bc a ab c N c ab - - - - -- ++ - -- + Ti n hành ghép t ng ph n trong ba h ng t trong bi u th c này thành ba c p theo các nhân t ,,. abbcc a - -- M t trong ba hng t m i s là 12 ( ) ( ). ( ,,) (, , ) ( ,, ) mn mn n nn ab a babc a b Gc a b N abc N bca -- + ⎡⎤ - - =- ⎢⎥ ⎣⎦ trong ó 1 . ( ,, ) . (, , ) (,,). (, , ). ( ,, ) n n n mn mn c a b a N b c a b N abc G cab N abc N b c a a b + -- - = - ây ta ã s dng ( ,, ) ( , ,). N bca N bac = Do c t s và mu s ca phân s trên u là nh ng a th c n a i x ng ba bi n ,, abc và i x ng hai bi n , ab nên (,,) G cab là hàm n a i x ng ba bin. Tìm tài liệu Toán ? Chuyện nh ỏ - www.toanmath.com111 y trong c ba tr ng h p ta u ch ra cách bin i thích h p a biu th c dng biu din cn thit. u này hoàn thành vic ch ng minh nh lý 2. Ni m tin v s t n t i bi u di n c s ã c kh ng nh. B. CÁC BÀI T P ÁP D NG. I. Bài t p có l i gi i. Bài 1. Cho ,,0 abc > th a 1 min{ , , } .max{ , , } 4 abc abc ≥ . Ch ng minh r ng 2 2222 1 1 1 9 1 () (). 4 16 ( ) ( ) ( ) () cyc ab ab bc ca ab bc c a ab ⎛⎞ ⎛⎞ - + + + + ≥+ ⎜⎟ ⎜⎟ ⎜⎟ ++++ ⎝⎠ ⎝⎠ ∑ Ch ng minh. Không m t tính t ng quát gi s 1 . 0. 4 cbac ≥≥≥> t 2 2 ,, 2 b c x a x yz c a y b x y z xyz a b z c x yz + = =- ++ ⎧⎧ ⎪⎪ + = ⇔= - +⇒ ⎨⎨ ⎪⎪ + = =+- ⎩⎩ là dài ba c nh ca mt tam giác Do 1 . 4 cbac ≥≥≥ nên 0 x yz ≥ ≥> và4( ) 355 x yz x yz y z x - ++ ≥+ - ⇒ + ≥ t ng th c cn ch ng minh tr thành 2 2 22 2222 2 2 22 2222 2 2 111 9 1 ( ) (222 ) . 4 16 4 44 1 1 1 1 () (222 ) 9 . 4 25 ()0 4 cyc cyc cyc xy xy yz zx x y z xyzz xy xy yz zx x y z xyzz xy xy z ⎛⎞ ⎛⎞ - + + - - - + + ≥+ ⎜⎟ ⎜⎟ ⎜⎟ ⎝⎠ ⎝⎠ ⎛⎞ ⎛⎞ - ⇔ + + - - - + + ≥+ ⎜⎟ ⎜⎟ ⎜⎟ ⎝⎠ ⎝⎠ ⎛⎞ ⇔- -≥ ⎜⎟ ⎝⎠ ∑ ∑ ∑ t 2 22 252525 ,, 4 44 xyz SSS yz zx xy x yz =- = - =- t ng th c c n ch ng minh t ng ng v i 222 ( ) ( ) ( )0 x yz S yz S zx S x y - + - + -≥ Do 0 x yz ≥ ≥> và355 y zx +≥ nên 0 x S > và850 y y xS ≥ ⇒≥ Ta ch ng minh Tìm tài liệu Toán ? Chuyện nh ỏ - www.toanmath.com112 22 22 33 0 225 2 4( )5 yz y S zS yz xz xy y z xyz +≥ ⇔ +≥ ⇔ +≥ Mà355 y zx +≥ nên ta ch c n ch ng minh 33 22 4( ) (3 5 ) ( )(4 4 )0 (ñuùng) y z y z yz y z yyzz + ≥+ ⇔ - + -≥ Ta có .( )0 y x z xy z - ≥ -≥ . Do ó 2 22 22 2 22 2 222 2 ( ) ( ) ( ) ( ) () . .() () ()() 0 x y z yz yz yz S y z S z x S xy S z x S xy y S xy S xy z x y yS zS z - + - + - ≥ - +- ≥ - +- -+ = ≥ ⇒ pcm. ng th c xy ra khi và ch khi x yz == ho c 5 . 8 yzx == . Bài 2. (Vasile Cirtoaje) ,,0 abc > th a 222 1 abc ++= . Ch ng minh r ng 1 1 19 1 1 12 ab bc ca + +≤ - -- Ch ng minh. t ng th c c n ch ng minh t ng ng v i 2 30 1 13 0 1 26 0 1 cyc cyc cyc ab ab ab ab ab ⎛⎞ -≥ ⎜⎟ - ⎝⎠ - ⇔≥ - - ⇔≥ - ∑ ∑ ∑ Tìm tài liệu Toán ? Chuyện nh ỏ - www.toanmath.com113 2 22 2 2 22 2 2 2 22 2 22 22 22 2( )6 0 1 3( )2 0 11 3() 0 1 11 3() 0 1 11 3( ) ( )() 1 (1 )(1) cyc cyc cyc cyc cyc cyc cyc cyc cyc cyc cyc a b c ab ab a b c ab ab ab a b c a bc ab ab ab a b ab ab ab bc ca ab ab a bc ab bc ca + +- ⇔≥ - - -- ⇔ +≥ -- - -- ⇔ + -≥ - -- - -- ⇔ + -≥ - -- - -+ ⇔ -≥ - -- ∑ ∑∑ ∑ ∑∑ ∑ ∑∑ ∑∑ 2 2 22 0 ( )(3 4 4 3 )0 cyc a b ac bc a bc ab c abc ⇔ - - - +++ ≥ ∑ t 2 22 2 22 222 3443 3443 3443 a b c S ab ac ab c abc a bc S ab bc a bc abc ab c S bc ac ab c a bc abc =- - + ++ =- - + ++ =- - +++ t ng th c c n ch ng minh t ng ng v i 222 ( )( )( ) 0 a bc S b c Sc a Sa b - + -+ -≥ Ta có 2 22 22 22 344 3( )44 3 3 44 22 32 3244 0 a S ab ac a b c ab ac aa b c ab ac ab ab ab ac >-- = + + -- ⎛ ⎞⎛⎞ = + + + -- ⎜ ⎟⎜⎟ ⎝ ⎠⎝⎠ ≥ + -- > Do ó 0 a S > ng t 0,0 bc SS >> 222 ( ) ( ) ( )0 a bc Sb c Sc a Sa b ⇒ - + -+ -≥ ⇒ pcm. ng th c xy ra khi và ch khi abc == . Tìm tài liệu Toán ? Chuyện nh ỏ - www.toanmath.com114 Bài 3. (Vietnam Team Selection Test 2006) Cho , , [1,2] xyz ∈ . Ch ng minh 111 ( )6 cyc x x yz x y z yz ⎛⎞ ⎛⎞ + + + +≥ ⎜⎟ ⎜⎟ ⎜⎟ + ⎝⎠ ⎝⎠ ∑ Ch ng minh. Ta có, bt ng th c cn ch ng minh t ng ng 22 2 2 2 2 3 3 2 22 222 ( )( )2 931 ( ) () 3 ( )() 2 .( )0 ( )() ( 2)() .( )0 ( )( )() ( 22 cyc cyc cyc cyc cyc x y z xy yz zx x xyz yz zxy xy xyz xz yz z xz yz xy xy xyxz yz z xz yz xy z x y xy xyz x y y z z x z x z y xyz x z y z x yz ⎛⎞ + + ++ -≥- ⎜⎟ + ⎝⎠ -- ⇔≥ ++ + +- ⇔ -≥ ++ ++-+ ⇔ -≥ + ++ ⇔ + + + + -- ∑ ∑∑ ∑ ∑ 22 2 )( )0 cyc xyz xy -≥ ∑ t 3 3 2 2222 22 3 3 2 2222 22 3 3 22222 22 2 22 2 22 2 22 x y z S x y x z x yz x y x z xy z xyz S y x y z xy z x y y z x yz xyz S z x z y xyz x z yz x yz xyz = + + + + -- = + ++ + -- =++ + + - - Khi ó b t ng th c c n ch ng tr thành 222 ( ) ( ) ( )0 x yz S yz S zx S x y - + - + -≥ Không m t tính t ng quát gi s xyz ≥≥ . Do , , [1,2] xyz ∈ nên . 2 x y z x yz + ≥ ≥ ≥≥ Ta có 3 3222 22 2 2 33 2 22 ( )( 2) 0 x S x y x z x yz x y x z xy z xyz x y x z x y z xy xz yz = + + + + -- = + + + +- > Tìm tài liệu Toán ? Chuyện nh ỏ - www.toanmath.com115 3 3 22222 22 2 22 3 3 2 2 22 22 2 22 2 22 ( )( 2) ( )( 2) ( )(2) 0 ( ) ( ) ( )2 ( ) ( )2 2 0 y yz S y x y z xy z x y y z x yz xyz y z x y xy yz zx y z x z xz z zx yzz x zx S S x y z yzy z x y z xyz xyz y z yz y z x yz xyz xyz xyz = + ++ + -- = + ++- ≥ + ++- = +- ≥ + = + + + + -- ≥ + + +- ≥ +- = Do ó theo tiêu chun 2, ta có pcm. ng th c xy ra khi và ch khi ( , , ) ( , , ),(2,1,1) ( [1,2]) x y z ttt t =∈ . Bài 4. ,,0 abc > . Ch ng minh rng 2 1 8 cyc ab bc ca a bc ++ ≥ + ∑ Ch ng minh. Ta có b t ng th c c n ch ng minh t ng ng v i 2 22 2 2222 2 22 2 22 22 0 8 ( ) (8) 0 8 622 0 8 ()2 20 88 cyc cyc cyc cyc cyc ab bc ca bc ab bc ca a bc ab bc ca bc a bc a bc a b a c a bc ab c abc a bc ab c b ca abc a bc a bc ++⎛⎞ -≥ ⎜⎟ ++ + ⎝⎠ ++ -+ ⇔≥ + + - ++ ⇔≥ + - +- ⇔ +≥ ++ ∑ ∑ ∑ ∑∑ Rõ ràng ta có 22 2 () 0 8 cyc a bc a bc - ≥ + ∑ . Do ó ta ch c n ch ng minh Tìm tài liệu Toán ? Chuyện nh ỏ - www.toanmath.com116 2 22 22 2 22 22 2 0 8 0 88 0 88 ( )(8 8) 0 (8 )(8 ) ( )(8 8 )(8 )0 cyc cyc cyc cyc cyc cyc cyc b ca a bc c a ab a bc a bc ab ab b ca a bc ab a bc a bc b ca a b ab c cab +- ≥ + -- ⇔ -≥ ++ -- ⇔-≥ ++ - +- ⇔≥ ++ ⇔ - + - +≥ ∑ ∑∑ ∑∑ ∑ ∑ Không m t tính t ng quát gi s 0 abc ≥≥> . t 2 2 2 (8 8 )(8 ) (8 8 )(8 ) (8 8 )(8 ) a b c S bc a abc S c a b b ca S ab c cab = +-+ = +-+ = +-+ Th thì ta có ,0 bc SS > . t ng th c c n ch ng minh t ng ng v i 222 ( )( )( ) 0 a bc S b c Sc a Sa b - + -+ -≥ Ta có 2 2 22 (8 ) (8 ) a bcab abc + ≥+ Do ó 222 2 22 22 22 22 (8 8 )(8 ) (8 8 )(8 ) (8 8 )(8 ) (8 8 )(8 ) (8 )(7 7 16 ) 0 ba aS bS a c a b bca b bc a a bc b c a b abc b bc a a bc b a bc a bc + = +- + + +-+ ≥ +- + + +-+ = + ++ > Do ó áp d ng tiêu chu n 4, ta có pcm. Tìm tài liệu Toán ? Chuyện nh ỏ - www.toanmath.com117 Bài 5. ,,0 abc > . Ch ng minh rng 2 2 () 0 cyc bc a bc + ≥ + ∑ Ch ng minh. Không m t tính t ng quát gi s 0 abc ≥≥> . Ta có b t ng th c c n ch ng minh t ng ng v i 2 22 2 2 2 22 22 22 22 22 22 2 0 0 0 ( ) ( )( )( ) 0 cyc cyc cyc cyc cyc cyc b ca a bc c a ab a bc a bc a b ab b ca a bc a b a ba b cc ab +- ≥ + -- ⇔ -≥ ++ -- ⇔ -≥ ++ ⇔ - + +- +≥ ∑ ∑∑ ∑∑ ∑ t 2 2 2 ( )( )() ( )( )() ( )( )() a b c S b c bca a bc S ca ca b b ca S ab abc c ab =+ +-+ =+ +-+ =+ +-+ Th thì ta có ,0 bc SS ≥ t ng th c c n ch ng minhh t ng ng v i 222 ( )( )( ) 0 a bc S b c Sc a Sa b - + -+ -≥ Ta ch ng minh 22 2 222 0 (*) ( )( )( ) ( )( )( ) ab b S aS a c ab c a b ca b abc bc a bc +≥ ⇔ +-+ + ≥ --+ + * N ua bc ≤+ bt ng th c (*) hin nhiên úng. * N ua bc >+ Ta có 2 2 22 0 0 ( ) ( )0 c ab abc c a bc a b ca b c ab ⎧ + - > --> ⎪ + ≥+> ⎨ ⎪ + ≥ +> ⎩ nên (*) úng. Tìm tài liệu Toán ? Chuyện nh ỏ - www.toanmath.com118 ây, áp d ng tiêu chun 4, ta suy ra c pcm. ng th c xy ra khi và ch khi ( , , ) ( , , ),( , ,0) ( 0). a b c t tt tt t => . Bài 6. (Crux Mathematicorum) ,, abc là dài ba c nh ca mt tam giác. Ch ng minh ( ) ( ) ( ) ( ) 2 22 2 4 .2 9 ab bc ca R r abc - + - +- ⎛⎞ ≤- ⎜⎟ ⎝⎠ ++ Ch ng minh. Ta có 2 2 22 4 2 2 ( )()() () ( )() R pabc r S abc abc c ab bc a ab c a b bca -=- =- + - + - +- - = + - +- ∑ Do ó b t ng th c c n ch ng minh t ng ng v i ( ) ( ) ( ) ( )( ) 2 2 2 2 22 4 () . 9()() 4 9()() 0 cyc cyc cyc ab ab bc a c ab abc a b a b a b c bc a c ab - - ≤ + - +- ++ ⎛⎞ ⎛⎞ ⇔ - + + + - +- +-≥ ⎜⎟ ⎜⎟ ⎝⎠ ⎝⎠ ∑∑ ∑ Do ,, abc là dài ba cnh ca mt tam giác nên ,, abc cng là dài ba nh ca mt tam giác. Do ó ( )( ) 22 2 22 4 9( )( ) 16 9 7 0 a b a b c bca c a b c c c + + + - +- +- > - => ng t ( )( ) ( )( ) 22 2 22 22 2 22 4 9( )( ) 16 9 7 0 4 9( )( ) 16 9 7 0 bc abc abccab a a a c a a b c bca ab c b b b + + + - +-+- > - => + + + - +- +- > - => ây, ta suy ra pcm. ng th c xy ra khi và ch khi abc == . Tìm tài liệu Toán ? Chuyện nh ỏ - www.toanmath.com119 Baøi 7. (Vasile Cirtoaje) ,, abc laø ñoä daøi ba caïnh cuûa moät tam giaùc. Khi ñoù, ta coù 3 23 a b c acb bc a cba ⎛ ⎞⎛⎞ ++ ≥ ++ + ⎜ ⎟⎜⎟ ⎝ ⎠⎝⎠ Chöùng minh. Ta coù 2 3 23 3 323 (5 5 3)( )0 cyc a bc acb b ca cba a bc acb b c a cba abca b ⎛⎞⎛⎞ + + ≥ + ++ ⎜⎟⎜⎟ ⎝⎠⎝⎠ ⎛⎞⎛⎞ ⇔ + + - ≥ ++- ⎜⎟⎜⎟ ⎝⎠⎝⎠ ⇔ - + -≥ ∑ Ñaët 553, 5 5 3, 5 5 3 abc S bcaS cabS ab c = - + =- + = -+ . Baát ñaúng thöùc caàn chöùng minh töông ñöông vôùi 222 ( )( )( ) 0 a bc S b c Sc a Sa b - + -+ -≥ + Tröôøng hôïp 1.abc ≤≤ . Khi ñoù, ta coù 0 b S ≥ vaø 8 2 0(do ) 8 20 ab cb S S b a ba S S cb +=->≥ + = -> Do ñoù 2 2 2 22 ( ) ( ) ( ) ( )( ) ( )( ) 0 a a c ab cb S b c S c a S ab S S bc S S ab - + -+ -≥ + - ++ -≥ ⇒ ñpcm. + Tröôøng hôïp 2.abc ≥≥ . Khi ñoù ta coù ,0 ac SS ≥ . Do ñoù neáu 0 b S ≥ thì ta coù ngay ñpcm, vì vaäy ta chæ caàn xeùt tröôøng hôïp 0 b S ≤ laø ñuû. + Tröôøng hôïp 2.1. ( ) 31 3 3() a c b a c bc + - ≤ ⇔-≤- Ta coù 3 14 10 12 12( ) 0 ab S S bca bca += + -≥ +- > Do ñoù 222 2 ( ) ( ) ( )( 3 )( )0 a a c ab S b c S c a S ab S S bc - + -+ -≥ + - ≥ ⇒ ñpcm. + Tröôøng hôïp 2.2. ( ) ( ) 31 3 31( ) a c b a b bc + - ≥ ⇔-≥-- + Tröôøng hôïp 2.2.1. 3 2 b a ≥ Ta coù 211 5 7 8()7 0 ab S S b c abca + = + - ≥ + -> Tìm tài liệu Toán ? Chuyện nh ỏ - www.toanmath.com120 3 2 13 5 13( ) 5 8 0 2 cb b S S b c ab ab aa ⎛⎞ + =+ - >+ - - =-≥ ⎜⎟ ⎝⎠ Do ñoù 2 2 2 22 ( ) ( ) ( )( 2 )( ) ( 2 )( )0 a a c ab cb S bc S c a S ab S S bc S S ab - + -+ -≥ + - + + -≥ ⇒ ñpcm. + Tröôøng hôïp 2.2.2. 3 2 b a ≤ + Tröôøng hôïp 2.2.2.1. 2 3 a a c bc + ≥ ⇒≥ Ta coù 211 5 7 8()7 0 2 2 13 5 13. 5 0 33 ab cb S S b c a bca aa SSbcaa + = + - ≥ + -> + = + - ≥ + -= Do ñoù 2 2 2 22 ( ) ( ) ( )( 2 )( ) ( 2 )( )0 a a c ab cb S bc S c a S ab S S bc S S ab - + -+ -≥ + - + + -≥ ⇒ ñpcm. + Tröôøng hôïp 2.2.2.2. 2 2() a c b a c bc +≤ ⇔-≤- Ta coù ( ) ( ) 22 4 3 1 (5 5 3 ) 3 1 17 15 17 abc S S S a b c b ca + + - = - + - + +- Do ( ) 313 a cb + -≥ neân ( ) 31 33 c a b - ≤+ Suy ra ( ) ( ) ( ) ( ) 5 31 5 55353 33 5 3 1 5 23 33 5 31 3 c a abcac ac a - -+≥ - - + -- =+ - > Do ñoù ( ) ( ) 3 2 5 31 4 3 1 17 15 17 3 abc a S S S b ca - + + - > + +- ( ) ( ) 3 5 31 16 17 3 a bca - ≥ + +- Tìm tài liệu Toán ? Chuyện nh ỏ - www.toanmath.com121 ( ) 3 5 31 0 3 a a - > -> Do ñoù ( ) 2 2222 () ( ) ( ) 4 31 () 0 a b c abc S bc S c a S a b S S S bc ⎛⎞ - + - + - ≥ + + - -≥ ⎜⎟ ⎝⎠ ⇒ ñpcm. Bài 8. ,,0 xyz > . Ch ng minh rng ( ) 2 22 22 2 2 2 2 2 x yz xy yz zx xyz yzx zx y + + +++ ≥ + + + + + Ch ng minh. Ta có, bt ng th c cn ch ng minh t ng ng v i ( ) ( ) 2 22 2 2 2 2 22 2 22 2 2 22 2 2 22 2( ) 2( ) 22 4() ( ) 2 2( ) () () ( )2 2() () ( )2 2() cyc cyc cyc cyc cyc cyc x y z xy yz zx x y z y z x z x y xy yz zx x y x y z xyz xyz xy y z yz zxy xy x y xy + + - + +≥ ≥ + + + + + - ++ ⇔ - ≥ + -+ - ⇔ -≥ + ++ - ⇔ -≥ + ++ ∑∑ ∑∑ ∑∑ Ta l i có 22 22 () () 2 () 2() theo bñt Bunhiacopxki cyc cyc zxy zxy xy x y xy -- ≤ + + ++ ∑∑ Do ó ta ch c n ch ng minh 2 2 () () cyc cyc zxy xy xy - -≥ + ∑∑ 2 1 ( )0 cyc z xy xy ⎛⎞ ⇔ - -≥ ⎜⎟ + ⎝⎠ ∑ t 1 , 1 ,1 x yz x yz S SS y z z x xy =- =- =- +++ Tìm tài liệu Toán ? Chuyện nh ỏ - www.toanmath.com122 t ng th c c n ch ng minh t ng ng v i 222 ( ) ( ) ( )0 x yz S yz S zx S x y - + - + -≥ Không m t tính t ng quát gi s 0 x yz ≥ ≥> . Khi ó ,0 yz SS > Ta có 22 2 2 22 22 20 yx x y xy xS yS xy xy xy xz yz + = + - - ≥+- ≥ ++ Do ó, áp d ng tiêu chu n 4, ta có pcm. ng th c xy ra khi và ch khi x yz == . Bài 9. (Hojoo Lee) ,,0 abc > . Ch ng minh rng 3332 2 22 2( ) 9( ) 33 a b c abc abc abc + + ++ +≥ ++ Ch ng minh. Ta có ( ) ( ) 22 3332 2 22 222 2( ) 9( ) 330 () cyc cyc a b cab a b c abc abc a b c abc a b c ⎛ ⎞⎛⎞ -- ⎜ ⎟⎜⎟ ⎜ ⎟⎜⎟ + + ++ ⎝ ⎠⎝⎠ +-=≥ ++ ++ ∑∑ ⇒ pcm. ng th c xy ra khi và ch khi abc == . Bài 10. (IMO 2005) ,, xyz là các s th c dng th a 1 xyz ≥ . Ch ng minh 5 2 5 2 52 5 2 2 5 22 52 2 0 xxyyzz x y z yz x zx y --- + +≥ + + + + ++ Ch ng minh. Không mt tính tng quát ta ch cn xét tr ng h p 1 xyz = là . Khi ó, ta có ( ) 52 52 4 2 42 22 5 22 4 22 4 222 5 22 . 2 () ( ) 2 () x x x x xyz x x yz x x y z x yz x yz yz x yz x y z xyz - - - -+ = =≥ + + ++ ++ ++ t 2 22 ,, ax b y cz = == . Khi ó, ta ch cn ch ng minh Tìm tài liệu Toán ? Chuyện nh ỏ - www.toanmath.com123 2 22 2 () 0 2() cyc a abc a bc -+ ≥ ++ ∑ 2 2 22 2 22 2 2 222 ()0 2 ( ) 2 () ().0 (2 ( ))(2 ( )) (ñuùng) cyc cyc ab ab abc bca c ac bc a b ab ab abc bca ⎛⎞ ⇔- -≥ ⎜⎟ + + ++ ⎝⎠ ++ ++ - ⇔-≥ + + ++ ∑ ∑ ⇒ pcm. ng th c xy ra khi và ch khi 1 abc x y z == ⇔ = == . Bài 11. (Moldova 2006) ,, abc là dài ba cnh ca mt tam giác. Ch ng minh rng 222 1 1 10 bca a bc c ab ⎛ ⎞ ⎛⎞ ⎛⎞ -+ -+ - ≥ ⎜ ⎟ ⎜⎟ ⎜⎟ ⎝ ⎠ ⎝⎠ ⎝⎠ Ch ng minh. + Cách 1. Ta có b t ng th c c n ch ng minh t ng ng v i 3 22 ( )0 1 ( )( ) 0() 2 ñuùng cyc cyc abbc c bca ab -≥ ⇔ +- -≥ ∑ ∑ ⇒ pcm. ng th c xy ra khi và ch khi abc == . + Cách 2. Ta có b t ng th c c n ch ng minh t ng ng v i Tìm tài liệu Toán ? Chuyện nh ỏ - www.toanmath.com124 2 2 2 2 2 2 2 1 2 () 2 ( )1 () 2 1 ()0 2 cyc cyc cyc cyc cyc cyc cyc ab a c ab bc ab a b c bca ab c c ab a ≥ ⎛⎞ ⇔ +- ≥- ⎜⎟ ⎝⎠ - ⇔ ≥- ⎛⎞ ⇔ - -≥ ⎜⎟ ⎝⎠ ∑∑ ∑∑ ∑∑ ∑ t 111 ,, 222 a bc a bc S SS bca =- = - =- t ng th c c n ch ng minh t ng ng v i 222 ( )( )( ) 0 a bc S b c Sc a Sa b - + -+ -≥ + Tr ng h p 1. . b c abc + > ≥≥ Th thì ta có ,0 ab SS > . Ta có 1 0( 0) do bc bc S S bc ca + =+- > ≥ > Do ó, áp d ng tiêu chu n 2, ta có pcm. + Tr ng h p 2. . abc a b ≤≤<+ Th thì ta có ,0 cb SS > . Ta có 2 () 1 1 20 ba b a b cb b c bc SS c b c b c b bc -- + = + - >+ - =+ - = ≥ Do ó, áp d ng tiêu chu n 2, ta có pcm. Tóm li, trong m i tr ng h p, ta luôn có 222 ( )( )( ) 0 a bc S b c Sc a Sa b - + -+ -≥ ( pcm) ng th c xy ra khi và ch khi abc == . Tìm tài liệu Toán ? Chuyện nh ỏ - www.toanmath.com125 Bài 12. ,, xyz là các s th c d ng. Ch ng minh rng 2 22 2 22 3 2 ( )( )( ) xyzy zx zxy yz zx xy + ++ + +≥ + ++ Ch ng minh. t ,, a yzb zxc xy =+=+=+ . Khi ó, ta có b t ng th c c n ch ng minh t ng ng v i 2 2 22 22 22 2 22 2 10 ( ) () 0 ( ) () 0 ( ) () 0 ()() 0 (ñuùng) cyc cyc cyc cyc cyc cyc cyc bc ca ab a a bc a cab aa bc a cab aa ca b ca b ba cab ab ab ⎛⎞ - -+ -≥ ⎜⎟ ⎝⎠ -- ⎛⎞ ⇔ -≥ ⎜⎟ ⎝⎠ -- ⇔ -≥ -- ⇔ -≥ -+ ⇔≥ ∑ ∑ ∑∑ ∑∑ ∑ ⇒ pcm. ng th c xy ra khi và ch khi abc == . Bài 13. (Gabriel Dospinescu) ,,0 abc > . Ch ng minh rng 2 2 22 2 2 2 222 2 222 ( )( )( )( ) 8 ( ) ab bc c a ab bc ca abc ab c + + + ++ ≥ ++ Ch ng minh. Ta có 2 2 22 2 2 2 222 2 222 622 2 2222222 ( )( )( )( ) 8 ( ) ( )( )2 ( ) ()0 (ñuùng) cyc cyc ab bc c a ab bc ca abc ab c a b c b bc c abcab bc c a ca b + + + ++ ≥ ++ ⎛⎞ ⇔ - + + + + + -≥ ⎜⎟ ⎜⎟ ⎝⎠ ∑∑ ⇒ pcm. ng th c xy ra khi và ch khi abc == . Tìm tài liệu Toán ? Chuyện nh ỏ - www.toanmath.com126 Bài 14. (Old And New Inequalities) ,,0 abc > . Ch ng minh rng 33 2 1 . () 4 cyc cyc ab ab ab - ≤- + ∑∑ Ch ng minh. t 2 2 ,, 2 b c x a x yz c a y b x y z xyz a b z c x yz + = =- ++ ⎧⎧ ⎪⎪ + = ⇔= - +⇒ ⎨⎨ ⎪⎪ + = =+- ⎩⎩ là dài ba cnh ca mt tam giác. Khi ó, b t ng th c c n ch ng minh t ng ng v i 3 2 () () cyc cyc xy xy z - ≤- ∑∑ Ta có 32 2 3 2 () ()( ) ()0 () ( ) (1) cyc cyc cyc cyc cyc xy xy yzx xy zz xy xy z - - +- -- =≥ - ⇒ -≥ ∑ ∑∑ ∑∑ Ta l i có 32 2 3 2 () ()( ) ()0 () ( ) (2) cyc cyc cyc cyc cyc xy xy z xy xy zz xy xy z - - +- -+ =≥ - ⇒ - ≥- ∑ ∑∑ ∑∑ (1) và (2) ta suy ra pcm. ng th c xy ra khi và ch khi abc == . Tìm tài liệu Toán ? Chuyện nh ỏ - www.toanmath.com127 Bài 15. (USA Team Selection Test 2004) ,,0 abc > . Ch ng minh rng ( ) ( ) ( ) { } 2 22 3 3 3max ,, abc abc ab bc ca ++- ≤ - -- Ch ng minh. t 6 66 , , ( , , 0) ax b y cz xyz ===> . Khi ó bt ng th c cn ch ng minh t ng ng v i 6 6 6 2 2 2 3 32 3 3 2 3 32 3 3max{( ),( )( ) } x y z xyz xy y z zx ++- ≤ - -- Do ó, ta ch cn ch ng minh 6 6 6 2 2 2 3 32 3 32 3 32 2 22 2 2222 3 ( )( )() ( )(2( ) ( )( )) 0 cyc x y z xyz x y yz zx xy x y xy x y z xy + + - ≤ - + - +- ⇔ - + +- + + +≥ ∑ t 22 2 2 22 2 4 4 22 3 3 2 2 22 2 22 2 2222 4 4 22 3 3 22 222 22 2 223 2 2 2 2 22 2( ) ( )( ) 4222 2( ) ( )() 4222 ( )( )32 2( )() x y z S yz yz x yz y z y z yz y z yz xy x z xyz S z x zx x y z zx z xzxxz xz xy yzxyz x z x y z x xz S x y xy xyz = ++ - +++ = + + + + - -- = ++ - + ++ =++ + + - - - =+ - ++ = + + - ++ 2 () xy + t ng th c c n ch ng minh t ng ng v i 222 ( ) ( ) ( )0 x yz S yz S zx S x y - + - + -≥ Không m t tính t ng quát gi s 0 x yz ≥ ≥> . Khi ó, rõ ràng ta có , 0. yz SS > Ta có 22232 ( ) 2()2( )() 0 xy S S x y z xy zxy xy + = -+ + + +- > Do ó, áp d ng tiêu chu n 2, ta suy ra pcm. ng th c xy ra khi và ch khi . x y z abc = =⇔ == Tìm tài liệu Toán ? Chuyện nh ỏ - www.toanmath.com128 Bài 16. (Ph m Kim Hùng) 0 abc ≥≥> . Ch ng minh rng 4 33 5 32 cyc a abc ab ++ ≥ + ∑ Ch ng minh. Ta có 4 33 4 33 4 33 2 22 33 5 32 25 5() 32 25 11 60 32 2( )(4 6) 0 32 cyc cyc cyc cyc a abc ab a abc ab a ab ab a b a abb ab ++ ≥ + ⇔ ≥ ++ + ⎛⎞ ⇔ - +≥ ⎜⎟ + ⎝⎠ - - ++ ⇔≥ + ∑ ∑ ∑ ∑ t 2 2 22 22 3 3 3 3 33 4 6 4 6 46 ,, 3 2 3 2 32 a bc b bc c c ca a a ab b SSS bc c a ab - + + - + + - ++ = == + ++ t ng th c c n ch ng minh t ng ng v i 222 2 ( )2 ( )2 ( ) 0 a bc S b c Sc a Sa b - + - + -≥ Rõ ràng ta có 0 b S ≥ . Ta s ch ng minh rng 22 2 0 (1) 2 0 (2) bc ba SS a S bS +≥ +≥ * Ch ng minh (1). Ta có bt ng th c (1) t ng ng v i 32 2 2 2 2 33 3 2 4 32 4 3 3 23 2(23 6)12( 2) 2( 4 ) 26 360 a a ab b c a ab b c a b a c b c a c ab c abc b c + + - + + -+ + +- ++ + + ≥ u này hi n nhiên úng do 0 abc ≥≥> . Vy (1) úng. * Ch ng minh (2). Ta có Tìm tài liệu Toán ? Chuyện nh ỏ - www.toanmath.com129 23322 23 322 (2) ( ) (3 2 )(6 4 ) 2 (3 2)(6 4) 0 fa a b c a acc b c a c bcb ⇔ = + + -+ + + + -≥ / 2 3 34 2 232 () 24 (3 2 2) (3 (7 8 ) 16 ( ) 53 ) 0 ( 0) do f a a ab ac b ac b ab ac bc c ab c ab c a b c = + -+ + + - + - + > ≥≥> ⇒ () fa ng bin. ⇒ 22 3 2 2 3 32 45 () () ( (2 7 3 12) 9 8 28) 0 f a f b b b b b c bc c b c bc c ≥ = ++- + + + ≥ ⇒ (2) úng. Tr l i bài toán c a ta. Ta có 222 222 2 2 2 ( )2 ( )2 ( ) ()( 2) ( 2 )( ) 0 ( .( ) 0) do a bc ba bc S b c Sc a Sa b b c a S bS a S S ab a c bc b b - + -+ -≥ -+ ≥ + - + ≥ - ≥ -≥ ⇒ pcm. Bài 17. (Ph m Kim Hùng) Cho caùc soá khoâng aâm , ,. abc Chöùng minh raèng 333 2 2 2 2 22 3 2 22 a b c abc ab bc c a ++ + +≥ + ++ Chöùng minh. Ta coù baát ñaúng thöùc caàn chöùng minh töông ñöông vôùi 2 22 2 2 2 2 22 2 22 .( ) .( ) .( ) 0 2 22 ba c b ac ab bc ca ab bc c a - -- - + - + -≥ + ++ Coù 2 tröôøng hôïp xaûy ra + Tröôøng hôïp 1. 0 abc ≥ ≥≥ . Khi ñoù, ta coù 2 2 22 2 2 22 2 2 22 4 0 22 22 0 22 4 22 0 22 bc a b ca aa a b ca b a ac a b ca -≥ ++ - +≥ ++ -- ⇒ +≥ ++ 22 2 2 22 422 .( ) .( ) 0 (1) 22 b a ac a b ca a b ca -- ⇒ - + -≥ ++ Tìm tài liệu Toán ? Chuyện nh ỏ - www.toanmath.com130 22 2 2 22 22 2 2 22 (4 2 ) (2 ) 0 22 422 .( ) .( ) 0 (2) 22 c b b a ca b c ca c b ac b c ca b c ca -- +≥ ++ -- ⇒ - + -≥ ++ Coäng caùc baát ñaúng thöùc (1) vaø (2) veá theo veá roâi chia caû hai veá cho 2, ta ñöôïc 2 22 2 2 2 2 22 2 22 .( ) .( ) .( ) 0 2 22 ba c b ac ab bc ca ab bc c a - -- - + - + -≥ + ++ + Tröôøng hôïp 2. 0. c ba ≥ ≥≥ + Tröôøng hôïp 2.1.2. b ca ≥+ Khi ñoù, ta seõ chöùng minh 2 2 22 2 4(2) 0 22 ba ac a b ca -- +≥ ++ Thaät vaäy, deã thaáy veá traùi laø haøm taêng cuûa c neân ta chæ caàn chöùng minh khi , cb = töùc laø chöùng minh 2 2 22 3 2 2 32 23 22 2 4(2 ) 0 22 4 2 2 16 8 8 40 3(5 2 2 )0 (ñuùng) b a ab a b ba bab ab a ab abb a a abb -- +≥ ++ ⇔ + - +-+ - ≥ ⇔ - +≥ Do ñoù 2 2 22 2 4(2) 0 22 ba ac a b ca -- +≥ ++ Vaäy 2 22 2 2 2 2 22 22 2 2 22 2 22 .( ) .( ) .( ) 2 22 22 .( ) .() 4(2 )2 0 ba c b ac ab bc c a a b b c ca b a ac ca ca a b ca - -- -+-+-≥ + ++ -- ≥ -+- ++ ≥ + Tröôøng hôïp 2.2.2. b ca ≤+ Khi ñoù, ta seõ chöùng minh 2 2 22 2 63 0 (3) 22 b a ac a b ca -- +≥ ++ Thaät vaäy, deã thaáy veá traùi laø haøm taêng cuûa c neân chæ caàn chöùng khi 2 c ba =- . Baát ñaúng thöùc (3) trôû thaønh 2 2 22 3 2 23 2 96 0 2 838 10 15 2 15 0 (ñuùng) b a ab a b b a ab b ba aba -- +≥ + +- ⇔- + +≥ Tieáp theo, ta seõ chöùng minh 2 2 22 2 32 . 0 (4) 2 22 cb ac b c ca -- +≥ ++ Tìm tài liệu Toán ? Chuyện nh ỏ - www.toanmath.com131 Thaät vaäy, vì veá traùi laø haøm giaûm theo a neân ta chæ caàn chöùng minh khiab = , baát ñaúng thöùc trôû thaønh 2 2 22 3 2 23 42 63 0 22 5 2 2 100 (ñuùng) c b bc b c cb c cb bc b -- +≥ ++ ⇔ +-+≥ Neáu 2 ca ≤ thì ta coù baát ñaúng thöùc caàn chöùng minh ñuùng. Neáu 2 ca ≥ thì töø 2 baát ñaúng thöùc treân, vôùi chuù yù raèng 2 22 3 ( ) 3( ) .( ), 2 ca ba cb - ≤ - +- ta coù 2 22 2 2 2 2 22 22 2 2 2 2 2 2 22 2 22 .( ) .( ) .( ) 2 22 2 3(2 ) 2 32 .( ) . .() 2 2 2 22 0 b a c b ac ab bc ca ab bc ca ba a c cb ac b a cb ab ca bc c a --- - + - + -≥ + ++ - - -- ⎛ ⎞⎛⎞ ≥ + -+ +- ⎜ ⎟⎜⎟ + + ++ ⎝ ⎠⎝⎠ ≥ Toùm laïi, ta luoân coù 2 22 2 2 2 2 22 2 22 .( ) .( ) .( ) 0 2 22 (ñpcm) ba c b ac a b b c ca ab bc c a - -- -+-+-≥ + ++ Ñaúng thöùc xaûy ra khi vaø chæ khi . abc == Bài 18. (Ph m V n Thu n) ,,0 abc > . Ch ng minh rng 2 22 8 2 ( )( )( ) a b c abc ab bc ca a b b c c a ++ +≥ + + + ++ Ch ng minh. Ta có b t ng th c c n ch ng minh t ng ng v i 2 2 1 ()0 2( ) ( )( )( ) ( ) (( )( ) ) 0 cyc cyc c ab ab bc ca a b b c c a a b a b c ab bc ca abc ⎛⎞ - -≥ ⎜⎟ + + + ++ ⎝⎠ ⇔ - +- + + -≥ ∑ ∑ t ( )() ( )() ( )() a b c S a b c ab bc ca abc S a b c ab bc ca abc S a b c ab bc ca abc =-++ + +- = -+ + +- = +- + +- t ng th c c n ch ng minh t ng ng v i 222 ( )( )( ) 0 a bc S b c Sc a Sa b - + -+ -≥ Tìm tài liệu Toán ? Chuyện nh ỏ - www.toanmath.com132 Không m t tính t ng quát gi s abc ≥≥ . Th thì ta có ,0 bc SS ≥ . Ta có 2 2 ( )0 ab S S c ab + = +≥ Do ó, áp d ng tiêu chu n 2, ta có ngay pcm. ng th c xy ra khi và ch khi abc == ho c ,0 a bc == và các hoán v. Bài 19. (Ph m V n Thu n) ,,0 abc > . Ch ng minh rng 2 33 3 222 2 22 ( )1 .2 2 a b c a b c abc abc ab bc ca abc ⎛⎞ + + + + ++ - -≤ ⎜⎟ ++ ++ ⎝⎠ Ch ng minh. Ta có b t ng th c c n ch ng minh t ng ng v i 2 2 22 41 ()0 cyc abc ab abc ab bc ca a bc ⎛⎞ ++ ⎛⎞ - + -≥ ⎜⎟ ⎜⎟ ⎜⎟ ++ ++ ⎝⎠ ⎝⎠ ∑ t ng th c này úng do 9 abc abc ab bc ca ++ ≥ ++ . ⇒ pcm. ng th c xy ra khi và ch khi abc == . Bài 20. (Ph m Kim Hùng) ,,0 abc > . Ch ng minh rng 222 222 2 52 52 5 21 4 ( ) ( ) () a bc b ca c ab b c c a ab +++ + +≥ + ++ Ch ng minh. Ta có b t ng th c c n ch ng minh t ng ng v i 2 2 2 22 2 2 57 0 4 () 8 776 0 () cyc cyc a bc bc abc bc bc ⎛⎞ + -≥ ⎜⎟ + ⎝⎠ - -+ ⇔≥ + ∑ ∑ Tìm tài liệu Toán ? Chuyện nh ỏ - www.toanmath.com133 ( ) 22 222 2 22 2 2 22 2 3 22 () 4 4 30 () () () 4( )( 2 ) 3 ()0 ( )( ) () (4( )( 2) 3( )( ))0 cyc cyc cyc cyc cyc a b c a bc bc b c bc ab ab c ab a c b c ab ab ab a b c a c bc - -- ⇔ --≥ + ++ ⎛⎞ + ++ ⇔ - -≥ ⎜⎟ + ++ ⎝⎠ ⇔ - + ++ - + +≥ ∑ ∑∑ ∑ ∑ t 3 22 3 22 3 22 4( )( 2)3( )() 4( )( 2 )3( )() 4( )( 2)3( )() a b c S bc bc a ab a c S ac a bc a b bc S a b ab c a c bc =+ ++- + + = + ++- ++ = + ++ - ++ t ng th c c n ch ng minh t ng ng v i 222 ( )( )( ) 0 a bc S b c Sc a Sa b - + -+ -≥ Không m t tính t ng quát gi s 0 abc ≥≥> . Khi ó, ta d dàng nh n th y ,0 bc SS ≥ . Ta có 3 22 3 22 222222 4( )( 2 )3( )() 4( )( 2)3( )() 8( )(( ) ( )) ( )( 4 2 2 2 ) 0 ba S S c a a bc a b bc bc bc a ab ac c a b a c b c a b a b ab ac bc c + = + + + - + ++ + + ++ - ++ = + + + + + - + + + +- ≥ Do ó, áp d ng tiêu chu n 2, ta có pcm. ng th c xy ra khi và ch khi ( , , ) ( , , ),( , ,0) ( 0). a b c t tt tt t => * Chú ý. 5 2 c ng là hng s t t nht c a bt ng th c 2 22 2 22 3( 1) 4 ( ) ( ) () a kbc b kca c kab k bc c a ab + + ++ + +≥ + ++ Tìm tài liệu Toán ? Chuyện nh ỏ - www.toanmath.com134 Bài 21. ,,0 abc > . Ch ng minh rng 2 2 2 222 3() abc a b c abc b c a abc ++ + + + ++≥ ++ Ch ng minh. Ta có b t ng th c c n ch ng minh t ng ng v i 2 222 2 3() 22 12 ()0 cyc cyc a abc b a abc b abc ab b abc ⎛⎞⎛⎞ ++ + - ≥ - -- ⎜⎟⎜⎟ ++ ⎝⎠⎝⎠ ⎛⎞ ⇔ - -≥ ⎜⎟ ++ ⎝⎠ ∑ ∑ t 1 2 1 2 12 ,, a bc S SS c abc a abc b abc = - = - =- ++ ++ ++ t ng th c c n ch ng minh t ng ng v i 222 ( )( )( ) 0 a bc S b c Sc a Sa b - + -+ -≥ Ta có 111 6 9 63 0 abc S SS a b c abc abc abc abc ++=++- ≥ - => + + ++ ++ ++ Ta l i có 2 3 2 3 2 ( )() () 3 () () 3 () () cyc ab bc ca cyc cyc cyc cyc aab c abc S S S S SS abcabc a abc ab a b abc a b c a abc ab a b abc a b c + - -+ + += ++ + -+ = ++ + -+ ≥ ++ ≥ ∑ ∑∑ ∑∑ 0( theo bñt Schur) ây, áp d ng tiêu chu n 5, ta có pcm. Tìm tài liệu Toán ? Chuyện nh ỏ - www.toanmath.com135 Bài 22. (Ph m Kim Hùng) Cho caùc soá döông , ,. abc Chöùng minh raèng 2 22 2 2 2 333 12( ) b c c abc aba cab abc ⎛ ⎞⎛ ⎞⎛ ⎞ ++ + + + + +≥ ⎜ ⎟⎜ ⎟⎜ ⎟ ++ ⎝ ⎠⎝ ⎠⎝ ⎠ . Ch ng minh. Ta coù 2 22 2 2 2 3 33 2 4 3 33 2 2 24 22 2 3 33 2 2 22 12( ) 12( ) 2 2 22 12( ) 2 4() cyc cyc cyc cyc cyc cyc cyc b c c abc a ba cab abc ab a abc a c abc b aba ac ab b a c b abc ab a abc abc ⎛ ⎞⎛ ⎞⎛ ⎞ ++ + + + + +≥ ⎜ ⎟⎜ ⎟⎜ ⎟ ++ ⎝ ⎠⎝ ⎠⎝ ⎠ ++ ⇔ + +≥ ++ ⎛ ⎞⎛⎞ ⇔ + - + + -+ ⎜ ⎟⎜⎟ ⎝ ⎠⎝⎠ ⎛⎞ ++ + - ≥ - ++ ⎜⎟ ⎜⎟ ++ ⎝⎠ ⇔ ∑ ∑∑ ∑∑ ∑∑ 2 2 2 4 2() ( ) 40 cyc b a ab bc abcc c ⎛⎞ + - + + -≥ ⎜⎟ ++ ⎝⎠ ∑ Ñaët 2 2 2 2 4 2() 4 4 2() 4 a b b a ab S abcc c c b bc S abca a + =+ +- ++ + =+ +- ++ 2 2 4 2() 4 c a c ca S abcb b + =+ +- ++ Baát ñaúng thöùc caàn chöùng minh töông ñöông vôùi 222 ( )( )( ) 0 a bc S b c Sc a Sa b - + -+ -≥ Coù 2 tröôøng hôïp xaûy ra + Tröôøng hôïp 1. 0. c ba ≥ ≥> Khi ñoù, ta coù 0. b S ≥ Ta coù 22 22 4( ) 2( ) 2( ) 80 ab b c ab a b bc SS abc c a ca + ++ + = + + + + -≥ ++ Vì 22 22 2 22 2, 4, 2 b c a cb c aa ca + ≥+ ≥≥ Tìm tài liệu Toán ? Chuyện nh ỏ - www.toanmath.com136 22 22 4( ) 2( ) 2( ) 80 cb a c b c ac bc SS abc b a ba + ++ + = + + + + -≥ ++ Vì 22 22 2 22 2, 4, 2 a c a bc b aa ba + ≥ + ≥≥ Do ñoù 2 2 2 22 ( ) ( ) ( )( )( ) ( )( )0 a b c ab cb S b c S c a S ab S S bc S S ab - + -+ -≥ + - ++ -≥ + Tröôøng hôïp 2. 0. abc ≥≥> Khi ñoù, ta coù 1, 1. ac SS ≥ ≥- Ta coù 22 22 2 8 4 2( ) 4() 2 120 ab b c b a ab bc SS abc c a ca + ++ + = + + + + -≥ ++ Vì 4 8 2 2 22 4, 4,4 a b a b ca abc c a ac + ≥ + ≥ +≥ ++ 22 22 22 22 4 16 4 2( ) 8( ) 4 20 4 8 4 2( ) 8( ) 16 () ab b c b a a b bc SS abc ca ca b c b a ab bc fb abc c a ca + ++ + =+ + + +- ++ + ++ ≥ + + + + -= ++ Deã daøng kieåm tra () fb laø haøm ñoàng bieán. Do ñoù 2 2 4 162 () () 9 2 32 91 c ca f b fc ac a ≥ = + + - ≥ -> + Khaû naêng 2.1. 2 2( ) 0 0. a c b bc ac bc ab +≤ ⇔ - ≥ - ≥ ∧ - ≥ -≥ Neáu 0 b S ≥ thì ta coù ngay ñpcm. Neáu 0 b S ≤ , thì 2 2 22 () ( ) ( ) ( 4 1)() 0 a b c ab S b c S c a S ab S S bc - + - + - ≥ +-- ≥ + Khaû naêng 2.2. 2. a cb +≥ . Khi ñoù, ta seõ chöùng minh 2 0. cb SS +≥ Thaät vaäy, ta coù 22 22 2 8 4 2( ) 4( ) 2 12 () cb a c b c a c bc S S gc abc ba ba + ++ + = + + + + -= ++ + Khaû naêng 2.2.1. 2 ab ≥ . Khi ñoù, do () gc laø haøm taêng neân 2 2 8 42 ( ) (0) 12 0 a b ba gcg a b ab b ≥ =+ ++ - ≥ + Vì 2 2 941 5, 4, 6, 3 a b ab aab b a b b a b ab b -- + ≥ +≥ +≥≥ ++ + Khaû naêng 2.2.2. 2 ab ≤ . Khi ñoù, do () gc laø haøm taêng neân 22 22 8 4 4 14 () (2 ) 02 33 (do ) a b ba gc g b a bab ab ba ≥ - = + + - - ≥ ≥≥ Vaäy Tìm tài liệu Toán ? Chuyện nh ỏ - www.toanmath.com137 2 2 2 22 () ( ) ( ) ( 2)() (2)( ) 0 a b c ab cb S b c S c a S ab S S bc S S ab - + -+ -≥ + - ++ -≥ Toùm laïi, trong moïi tröôøng hôïp, ta luoân coù 222 ( ) ( ) ( )0 (ñpcm) a bc S b c Sc a Sa b - + - + -≥ Ñaúng thöùc xaûy ra khi vaø chæ khi . abc == Bài 23. ,,0 abc > . Ch ng minh rng 33 2 29 4() 6 cyc ab abc a ab - ≤ ++ + ∑ Ch ng minh. Ta có b t ng th c c n ch ng minh t ng ng v i 33 2 3 23 2 22 2 22 2 2 2 29 40 6 45 0 6 ( )(5 ) 0 6 ( )() ( )0 6 ( )() 0 6 (ñuùng) cyc cyc cyc cyc cyc cyc ab a a ab b aba a ab ba a abb a ab b aba ba a ab b a ab a ab ⎛⎞ - -≤ ⎜⎟ + ⎝⎠ +- ⇔≥ + - ++ ⇔≥ + -- ⇔ + -≥ + -+ ⇔≥ + ∑ ∑ ∑ ∑∑ ∑ ⇒ pcm. ng th c xy ra khi và ch khi abc == . Bài 24. ,,0 abc > . Ch ng minh rng 3 3 22 2 22 4 5 3 10 5() 3 cyc a b a b ab a b c ab bc ca ab + -+ ≥ ++ - -- + ∑ Ch ng minh. Ta có b t ng th c c n ch ng minh t ng ng v i 332 2 22 4 5 3 10 ( 4 )0 3 cyc a b a b ab a b ab ab ⎛⎞ + -+ - + -≥ ⎜⎟ + ⎝⎠ ∑ Tìm tài liệu Toán ? Chuyện nh ỏ - www.toanmath.com138 3 3 2 2 22 3 3 22 2 4 5 3 10 (3 )( 4 ) 0 3 0 3 ( )() 0 3 (ñuùng) cyc cyc cyc a b a b ab a b a b ab ab a b a b ab ab b a ab ab + - + - + +- ⇔≥ + + -- ⇔≥ + -+ ⇔≥ + ∑ ∑ ∑ ⇒ pcm. ng th c xy ra khi và ch khi abc == . Bài 25. ,,0 abc > . Ch ng minh rng 33 2 22 37 3() 23 cyc ab a b c ab bc ca ab + ≥ + + - -- + ∑ Ch ng minh. Ta có b t ng th c c n ch ng minh t ng ng v i 33 22 3 3 22 3 3 22 37 ( 2 )0 23 37(2 3)( 2 ) 0 23 0 23 cyc cyc cyc ab a b ab ab a b a b a b ab ab abab ab ab ⎛⎞ + - + -≥ ⎜⎟ + ⎝⎠ +-+ + - ⇔≥ + + -- ⇔≥ + ∑ ∑ ∑ 2 ( )() 0 23 (ñuùng) cyc b a ab ab -+ ⇔≥ + ∑ ⇒ pcm. ng th c xy ra khi và ch khi abc == . Tìm tài liệu Toán ? Chuyện nh ỏ - www.toanmath.com139 Bài 26. ,,0 abc > . Ch ng minh rng 4 33 2 cyc a abc ab ≥ ++ + ∑ Ch ng minh. Ta có b t ng th c c n ch ng minh t ng ng v i 4 33 4 33 222 33 4 2() 4 5 30 ()(3) 0 cyc cyc cyc a abc ab a ab ab ab b aba ab ≥ ++ + ⎛⎞ ⇔ - +≥ ⎜⎟ + ⎝⎠ - +- ⇔≥ + ∑ ∑ ∑ t 22 33 22 33 22 33 3 3 3 a b c c bcb S bc a acc S ca b aba S ab +- = + +- = + +- = + t ng th c c n ch ng minh t ng ng v i 222 ( )( )( ) 0 a bc S b c Sc a Sa b - + -+ -≥ + Tr ng h p 1.abc ≤≤ . Khi ó, d th y ,0 ca SS ≥ . Ngoài ra, ta c ng d dàng ch ng minh c 20, 20 cb ab S S SS + ≥ +≥ . Do ó áp d ng tiêu chu n 3, ta suy ra pcm. + Tr ng h p 2.abc ≥≥ . Khi ó, d th y 0 b S ≥ . Ngoài ra, ta cng d dàng ch ng minh c 22 20 20 bc ba SS aS bS +≥ +≥ Do ó Tìm tài liệu Toán ? Chuyện nh ỏ - www.toanmath.com140 ( ) 222 2 2 22 2 2 ( )2 ( )2 ( ) () 2 ( ) .( 2) 0 ( .( ) 0) do a bc b c ba S b c Sc a Sa b bc S S a b a S bS b a a c bc b - + - + -≥ - ≥+ -++ ≥ - ≥ -≥ ⇒ pcm. ng th c xy ra khi và ch khi abc == . Bài 27. ,,0 xyz > . Ch ng minh rng 22 0 cyc xz yz - ≥ + ∑ Ch ng minh. Ta có b t ng th c c n ch ng minh t ng ng v i ( ) ( ) 22 22 22 2222 222 22 2 22 44 44 2() 2() 44 3 4 2 () 4 2 2 () () 4 22 cyc cyc cyc cyc cyc cyc cyc cyc cyc cyc cyc xz yz yz xz x yz x yz yz yz xz y z yz y z yz x y z yz yz yz yz x y z yz yz yz yz yz x yz y ≥ ++ ⇔ - + + ≥ - ++ ++ ⎛ ⎞⎛⎞ ⇔ - + ≥ +- ⎜ ⎟⎜⎟ ++ ⎝ ⎠⎝⎠ ---- ⇔≥ ++ -- -- ⇔ +≥ + ++ -- ⇔ + ∑∑ ∑∑ ∑∑ ∑∑ ∑ ∑∑ 2 22 2 2 22 2 2 22 0 2 0 0 0 cyc cyc cyc cyc cyc cyc z x yz yz x y zx y z yz xy xy y z xz ≥ -- ⇔≥ + -- ⇔ -≥ ++ -- ⇔ -≥ ++ ∑ ∑ ∑∑ ∑∑ Tìm tài liệu Toán ? Chuyện nh ỏ - www.toanmath.com141 2 ( )() 0 ( )() cyc x y xy y z xz -+ ⇔≥ ++ ∑ ây là u hin nhiên úng. Vy bt ng th c c ch ng minh hoàn toàn. ng th c xy ra khi và ch khi . xyz == . Bài 28. ,,0 abc > . Ch ng minh rng 22 2 22 4() 3 ()(2) cyc abc b c abc + ≤ + ++ ∑ Ch ng minh. Ta có b t ng th c c n ch ng minh t ng ng v i 22 22 2 22 2 22 2 22 2 22 2 22 2 22 2 22 2 2 2 2 22 2 4( ) 2( ) 2( ) 3 ( )(2 ) ( )(2 ) ( )(2 ) 2( ) ( ) () ( )(2 )2 1 12 () 2 2 ( )( 2) cyc cyc cyc cyc cyc cy ab c ab c ab c b c abc b c abc b c abc ab c a b ac b c abc abc c ab a b c a b caba bc + ++ - ≤- + ++ + ++ + ++ - - +- ⇔≤ + ++ ++ ⎛⎞ ⇔- +- ⎜⎟ + + + + + ++ ⎝⎠ ∑ ∑∑ ∑∑ 0 c ≥ ∑ t 2 22 2 2 2 2 22 2 2 2 2 2 2 2 22 2 22 2 22 2 2 2 1 12 2 2()(2) 1 12 2 2()( 2 ) 1 12 2 2 ( )( 2) a b c a S a bc ab c b c a bc b S abc ab c c aa bc c S a bc a b c a bab c =+- + + + + + ++ = +- ++ ++ + + + = +- ++ + + + ++ t ng th c c n ch ng minh t ng ng v i 222 ( )( )( ) 0 a bc S b c Sc a Sa b - + -+ -≥ Không m t tính t ng quát gi s abc ≥≥ . Ta có 2 2 2 2 2 2 2 22 1 12 2 2()( 2 ) b b S abc ab c c aa bc = +- ++ ++ + + + Tìm tài liệu Toán ? Chuyện nh ỏ - www.toanmath.com142 2 22 2 2 2 2 22 2 22 2 2 2 2 2 2 2 22 2 22 2 22 2 22 2 2 2 2 22 2 2 2 2 22 1 1 21 . 2 22 1 1 11 .. 2 2 22 ()( ) 11 . (2 )(2 ) 22 11 . 22 1 b a a bc ab c a bc bb aa a bc a bc ab c a bc a bab cab b a a a bc a b c ab c a bc b a a b c a bc a ≥ +- + + + + ++ ⎛⎞⎛⎞ = - +- ⎜⎟⎜⎟ + + ++ + + ++ ⎝⎠⎝⎠ - + +-⎛⎞ = +- ⎜⎟ + + ++ + + ++ ⎝⎠ ≥- + + ++ ≥ 2 2 2 2 22 1 22 0 b c a bc - + + ++ ≥ 2 22 2 22 2 2 2 22 2 2 2 2 22 2 2 2 2 2 22 2 22 2 22 2 22 1 12 2 2 ( )( 2) 111 2 22 41 332 2 6 ( 2)(332) 0 c c S a bc a b c a bab c ab c a bc ab c a b c a bc a bc a b c a bc = +- ++ + + + ++ ≥ +- ++ + + ++ ≥- + + ++ ++ = + + ++ > Do ó ,0 bc SS ≥ . Ta l i có 2 2 2 2 22 2 2 2 2 2 22 2 2 22 2 2 2 2 2 22 2 2 22 2 2 2 2 2 22 222 2 2 2 1 12 (2 )( 2)()(2) 1 12 (2 ) ( 2) ( )( 2) 1 12 (2 )( 2)(2) 11 (2 )( ab SS a b a a bc a ab c ab c abc bab c b ab c bc aa bc aab caa b caba b c b a b c ba += + -+ + + + + + ++ ++- + + + + + ++ ≥+-+ + + ++ ++ ++ ++ 2 2 2 22 2 2 22 2 2 2 2 2 22 2 2) ( 2) 112 ( 2) ( 2) ( 2) bcaba b c aab c ba bcabab c - + + ++ ⎛⎞ = + -+ ⎜⎟ ++ + + ++ ⎝⎠ Tìm tài liệu Toán ? Chuyện nh ỏ - www.toanmath.com143 2 2 2 2 2 222 222 112 0 ( 2)(2)(2) a ab c babc ab abc ⎛⎞ + + -≥ ⎜⎟ + + ++ ++ ⎝⎠ ⇒ 22 0. ba a S bS +≥ Do ó, áp d ng tiêu chu n 4, ta có ngay pcm. ng th c xy ra khi và ch khi abc == . Bài 29. ,,0 abc > và 1 ab bc ca + += . Ch ng minh r ng 222 335 . 44 a b c abc +++≥ Ch ng minh. t ng th c c n ch ng minh t ng ng v i 2222 3 22 4( )()33()5() 4 5 6( ) 3 3( )0 sym cyc a b c ab bc ca abc ab bc ca ab bc ca a b a b a b c abc abc ab bc ca ++ ++ + ++ ≥ ++ ⇔ - - ++ + + +≥ ∑∑ Áp dng bt ng th c AM-GM, ta có 3 22 5 .5 2 sym cyc a b ab ≥ ∑∑ Do ó, ta ch cn ch ng minh ( ) 3 3 3 2 2 2 3 . 3 3( )6() 2 2 3( )4() 2 3( ) 2() . () ( )() 3() () 3() sym sym sym cyc cyc a b abc ab bc ca abc a b c a b abc ab bc ca abc a b c abc a b c ab bc ca a b abc a b c abc a b ab ac b c a b c ab bc ca abc b c ab ac a b c ab bc ca + + + ≥ ++ ⇔ + + + ≥ ++ ⇔ ++ - + + ≤ - ++ - ⇔ ≤ +- +++ + + ⎛ ⇔ - +- + + + ++ ∑ ∑ ∑ ∑ ∑ 0 cyc ⎞ ≥ ⎜⎟ ⎜⎟ ⎝⎠ ∑ u này rõ ràng úng vì Tìm tài liệu Toán ? Chuyện nh ỏ - www.toanmath.com144 min{ , , } 3() abc ab bc ca abc abbcca < + + + ++ ⇒ pcm. ng th c xy ra khi và ch khi abc == . Bài 30. (Vasile Cirtoaje) ,,0 abc > và 3 abc + += . Ch ng minh r ng 3 2 cyc a a bc ≥ + ∑ Ch ng minh. Ta có b t ng th c c n ch ng minh t ng ng v i 22 22 2 2 2 22222 222222 2 22 2 ( )()3()( )() 33 1 3 . ( )3 2 3 9 . () 9 () 2 3 () 2 cyc cyc cyc cyc cyc cyc cyc cyc a b ca c ab a bc b ca c ab abc a b a b c abc a abc c a b a b c abc a a abc c a b a b c abc a b ca a b abc ++ ≥ +++ ⎛⎞ ⇔ + ≥+ ⎜⎟ ⎜⎟ ⎝⎠ ⎛⎞ ⇔+ -≥ +- ⎜⎟ ⎜⎟ ⎝⎠ ⎛⎞ ⇔+ -≥ +- ⎜⎟ ⎜⎟ ⎝⎠ ⇔ - +- ∑ ∑∑ ∑ ∑∑ ∑∑ 0 2 cyc bc ⎛⎞ ≥ ⎜⎟ ⎝⎠ ∑ t 222 2 22 333 ,, 2 2 2 2 22 a bc a abc b abc c abc S a bc S ab c S abc = + - = + - = +- t ng th c c n ch ng minh t ng ng v i 222 ( )( )( ) 0 a bc S b c Sc a Sa b - + -+ -≥ Không m t tính t ng quát gi s abc ≥≥ . Khi ó, d th y 0 a S > . Ta có 2 22 3 .(3 ) 0 2 22 3() 3 (3 )0 2 b bc b abc b S b ac bc S S abc bc abc bc a >-= - > + + > - ≥ - = -> Tìm tài liệu Toán ? Chuyện nh ỏ - www.toanmath.com145 Do ó áp d ng tiêu chu n 2, ta có pcm. ng th c xy ra khi và ch khi abc == . Bài 31. (Nguy n Anh C ng) ,,0 abc > . Ch ng minh rng 2 22 2 6 2 cyc a b c ab bc ca a bc + + + ++ ≥ + ∑ Ch ng minh. Ta có b t ng th c c n ch ng minh t ng ng v i 2 22 2 2 22 2 2 22 2 2() 40 2 6 22 2 22 0 2 (3 2 )( ) (3 2 )( ) 0 2 (3 2 )( ) (3 2 )( ) 0 22 (2 3 )( ) (3 2 cyc cyc cyc cyc cyc cyc a b c ab bc ca a bc a b c ab bc ca a bc ab c c a a bc ab a bc ab c c a a bc ab abc abc abcaba b ca ⎛⎞ + + + ++ -≥ ⎜⎟ + ⎝⎠ - ++ + -+ ⇔≥ + -+ - - + -- ⇔≥ + -+ - + -- ⇔-≥ ++ + -- ⇔- + ∑ ∑ ∑ ∑∑ ∑ 2 22222 2 )() 0 2 ( ) (4 4 6 5 5 )(2 ) 0 cyc cyc bc ab a bc a b a b c ab bc ca c ab + -- ≥ + ⇔ - + + + - - +≥ ∑ ∑ t 2222 2222 2222 ( 4 4 6 5 5)(2) (4 4 6 5 5 )(2 ) (4 4 6 5 5 )(2 ) a b c S a b c bc ab ca a bc S a b c ca ab bc b ca S a b c ab ac bc c ab =+++--+ = + + +-- + = +++--+ t ng th c c n ch ng minh t ng ng v i 222 ( )( )( ) 0 a bc S b c Sc a Sa b - + -+ -≥ Không m t tính t ng quát gi s abc ≥≥ . Khi ó, d th y ,0 bc SS ≥ . Ta có 2 2 222 2 2 22222 (4 4 6 5 5)(2) ( 4 4 6 5 5 )(2 )0 ba a S b S a a b c ca ab bc b ca b a b c bc ab ca a bc + = + + +-- + + + + + + - - +≥ Tìm tài liệu Toán ? Chuyện nh ỏ - www.toanmath.com146 Vì 2 2 22 (2 ) (2 ) 0 a b cababc +≥ +> và 2 22 2 22 (4 4 6 55)( 4 4 6 55)0 a b c ca ab bc a b c bc ab ca + + +-- + + + +-- ≥ Do ó, áp d ng tiêu chu n 4, ta có ngay pcm. ng th c xy ra khi và ch khi abc == ho c ,0 a bc == và các hoán v. Bài 32. ,,0 abc > th a 1 ab bc ca + += . Ch ng minh r ng 22 33 8 2 cyc a bc ≥ ++ ∑ Ch ng minh. Áp dng b t ng Bunhiacopxki, ta có 2 22 22 222 () () 3( )3 2 2 2() cyc cyc sym a a abc abc a b c abc b c a ab ac a b c a b ++ ++ = ≥= + +- + + + + + ++ ∑∑ ∑ Do ó, ta ch cn ch ng minh ( ) 2 2 2 2 ( ) 33 3( )38 8()93 93() 8( ) 9 3 9 3( )( ) 8() 3(( )( ) 9 ) 8() . () 3() cyc abc a b c abc a bc abc abc a b c ab bc ca abc a b c ab bc ca a b c ab bc ca a b c ab bc ca abc ab bcca abc a b c ab bc ca ab a b c ab bc ca ++ ≥ + +- ⇔ ++ + ≥ ++ ⇔ + + ++ + ≥ + + ++ ⇔ + + ++ + + - ++ ≥ ≥ ++ + +- ⎛ + + ++ ⇔- + + + ++ ⎝ ∑ 2 2 3 () 8() ( ) 30 3() cyc cyc cab a b c ab bc ca abc ab c ab bcca ⎞⎛⎞ ≥- ⎜ ⎟⎜⎟ ⎜ ⎟⎜⎟ ⎠⎝⎠ ⎛⎞ + + ++ ⇔- -≥ ⎜⎟ ⎜⎟ + + + ++ ⎝⎠ ∑ ∑ t 8() 3 3() a a b c ab bc ca Sa a b c ab bc ca + + ++ =- + + + ++ Tìm tài liệu Toán ? Chuyện nh ỏ - www.toanmath.com147 8() 3 3() 8() 3 3() b c a b c ab bc ca Sb abc ab bcca a b c ab bc ca Sc abc ab bcca + + ++ =- + + + ++ + + ++ =- + + + ++ t ng th c c n ch ng minh t ng ng v i 222 ( )( )( ) 0 a bc S b c Sc a Sa b - + -+ -≥ Không m t tính t ng quát gi s abc ≥≥ . Khi ó, d th y ,0 bc SS ≥ . Ta ch ng minh ( ) 22 2 2 2 2 33 2 2 22 2 2 33 2 2 22 0 4() 3 ()3() 4() ( ) 3( )() ba aS bS a b b c ab ca a b ab bc ca a b c a b ab a b ab ab bc ca b c ca a b ab bc ca ab ab ab bcca abc abab +≥ ⇔ + + + ++ ++ ≥ ≥ + + + + + ++ ⇔ + + + + ++ + + ++≥ +++ Ta d dàng ch ng minh c 3 3 2 2 22 4( ) ( ) 3( )( ) (1) a b ab a b ab ab a b a b ab + + + > ++ Và 2 22 4 3 ( ) (2) a c ab bc ca c a b ab ++>+ (1) và (2), ta suy ra 22 0 ba aS bS +≥ . ây, áp d ng tiêu chu n 4, ta có ngay pcm. ng th c xy ra khi và ch khi abc == . Bài 33. ,,0 xyz > th a 1 xyz = . Ch ng minh r ng 3 cyc yz xyz x + ≥ + ++ ∑ Ch ng minh. t 2 22 , , ( , , 0) x a y b z c abc = ==> thì 1 abc = và b t ng th c c n ch ng minh ng ng v i Tìm tài liệu Toán ? Chuyện nh ỏ - www.toanmath.com148 22 22 3 22 23 cyc cyc bc abc a bc a b c abc a + ≥ + ++ + ⇔ - --≥ - - - ∑ ∑ dng bt ng th c AM-GM, ta có03 abc ≥ - -- . Do ó, ta ch cn ch ng minh 22 2 22 2 2 22 2 2 22 2 2 2 20 2 0 0 0 ( )() 0 (ñuùng) cyc cyc cyc cyc cyc cyc cyc bc abc a b ca a c a ab aa ab ab ba a b ab ab + - --≥ +- ⇔≥ -- ⇔-≥ -- ⇔ -≥ -+ ⇔≥ ∑ ∑ ∑∑ ∑∑ ∑ ⇒ pcm. ng th c xy ra khi và ch khi 1 1. abc x y z = = = ⇔ = == Bài 34.(Vasile Cirtoaje) ,,0 abc > . Ch ng minh rng 2 22 2 cyc a bc b bcc + ≥ ++ ∑ Ch ng minh. * B .u , , , ,, ab cx yz là sáu s th c không âm th a mãn u ki nabc ≥≥ và xyz ≥≥ (ho cxyz ≤≤ ) thì ( )( ) ()( ) ( )()0 xa b ac yb c ba zca c b - - + - -+ - -≥ Ch ng minh. + Tr ng h p 1. 0 x yz ≥ ≥≥ . Ta có Tìm tài liệu Toán ? Chuyện nh ỏ - www.toanmath.com149 0 ( ) ( )0 (do ) a cb c abc xa c yb c - ≥ - ≥ ≥≥ ⇒ -≥ -≥ Mà 0 ab -≥ nên ( )( ) ( )( )0 ( )() ( )()0 xac ab yb c ab xac ab ybc ba - -≥ - -≥ ⇔ - - + - -≥ t khác, doabc ≥≥ và 0 z ≥ nên ( )( )0 zc a cb - -≥ Do ó ( )( ) ()( ) ( )()0 xa b ac yb c ba zca c b - - + - -+ - -≥ + Tr ng h p 2. 0 x yz ≤ ≤≤ . Ta có 0 ( ) ( )0 (do ) a c a b abc za c yab - ≥- ≥ ≥≥ ⇒ - ≥ -≥ Mà 0 bc -≥ nên ( )() ( )()0 ( )( ) ( )( ) 0 za c bc ya b bc zc a cb ybc ba - -≥ - -≥ ⇔ - - + - -≥ t khác, doabc ≥≥ và 0 x ≥ nên ( )( )0 xac ab - -≥ Do ó ( )( ) ()( ) ( )()0 xa b ac yb c ba zca c b - - + - -+ - -≥ c ch ng minh hoàn toàn. Tr l i bài toán c a ta. Không m t tính t ng quát gi s abc ≥≥ . Khi ó, ta có 2 22 22 2 2 22 22 2 0 1 11 0 b bc c a ac c a ab b b bc c a ac c a ab b < + + ≤+ + ≤ ++ ⇒ ≥ ≥> + + + + ++ Áp dng B trên v i 2 2 2 2 22 1 11 ,, x yz b bc c a ac c a ab b === + + + + ++ ta suy ra Tìm tài liệu Toán ? Chuyện nh ỏ - www.toanmath.com150 22 2 22 22 2 22 22 ( )() 0 cyc cyc cyc cyc cyc a b ac b bcc a ab ac bc b bcc b bcc a bc ab ac b bcc b bcc -- ≥ ++ +- ⇒≥ ++ ++ ++ ⇒≥ ++ ++ ∑ ∑∑ ∑∑ Do ó, ta ch cn ch ng minh 22 22 22 22 22 2 2 22 2 2 22 2 2 0 ( ) () 0 ( ) () 0 ( ) () 0 ( )() ( )( cyc cyc cyc cyc cyc cyc cyc ab ac b bcc abaca abc b bcc aba b cac a b bcc aba b cac a b bcc b bcc aba b aba b b bcc a acc abab abc b bc c a ac + ≥ ++ +⎛⎞ ⇔ -≥ ⎜⎟ ++ ++ ⎝⎠ - -- ⇔≥ ++ -- ⇔ -≥ ++ ++ -- ⇔ -≥ + + ++ - ++ ⇔ + + ++ ∑ ∑ ∑ ∑∑ ∑∑ 2 0 ) (ñuùng) cyc c ≥ ∑ ⇒ pcm. ng th c xy ra khi và ch khi abc == . Bài 35. ,,0 abc > . Ch ng minh rng 22 22 2 2 22 3() 3() cyc ab abc abc ab abc + ++ ++≤≤ + ++ ∑ Ch ng minh. Tr c tiên ta ch ng minh bt ng th c 22 2 22 3( ) (*) cyc ab abc ab + + +≤ + ∑ Ta có Tìm tài liệu Toán ? Chuyện nh ỏ - www.toanmath.com151 ( ) ( ) 22 2 22 22 222 2 2 222 2 222 2 3() () 3() 2 () () 2() 3() 1 12 () 2 3() 1 11 ( 2 theo bñt Bunh cyc cyc cyc cyc cyc cyc ab abc ab a b ab a b c abc ab ab ab ab a b c abc ab ab a b c abc ab ab abc + - + += + ⎛⎞ ++ = - - + + - -- ⎜⎟ + ⎝⎠ - - =- + + + + ++ ⎛⎞ ⎜⎟ = -- ⎜⎟ + + + + ++ ⎝⎠ ⎛⎞ ≥ -- ⎜⎟ + ++ ⎝⎠ ∑ ∑ ∑ ∑ ∑ ∑ ) 0 iacopxki ≥ ⇒ (*) úng. ng th c xy ra khi và ch khi abc == . Tip theo, ta ch ng minh bt ng th c 22 22 2 3() cyc ab abc ab abc + ++ ≤ + ++ ∑ + Cách 1. Ta có 22 22 2 22 2 22 22 2 22 22 2 3() ( ) 3() () () 0 ( ) () 0 cyc cyc cyc cyc cyc cyc ab abc ab abc ab a b c abc ab cab abc ab cab c ab cac a bcbc ab ab + ++ ≤ + ++ ⎛⎞ + ⇔+ + ≤ ++ ⎜⎟ ⎜⎟ + ⎝⎠ + ⇔ ≤ ++ + ⎛⎞ + ⇔ -≥ ⎜⎟ + ⎝⎠ -- ⇔ -≥ ++ ∑ ∑ ∑ ∑ ∑∑ Tìm tài liệu Toán ? Chuyện nh ỏ - www.toanmath.com152 2 () () 0 () 0() ( )() ñuùng cyc cyc cyc abab abab b c ca abab b c ca -- ⇔ -≥ ++ - ⇔≥ ++ ∑∑ ∑ ⇒ pcm. + Cách 2. Ta có 22 22 2 22 22 2 2 2 2 2 2 3() ( )3( )() 2 () () 2() 21 ()0 ( )() 0 cyc cyc cyc cyc cyc cyc ab abc ab abc a b ab a b c abc ab abc ab ab ab abc ab ab c ab ab abc ab + ++ ≤ + ++ ⎛⎞ + + + + - ++ ⇔ -≤ ⎜⎟ + ++ ⎝⎠ - - ⇔≤ + ++ ⎛⎞ ⇔ - -≥ ⎜⎟ +++ ⎝⎠ - +- ⇔≥ + ∑ ∑ ∑ ∑ ∑ ∑ t ,, abc bc a c ab ab c S SS bc c a ab + - + - +- === + ++ t ng th c c n ch ng minh t ng ng v i 222 ( )( )( ) 0 a bc S b c Sc a Sa b - + -+ -≥ Không m t tính t ng quát gi s abc ≥≥ . Khi ó, d th y ,0 bc SS ≥ . Ta có 2 2 22 2 2 22 22 2 ( )0 ab a b ab a b ab bS aS a b a b ab acbc ab + = +- - > + - - =- ≥ ++ ây, áp d ng tiêu chu n 4, ta suy ra pcm. ng th c xy ra khi và ch khi abc == ho c ,0 a bc == và các hoán v. Tìm tài liệu Toán ? Chuyện nh ỏ - www.toanmath.com153 Bài 36. (Hojoo Lee) ,,0 abc > . Ch ng minh rng 2 22 1 22 cyc cyc a bc a bc a bc ≥≥ ++ ∑∑ Ch ng minh. * Ch ng minh 2 1 (*) 2 cyc bc a bc ≥ + ∑ * B .u , , , ,, ab cx yz là sáu s th c không âm th a mãn u ki nabc ≥≥ và xyz ≥≥ (ho cxyz ≤≤ ) thì ( )( ) ()( ) ( )()0 xa b ac yb c ba zca c b - - + - -+ - -≥ Ch ng minh. + Tr ng h p 1. 0 x yz ≥ ≥≥ . Ta có 0 ( ) ( )0 (do ) a cb c abc xa c yb c - ≥ - ≥ ≥≥ ⇒ -≥ -≥ Mà 0 ab -≥ nên ( )( ) ( )( )0 ( )() ( )()0 xac ab yb c ab xac ab ybc ba - -≥ - -≥ ⇔ - - + - -≥ t khác, doabc ≥≥ và 0 z ≥ nên ( )( )0 zc a cb - -≥ Do ó ( )( ) ()( ) ( )()0 xa b ac yb c ba zca c b - - + - -+ - -≥ + Tr ng h p 2. 0 x yz ≤ ≤≤ . Ta có 0 ( ) ( )0 (do ) a c a b abc za c yab - ≥- ≥ ≥≥ ⇒ - ≥ -≥ Mà 0 bc -≥ nên ( )() ( )()0 za c bc yab bc - -≥ - -≥ Tìm tài liệu Toán ? Chuyện nh ỏ - www.toanmath.com154 ( )( ) ( )( ) 0 zc a cb ybc ba ⇔ - - + - -≥ t khác, doabc ≥≥ và 0 x ≥ nên ( )( )0 xac ab - -≥ Do ó ( )( ) ()( ) ( )()0 xa b ac yb c ba zca c b - - + - -+ - -≥ c ch ng minh hoàn toàn. Tr l i bài toán c a ta. Ta có ( )( ) 2 2 3 (*)0 2 ( )() 0 2 ( )() .0 2 cyc cyc cyc bc bc ab bc ca a bc bca b ac ab bc ca a bc abc a b a c ab bc ca a abc ⎛⎞ ⇔ -≥ ⎜⎟ ++ + ⎝⎠ -- ⇔≥ +++ -- ⇔≥ ++ + ∑ ∑ ∑ Không m t tính t ng quát, gi s abc ≥≥ . Khi ó, ta có 3 33 3 33 2 2 20 1 11 0 2 22 a abc b abc c abc c abc b abc a abc + ≥+ ≥+> ⇒ ≥ ≥> + ++ Áp dng b trên v i 3 33 1 11 ,, 2 22 x yz a abc b abc c abc = == + ++ ta suy ra c 3 ( )() 0 2 cyc a b ac a abc -- ≥ + ∑ y (*) úng. * Ch ng minh 2 2 1 (**) 2 cyc a a bc ≥ + ∑ Ta có 2 2 (**) 0 2 cyc aa abc a bc ⎛⎞ ⇔ -≥ ⎜⎟ ++ + ⎝⎠ ∑ Tìm tài liệu Toán ? Chuyện nh ỏ - www.toanmath.com155 2 22 22 22 ( 2) 0 2 ( ) () 0 22 () () 0 22 ( ) (2 2 )( 2 )0 cyc cyc cyc cyc cyc cyc a ab ac bc a bc ca a b ab c a a bc a bc caab bcab a bc b ca a b c bc ca ab c ab +- ⇔≥ + -- ⇔-≥ ++ -- ⇔ -≥ ++ ⇔ - + - +≥ ∑ ∑∑ ∑∑ ∑ t 2 2 2 (2 2 )( 2) (2 2 )( 2) (2 2 )( 2) a b c S a ab ca bc a bc S b ab bc ca b ca S c bc ca ab c ab = + -+ = + -+ = + -+ t ng th c c n ch ng minh t ng ng v i 222 ( )( )( ) 0 a bc S b c Sc a Sa b - + -+ -≥ Không m t tính t ng quát gi s abc ≥≥ . Khi ó, d th y ,0 ab SS ≥ . th y 22 ( 2 ) ( 2 )0 b b ca c c ab + ≥ +≥ nên 2 2 ( 2)(2 2 2 2) ( 2)( 4) 0 bc vS S c c ab ab bc ca bc ca ab c c ab ab bc ca + ≥ + +- ++- = + ++ ≥ Do ó, áp d ng tiêu chun 2, ta suy ra ngay pcm. Bài 37. (Hojoo Lee) ,,0 abc > . Ch ng minh rng 2 cyc a bc abc bc + ≥ ++ + ∑ Ch ng minh. Ta có 2 cyc a bc abc bc + ≥ ++ + ∑ Tìm tài liệu Toán ? Chuyện nh ỏ - www.toanmath.com156 2 2 22 2 2 22 2 2 22 2 () 0 2 2 0 0 0 ( )() 0 ( )() (ñuùng) cyc cyc cyc cyc cyc cyc cyc a bc bc bc abc bc a b ca bc bc ab ab b c ac a b ab b c ac ⎛⎞ ++ ⇔ -≥ ⎜⎟ + ⎝⎠ -- ⇔≥ + -- ⇔ -≥ ++ -- ⇔ -≥ ++ -+ ⇔≥ ++ ∑ ∑ ∑∑ ∑∑ ∑ ng th c xy ra khi và ch khi abc == . Bài 38. (Gabriel Dospinescu) ,,0 xyz > . Ch ng minh rng 1 31 31 1 21 cyc sym x xy z xy +≥ + ++ + ++ ∑∑ Ch ng minh. t 1 11 ,, 3 33 a x b y cz = + = + =+ . Khi ó, b t ng th c c n ch ng minh t ng ng v i 1 31 32 1 1 11 0 3 22 (4 )((2 )(2 ) 3 ( )) 0 ( )(2 )(2 ) ( )( ) ( )() 0 (2 ) (2) ( )( ) ( )() (2)( cyc sym cyc cyc cyc cyc cyc a a bc ab a ab c ab ac abc ab ac aabc aabc ab a c ab a c ab ac aab aac ab ac ba bc a a b ba +≥ +++ ⎛⎞ ⇔ + - -≥ ⎜⎟ ++ ++ ⎝⎠ ++ + + - ++ ⇔≥ ++ ++ -- -- ⇔ -≥ ++ -- -- ⇔- + ∑∑ ∑ ∑ ∑∑ ∑ 2 0 2) 22 ().0 (2 )( 2) cyc cyc b bc ca ab ab abab a b ≥ + +- ⇔-≥ ++ ∑ ∑ Tìm tài liệu Toán ? Chuyện nh ỏ - www.toanmath.com157 t 2 2 2 2 22 ,, (2 )( 2 ) (2 )( 2 ) (2 )( 2 ) a bc ab ca bc ab bc ca bc ca ab S SS bc bc b c caa c a c aba b a b +- +- +- = == + + + + ++ t ng th c c n ch ng minh t ng ng v i 222 ( )( )( ) 0 a bc S b c Sc a Sa b - + -+ -≥ Không m t tính t ng quát gi s abc ≥≥ . Khi ó, d th y ,0 ab SS ≥ . Dobc ≥ nên (2 )(2) (2 )(2) 0 11 0 (2 )(2) (2 )(2) 2 2 22 (2 )(2) (2 )(2) 2 2 22 (2 )( 2) (2 )( 2) 4 (2 )( 2) 0 bc b a bab c a cac c a cac b a bab ab bc ca bc ca ab SS ca a cacab a bab ab bc ca bc ca ab ba ab a b abab a b ab bc ca ba a bab + + ≥ + +> ⇒ ≥> + + ++ + - +- ⇒+=+ + + ++ + - +- ≥+ ++ ++ ++ = ++ ≥ Do ó áp d ng tiêu chun 2, ta có ngay pcm. Bài 39. (Iran 1996) ,,0 abc > . Ch ng minh rng 2 22 1 1 19 () 4 ( ) ( ) () ab bc ca ab bc ca ⎛⎞ ++ + +≥ ⎜⎟ + ++ ⎝⎠ Ch ng minh. * Cách 1. Không m t tính t ng quát gi s 0 c ba ≥ ≥> . t 2 2 ,, 2 b c x a x yz c a y b x y z xyz a b z c x yz + = =- ++ ⎧⎧ ⎪⎪ + = ⇔= - +⇒ ⎨⎨ ⎪⎪ + = =+- ⎩⎩ là dài ba c nh ca mt tam giác. Do 0 c ba ≥ ≥> nên 0 x yz ≥ ≥> t ng th c cn ch ng minh tr thành Tìm tài liệu Toán ? Chuyện nh ỏ - www.toanmath.com158 2 22 2 22 2 22 2 22 2 2 111 9 (222 ) 4 4 44 111 (222 ) 9 21 ()0 cyc xy yz zx x y z xyz xy yz zx x y z x yz xy xy z ⎛⎞ + + --- + +≥ ⎜⎟ ⎝⎠ ⎛⎞ ⇔ + + - - - + +≥ ⎜⎟ ⎝⎠ ⎛⎞ - -≥ ⎜⎟ ⎝⎠ ∑ t 2 22 21 21 21 ,, x yz SSS yzzxxy xyz =- = - =- t ng th c c n ch ng minh t ng ng v i 222 ( ) ( ) ( )0 x yz S yz S zx S x y - + - + -≥ Do 0 x yz ≥ ≥> vày zx +> nên 0 x S ≥ và 0 y S ≥ Ta ch ng minh 22 33 0 yz y S zS y z xyz +≥ ⇔ +≥ Mày zx +> nên ta ch c n ch ng minh 33 2 () ()()0 (ñuùng) y z y z yz y z yz + ≥+ ⇔ - +≥ ây, áp d ng tiêu chu n 4, ta suy ra pcm. * Cách 2. . N u , , , ,, ab cx yz là 6 s th c không âm th aabc ≥≥ vàxyz ≤≤ thì 2 2 22 22 ( ) (3 ) ( ) (3 ) ( )(3 )0 x b c bc ca ab a y c a ca ab bc b z a b ab bc ca c - ++ - + - + +- + + - + + -≥ Ch ng minh B . Doabc ≥≥ nên 2 2 30 30 ca ab bc b ab bc ca c + +-≥ ++- ≥ Do ó + Nu 2 30 bc ca aba + + -≥ thì b hin nhiên úng. Tìm tài liệu Toán ? Chuyện nh ỏ - www.toanmath.com159 + Nu 2 30 bc ca aba + + -≤ thì 22 22 22 ( )(3 )0 ( )(3 ) ( )(3 ) b c bcca aba x b c bc ca ab a y b c bc ca ab a - + + -≤ ⇒ - ++ - ≥ - ++ - i cózy ≥ nên 2 2 22 ()(3)()(3) z a b ab bc ca c y a b ab bc ca c - ++ - ≥ - ++ - Do ó 22 22 2 ()(3)()(3) 4 () 0 cyc cyc cyc xbc bc ca aba y bc bc ca aba y abab ⎛⎞ - + +- ≥ - + +- ⎜⎟ ⎜⎟ ⎝⎠ ⎛⎞ =- ⎜⎟ ⎜⎟ ⎝⎠ ≥ ∑∑ ∑ c ch ng minh. Tr l i bài toán c a ta. Ta có 2 22 22 22 2 2 4() (*) 30 () (3 )( ) (3 )( ) 0 () () (3 )( ) (3 )( ) 0 () () (3 )( ) (3 )( ) 0 ( ) () 3 ( ). () cyc cyc cyc cyc cyc cyc ab bc ca ab a bc a a cb c a b ab ab ca ac bc ab ab ab ca ac ab a b ac ab bc ca c ab bc ⎛⎞ ++ ⇔ -≥ ⎜⎟ + ⎝⎠ ⎛⎞ +- +- ⇔ -≥ ⎜⎟ ++ ⎝⎠ + - +- ⇔ -≥ ++ + - +- ⇔ -≥ ++ ++- ⇔- + ∑ ∑ ∑∑ ∑∑ 22 222 0 () ( )( )(3 ) 0 cyc cyc ac a b a b ab bc ca c ≥ + ⇔ - + + + -≥ ∑ ∑ Không m t tính t ng quát gi s abc ≥≥ . Khi ó, ta có 2 22 ( )( )( )0 ab c a bc + ≥ + ≥ +> Áp dng B trên v i 2 22 ( ), ( ), ( ) z ab y c a x bc = + = + =+ Tìm tài liệu Toán ? Chuyện nh ỏ - www.toanmath.com160 ta suy ra c 222 ( )( )(3 ) 0 cyc ab a b abbc cac - + ++- ≥ ∑ ⇒ pcm. Bài 40. (Komal) ,,0 abc > th a 1 abc = . Ch ng minh r ng 2 2 2 2 22 111 3 1112 . a b c abc a b c abc ⎛⎞ + + - ≥ ++ ⎜⎟ ++ ++ ⎝⎠ Ch ng minh. Ta có 2 222 2 2 2 2 2 2 22 2 22 2 2 2 2 22 2 22 2 22 2 22 111 3 1112 . 32() 9 2( )6 () ()( 2) ( )() cyc c abc a b c abc a b c abc abbcca ab bc ca abc abc abc a b b c c a abc ab bc ca abc abc abc c a b c a b c bc ca ab abc a b c abc ⎛⎞ + + - ≥ ++ ⎜⎟ ++ ++ ⎝⎠ ++ ⇔ ++- ≥ ++ ++ ++ ⇔ ++- ≥ - ++ ++ ++ - - ++- ⇔≥ ++ + + ++ ∑ 2 22 ( )( 2 )0 yc cyc c a b a b ab bc ca ⇔-++ -- ≥ ∑ ∑ t 22 22 22 ( 2) ( 2) ( 2) a b c S a b c bc ca ab S b c a ca ab bc S c a b ab bc ca = + + -- = + + -- = ++ -- t ng th c c n ch ng minh t ng ng v i 222 ( )( )( ) 0 a bc S b c Sc a Sa b - + -+ -≥ Không m t tính t ng quát gi s abc ≥≥ . Khi ó, d th y ,0 bc SS ≥ . Ta có 2 2 2 22 22 (( ) ( ) 2 ( ) ( )) 0 ba aS bS aba b a b ca bab cab + = - + + + - + +> ây, áp d ng tiêu chu n 4, ta suy ra pcm. Tìm tài liệu Toán ? Chuyện nh ỏ - www.toanmath.com161 II. Bài t p ngh. i các bn gii các bài toán sau làm quen v i ph ng pháp trên và nu có th các bn hãy th gii các bài toán này bng ph ng pháp khác nhé! Bài 1. a) (Old and New Inequalities) ,,0 abc > và 1 abc ++= . Ch ng minh r ng 2 2 cyc ab bc + ≥ + ∑ b) (Võ Qu c Bá C n),,0 abc > và 1 abc ++= . Ch ng minh r ng 2 5 8 cyc ab bc + ≥ + ∑ c) (Võ Quc Bá C n) i nh ng u kin nh trên, hãy tìm hng s k l n nht cho bt ng th c 2 31 2 cyc a kbk bc ++ ≥ + ∑ Bài 2. (Võ Qu c Bá C n) a) ,,0 abc > . Ch ng minh r ng 2 2 22 2 2 22 25 2159( )()( ) 4 ( ) 4( )( )( ) cyc a bc ab bc ca bc ab bc c a + - -- ≥+ + + ++ ∑ b) V i nh ng u kin nh trên, hãy tìm hng s k l n nht cho bt ng th c 2 2 22 2 2 22 25 21 ( )()( ) 4 ( ) ( )( )() cyc a bc kab bc ca bc ab bc c a + - -- ≥+ + + ++ ∑ Bài 3. (Võ Qu c Bá C n) a) ,,0 abc > . Ch ng minh r ng 2 2 3() 2 cyc a bc abc bc + ++ ≥ + ∑ b) V i nh ng u kin nh trên, hãy tìm hng s k l n nht cho bt ng th c Tìm tài liệu Toán ? Chuyện nh ỏ - www.toanmath.com162 2 (1)() 2 cyc a kbc k a b c bc + + ++ ≥ + ∑ Bài 4. (Võ Qu c Bá C n) ,,0 abc > và 1 ab bc ca + += . Ch ng minh r ng 222 2 22 2( ) 32 abc abc b ca + + - + + ≥- Bài 5. (Võ Qu c Bá C n) ,,0 abc > . Tìm hng s k l n nh t sao cho b t ng th c sau úng 2 22 111() ()9 k ab bc ca abc k abc abc ++ ⎛⎞ + + + + + ≥+ ⎜⎟ ++ ⎝⎠ Bài 6. ( Võ Qu c Bá C n) ,,0 abc > . Ch ng minh rng 4 4 4 3 33 4( )3( )7() a b c abcabc ab bcca + + + ++ ≥ ++ Bài 7. (Mathlinks) ,,0 abc > . Ch ng minh rng 2 3 33 2 22 3() 2() cyc a abc bc abc ++ ≥ + ++ ∑ Bài 8. (Mathnfriends) ,,0 abc > . Ch ng minh rng 2 2 2 22 1 21 2( ) 5( ) cyc a ab b a b c ab bc ca ≥ + + + + + ++ ∑ Bài 9. (Olympic 30 - 4 - 2006) ,,0 abc > . Ch ng minh rng 22 ( )6 5 () cyc abc a bc + ≤ ++ ∑ Tìm tài liệu Toán ? Chuyện nh ỏ - www.toanmath.com163 Bài 10. (Mathlinks) ,,0 abc > . Ch ng minh rng 22 22 cyc cyc aa a b ab ≥ ++ ∑∑ Bài 11. (Stronger than Vietnam TST 2006 - Võ Qu c Bá C n) a) , , [1,2] xyz ∈ . Ch ng minh r ng 2 22 1 1 1 9( )( )( ) ( )6 ( )( )( ) xyz xyyzzx x yz x y z y z zx xy xyzxy yz zx ⎛⎞⎛⎞ - -- ++ ++≥ + ++ ⎜⎟⎜⎟ + + + + ++ ⎝⎠⎝⎠ b) V i các u kin nh trên, hãy tìm hng s k t t nht cho bt ng th c 2 22 1 1 1 ( )( )( ) ( )6 ( )( )() x y z kx y yz zx x yz x y z yz z x x y xyzx y yz zx ⎛⎞⎛⎞ - -- ++ ++≥ + ++ ⎜⎟⎜⎟ + + + + ++ ⎝⎠⎝⎠ Bài 12. (Mathlinks) ,,0 abc > . Ch ng minh rng 22 3 ()2 cyc cyc bca ab c bc + -≥ ++ ∑∑ Bài 13. (Diendantoanhoc) ,,0 abc > . Ch ng minh rng 2 22 19 () cyc a ab b abc ≥ + + ++ ∑ Bài 14. (Gabriel Dospinescu) ,,0 abc > . Ch ng minh rng 2 22 1 11 27 2 2 2 6( ) a bc abc bc ca ab a b c ⎛ ⎞⎛ ⎞⎛⎞ ⎛⎞ + + + + ≥ + + ++ ⎜ ⎟⎜ ⎟⎜⎟ ⎜⎟ ⎝⎠ ⎝ ⎠⎝ ⎠⎝⎠ Bài 15. (Belarus 1997) ,,0 abc > . Ch ng minh rng cyc cyc a ac b bc + ≥ + ∑∑ Tìm tài liệu Toán ? Chuyện nh ỏ - www.toanmath.com164 Bài 16. (Belarus 1998) ,,0 abc > . Ch ng minh rng 1 cyc a a b bc b b c ab ++ ≥ ++ ++ ∑ Bài 17. (Mathlinks) ,,0 abc > . Ch ng minh rng cyc cyc a bc b ac + ≥ + ∑∑ Bài 18. (Mathlinks) ,,0 abc > . Ch ng minh rng 4 cyc cyc bca a bc ⎛⎞ + ≥ ⎜⎟ ⎜⎟ + ⎝⎠ ∑∑ Bài 19. (Mildorf) ,,0 abc > . Ch ng minh rng 6 33 22 22 2 16 9 () cyc cyc cyc a ab ab ab + ≥+ ∑ ∑∑ Bài 20. (Vasile Cirtoaje) ,,0 abc > . Ch ng minh rng 2 22 22 7() 9 44 cyc abc b bcc ++ ≥ -+ ∑ Bài 21. (Mathlinks) ,,0 abc > và 1 ab bc ca + += . Ch ng minh r ng 22 2 15 2 () cyc ab ab + ≥ + ∑ Bài 22. (Diendantoanhoc) ,,0 abc > và 1 3 ab bc ca + += . Ch ng minh r ng 2 1 3 1 cyc a bc ≤ -+ ∑ Tìm tài liệu Toán ? Chuyện nh ỏ - www.toanmath.com165 Bài 23. (Japan 2004) ,,0 abc > và 1 abc ++= . Ch ng minh r ng 1 2 1 cyc cyc aa ba ⎛⎞ + ≥ ⎜⎟ ⎜⎟ - ⎝⎠ ∑∑ Bài 24. (Vasile Cirtoaje) ,,0 abc > , t (,,) ( )() cyc Eabc aab ac = -- ∑ . Ch ng minh r ng a) 2 ( )(,,) () cyc a b c E abc ab a b ++ ≥- ∑ b) 2 1 11 (,,) cyc cyc Eabcaab abc ⎛⎞ ++ ≥- ⎜⎟ ⎝⎠ ∑∑ Bài 25. (Vasile Cirtoaje) ,, 0,1 x y z xyz >= . Ch ng minh rng ( )( )( ) 7 5( ) x y yz z x x yz + + + + ≥ ++ Bài 26. (Vasile Cirtoaje) a),,0 xyz > . Ch ng minh r ng 4 3 22 3 () cyc cyc cyc x xy z xy ⎛⎞ - ≥- ⎜⎟ ⎜⎟ ⎝⎠ ∑ ∑∑ b) Ch ng minh r ng b t ng th c trên c ng úng cho ,, xyz ∈R . Bài 27. (Mathlinks) ,,0 abc > và 1 3 ab bc ca + += . Ch ng minh r ng 2 1 1 cyc a abc a bc ≥ ++ -+ ∑ Bài 28. (Võ Qu c Bá C n) a) ,,0 abc > . Ch ng minh r ng 2 22 29 2 cyc a bc bc + ≥ + ∑ b) V i các u kin nh trên, tìm hng s k t t nht cho bt ng th c Tìm tài liệu Toán ? Chuyện nh ỏ - www.toanmath.com166 2 22 3( 1) 2 cyc a kbc k bc ++ ≥ + ∑ c) i các u kin nh trên, tìm hng s k t t nht cho bt ng th c 2 2 22 22 2 2 22 2 2 2 9 ( )( )() 2( )( )() cyc a bc kab bc ca b c ab bc ca + - -- ≥+ + + ++ ∑ Bài 29. (Mathlinks) ,,0 abc > . Ch ng minh rng 2 12 3 cyc cyc a bc a bc ≥ + + ∑∑ Bài 30. ,,0 abc > . Ch ng minh rng 2 22 cyc cyc aa bc bc ≥ + + ∑∑ Bài 31. (VMO 2006B) ,, 0,1 a b c abc >= . Tìm k max sao cho 222 111 3(1)() k k abc abc + + + ≥ + ++ Bài 32. (Võ Qu c Bá C n) a) ,,0 abc > . Ch ng minh r ng 2 2 () 4 2 cyc bc a bc + ≥ + ∑ b) i các u kin nh trên, tìm hng s k t t nht cho bt ng th c 2 2 22 2 222 ( ) ( )( )() 4 2 ( 2 )( 2 )( 2) cyc bc kab bc c a a bc a bc b ca c ab + - -- ≥+ + + ++ ∑ Bài 33. a) (Mathlinks) ,,0 abc > . Ch ng minh r ng 2 1 (2 )(2 ) 3 cyc a a b ac ≤ ++ ∑ Tìm tài liệu Toán ? Chuyện nh ỏ - www.toanmath.com167 b) (Võ Qu c Bá C n) V i các u ki n nh trên, tìm h ng s k t t nh t cho t ng th c 2 2 22 1 ( )( )() (2 )(2 ) 3 (2 )(2 )(2 )(2 )(2 )(2 ) cyc a kab bc c a a b a c ab bc c a ac c b ba - -- ≤- + + + + + + ++ ∑ Bài 34. (Mathinks) ,,0 abc > và 37 p≥+ . Ch ng minh r ng 2 19 (1)() cyc p ab bc ca pa bc ≥ + ++ + ∑ Bài 35. (Mathlinks) ,,0 abc > và 2 p >- . Ch ng minh r ng 22 ( 1) 3( 1) 2 cyc ab p bc ca p p b pbc c +-++ ≥ + ++ ∑ Bài 36. (Stronger than Schur - Nguy n Anh C ng) ,,0 abc > . Ch ng minh rng 33 3 2 2 22 2 2 3 2( ) 2( ) 2( ) a b c abc ab a b bc b c ca c a +++ ≥ ++ +++ Bài 37. (JBMO 2002) ,,0 abc > . Ch ng minh rng 32 2 cyc cyc aa b b ≥ ∑∑ Bài 38. (Mathlinks) ,,0 abc > . Ch ng minh rng 2 22 3 5 3 cyc ab a bc ≤ ++ ∑ Bài 39. (Ph m Kim Hùng) a) ,,0 abc > . Ch ng minh r ng 4 33 2 cyc cyc cyc a a b ab ⎛⎞ +≥ ⎜⎟ ⎜⎟ ⎝⎠ ∑ ∑∑ b) Ch ng minh r ng b t ng th c trên c ng úng cho ,, abc ∈ Rr . Tìm tài liệu Toán ? Chuyện nh ỏ - www.toanmath.com168 Bài 40. (Diendantoanhoc) ,,0 abc > . Ch ng minh rng ( ) 4 33 2 21 cyc cyc cyc a a b ab ⎛ ⎞ ⎛⎞ + ≥+ ⎜ ⎟ ⎜⎟ ⎜ ⎟ ⎜⎟ ⎝ ⎠ ⎝⎠ ∑ ∑∑ Bài 41. (Võ Qu c Bá C n) a) ,,0 abc > . Ch ng minh r ng 2 22 23 5 cyc a bc b bcc + ≥ ++ ∑ b) V i các u kin nh trên, tìm hng s k t t nht cho bt ng th c 2 22 1 cyc a kbc k b bcc + ≥+ ++ ∑ c) i các u kin nh trên, tìm hng s k t t nht cho bt ng th c 2 2 22 2 2 2 22 222 2 3 ( )( )() 5 ( )( )( ) cyc a bc kab bc c a b bc c a ab b b bc c c ca a + - -- ≥ ++ + + + + ++ + ∑ Bài 42. (Vasile Cirtoaje) ,,0 abc > , p ∈ R . Ch ng minh r ng ( )( )( )( ) 0 cyc a pb a pc ab ac - - - -≥ ∑ Bài 43. (Mathlinks) ,,0 abc > . Ch ng minh rng 2 22 ( )() 3 () cyc b c a b c ab bc ca a abc a b c + + + ++ ≥+ ++ ∑ Bài 44. (Diendantoanhoc) ,,0 abc > . Ch ng minh rng 2 22 61 cyc b c ab bc ca a abc + ++ + ≥+ ++ ∑ Tìm tài liệu Toán ? Chuyện nh ỏ - www.toanmath.com169 Bài 45. (Mathlinks) ,,0 abc > . Ch ng minh rng 2 3 2 3 cyc ab bc ca ab bc ca a ++ ≥ +++ ∑ Bài 46. (Mathlinks) ,,0 abc > và 3 abc + += . Ch ng minh r ng 2 22 222 111 abc abc + + ≥ ++ Bài 47. (Mathlinks) ,,0 abc > . Ch ng minh rng 2 2 22 2 22 18 . 5 ( ) () cyc abc abc a bc abc + ++ ≤ + + ++ ∑ Bài 48. (Mathnfriend) ,,0 abc > . Ch ng minh rng 2 22 2 2 2 2 22 1 1 1 915( )( )() (). 44 ( ) ( ) () ( )( )() ab bc ca ab bc ca ab bc ca ab bc ca ⎛⎞ - -- + + + + ≥+ ⎜⎟ + + + + ++ ⎝⎠ Bài 49. (Mathnfriend) ,, abc là dài ba cnh ca mt tam giác. Ch ng minh rng 2 22 2 2 2 2 22 111 947( )()() (). 44 ( ) ( ) ( ) ( )( )( ) ab bc ca ab bc ca ab bc ca ab bc ca ⎛⎞ - -- + + + + ≥+ ⎜⎟ + + + + ++ ⎝⎠ Bài 50. (Vasile Cirtoaje). a) ,,0 abc > . Ch ng minh r ng 2 22 4 30 () cyc b ca abc +- +≥ + ∑ b) V i các u kin nh trên, tìm hng s k t t nht cho bt ng th c 2 22 3( 2) 0 ( )2 cyc bcka k abc +-- +≥ + ∑ Tìm tài liệu Toán ? Chuyện nh ỏ - www.toanmath.com170 Bài 51. (Toán H c Tu i Tr 1998) ,,0 abc > . Ch ng minh rng 2 22 2 22 11 2. 22 a b c a b c ab bc ca ab bc ca b c c a a b abc ++ ++ + ≥++ ≥- ++ + ++ ++ Bài 52. (Mathlinks) ,,0 abc > và 1 abc ++= . Ch ng minh r ng 2 2 2 22 1 8() 2 (1 )( 1 )(1 ) cyc abc a abc ++ ⎛⎞ -≥ ⎜⎟ --- ⎝⎠ ∑ Bài 53. (Mathlinks) ,,0 abc > . Ch ng minh rng 3 322 2 9 12 cyc cyc cyc cyc cyc a ab a aa ⎛⎞ ⎛ ⎞⎛⎞ + +≥ ⎜⎟ ⎜ ⎟⎜⎟ ⎜⎟ ⎜ ⎟⎜⎟ ⎝⎠ ⎝ ⎠⎝⎠ ∑ ∑ ∑ ∑∑ Bài 54. (Mathlinks) ,,0 abc > và 1 abc ++= . Ch ng minh r ng 222 1 1 10 abc ab bc ca abc +≥ ++ ++ Bài 55. (Mathnfriend) ,,0 abc > . Ch ng minh rng 3 222 4 2 cyc a abc abc ++ ≥ ++ ∑ Bài 56. (Mathlinks) ,,0 abc > . Ch ng minh rng 2 2 22 1 21 2 cyc ab bc ca a bc abc ≥+ ++ + ++ ∑ Bài 57. (Mathlinks) a) ,,0 abc > . Ch ng minh r ng 2 2 3() cyc a a bc ab bc ca + ≥ ++ ∑ Tìm tài liệu Toán ? Chuyện nh ỏ - www.toanmath.com171 b) ,,0 abc > . Ch ng minh r ng 2 32() cyc a a bc ab bc ca + ≥ ++ ∑ c) ,,0 abc > . Tìm hng s k t t nht cho bt ng th c 2 1() cyc a a kbc k ab bc ca + ≥ + ++ ∑ Bài 58. ,,0 abc > và 1 abc ++= . Ch ng minh r ng ( ) 5 .( ) ( ) ( )2 2 ab bc ca a b ab b c bc c a ca +++ + ++ ++≤ Bài 59. a) (Mathlinks) ,,0 abc > . Ch ng minh r ng 2 6 2 cyc bc abc a bc + ≥ ++ + ∑ b) (Võ Quc Bá Cn) i các u kin nh trên, tìm hng s k tt nht cho t ng th c 2 22 2 222 ( )( )() ()6 2 (2)(2 )(2) cyc bc kab bc ca abc a bc a bc b ca c ab ⎛⎞ + - -- + + ≥+ ⎜⎟ ⎜⎟ + +++ ⎝⎠ ∑ Bài 60. (Mathlinks) ,,0 abc > . Ch ng minh rng 3 1 () () cyc cyc bc abc a bc ≤ + + ∑∑ Bài 61. (Mathlinks) ,,0 abc > . Ch ng minh rng 2 2 2 22 4 3() cyc a abc bc ⎛⎞ ≥ ++ ⎜⎟ ⎜⎟ + ⎝⎠ ∑ Tìm tài liệu Toán ? Chuyện nh ỏ - www.toanmath.com172 Bài 62. (Japan 1997) ,,0 abc > . Ch ng minh rng 2 22 ( )3 5 () cyc bca a bc +- ≥ ++ ∑ Bài 63. (USA 2003) ,,0 abc > . Ch ng minh rng 2 22 (2) 8 2() cyc abc a bc ++ ≤ ++ ∑ Bài 64. (Poland 1992) ,, abc ∈ R . Ch ng minh rng 222222222222 ( )( )( )()()() a b c b c a c a b abc bc a c ab + - + - + - ≥ + - + - +- Bài 65. (Mathlinks) ,,0 abc > . Ch ng minh rng 222 1 1 13 2 11 11 11 ab bc ca a bc b ca c ab + +≥ ++ +++ Bài 66. a) (Mathinks) ,,0 abc > và 1 ab bc ca + += . Ch ng minh rng 2 2 2 2 22 2 22 1 1 15 2 ( )( )( ) a b b c ca a b b c ca + ++ + +≥ + ++ b) (Võ Qu c Bá C n) V i các u ki n nh trên, hãy tìm h ng s k t t nh t cho bt ng th c 22 22 22 2 22 2 2 2 222 1 1 1 5 ( )( )() 2 ( ) () ( ) ( )()( ) ab bc ca kab bc ca ab b c c a ab bc c a + + + - -- + + ≥+ + + + + ++ Bài 67. (Ph m Kim Hùng) ,,0 abc > . Ch ng minh rng 3 33 5 3 2() cyc a abc bc abc +≥ + ++ ∑ Tìm tài liệu Toán ? Chuyện nh ỏ - www.toanmath.com173 Bài 68. (Ph m Kim Hùng) ,,0 abc > . Ch ng minh rng 333 3 54 5 () a b c abc abc abc ++ +≥ ++ Bài 69. (Ph m Kim Hùng) ,,0 abc > . Ch ng minh rng 4 44 2 22 32 .() 3 a b c abc abc ab bc ca a b c ++ + ≥ ++ + + ++ Bài 70. (Ph m Kim Hùng) ,,0 abc > và 1 ab bc ca + += , 23 k≥+ . Ch ng minh r ng 2 22 111 12 1 bc ca ab k ka bc kb ca kc ab +++ + +≥ + +++ Bài 71. (Võ Qu c Bá C n) a) ,,0 abc > . Ch ng minh r ng 2 2 2 22 19 4 47() cyc b bc c abc ≥ - + ++ ∑ b) i các u kin nh trên, tìm hng s k t t nht cho bt ng th c 2 2 222 19 ( 2)( ) cyc b kbc c k abc ≥ + + + ++ ∑ Bài 72. (VMO 1991) 0 x yz ≥ ≥> . Ch ng minh rng 2 22 2 22 x y y z zx x yz z xy + + ≥ ++ Bài 73. (Mathinks) ,,0 abc > . Tìm hng s k t t nh t cho b t ng th c 2 22 ( )3 2 cyc a k ab bc ca k bc abc ++ + ≥+ + ++ ∑ Tìm tài liệu Toán ? Chuyện nh ỏ - www.toanmath.com174 Bài 74. ,,0 abc > . Ch ng minh rng ( ) 2 ()43() abc a b c a bc abcabc + + + ++ ≥ ++ Bài 75. (Ph m Kim Hùng) ,,0 abc > . Tìm hng s k t t nh t cho b t ng th c 333 2 ( )3 ( )( )( ) 8 3 () a b c k ab bc ca k ab bc c a abc + + ++ + ≥+ + ++ ++ Bài 76. (Vasile Cirtoaje) ,,,0 a b ck > . Ch ng minh rng 2 ( 3) 3( 1) () cyc ab k bc ca k k b c kbc +- +- ≥ -+ ∑ Bài 77. (Võ Qu c Bá C n) Ch ng minh r ng v i m i ,, 0,1 abck >≥ ta luôn có 2 ( )6 2 2 cyc abc k b kbcc + ≥ + ++ ∑ Bài 78. (Võ Qu c Bá C n) ,,0 abc > . Tìm hng s k t t nh t cho b t ng th c 2 (2 ) 12 1 cyc aabc k ka bc ++ ≥ + + ∑ Bài 79. (Toán H c Tu i Tr 2002) ,,0 abc > . Ch ng minh rng 3 222 ()()()( )27 abc ab c ab c bca abc ++ +- - + + - ≤ Bài 80. (Manlio Marangelli) ,,0 abc > . Ch ng minh rng 2222223 3( )( ) () ab bcca ab bc ca abcabc + + + + ≥ ++ Tìm tài liệu Toán ? Chuyện nh ỏ - www.toanmath.com175 Bài 81. (Võ Qu c Bá C n) ,,0 abc > . Ch ng minh rng 2 2 2 2 22 22 2 222 1 222 bc a c ab ab c a bc b ca c ab + - + - +- + +≥ + ++ Bài 82. (Toán H c Tu i Tr 2005) ,,0 abc > . Ch ng minh rng 222 1 11 bc c a ab abc a bc b ca c ab + ++ ++≥ ++ + ++ Bài 83. (Võ Qu c Bá C n) ,,0 abc > . Tìm hng s k t t nh t cho b t ng th c 3 33 ( )( )( ) 8 ( )( ) 93 abbcca kabck a b c ab bc ca abc + ++ + ≥+ + + ++ ++ Bài 84. (Võ Qu c Bá C n) ,,0 abc > . Tìm hng s k t t nh t cho b t ng th c 3 33 3 23 a b c kabck bc ca ab abc + + + ≥+ + ++ ++ Bài 85. (Mathlinks) ,,0 abc > . Ch ng minh rng 2 2 2 22 11 55 cyc a ab b abc ≥ - + ++ ∑ Tìm tài liệu Toán ? Chuyện nh ỏ - www.toanmath.com176 T TÌM TÒI NH V B T NG TH C Voõ Quoác Baù Caån Baát ñaúng thöùc laø moät trong nhöõng lónh vöïc hay, khoù vaø loâi cuoán nhaát cuûa toaùn hoïc. Baïn coù theå deã daøng kieåm chöùng ñöôïc ñieàu naøy qua caùc trang web toaùn hoïc, trong forum baát ñaúng thöùc cuûa caùc trang web naøy, luoân chieám soá löôïng baøi vieát nhieàu nhaát. Baøi vieát sau ñaây, toâi xin giôùi thieäu moät phöông phaùp hay, khaù hieäu quaû ñeå chöùng minh baát ñaúng thöùc ñoái xöùng ba bieán maø toâi tình côø tìm ñöôïc qua vieäc giaûi toaùn. Do trình ñoä coøn haïn heïp vaø ñaây chæ laø moät tìm toøi nhoû cuûa toâi neân khoù loøng traùnh khoûi nhöõng sai soùt, mong baïn ñoïc thoâng caûm. Phöông phaùp naøy raát ñôn giaûn nhöng khaù hieäu quaû vaø noù ñaõ giuùp toâi giaûi ñöôïc khaù nhieàu baøi toaùn khoù maø nhöõng phöông phaùp maïnh khaùc nhö S.O.S, doàn bieán… ñaønh baát löïc. Xin ñöôïc noùi sô qua veà cô sôû cuûa phöông phaùp naøy, noù ñöôïc xaây döïng hoaøn toaøn döïa treân 2 Boå ñeà raát cô baûn sau * Boå ñeà 1. (baát ñaúng thöùc Schur) ,,0 thì abc ∀≥ 3 4 9 pqp r - ≥ trong ñoù , ,. p a b c q ab bc ca r abc =++ = + += * Boå ñeà 2. ,, abc ∀∈ R thì toàn taïi caùc soá thöïc 0 011 , ,, x y xy sao cho 0 0 11 22 0 0 0 1 11 22 00 11 22 22 p a b c x y xy q ab bc ca x x y x x y xy r abc xy =++= + =+ = + +=+ =+ ≤=≤ Ngoaøi ra, neáu,,0 abc ≥ thì 011 ,,0 x xy ≥ . Trong ñoù 2 0 2 0 40 40 Neáu thì Neáu thì p qy p qy + ≥≤ + ≤≥ Tìm tài liệu Toán ? Chuyện nh ỏ - www.toanmath.com177 Caùc keát quaû treân chöùng minh töông ñoái ñôn giaûn, caùc baïn neân töï chöùng minh laáy, xem nhö laø baøi taäp. Ñeå hieåu roõ hôn tính hieäu quaû cuûa phöông phaùp naøy, caùc baïn haõy cuøng theo doõi caùc ví duï sau Ví duï 1. (Vasile Cirtoaje) Cho 2 22 ,,0. thoûa abc a b c abc > + + = ++ Chöùng minh raèng 2 2 2 2 22 ab bc ca a b b c c a + + ≥ ++ Lôøi giaûi. Ta coù baát ñaúng thöùc caàn chöùng minh töông ñöông vôùi 2 2 22 2 2 2 2 22 ( ). abc ab bc ca a b b c c a abc ⎛⎞ ++ + + ≥ ++ ⎜⎟ ++ ⎝⎠ Do caû 2 veá cuûa baát ñaúng thöùc naøy ñoàng baäc neân khoâng maát tính toång quaùt, ta coù theå giaû söû 1. abc + += Ñaët 1 ,0 3 q ab bc ca r abc q = + + = ⇒ ≤≤ . Khi ñoù, baát ñaúng thöùc caàn chöùng minh trôû thaønh 22 (12)2 () 189(41)(1) 0 (*) q q qr fr r qqq - ≥- = + - -≥ * Tröôøng hôïp 1.41 q ≤ thì (*) hieån nhieân ñuùng. * Tröôøng hôïp 2.41 q ≥ , theá thì theo Boå ñeà 1, ta coù 41 0 9 q r - ≥≥ Do ñoù ( ) 18 9 (4 1)( 1) 2(4 1) 9 (4 1)( 1) (4 1)(2 3)(1 3) 0 fr r qq q q qq q q qq = +- -≥ -+- - = - - -≥ (*) ñuùng. ñpcm. ⇒ ⇒ Tìm tài liệu Toán ? Chuyện nh ỏ - www.toanmath.com178 Ví duï 2. (Vasile Cirtoaje) Cho,,03 thoûa . abc abc > ++= Chöùng minh raèng 3 ( , ,) 2 a bc Pabc a bc b ca c ab =+ +≥ +++ Lôøi giaûi. Ñaët , 0 3. q ab bc ca r abc q = + + = ⇒ ≤≤ Ta coù baát ñaúng thöùc töông ñöông vôùi: 22 3 2(6)0 r r qq + - -≤ Töø Boå ñeà 1, ta coù 2 22222 1 1 11 3 2 (6 ) 3( ) ( )(6 ) r r q q xy xy q q + -- ≤ + -- Do ñoù, ñeå chöùng minh baát ñaúng thöùc ñaõ cho, ta chæ caàn chöùng minh 2222 11 11 1 11 1 2 1 11 1 2 1 1 1 1 2 1 11 2 11 3() ()(6 ) 0 3 ( , ,) 2 23 12 23 12 3 2 2 43 12 29 ( 1)(3 )0 (ñuùng) ñpcm. xy xy q q Px xy y y yx y y y y y y yy yy + - -≤ ⇔≥ ⇔ +≥ + + ⇔+≥ + - ⎛⎞ + ⎜⎟ ⎝⎠ ⇔+≥ + -+ ⇔ - -≥ ⇒ Ñaúng thöùc xaûy ra khi vaø chæ khi 3,0 hoaëc vaø caùc hoaùn vò. abc a bc = = = =→ Tìm tài liệu Toán ? Chuyện nh ỏ - www.toanmath.com179 Ví duï 3. (Phaïm Kim Huøng) Cho,,0 abc ≥ thoûa 1. abc ++= Chöùng minh raèng 22 2222 2 22 8( )( 16 ) ab bc ca a b b c c a a b c abc + + ≥ + + +++ Lôøi giaûi. Ñaët ,0 , 1 3 qr q ab bc ca r abc q ≥ ⎧ ⎪ = + + =⇒ ⎨ ≤ ⎪ ⎩ theá thì theo baát ñaúng thöùc Schur, ta coù 41 9 q r - ≥ . Töø caùch ñaët, ta coùù 22 22 222 222 2 12 ab bc ca q r abcq + + =- + + =- Do ñoù, baát ñaúng thöùc caàn chöùng minh trôû thaønh 2 2 8( 2 )(16 1 2 ) ( ) 8(2 )(16 1 2 ) 0 q q r rq fr r q r qq ≥ - +- ⇔ = - + - +≥ Ta coù / ( ) 6(32 (4 1)(2 1)) fr r qq = - -+ Coù 2 tröôøng hôïp xaûy ra * Tröôøng hôïp 1. / 1 4 ()0 q fr ≥ ⇒≥ ⇒ () fr laø haøm ñoàng bieán 0. r ∀≥ * Tröôøng hôïp 2. 41 410 9 q qr - ≥⇒≥≥ . Do ñoù / 32(4 1) ( ) 6(32 (4 1)(2 1)) 6 (4 1)(2 1) 9 2(4 1)(23 18 ) 3 0 q fr r qq qq qq -⎛⎞ = - -+ ≥ - -+ ⎜⎟ ⎝⎠ -- = ≥ ⇒ () fr laø haøm ñoàng bieán 0. r ∀≥ Toùm laïi, trong moïi tröôøng hôïp, ta luoân coù () fr laø haøm ñoàng bieán 0. r ∀≥ Do ñoù 2 ( ) (0) (4 1) 0 ñpcm. fr f qq ≥=- ≥ ⇒ Tìm tài liệu Toán ? Chuyện nh ỏ - www.toanmath.com180 * Chuù yù. Caùc baïn neân chuù yù raèng phöông phaùp naøy chæ ñaëc bieät coù hieäu quaû ñoái vôùi nhöõng baát ñaúng thöùc maø daáu baèng xaûy ra khiabc == hoaëc trong ba soá ,, abc coù moät soá baèng 0, hai soá coøn laïi baèng nhau. Tìm tài liệu Toán ? Chuyện nh ỏ - www.toanmath.com181 BAØI TAÄP Baøi 1. (Iran 1996) Cho , , 0. Chöùng minh raèng abc > 2 22 1 1 19 () 4 ( ) ( ) () ab bc ca ab bc ca ⎛⎞ ++ + +≥ ⎜⎟ + ++ ⎝⎠ Baøi 2. (Phaïm Kim Huøng) Cho ,, abc laø caùc soá thöïc döông thoûa maõn 1 abc = . Chöùng minh raèng 3336 (1 )(1 )(1 ) ( ) 64 a b c abc + + + ≤ ++ Baøi 3. Cho 2 22 , , 9. thoûa Tìm giaù trò lôùn nhaát cuûa bieåu thöùc abc abc ∈ ++= R 2() P abc abc = ++- Baøi 4. Cho , , 0. Chöùng minh raèng abc > 33 3 2 22 2 3 3 3 2 22 2 3 1 44 a) b) abc ab bc ca a b c abc a b c abc abc ab bc ca ++ +≥ + + ++ ⎛⎞ + + ++ +≥ ⎜⎟ ++ ⎝⎠ Baøi 5. Cho,,0. Chöùng minh raèng xyz > 4 44 22 22 22 2 22 2() 12 x y z xy yz zx xy yzzx x yz + + ++ + ≥+ + + ++ Baøi 6. (Vietnam TST 1996) Cho,,. abc ∈ R Chöùng minh raèng 4 4 4 4 44 4 ( )( )( ) .( ) 7 ab bc c a abc + + + + + ≥ ++ Tìm tài liệu Toán ? Chuyện nh ỏ - www.toanmath.com182 Baøi 7. Cho , , 0. Chöùng minh raèng abc > 111 9 1 11 4 a b c ab c ab bc c a ⎛⎞ +++ ≥ ++ ⎜⎟ ++ + ++ ⎝⎠ Baøi 8. Cho,,03 thoûa . Chöùng minh raèng a b c ab bc ca > + += 2 22 111 1 2 22 abc ++≤ + ++ Baøi 9. (Vasile Cirtoaje) Cho,,03 thoûa . Chöùng minh raèng a b c ab bc ca > + += 2 22 1 1 13 2 1 11 abc ++≥ + ++ Baøi 10. (Kvant 1993) Cho,,,01 thoûa .Chöùng minh raèng abcd abc > ++= 3 33 11 min, 4 9 27 d a b c abcd ⎧⎫ +++ ≥+ ⎨⎬ ⎩⎭ Baøi 11. (Mihai Piticari, Dan Popescu) Cho,,01 thoûa .Chöùng minh raèng abc abc > ++= 2 22 33 3 5( ) 6( )1 a b c abc + + ≤ + ++ Baøi 12. Cho , , 0. Chöùng minh raèng abc > 2 22 8 2 ( )( )( ) a b c abc ab bc ca a b b c c a ++ +≥ + + + ++ Baøi 13. Cho,,01 thoûa . Chöùng minh raèng xyz x yz > + += 1 2 xy yz zx xy yz yz zx zx xy + +≤ + ++ Tìm tài liệu Toán ? Chuyện nh ỏ - www.toanmath.com183 Baøi 14. (Phaïm Vaên Thuaän) Cho , , 0. Chöùng minh raèng abc > 2 33 3 2 22 2 22 ( )1 .4 2 a b c a b c abc abc ab bc ca abc ⎛⎞ + + + + ++ + -≥ ⎜⎟ ++ ++ ⎝⎠ Baøi 15. (Toaùn Hoïc Tuoåi Treû 2002) Cho 2 22 , , 1. thoûa abc abc ∈ ++= R Tìm giaù trò lôùn nhaát cuûa bieåu thöùc: 3( ) 22 P a b c abc = + +- Baøi 16. (Voõ Quoác Baù Caån) Cho,,03 thoûa . Chöùng minh raèng abc abc > + += 111 (1 )(1 )(1 ) (1 )(1 )(1 ) 1 k k k k kk abc a b c k +++ + + + ≥ + + + ∀≥ Baøi 17. (Vasile Cirtoaje) Cho 222 ,,03 thoûa . Chöùng minh raèng abc abc ≥ ++= 1297() abc ab bc ca + ≥ ++ Baøi 18. (Vasile Cirtoaje) Cho,,03 thoûa . Chöùng minh raèng a b c ab bc ca ≥ + += 333 7 10 a b c abc +++≥ Baøi 19. Cho,,03 thoûa . Chöùng minh raèng abc abc ≥ + += 1 1 13 66 65 ab bc ca + +≤ - -- Baøi 20. (Vasile Cirtoaje) Cho 222 ,,03 thoûa . Chöùng minh raèng abc abc ≥ ++= 3 5( ) 18 abc abc ++ +≥ Tìm tài liệu Toán ? Chuyện nh ỏ - www.toanmath.com184 HAØM LOÀI (LOÕM), HAØM NÖÛA LOÀI NÖÛA LOÕM VAØ BAÁT ÑAÚNG THÖÙC Voõ Quoác Baù Caån I. Caùc ñònh nghóa. 1. Ñònh nghóa haøm loài (loõm). Haøm soá () fx ñöôïc goïi laø loài treân taäp[,] ab ⊂R neáu vôùi moïi , [,] xy ab ∈ vaø vôùi moïi caëp soá khoâng aâm , αβ coù toång baèng 1, ta ñeàu coù ( ) ( ) () f x y f x fy αβ αβ + ≥+ Haøm soá () fx ñöôïc goïi laø loõm treân taäp[ ,] ab ⊂R neáu vôùi moïi , [ ,] x y ab ∈ vaø vôùi moïi caëp soá khoâng aâm , αβ coù toång baèng 1, ta ñeàu coù ( ) ( ) () f x y f x fy αβ αβ + ≤+ Keát quaû sau ñaây chuùng ta thöôøng duøng ñeå nhaän bieát moät haøm laø loài hay loõm Neáu () fx khaû vi baäc hai treân[,] ab thì () fx loài (loõm) treân[,] ab khi vaø chæ khi // // ()0( ()0) f x fx ≤≥ [,] x ab ∀∈ . 2. Ñònh nghóa haøm nöûa loài nöûa loõm. Haøm soá () fx ñöôïc goïi laø nöûa loài nöûa loõm treân[,] ab ⊂R neáu toàn taïi duy nhaát haèng soá() cacb << sao cho () fx loài treân[,] ac vaø loõm treân[,] cb (hoaëc ngöôïc laïi). II. Moät soá tính chaát. 1. Tính chaát 1. Neáu () fx laø moät haøm loõm treân[ ,] ab thì vôùi moïi bxz ya x y za ≥ ≥ ≥≥ ⎧ ⎨ + -≥ ⎩ ta coù ()() ()( ) fx f y fz fx y z + ≥ + +- Chöùng minh. Tìm tài liệu Toán ? Chuyện nh ỏ - www.toanmath.com185 Ta coù h ∀ thoûa 0 h xy ≤ ≤- thì toàn taïi [0,1] α∈ sao cho () h xy α =- Do ñoù (1 ) [ ,] x h x y ab αα - = - +∈ neân theo ñònh nghóa haøm loõm, ta coù ( ) ((1 ) ) (1 ) ( ) () fx h f x y fx f y α α αα -= - + ≤-+ Töông töï, ta coù (1 ) [,] y h x y ab αα += +-∈ neân theo ñònh nghóa haøm loõm, ta cuõng coù ( ) ( (1 )) ( ) (1 ) () fy h f x y f x fy α ααα + = + - ≤ +- Do ñoù ( )( ) ()() (*) fx h f y h fx f y -+ +≤+ Roõ raøng ta coù 0 x z xy ≤- ≤- neân aùp duïng (*) vôùi , h xz =- ta ñöôïc ()() ()( ) fx f y fz fx y z + ≥ + +- Tính chaát 1 ñöôïc chöùng minh hoaøn toaøn. Töø tính chaát 1 ta suy ra ñöôïc tính chaát 2 nhö sau 2. Tính chaát 2. Neáu () fx laø moät haøm loõm treân[,] ab thì vôùi moïi 12 , ,..., [ , ] n x x x ab ∈ thoûa maõn 12 ... ( 1) n x x x n ab + + + - -≤ thì ta coù 1 2 12 ( ) ( ) ... ( ) ( 1) ( ) ( ... ( 1) ) nn fx fx fx n f a fx x x n a + + + ≤ - + ++ +- - Chöùng minh. Ta chöùng minh baèng quy naïp theo n. Deã thaáy khaúng ñònh ñuùng cho 1 bieán soá. Giaû söû khaúng ñònh ñuùng cho n bieán soá, töùc laø ta coù 1 2 12 ( ) ( ) ... ( ) ( 1) ( ) ( ... ( 1) ) nn fx fx fx n f a fx x x n a + + + ≤ - + ++ +- - Ta seõ chöùng minh khaúng ñònh ñuùng cho 1 n + bieán soá, töùc laø chöùng minh 1 2 1 121 ( ) ( ) ... ( ) ( ) ( ... ) nn fx fx fx nf a fx x x na ++ + ++ ≤ + + ++ - Khoâng maát tính toång quaùt, ta coù theå giaû söû 1 121 max{ , ,..., } nn x xxx ++ = . AÙp duïng giaû thieát quy naïp, ta coù Tìm tài liệu Toán ? Chuyện nh ỏ - www.toanmath.com186 1 2 12 ( ) ( ) ... ( ) ( 1) ( ) ( ... ( 1) ) nn fx fx fx n f a fx x x n a + + + ≤ - + ++ +- - Do ñoù, ñeå chöùng minh khaúng ñònh ñuùng cho 1 n + bieán soá, ta chæ caàn chöùng minh 12 1 12 1 ( ... ( 1) ) ( ) ( ) ( ... ) n nn fx x x n a fx f a fx x x na ++ ++ + - - + ≤ + ++ + - Do 1 121 max{ , ,..., } nn x xxx ++ = vaø 1, i bx a in ≥≥∀= neân ta coù 12 11 ... nn bxx x naxa ++ ≥ + ++ - ≥≥ Do ñoù theo tính chaát 1, ta coù 121 12 1 11 121 ()( ...) (( ... ) ) ( ) ( ... ( 1)) () n n nn nn f a f x x x na fx x x na ax fx fx x x n a fx + + ++ + + + ++ -≥ ≥ + ++ - +-+ = + ++ - -+ Vaäy khaúng ñònh ñuùng cho 1 n+ bieán soá. Theo nguyeân lyù quy naïp, khaúng ñònh ñuùng vôùi moïi 1. n ≥ Tính chaát 2 ñöôïc chöùng minh. 3. Tính chaát 2’. Neáu () fx laø moät haøm loài treân[ ,] ab thì vôùi moïi 12 12 , ,..., [ , ] ... ( 1) n n x x x ab x x x n ab ∈ ⎧ ⎨ ++ +- - ≤ ⎩ thì ta coù 1 2 12 ( ) ( ) ... ( ) ( 1) ( ) ( ... ( 1) ) nn fx fx fx n f a fx x x n a + + + ≥ - + ++ +- - 4. Tính chaát 3. (Heä quaû cuûa ñònh lyù Larange) + Neáu () fx khaû vi baäc 2 treân[ ,] ab vaø loõm treân[ ,] ab thì vôùi moïi 0 , [ ,] x x ab ∈ ta coù / 0 00 () ( )( ) () fx f x x x fx ≥ -+ + Neáu () fx khaû vi baäc 2 treân[,] ab vaø loài treân[,] ab thì vôùi moïi 0 , [,] xx ab ∈ ta coù / 0 00 () ( )( ) () fx f x x x fx ≤ -+ Töø tính chaát treân, ta suy ra ñöôïc baát ñaúng thöùc Jensen noåi tieáng. Caùc baïn haõy thöû chöùng minh laïi baèng caùch söû duïng tính chaát 3 xem nhö laø baøi taäp. Tìm tài liệu Toán ? Chuyện nh ỏ - www.toanmath.com187 III. ÖÙng duïng vaøo baát ñaúng thöùc. Caùc ñònh lyù sau ñaây coù theå xem nhö laø moät phöông phaùp chöùng minh baát ñaúng thöùc khaù hieäu quaû. (baïn cuõng neân töï chöùng minh laáy xem nhö laø baøi taäp, löu yù laø ñeå chöùng minh chuùng, ta chæ caàn duøng caùc tính chaát treân laø ñuû). 1. Ñònh lyù 1. 12 , ,..., n xxx laø n soá thöïc thoûa maõn 12 12 ... [ , ] 1, ...( i) ii) iii) laø haèng soá) n i n xxx x ab in x x x CC ≤ ≤≤ ∈ ∀= + + += vaø f laø moät haøm treân[ ,] ab thoûa maõn f loài treân[,] ac vaø loõm treân[,] cb . Ñaët 12 ( ) () ... () n F fx fx fx = + ++ Khi ñoù 123 11 ... , ... [ , ] ( 1,2,..., ) ñaït min khi k kn F xxx xax xabkn -+ ==== = ==∈= 1 2 1 12 ... [ , ], ... ( 1,2,..., ) ñaït max khi k k kn F xx x abx x xbkn - ++ == = ∈ = = == = 2. Ñònh lyù 1’. 12 , ,..., n xxx laø n soá thöïc thoûa maõn 12 12 ... [ , ] 1, ...( i) ii) iii) laø haèng soá) n i n xxx x ab in x x x CC ≤ ≤≤ ∈ ∀= + + += vaø f laø moät haøm treân[ ,] ab thoûa maõn f loõm treân[,] ac vaø loài treân[,] cb . Ñaët 12 ( ) () ... () n F fx fx fx = + ++ Khi ñoù 123 11 ... , ... [ , ] ( 1,2,..., ) ñaït max khi k kn F xxx xax xabkn -+ ==== = ==∈= 1 2 1 12 ... [ , ], ... ( 1,2,..., ) ñaït min khi k k kn F xx x abx x xbkn - ++ == = ∈ = = == = Tìm tài liệu Toán ? Chuyện nh ỏ - www.toanmath.com188 3. Ñònh lyù 2. 12 , ,..., n xxx laø n soá thöïc thoûa maõn 12 12 ... ( , ) 1, ...( i) ii) iii) laø haèng soá) n i n xxx x in x x x CC ≤ ≤≤ ∈ -∞ +∞ ∀ = + + += vaø f laø moät haøm treân[ ,] ab thoûa maõn f loài treân( ,] c -∞ vaø loõm treân[,) c +∞ . Ñaët 12 ( ) () ... () n F fx fx fx = + ++ Khi ñoù, F ñaït min khi 1 23 ... n xxxx ≤ = == F ñaït max khi 121 ... nn x x xx - = = =≤ . 4. Ñònh lyù 2’. 12 , ,..., n xxx laø n soá thöïc thoûa maõn 12 12 ... ( , ) 1, ...( i) ii) iii) laø haèng soá) n i n xxx x in x x x CC ≤ ≤≤ ∈ -∞ +∞ ∀ = + + += vaø f laø moät haøm treân[ ,] ab thoûa maõn f loõm treân( ,] c -∞ vaø loài treân[,) c +∞ . Ñaët 12 ( ) () ... () n F fx fx fx = + ++ Khi ñoù, F ñaït max khi 1 23 ... n xxxx ≤ = == F ñaït min khi 121 ... nn x x xx - = = =≤ . IV. Moät soá aùp duïng. Ví duï 1. (VMEO 2004) Cho tam giaùc nhoïn . ABC Tìm giaù trò nhoû nhaát cuûa bieåu thöùc 25 tg tg tg P ABC = ++ Lôøi giaûi. Tìm tài liệu Toán ? Chuyện nh ỏ - www.toanmath.com189 Xeùt haøm soá () tg fxx = vôùi 0,. 2 x π ⎛⎞ ∈ ⎜⎟ ⎝⎠ Ta coù /2 //2 ()1 () 2 ( 1)0 tg tg tg fxx f x xx =+ = +> ⇒ () 0,. 2 laø haøm loõm treân fx π ⎛⎞ ⎜⎟ ⎝⎠ Do ñoù, theo tính chaát haøm loõm, ta coù / ( ) ( 3) ( 3)( 3) 3 10( 3) arctg arctg arctg arctg fAf f AA ≥ + - =+- Töông töï, ta coù / ( ) ( 2) ( 2)( 2) 2 5( 2) 2 ( ) 4 10( 2) arctg arctg arctg arctg arctg fBf f BB fBB ≥ + - =+- ⇒ ≥+- / ( ) ( 1) ( 1)( 1) 1 2( 1) 5 ( ) 5 10( 1) arctg arctg arctg arctg arctg fCf f CC fCC ≥ + - =+- ⇒ ≥+- Do ñoù () 2 () 5( ) 12 10( 3 2 1) 12( 3 2 1) arctg arctg arctg vì arctgarctgarctg PfAfBfC ABC ABC π =++ ≥+++--- =++=++= Ñaúng thöùc xaûy ra khi vaø chæ khi 3 2 4 arctg arctg A B C π ⎧ ⎪ = ⎪ = ⎨ ⎪ ⎪ = ⎩ . Vaäy min 12. P = Tìm tài liệu Toán ? Chuyện nh ỏ - www.toanmath.com190 Ví duï 2. Cho caùc soá döông ,, abc thoûa 21 2 8 12. ab bc ca ++≤ Tìm giaù trò nhoû nhaát cuûa bieåu thöùc 1 23 P abc = ++ Lôøi giaûi. Ñaët 1 23 , ,, x yz a bc = == baøi toaùn chuyeån veà ,,0 xyz > thoûa2 4 7 2. x y z xyz + +≤ Tìm giaù trò nhoû nhaát cuûa bieåu thöùc P x yz =++ Khoâng maát tính toång quaùt, ta chæ caàn xeùt tröôøng hôïp2 4 72 x y z xyz + += laø ñuû (taïi sao?). Ñaët 7 27 7, .,. 27 x my nzp = == thì ta coùm npmnp + += . Do ñoù, toàn taïi tam giaùc nhoïn ABC sao cho , ,. tg tg tg mAnBp C = == Khi ñoù, ta coù 7 .(14 7 4 ) 14 tg tg tg P A BC = ++ Xeùt haøm soá () tg fxx = vôùi 0,. 2 x π ⎛⎞ ∈ ⎜⎟ ⎝⎠ Ta coù /2 //2 ()1 () 2 ( 1)0 tg tg tg fxx f x xx =+ = +> ⇒ () 0,. 2 laø haøm loõm treân fx π ⎛⎞ ⎜⎟ ⎝⎠ Do ñoù, theo tính chaát haøm loõm, ta coù / 37 37 37 () 7 77 arctg arctg arctg fAf fA ⎛ ⎞⎛ ⎞⎛⎞ ≥ +- ⎜ ⎟⎜ ⎟⎜⎟ ⎝ ⎠⎝ ⎠⎝⎠ Tìm tài liệu Toán ? Chuyện nh ỏ - www.toanmath.com191 37 16 37 . 777 37 14 ( ) 6 7 32 7 arctg arctg A fAA ⎛⎞ = +- ⎜⎟ ⎝⎠ ⎛⎞ ⇒ ≥ +- ⎜⎟ ⎝⎠ Töông töï, ta coù / 57 57 57 () 7 77 5 7 32 57 . 777 57 7 ()5732 7 arctg arctg arctg arctg arctg fBf fB B fBB ⎛⎞⎛⎞⎛ ⎞ ≥ +- ⎜⎟⎜⎟⎜ ⎟ ⎝⎠⎝⎠⎝ ⎠ ⎛⎞ = +- ⎜⎟ ⎝⎠ ⎛⎞ ⇒ ≥ +- ⎜⎟ ⎝⎠ ( ) ( )( ) ( ) ( ) / () 7 77 787 4()47327 arctg arctg arctg arctg arctg fCf fC C fCC ≥ +- = +- ⇒ ≥ +- Do ñoù 7 .(14( ) 7( ) 4()) 14 7 37 57 .157327 14 77 arctg arctg arctg P f A f B fC A BC = ++ ⎛⎞ ⎛⎞ ≥ + ++--- ⎜⎟ ⎜⎟ ⎜⎟ ⎝⎠ ⎝⎠ 15 3 7 57 ( 7) 2 77 vì arctg arctg arctg ABC π =++= ++= Ñaúng thöùc xaûy ra khi vaø chæ khi 37 1 3 7 3 57 54 . 7 25 2 3 7 2 arctg arctg arctg A a m B nb p C c ⎧ ⎧ = ⎪ = ⎪ = ⎧ ⎪ ⎪ ⎪ ⎪ ⎪ ⎪⎪ = ⇔ = ⇔= ⎨ ⎨⎨ ⎪ ⎪⎪ = ⎪ ⎪⎪ = ⎩ = ⎪⎪ ⎩ ⎪ ⎩ Vaäy Tìm tài liệu Toán ? Chuyện nh ỏ - www.toanmath.com192 15 min. 2 P = Ví duï 3. (Phaïm Kim Huøng) Cho 12 , ,..., 0 n aaa > thoûa 12 ...1 n aaa = . chöùng minh raèng vôùi moïi 0 k > thì 12 1 11 ... min 1, (1 ) (1 ) (1 ) 2 k k kk n n a aa ⎧⎫ + ++≥ ⎨⎬ + ++ ⎩⎭ Chöùng minh. Neáu 1 n = thì baát ñaúng thöùc ñaõ cho hieån nhieân ñuùng. Neáu 2 n = + Neáu01 k << thì ta coù 1 2 1 21 1 1 1 1 1 11 1 1 (1 )(1 )1 11 1 kk a a a aa a + ≥+ =+ = + + + ++ + + Neáu 1 k≥ thì ta coù 1 1 1 21 1 1 11 ( (1 ) (1 ) (1)2 theo baát ñaúng thöùc Holder) k k k kk a a aa - + + =≥ + ++ Xeùt 3 n ≥ Ta chöùng minh baát ñaúng thöùc ñuùng cho giaù trò tôùi haïn 1 2 2 log k n kn = ⇔= . Do 3 n ≥ neân 11 nk - >> . Khi ñoù , ta coù mk +∀≥ 11 11 1 1 11 .. (1 ) (1 ) 2 2 m m nn k k m m k m km ii ii kk nn aa nn -- == ⎛⎞ ⎛⎞ ≥ ≥= ⎜⎟ ⎜⎟ ++ ⎝⎠ ⎝⎠ ∑∑ , ta coù mk +∀≤ 1 11 1 11 11 (1 ) (1 ) (1 ) k n nn m m km i ii i ii a aa = == ⎛⎞ ≥ ≥⇒≥ ⎜⎟ + ++ ⎝⎠ ∑ ∑∑ Tìm tài liệu Toán ? Chuyện nh ỏ - www.toanmath.com193 Khoâng maát tính toång quaùt giaû söû 12 0 ... n aaa < ≤ ≤≤ . Ñaët 12 1 122 1 2 12 ... ln , ln ,..., ln ... 0 ... 1) thì (do n nn nn xxx x ax a xa x x x aaa ≤ ≤≤ ⎧ = == ⎨ +++== ⎩ Xeùt haøm soá 1 () ( 1) xk fx e = + Ta coù // 2 // .( 1) () ( 1) ( ) 0 ln xx xk ke ke fx e fx xk + - = + = ⇔ =- Töø ñoù, ta coù f loài treân( , ln] k -∞- vaø loõm treân[ln, ) k - +∞ ⇒ Theo Ñònh lyù 2, ta coù 11 11 (1) (1) i nn kx k ii i P a e == == + + ∑∑ ñaït min khi 1 23 ... n xxxx ≤ = == ( 1) ( 1) 1 11 min min ( 0) ( 1) ( 1) 1 min ( 1) (1) ( 1) ( 1) t k ntk nk t k nk n Pt ee nx xe xx -- - - ⎧⎫ - ⇒≥ +≥ ⎨⎬ ++ ⎩⎭ ⎧⎫ - = + =≥ ⎨⎬ ++ ⎩⎭ Tieáp theo, ta seõ tìm min cuûa haøm soá ( 1) 1 1 ()1 ( 1) ( 1) vôùi nk k nk nx gxx xx - - - =+≥ ++ Ta coù (1)1 / 1 11 1 ( ) ( 1) ( 1) ( 1) nk n kk x gx nk xx -- - ++ ⎛⎞ =-- ⎜⎟ ++ ⎝⎠ / 1)1 1 11 ( ) 0 .( 1) ( 1) nk k nk gx x xx (-- + -+ =⇔ + =+ 1)1 1 1 .( 1) 1 (2) nk n k x xx (-- - + ⇔ +=+ Ñaët 1 1 1 k txt + = ⇒≥ . Khi ñoù, phöông trình (2) trô ûthaønh 1) 1 1 ( 1)( 1) (1)( 1) 1)1 .(1)1 10 n k k nk n k nk nk t tt t tt (- - + -+ - + (-- +=+ ⇔ - - += Tìm tài liệu Toán ? Chuyện nh ỏ - www.toanmath.com194 t / () ht 1 0 () ht _ + 0 t +∞ +∞ 0 1 1 x / () gx 1 0 () gx _ + 1 1 k t + +∞ Xeùt haøm soá ( 1)( 1) 1) 1 ()1 n k nk nk ht t tt - + (-- = --+ vôùi 1 t ≥ Ta coù /(1)21 ( ) .(( 1)( 1) ( 1) 1) nk nk h t t n k t nkt n k --+ = - + - - -+ / ()0 ht =⇔ 1 ( 1)( 1) ( 1) 1 0 nk n k t nkt nk + - + - - - += Xeùt tieáp haøm soá 1 () ( 1)(1) ( 1)1 1 vôùi nk mtn ktnkt nkt + =- + - --+≥ Ta coù /1 ( ) ( 1) (( 1) ) k nk mtnk tntk -- = + -- Chuù yù raèng 1 nk -> neân / ( ) ( 1) (( 1) ) 0 k mtnk tnk ≥ + - -> () mt ⇒ laø haøm ñoàng bieán treân[1,) +∞ Ta laïi coù (1) ( 1)( 1) ( 1) 1 (1 ) 0, lim ( ) t m n k nk n k n k mt →+∞ =- + - - - += -< =+∞ Neân phöông trình ()0 mt = coù nghieäm duy nhaát 0 1 t > ⇒ Phöông trình / ()0 ht = coù nghieäm duy nhaát 0 1 t > Baûng bieán thieân cuûa () ht Caên cöù vaøo baûng bieán thieân, ta coù 10 ()01 coù 2 nghieäm phaân bieät laø 1 vaø ht tt = >> Do ñoù / ()0 gx = coù 2 nghieäm phaân bieät laø 1 vaø 1 1 1 k t + > . Baûng bieán thieân cuûa () gx Caên cöù vaøo baûng bieán thieân, ta suy ra Tìm tài liệu Toán ? Chuyện nh ỏ - www.toanmath.com195 { } ( ) min (1),lim ( ) 1 1 (3) x gx g gx x →∞ ≥ = ∀≥ Töø (1) vaø (3), ta suy ra ñpcm. Ví duï 4. (Vasile Cirtoaje) Cho 2 21 3, ,0 ( 1) n n nNk n - ≥ ∈ <≤ - vaø 12 , ,..., 0 n aaa > thoûa 12 ...1 n aaa = . Chöùng minh raèng 12 1 11 ... 1 1 11 n n ka ka ka k + ++≤ + + ++ Lôøi giaûi. Ñaët 12 ( 1, ) ... n iin y kai n yy yk = =⇒= vôùi 12 2 21 ... ( 1) n n n k yyy n - =≤ - . Khi ñoù, baát ñaúng thöùc caàn chöùng minh trôû thaønh 12 1 11 ... 1 111 n n y y yk + ++≤ + + ++ Khoâng maát tính toång quaùt giaû söû 12 0 ... n yyy < ≤ ≤≤ . Ñaët 12 1 122 1 2 12 ... ln , ln ,..., ln ... ln ... 1) thì (do n nn nn xxx x yxy xy x x x n k aaa ≤ ≤≤ ⎧ = == ⎨ +++== ⎩ Xeùt haøm soá 12 1 () ( 1) x fx e = + Ta coù // // 52 .( 2) ( ) ; ( ) 0 ln2 4( 1) xx x ee fx fx x e - = =⇔= + Töø ñoù, ta coù f loài treân ( ,ln2] -∞ vaø loõm treân [ln2, ) +∞ ⇒ Theo Ñònh lyù 2, ta coù 12 12 11 11 (1) (1) i nn x ii i P y e == == + + ∑∑ ñaït max khi 121 .... nn x x xx - = ==≤ ln ( 1) 11 max max ( ln ) 11 t n k nt n P tk ee -- ⎧⎫ - ⇒≤ +≤ ⎨⎬ ++ ⎩⎭ Tìm tài liệu Toán ? Chuyện nh ỏ - www.toanmath.com196 1 2 1 1 max ( ) (1) 1 n t nn nx x ek x xk - - ⎧⎫ -⎪⎪ = + =≤ ⎨⎬ + + ⎪⎪ ⎩⎭ Tieáp theo, ta seõ tìm max cuûa haøm soá 1 2 1 1 () 1 vôùi n nn nx gx xk x xk - - - =+≤ + + Ta coù 3 2 / 33 1 22 11 (). 2 ( ) ( 1) n n nn n kx gx x kx - - ⎛⎞ - ⎜⎟ =- ⎜⎟ ⎜⎟ ++ ⎝⎠ 3 33 /1 2 22 23 1 33 ()0 .(1)() .( 1) (2) n n nn nn nn gx kx x xk kx x xk - - - - =⇔ + =+ ⇔ +=+ Ñaët 22 33 t x tk = ⇒≤ . Khi ñoù, phöông trình (2) trô ûthaønh 2 3 3 3( 1) 3222 22 3(1)3 33 2 22 .1 0 nnn n nn n nn n kt t tk t kt kt k -- -- ⎛⎞ +=+ ⎜⎟ ⎝⎠ ⇔ - - += Xeùt haøm soá 22 3(1)3 33 2 22 () nn n nn n ht t kt ktk -- = - -+ vôùi 2 3 tk ≤ Ta coù 5 22 3 / 3 33 2 1 ( ) . . 3( 1) ( 3) 2 n nn n ht t n t nkt nk - ⎛⎞ = - - -- ⎜⎟ ⎜⎟ ⎝⎠ / ()0 ht =⇔ 22 3 33 2 3(1) (3)0 nn n n t nkt nk - - --= Xeùt tieáp haøm soá 2 22 3 3 33 2 ( ) 3( 1) ( 3) vôùi nn n mt n t nk t n k tk =-- --≤ Ta coù 2 1 23 / 3 22 3 ( ) . 2( 1) 2 n n n mtt n tk - ⎛⎞ = -- ⎜⎟ ⎜⎟ ⎝⎠ 2 2 23 3 / 0 ()0 2( 1) n n k mtt n - ⎛⎞ ⎜⎟ = ⇔= ⎜⎟ - ⎜⎟ ⎝⎠ Tìm tài liệu Toán ? Chuyện nh ỏ - www.toanmath.com197 t / () ht 0 0 () ht _ + 1 t 23 k 0 n k () gx x / () gx 0 0 _ + 32 2 t k 0 Do 2 21 0 ( 1) n k n - <≤ - neân 2 3 0 tk < . Qua 0 t thì / () mt ñoåi daáu töø aâm sang döông neân () mt nghòch bieán treân 0 (0,] t vaø ñoàng bieán treân 2 3 0 , tk ⎡⎞ ⎟ ⎢ ⎟ ⎣⎠ . Ta laïi coù 22 33 2 2 -1 (0)3 0, (2 )02 ( -1) do n n mnmknkkk n ⎛⎞ ⎛⎞ =-≤ = -> >≥ ⎜⎟ ⎜⎟ ⎜⎟ ⎝⎠ ⎝⎠ Neân phöông trình ()0 mt = coù nghieäm duy nhaát 2 3 1 0 tk << . ⇒ Phöông trình / ()0 ht = coù nghieäm duy nhaát 2 3 1 0 tk << Baûng bieán thieân cuûa () ht Caên cöù vaøo baûng bieán thieân, ta coù 23 21 ()0 coù 2 nghieäm döông phaân bieät laø vaø ht k tt =< . Do ñoù / ()0 gx = coù 2 nghieäm döông phaân bieät laø k vaø 32 32 21 ttk << . Baûng bieán thieân cuûa () gx Caên cöù vaøo baûng bieán thieân, ta suy ra { } ( ) max (0), ( ) (3) 1 n gx g gk xk k ≤ = ∀≤ + Töø (1) vaø (3), ta suy ra ñpcm. Tìm tài liệu Toán ? Chuyện nh ỏ - www.toanmath.com198 Ví duï 5. (Voõ Quoác Baù Caån) Cho caùc soá thöïc ,, xyz thoûa ( ) , , 3,3 1 xyz xyz ⎧ ∈- ⎪ ⎨ + += ⎪ ⎩ . Tìm giaù trò lôùn nhaát vaø giaù trò nhoû nhaát cuûa bieåu thöùc 2 22 ( , ,) 1 11 x yz Pxyz x yz =++ + ++ Lôøi giaûi. Khoâng maát tính toång quaùt, ta coù theå giaû söûx yz ≤≤ Xeùt haøm soá 2 () 1 x fx x = + vôùi ( ) 3,3 x∈- Ta coù 2 // 23 // ( 3) () ( 1) ()0 03 xx fx x fx xx - = + =⇔ = ∨=± Qua 0 thì // () fx ñoåi daáu töø döông sang aâm neân () fx loõm treân ( 3,0 ⎤ - ⎦ vaø loài treân ) 0,3 ⎡ ⎣ . Do ñoù, theo Ñònh lyù 1’, ta coù (, , ) ( ) ( ) () Pxyz f x f y f z = ++ ñaït max khi 3,3 1 11 x yz x y z xy xyz xy z xy z ⎧⎧ == ⎧ = - = = =- ⎪⎪ ∨∨ ⎨ ⎨⎨ + += + += + += ⎪⎪ ⎩ ⎩⎩ Hay 3 3 1 ( 31 3 1 23 2 loaïi) x xy xyz z yz ⎧ =- ⎧ = =- ⎪⎪ ===∨∨ ⎨⎨ + =+ == ⎪ ⎪ ⎩ ⎩ Ta laïi coù 1119 ,, 333 10 P ⎛⎞ = ⎜⎟ ⎝⎠ Tìm tài liệu Toán ? Chuyện nh ỏ - www.toanmath.com199 ( ) 31 31 5439 3,, 2 2 10 443 P ⎛⎞ + ++ - =< ⎜⎟ + ⎝⎠ Do ñoù 9 max(,,). 10 Pxyz = Cuõng theo Ñònh lyù 1’, ta coù (, , ) ( ) ( ) () Pxyz f x f y f z = ++ ñaït min khi ,33 1 11 x yz x yz yz xyz x yz x yz ⎧⎧ == ⎧ = = == ⎪⎪ ∨∨ ⎨ ⎨⎨ + += + += + += ⎪⎪ ⎩ ⎩⎩ Hay 13 1 23 1 2 3 3 3 (loaïi) x xy xyz yz z ⎧ - ⎧ =- == ⎪⎪ ===∨∨ ⎨⎨ == ⎪ ⎪ ⎩ = ⎩ Ta laïi coù ( ) 111 9 ,, 333 10 1 31 3 5 43 , ,3 22 443 P P ⎛⎞ = ⎜⎟ ⎝⎠ ⎛⎞ --- = ⎜⎟ - ⎝⎠ Vaäy ( ) 5 43 min(,,). 443 Pxyz - = - Keát luaän 9 max (, ,) 10 Pxyz = ( ) 5 43 min ( , ,) 443 Px yz - = - Tìm tài liệu Toán ? Chuyện nh ỏ - www.toanmath.com200 Ví duï 6. (Crux mathematicorum) Cho caùc soá khoâng aâm 12 , ,..., n xxx ( 2) n ≥ thoûa 12 ... 1. n xxx + + += Chöùng minh raèng 12 12 1 112 ... 2 1 11 3 n n x xx Pn x xx - -- = + + + ≤ -+ + ++ Chöùng minh. Xeùt haøm soá 1 () 1 x fx x - = + vôùi [0,1] x ∈ . Ta coù 2 // 33 // (1 ) (1 2 ) () (1)(1)(1)(1) 1 ()0 2 xx fx xx xx fxx +- = +- +- = ⇔= Qua 1 2 thì // () fx ñoåi daáu töø döông sang aâm neân () fx loõm treân 1 0, 2 ⎡⎤ ⎢⎥ ⎣⎦ vaø loài treân 1 ,1 2 ⎡⎤ ⎢⎥ ⎣⎦ . Do ñoù, theo Ñònh lyù 1’, ta coù 12 ( ) ( ) ... () n P fx fx fx = + ++ ñaït max khi 1 2 12 12 ... 0, ... ( 0, 1) ...1 m m mn n x x x x x x mn xxx ++ ⎧ = == = = == = - ⎪ ⎨ + + += ⎪ ⎩ Hay 12 12 ...0 ( 0, 1) 1 ... m m mn xxx mn xxx nm ++ = = == ⎧ ⎪ =- ⎨ = = == ⎪ - ⎩ + Neáu 1 mn =- thì ta coù 1 21 ()()... ( )() ( 1) (0) (1) 1 2 2 (1) 3 nn P fx fx fx fx n ff n n - = + +++ ≤-+ =- < -+ Tìm tài liệu Toán ? Chuyện nh ỏ - www.toanmath.com201 + Neáu 1 mn <- thì ta coù 1 21 ()()... ( )() 1 (0)() 1 () 1 1 1 nn P fx fx fx fx mf n mf nm nm m nm nm t n tt t - = + +++ ⎛⎞ ≤ +- ⎜⎟ - ⎝⎠ -- = +- -+ - = -+ + () gt = Trong ñoù [2,] tnmn = -∈ . Ta coù ( ) ( ) 23 / 3 43 3 23 32 3 23 ( 1) ( 1) () ( 1) ( 1) ( 1) ( 1) ( 1) ( 1) ( 1) ( 1) 2 21 ( 1) ( 1) ( 1) ( 1) 02 (do ) ttt gt tt ttt tt t tt tt tt t tt t - +- = +- -+- = + - + +- - ++ = + - + +- <≥ () gt ⇒ laø haøm nghòch bieán treân [2, ]. n 2 ( ) (2) 2 [2, ] 3 gt g n tn ⇒ ≤ = - + ∀∈ 2 2 (2) 3 Pn ⇒ ≤ -+ Töø (1) vaø (2) suy ra trong moïi tröôøng hôïp, ta luoân coù 2 2 3 Pn ≤ -+ ⇒ ñpcm. Tìm tài liệu Toán ? Chuyện nh ỏ - www.toanmath.com202 Ví duï 7. Cho caùc soá thöïc döông ,, abc thoûa 1. abc = Chöùng minh raèng 22 1 11 3 1 11 aabbcc 2 + +≤ - + - + -+ Nhaän xeùt. Ta coù theå ñaët , , (,,) x yz a ebece xyz === ∈R thì ta coù 0 x yz ++= . Ñeán ñaây, neáu laøm theo caùch laøm treân, ta seõ xeùt haøm soá 2 1 () 1 tt ft ee = -+ ñeå xem () ft coù laø haøm nöûa loài nöûa loõm hay khoâng. Nhöng ruûi thay, () ft laïi khoâng phaûi laø haøm nöûa loài nöûa loõm. Thaät vaäy, ta coù 32 // 23 (433 1) () ( 1) t t tt tt ee ee ft ee - -+ = -+ . Deã thaáy // () ft coù 2 nghieäm phaân bieät neân () ft khoâng phaûi laø haøm nöûa loài nöûa loõm. Vaäy phaûi laøm sao baây giôø? Laøm theá naøo ñeå vöôït qua noù ñaây? Sau ñaây giaûi phaùp cuûa toâi cho vaán ñeà treân Chöùng minh. Ta coù Boå ñeà sau Boå ñeà. Vôùi moïi soá thöïc döông ,, abc thoûa 1 abc = , ta coù 22 1 11 (,,)1 1 11 Pabc aabbcc 2 = ++≥ + + + + ++ Chöùng minh. Do ,,0 abc > vaø 1 abc = neân toàn taïi ,, xyz∈R sao cho ,, x yz a ebe ce === . Khi ñoù, ta coù 0 x yz ++= vaø (, , ) () ( ) () Pabc fx f y fz =++ vôùi 2 1 () 1 tt ft ee = ++ . Ta coù 32 // 23 (4 3 3 1) () ( 1) t t tt tt ee ee ft ee + -- = ++ Tìm tài liệu Toán ? Chuyện nh ỏ - www.toanmath.com203 Deã thaáy // ()0 ft = coù duy nhaát moät nghieäm 0 t vaø qua 0 t thì // () ft ñoåi daáu töø aâm sang döông neân () ft loài treân 0 ( ,] t -∞ vaø loõm treân 0 [,) t +∞ neân theo Ñònh lyù 2, ta coù ( ,,) Pabc ñaït min khixyz ≤= . Do ñoù, ta chæ caàn chöùng minh 22 () 2 ()1 12 1 (*) 11 x x yy f x fy e e ee +≥ ⇔ +≥ + + ++ Vôùi , xy ∈R thoûa 20 xy += . Ta coù 2 42 4 2 42 4 2 22 2 4 42 2 21 (*) 1() 11 1 1 2 1 11 2 1 1 ( 1)( 1) 2(1)1 ( 1) 0() ñuùng y me mm mm m m m mm m mm mm mm m m m mm m ⇔ + ≥= ++ ++ ⇔ +≥ + + ++ ⇔+≥ + + ++ -+ ⇔ - + + ≥ ++ ⇔ -≥ Töø ñaây, ta suy ra ñöôïc (,,)1 Pabc ≥ Boå ñeà ñöôïc chöùng minh hoaøn toaøn. Ñaúng thöùc xaûy ra khi vaø chæ khi 1. abc === Trôû laïi baøi toaùn cuûa ta Töø Boå ñeà treân, thay ,, abc laàn löôït bôûi 2 22 1 11 ,, abc , ta ñöôïc 4 2 4 2 42 4 44 4 2 4 2 42 1 11 1 1 1 1 1 11 1 11 1 1 11 a a b b cc a bc a a bb cc ++≥ + + + + ++ ⇔ + +≥ + + + + ++ Tìm tài liệu Toán ? Chuyện nh ỏ - www.toanmath.com204 2 22 4 2 4 2 42 2 22 4 2 4 2 42 2 2 22 2 2 22 22 22 2 1 11 2 1 11 2( 1) 2( 1) 2( 1) 4 1 11 ( 1) ( 1) ( 1) ( 1) ( 1)( 1) ( 1)( 1) ( 1) ( 1) 4 ( 1)( 1) 1 1 cy abc a a b b cc abc a a b b cc a a a a bb bb a a a a bb bb c c cc c c cc aa + ++ ⇔ + +≤ + + + + ++ + ++ ⇔ + +≤ + + + + ++ + ++ - + + ++ - + ⇔ ++ + + - + ++ -+ + + + -+ +≤ ++ -+ ⇔ -+ 2 1 4 (**) 1 c cyc aa ⎛ ⎞⎛⎞ +≤ ⎜ ⎟⎜⎟ ⎜ ⎟⎜⎟ ++ ⎝ ⎠⎝⎠ ∑∑ Laïi aùp duïng Boå ñeà treân, ta coù 2 1 1 1 cyc aa ≥ ++ ∑ Neân töø (**), ta suy ra ñöôïc 2 1 3 1 cyc aa ≤ -+ ∑ ⇒ ñpcm. Ñaúng thöùc xaûy ra khi vaø chæ khi 1. abc === Tìm tài liệu Toán ? Chuyện nh ỏ - www.toanmath.com205 Baøi taäp. UBaøi 1U. (VMEO 2005) Cho ,, abc laø caùc soá thöïc döông cho tröôùc vaø ,, xyz laø caùc soá thöïc döông thoûa maõn . ax by cz xyz + += Tìm giaù trò nhoû nhaát cuûa bieåu thöùc P x yz =++ UBaøi 2U. (Phaïm Kim Huøng) Cho ,, abc laø caùc soá thöïc khoâng aâm. Chöùng minh raèng 3 min 2, 0 2 kkk k a bc k b c c a ab ⎛ ⎞⎛ ⎞⎛ ⎞ ⎧ ⎫ + + ≥ ∀≥ ⎨⎬ ⎜ ⎟⎜ ⎟⎜ ⎟ + ++ ⎝ ⎠⎝ ⎠⎝ ⎠ ⎩ ⎭ UBaøi 3U. (Crux mathematicorum) Chöùng minh raèng vôùi moïi soá khoâng aâm ,, abc ta coù 48 48 48 1 1 1 15 a bc b c c a ab + ++ ++ ≥ + ++ UBaøi 4U. Cho tam giaùc khoâng tuø . ABC Chöùng minh raèng 222 cos cos cos 1 cos 1 cos 1 cos 1 2 ABC ABC + +≥ + ++ UBaøi 5U. Xeùt caùc soá thöïc khoâng aâm 12 , ,..., n xxx thoûa maõn 12 ... 1 n xxx n r nn + ++ - =≥ Chöùng minh raèng 2 2 22 12 1 11 ... 1 1 11 n n x x xr + ++≥ + + ++ Tìm tài liệu Toán ? Chuyện nh ỏ - www.toanmath.com206 UBaøi 6U. Xeùt caùc soá thöïc khoâng aâm 12 , ,..., n xxx thoûa maõn 12 2 ... 1 1 n xxx n r n nn + ++ - =≤ -+ Chöùng minh raèng 2 2 22 12 1 11 ... 1 1 11 n n x x xr + ++≤ + + ++ UBaøi 7U. Xeùt caùc soá thöïc khoâng aâm 12 , ,..., n xxx thoûa maõn 12 1 ... 1 n n xx xp n =≤ - Chöùng minh raèng 12 1 11 ... 1 1 11 n n x x xp + ++≤ + + ++ UBaøi 8U. Xeùt caùc soá thöïc khoâng aâm 12 , ,..., n xxx thoûa maõn 12 ...1 1 n n n xx xp n =≤- - Chöùng minh raèng 2 2 22 12 1 11 ... (1 ) (1 ) (1 ) (1 ) n n x x xp + ++≤ + + ++ UBaøi 9U. Cho tam giaùc . ABC Tìm giaù trò nhoû nhaát cuûa caùc bieåu thöùc 23 sin .sin .sin P A BC = sin .sin .sin , , ( laø caùc soá thöïc döông cho tröôùc) m np Q A B C mnp = Tìm tài liệu Toán ? Chuyện nh ỏ - www.toanmath.com207 CAÙC BAØI TOAÙN CHOÏN LOÏC ----oOo---- Baøi toaùn 1. (Phaïm Kim Huøng) ,, Cho laø caùc soá thöïc khoâng aâm. Chöùng minh raèng abc 44 44 449 ab bc ca a b c a bc b ca cab ++ + +≤ + + + + ++ Lôøi giaûi. * Caùch 1. Ta coù baát ñaúng thöùc caàn chöùng minh töông ñöông vôùi 4 3 3 222 4 3 222 9 (4 4 )(4 4 ) ( )(4 4 )(4 4 )( 4 4 ) 4 16 11 3 6 4 11( )( )( )( ) 5 3 6 cyc cyc cyc cyc cyc cyc cyc cyc cyc cyc aba b c ab c abc abc abcabc a ab ab ab abc a a b b c c a a b c ab a b a bc + + ++≤ ≤++++++++ ⇔ + ≥ ++ ⇔ + --- +++ ≥+ ∑ ∑ ∑∑ ∑∑ ∑ ∑ ∑∑ Khoâng maát tính toång quaùt, giaû söû { } min , , a abc = Ñaëêt , ba xcay =+ =+ Khi ñoù, ta coù 4 4 32 223 3 4 4 3 4 3 2 22 3 3 23 22 4 32 2 22 2 22 3 4( ) 6( ) 4( ) 3 4( ) 3( ) ( 3) 3 4( ) 2( 2 ) 2( ) cyc cyc cyc a a x ya xya x ya xy ab a x y a x y xy a x y xy a xy ab a x ya x y xya xyxy axy = + + + + + + ++ = + + + ++ ++++ = + + + ++ +++ ∑ ∑ ∑ 2 4 32 2 22 2 33 34( ) ( 5)( ) ( )( )( )( ) ( )(3 ) 3() cyc a bc a x y a x y xy a x y xy a abb c ca abc xyx y a x y xy x y a x y xy = + + + + + ++ - - - + + =- - ++ =- - -+ ∑ Tìm tài liệu Toán ? Chuyện nh ỏ - www.toanmath.com208 Do ñoù, baát ñaúng thöùc caàn chöùng minh töông ñöông vôùi 22 2 3 3 22 4 4 3 223 27( ) (21 21 45 36 ) 4 4 11 3 16 0 x y xy a x y x y xy a x y x y x y xy +- + + - ++ + + - - +≥ 2 2 2 332 2 222 27( ) (21 21 45 36 ) ( 2)(4 5 )0 (ñuùng) x y xya x y xy xya x y x xyy ⇔ +- + + - ++ + - + +≥ ñpcm. ⇒ Ñaúng thöùc xaûy ra khi vaø chæ khi 0,2 hoaëc abc a b c == == vaø caùc hoaùn vò töông öùng. * Caùch 2. Ta coù 3 4( ). 44 44 cyc cyc abb b abc a bc abc ⎛⎞ + = ++ ⎜⎟ ++ ++ ⎝⎠ ∑∑ Do ñoù, baát ñaúng thöùc caàn chöùng minh töông ñöông vôùi 1 (*) 44 44 443 a bc a bc b c a c ab + +≤ + + + + ++ Khoâng maát tính toång quaùt, ta coù theå giaû söû 3 abc + += . Khi ñoù, ta coù 2 22 (*)1 4 44 (4 )(4 ) (4 )(4 )(4 ) 4 cyc a bc c ab a ab abc a b b c c a abc ⇔++≤ - -- ⇔ - - ≤ - -- ⇔ + + +≤ ∑ Nhö vaäy, ñeå chöùng minh baát ñaúng thöùc ñaõ cho, ta caàn phaûi chöùng minh 2 22 4 (**) a b b c c a abc + + +≤ Khoâng maát tính toång quaùt, ta coù theå giaû söû b naèm giöõa a vaø . c Do ñoù 2 22 222 222 ( )( )0 2 () cab cb b c c a bc abc a b b c c a abc a b bc abc b a c - -≤ ⇔ +≤+ ⇔ + + + ≤ + + =+ Tìm tài liệu Toán ? Chuyện nh ỏ - www.toanmath.com209 Maët khaùc, aùp duïng baát ñaúng thöùc AM-GM, ta coù 2 2 2 2 ( )() 2 ( ) 2 .( ).( ) 8 3 ( )4 b a c ac bac bac ac bac ++ ++ ⎛⎞ +=++≤= ⎜⎟ ⎝⎠ ⇒ +≤ Vaäy 2 22 4 ñpcm. a b b c c a abc + + +≤ ⇒ Ñaúng thöùc xaûy ra khi vaø chæ khi 0,2 hoaëc abc a b c == == vaø caùc hoaùn vò töông öùng. Baøi toaùn 2. (Phaïm Kim Huøng) Cho,,3 laø caùc soá thöïc khoâng aâm thoûa maõn abc abc ++= . Chöùng minh raèng 333 333333 36( ) ( )( ) ab bc ca a b c ab bc ca + + ≥ + + ++ Lôøi giaûi. Ñaët 3 33 33 3333 ( , , ) 36( ) ( )( ) f abc ab bc ca a b c ab bc ca = + + - + + ++ Khoâng maát tính toång quaùt, giaû söû 0 abc ≥ ≥≥ Khi ñoù, ta coù ( ) ( ) 3 3 3 33 3 3 3 3 33 3 2 3 2 3 3 3 3 33 3 3 3 3 33 3 33 3333 33 3 33 3333 3 333 () ()0 () 330 () () ( )( ) 36( ) ( )( ) 36() () () ab bcca abc a b c abc a b c ab ac abc abc ab ac bc a b c ab c abc ab acbc ab bc ca abc ab acbc abc a b c a bc + + ≥+ + + ≥ ++≥ + = ++ + ≥ ++≥ ⇒ + + + ≥ + + ++ ⇒ ++ - ++ + +≥ ≥ +-+++ Do ñoù 3 333 (, , ) (, ,0) ( ,3 ,0) 36 (3 ) (3 ) ( (3 ) ) f abc f ab c faa aaa aaa ≥+ =- = - - - +- 22 9 (3 )( 3 2) ( (3 ) 1) 0 a aa a aa = - - + - +≥ Tìm tài liệu Toán ? Chuyện nh ỏ - www.toanmath.com210 (, ,) 0 (ñpcm) f abc ⇒≥ Ñaúng thöùc xaûy ra khi vaø chæ khi ( , , ) (2,1,0). abc = Baøi toaùn 3. (Phaïm Kim Huøng) Cho,,,0 abcd ≥ thoûa maõn 4 a bcd + + += . Chöùng minh raèng 4 4 4 4 3 3 33 (1 )(1 )(1 )(1 ) (1 )(1 )(1 )(1 ) ab c d a bcd + + ++ ≥+ ++ + Lôøi giaûi. + Caùch 1. Khoâng maát tính toång quaùt giaû söû 0 abcd ≥ ≥ ≥≥ * Tröôøng hôïp 1. 11 3 10 1,0 ,0. 23 a bcd b cd ≥⇒+ + ≤ ⇒ ≤ ≤ ≤ ≤ ≤ ≤ 4 4 4 4 3 3 33 33 4 33 (1 )(1 )(1 )(1 ) (1 )(1 )(1 )(1 ) 11 (1 ) (1 )(1 1 ) 1 1 23 a b c d abcd aa ⇒ + + + +-+ + + +≥ ⎛ ⎞⎛⎞ ⎛ ⎞ ⎛⎞ ≥+-+ + ++ ⎜ ⎟⎜⎟ ⎜ ⎟ ⎜⎟ ⎜ ⎟⎜⎟ ⎝ ⎠ ⎝⎠ ⎝ ⎠⎝⎠ 43 33 3 4 44 4 33 3 3 74 . 33 74 3. 33 2 .( 2)0 3 (1 )(1 )(1 )(1 ) (1 )(1 )(1 )(1 ) aa aa a a b c d abcd = -- ≥ -- = -> ⇒+ + ++ ≥+ ++ + * Tröôøng hôïp 2.30 abcd ≥ ≥ ≥ ≥≥ Ta coù baát ñaúng thöùc caàn chöùng minh töông ñöông vôùi ( ) 43 (1 ) 2(1 )0 2ln ln cyc aa + - +≥ ∑ Xeùt haøm soá 43 () (1 )2(1 ) 1 03 2ln ln vôùi fx x xxx = + - + - + ≤≤ Ta coù 3 2 654 2 / 4 3 43 8 6 ( 1)( 7 1) ()1 1 1 ( 1)( 1) x x x xxx xx fx x x xx - - + + + ++ = - -= + + ++ Tìm tài liệu Toán ? Chuyện nh ỏ - www.toanmath.com211 { } / 0 // 0 ( ) 0 (2,3). () () ( ) min (1), (3) min 0,2( Deã thaáy chæ coù 2 nghieäm döông phaân bieät laø 1 vaø Qua 1 thì ñoåi daáu töø aâm sang döông,qua thì ñoåi daáu töø döông sang aâm neân ln41 ln fxx fx x fx fx ff =∈ ≥ =- { } [ ] 1) 0 0,3 14 x - = ∀∈ [ ] 43 2 (1 ) 2 (1 ) 1 0,3 ln ln x x xx ⇒ + - + ≥-∀∈ ( ) 43 (1 ) 2 (1 ) ( 1) 0 2ln ln cyc cyc a aa ⇒ + - + ≥ -= ∑∑ ñpcm. ⇒ Ñaúng thöùc xaûy ra khi vaø chæ khi 1. abcd = = == * Nhaän xeùt. Baèng caùch laøm hoaøn toaøn töông töï, ta coù keát quaû sau Cho,,,0 abcd ≥ thoûa maõn 4 a bcd + + += . Khi ñoù, ta coù 1 1 11 (1 )(1 )(1 )(1 ) (1 )(1 )(1 )(1 ) 2. k k k k k k kk abc d a b c d k + + ++ + + + + ≥ + + + + ∀≥ + Caùch 2. Ta seõ chöùng minh baát ñaúng thöùc ñaõ baèng phöông phaùp phaûn chöùng. Giaû söû ngöôïc laïi toàn taïi boán soá khoâng aâm(,,,) abcd thoûa 4 a bcd + + += sao cho 4 4 4 4 3 3 33 (1 )(1 )(1 )(1 ) (1 )(1 )(1 )(1 ) ab c d a bcd + + ++ <+ ++ + Khoâng maát tính toång quaùt, ta coù theå giaû söû . abcd ≤≤≤ Ñaët (1 )(1 )(1 )(1 ) k k kk k F a bcd =+ + ++ . Theá thì theo baát ñaúng thöùc Bunhiacopxki, ta coù 2 22 42 3 3 1 2 20 1 . , . , . (1) FF F FF F FF F ≥ ≥≥ Theo giaû thieát phaûn chöùng thì 43 (2) FF < Töø (1) vaø (2), ta suy ra ñöôïc 432 1 0 16 (3) FF FF F < << <= Tìm tài liệu Toán ? Chuyện nh ỏ - www.toanmath.com212 Töø (3), ta coù 2. d < Ñeå daãn tôùi maâu thuaãn vôùi (3), ta seõ chöùng minh 31 (4) FF ≥ Thaät vaäy 2 2 22 2 2 22 4 2 2 22 22 (4) (1 )(1 )(1 )(1 ) 1 3 (2 1) 3 (2 1) 3 (2 1) 3 (2 1) 1 4 4 4 4 4 4 44 (2 1) (2 1) (2 1) (2 1) 4 1 1 11 3 3 3 33 (1 )(1 )(1 a a b b c c dd abcd abc d xy ⇔ -+ -+ -+ -+≥ ⎛ ⎞⎛ ⎞⎛ ⎞⎛ ⎞ - - -- ⇔+ + + +≥ ⎜ ⎟⎜ ⎟⎜ ⎟⎜ ⎟ ⎝ ⎠⎝ ⎠⎝ ⎠⎝ ⎠ ⎛ ⎞⎛ ⎞⎛ ⎞⎛ ⎞ - - -- ⎛⎞ ⇔+ + + +≥ ⎜ ⎟⎜ ⎟⎜ ⎟⎜ ⎟ ⎜⎟ ⎝⎠ ⎝ ⎠⎝ ⎠⎝ ⎠⎝ ⎠ ⇔+ ++ 4 2 22 )(1 ) 1 (5) 4 x y zt zt ⎛⎞ + ++ ⎛⎞ + ≥+⎜⎟ ⎜⎟ ⎜⎟ ⎝⎠ ⎝⎠ Trong ñoù 2 1 2 1 2 1 21 , ,, 3 3 33 a b cd x y zt - --- = = == Töø ñoù, xeùt baát ñaúng thöùc 2 2 22 (1 )(1 ) 1 (6) 2 AB AB ⎛⎞ + ⎛⎞ + + ≥+⎜⎟ ⎜⎟ ⎜⎟ ⎝⎠ ⎝⎠ 2 22 1 .( )(8 6 )0 8 A B A ABB ⇔ - - - -≥ Ta thaáy neáu 2 AB +≤ thì baát ñaúng thöùc treân ñuùng. Töø 2, abcd ≤ ≤ ≤< ta deã daøng chöùng minh ñöôïc 2 . 2 xt yz +< ⎧ ⎨ +< ⎩ Do ñoù, theo (6), ta coù 2 2 22 2 2 22 (1 )(1 )1 2 (1 )(1 ) 1 2 xt xt yz yz ⎛⎞ + ⎛⎞ + + ≥+⎜⎟ ⎜⎟ ⎜⎟ ⎝⎠ ⎝⎠ ⎛⎞ + ⎛⎞ + + ≥+⎜⎟ ⎜⎟ ⎜⎟ ⎝⎠ ⎝⎠ Tìm tài liệu Toán ? Chuyện nh ỏ - www.toanmath.com213 2 22 2 2 22 (1 )(1 )(1 )(1 ) 1 1 22 x t yz x y zt ⎡⎤ ⎛ ⎞⎛⎞ ++ ⎛ ⎞ ⎛⎞ ⇒++++≥++ ⎢⎥ ⎜ ⎟⎜⎟ ⎜ ⎟ ⎜⎟ ⎜ ⎟⎜⎟ ⎝ ⎠ ⎝⎠ ⎢⎥ ⎝ ⎠⎝⎠ ⎣⎦ Töø 2 2 xt yz +< ⎧ ⎨ +< ⎩ ta coù 2 22 x t yz ++ +< . Do ñoù, theo (6), ta laïi coù 2 2 22 1 11 2 24 x t yz x yz t ⎛ ⎞⎛ ⎞⎛⎞ + + + ++ ⎛⎞ ⎛⎞ ⎛⎞ + + ≥+ ⎜ ⎟⎜ ⎟⎜⎟ ⎜⎟ ⎜⎟ ⎜⎟ ⎜ ⎟⎜ ⎟⎜⎟ ⎝⎠ ⎝⎠ ⎝⎠ ⎝ ⎠⎝ ⎠⎝⎠ Do ñoù 4 2 2 2 22 (1 )(1 )(1 )(1 ) 1 4 x y zt x y zt ⎛⎞ + ++ ⎛⎞ + + + + ≥+⎜⎟ ⎜⎟ ⎜⎟ ⎝⎠ ⎝⎠ ⇒ (5) ñuùng ⇒ (4) ñuùng. Töø ñaây daãn ñeán maâu thuaãn. Vaäy ta phaûi coù 4 4 4 4 3 3 33 (1 )(1 )(1 )(1 ) (1 )(1 )(1 )(1 ) (ñpcm) ab c d a bcd + +++ ≥+ + + + Ñaúng thöùc xaûy ra khi vaø chæ khi 1. abcd = = == * Ghi chuù. Ngoaøi 2 caùch chöùng minh treân, ta coøn coù moät caùch chöùng minh nöõa laø chöùng minh baát ñaúng thöùc maïnh hôn nhö sau Cho,,,0 abcd ≥ thoûa maõn 4 a bcd + + += . Khi ñoù, ta coù 2 2 22 (1 )(1 )(1 )(1 ) (1 )(1 )(1 )(1 ) a b c d abcd + + + + ≥ + + ++ Chöùng minh. Ta coù baát ñaúng thöùc caàn chöùng minh töông ñöông vôùi 2 2 22 ( , , , ) (1 )(1 )(1 )(1 ) (1 )(1 )(1 )(1 ) 0 f abcd a b c d a b c d = + + + + - + + + +≥ Ta coù Nhaän xeùt sau Nhaän xeùt. Neáu 2 vaø thì a b a xb + ≤ ≥≥ Tìm tài liệu Toán ? Chuyện nh ỏ - www.toanmath.com214 (, , , ) (, ,, ) f abcd f xa b xcd ≥ +- Thaät vaäy 2 22 (, ,, ) (, ,, ) ( )( )(( 1)( 1) ( 1)( 1)( 2)) f abcd f xa b xcd a x x b c d c d ab x ax bx - +-= = - - + + - + + -+ +- Töø ñaây, söû duïng giaû thieát, ta deã daøng chöùng minh ñöôïc (, , , ) (, ,, ) f abcd f xa b xcd ≥ +- Nhaän xeùt ñöôïc chöùng minh. Trôû laïi baøi toaùn cuûa ta Khoâng maát tính toång quaùt, ta coù theå giaû söûabc d ≤≤≤ vaø ñaët 3 abc x ++ = theá thì ta coù 2. vaø a c c xa + ≤ ≥≥ Do ñoù, theo Nhaän xeùt treân, ta coù ( ,, , ) ( ,, , ) (1) f abcd f a c xbxd ≥ +- Chuù yù raèng () 3 a c x bx x + - ++ = neân neáu min{ , , } max{,,} x xba c x x xba cx = +- ⎡ ⎢ = +- ⎣ thì ta coùxb == a cx = +- neân ( , , ,) (, , ,) f a c xbxd f xxxd +-= Giaû söû ngöôïc laïi, khi ñoù coù 2 tröôøng hôïp xaûy ra (2) (3) b x a cx b x a cx < < +- > > +- Laïi söû duïng Nhaän xeùt, ta ñöôïc (2) ( , ,, ) ( , 2,, ) (,,, ) (3) ( , , ,) ( 2, , ,) (, , ,) f a c xbxd f xa b c xxd f xxxd f a c xbxd f a b c xxxd f xxxd ⇒ +- ≥ + +- = ⇒ +- ≥ + +- = Toùm laïi, trong moïi tröôøng hôïp, ta luoân coù ( , , ,) (, , ,) (, ,,) (, , ,) (4) f a c xbxd f xxxd f abcd f xxxd +-≥ ⇒≥ Maët khaùc, ta coù (,,, ) 444 ,,, 333 f xxxd ddd fd = --- ⎛⎞ = ⎜⎟ ⎝⎠ Tìm tài liệu Toán ? Chuyện nh ỏ - www.toanmath.com215 26 5 4 32 1 .( 1) ( 22 223 1268 4210 7564 6364) 729 0 d d d d ddd = - - + - + -+ ≥ Neân töø (4), ta suy ra ñöôïc (,,,)0 ñpcm. f abcd ≥ ⇒ Baøi toaùn 4. (Phaïm Kim Huøng) 12 ,..., Cho , laø caùc soá thöïc döông thoûa maõn n xxx 1 1 i=1 nn i i i x x = = ∑∑ Chöùng minh raèng 2 1 1 1 1 n i i xn = ≤ +- ∑ Lôøi giaûi. Neáu 1,2 nn == thì baát ñaúng thöùc ñaõ cho trôû thaønh ñaúng thöùc. Xeùt 3 n ≥ Ñaët 11 11 (1,). thì nn ii ii ii y iny xy == === ∑∑ Khi ñoù, baát ñaúng thöùc caàn chöùng minh töông ñöông vôùi 2 2 1 2 1 1 ( 1)1 1 1 ( 1)1 n i i i n i i y ny ny = = ≤ -+ ⇔≥ -+ ∑ ∑ Giaû söû ngöôïc laïi 2 1 1 1. ( 1)1 n i i ny = < -+ ∑ Ñaët 2 1 ( 1,) ( 1)1 i i a in ny == -+ thì 1 0 1,,1 n ii i a i na = >∀=< ∑ vaø 1 1, ( 1) i i i a y in na - = ∀= - Tìm tài liệu Toán ? Chuyện nh ỏ - www.toanmath.com216 Ñaët 1 1,, 1 j n ji ii i a b i naa n ≠ = = ∀== - ∑ ∑ Ta coù 1 1 1 1 ( 1) 1, 1, ( 1) n ii i ij ii i ji ii ab a a a n b i n y in n aa =≠ - < ⇒ - > = - ∀= ⇒ = > ∀= - ∑∑ 11 nn i i ii i b y a == ⇒> ∑∑ Ta chöùng minh 11 (*) nn ii iiii ba ab == ≥ ∑∑ Thaät vaäy, ta coù 11 ( 1) ( 1) 1. ( 1) nn i i ii ii ii ii b a n b na n ab anb == ⎛⎞ - -- - -= ⎜⎟ ⎜⎟ - ⎝⎠ ∑∑ 1 ( 1) () ji n ji i ii a na aaa ≠ = -- = - ∑ ∑ ( ) ( ) 1 2 , 1 2 1 11 () ( ) () ( ). ( )(). ( ) () 0 ( )() ( )(). ( ) () ij i jn j j ii ijk k ikj i jn iji j ii j j ij ij i jn iji j ii j j aa a a a aaa aaa aa aa aa aaa a aa aa a aa aa aa aa aaa a aa ≤<≤ ≠≠ ≤<≤ ≤<≤ ⎛⎞ ⎜⎟ = -- ⎜⎟ -- ⎝⎠ - = - - -+- ≥ - -- = - - -+- ∑ ∑ ∑ ∑ 1 11 0 (*) ñuùng. n ii i ii nn ii iiii ba ab ba ab = == ⎛⎞ ⇒ -≥ ⎜⎟ ⎜⎟ ⎝⎠ ⇒≥ ⇒ ∑ ∑∑ Tìm tài liệu Toán ? Chuyện nh ỏ - www.toanmath.com217 1 111 2 1 1 . 1 1 ( 1)1 Vaäy Ñieàu naøy traùi vôùi giaû thieát. Vaäy ta phaûi coù (ñpcm). nn nn ii i iiii i ii n i i ba y a by ny ==== = > ≥> ≥ -+ ∑∑∑∑ ∑ Baøi toaùn 5. (Phaïm Kim Huøng) Cho ,, abc laø caùc soá thöïc döông thoûa maõn 1 abc = . Chöùng minh raèng 2224 3336 (1 )(1 )(1 ) 8( ) (1 )(1 )(1 ) ( ) a b c abc a b c abc + + + ≤ ++ + + + ≤ ++ i. i 64 i. 81 Lôøi giaûi. i. Ñaët 4 2 22 ( , , ) 8( ) 81(1 )(1 )(1 ) fabc abc a b c = ++ - + ++ Tröôùc heát ta chöùng minh raèng 2 4 22 24 3 4 4 224 2 10 9 87 6 5 4 32 1 ,,0 11 82 81(1)10 8(2 1) 81(1)(1)0 ( 1) (47 94 21 120 99 78 87 96 24 16 8) 0 (ñuùng) f xx x xx xx x x xx x x x xx xx x xx x ⎛⎞ ≥ ⎜⎟ ⎝⎠ ⎛ ⎞ ⎛⎞ ⇔ + - + +≥ ⎜ ⎟ ⎜⎟ ⎝ ⎠ ⎝⎠ ⇔ + - + +≥ ⇔- + - ++ +++ + + + +≥ Vaäy 2 1 , , 0 (*) f xx x ⎛⎞ ≥ ⎜⎟ ⎝⎠ Tieáp theo, khoâng maát tính toång quaùt giaû söûabc ≥≥ . Ta chöùng minh ( ) (,,) ,, f a b c f ab ab c ≥ ( ) ( ) ( ) ( ) 4 2 22 4 22 2 22 2 22 8( ) (1 )(1 )(1 ) 8 2 (1 ) (1 ) 8 ()22 81( ) (1 ) 81 81 abc abc ab c ab c a b abc c ab a b c abc ⇔ ++ - + + +≥ ≥ + - ++ ⎛ ⎞⎛⎞ ⇔ - ++ + + + +≥ ⎜ ⎟⎜⎟ ⎝ ⎠⎝⎠ ≥ -+ Tìm tài liệu Toán ? Chuyện nh ỏ - www.toanmath.com218 ( ) ( ) ( ) ( ) ( ) ( ) ( ) 22 2 2 2 2 2 22 2 2 8()22 81 (1 )0 8 2 22 81 (1 ) 0 (**) abc c ab a bc a bc ab c ab c ab ab c a bc ⎛ ⎞⎛⎞ ⇔ ++ + + + +- ⎜ ⎟⎜⎟ ⎝ ⎠⎝⎠ - + +≥ ⎛⎞ ⎛ ⎞ ⎛⎞ ⇔ ++- ++ ++ - ⎜ ⎟ ⎜⎟ ⎜⎟ ⎝ ⎠ ⎝⎠ ⎝⎠ - + +≥ Ñaët ( ) 2 4 44 t ab t abc = + ⇒ ≥ ≥≥ Xeùt haøm soá ( ) ( ) 22 2 ()8 2 2 ( 2)81(1) gt tcab c ab t ct c ⎛⎞ = +- ++ + -+ ⎜⎟ ⎝⎠ ()0 Ta caàn chöùng minh Ta coù gt ≥ ( ) ( ) ( ) ( ) ( ) ( )( ) ( ) ( ) 22 /2 22 2 2 2 2 ( ) 8 2 2 16( 2 ) 2 81(1 ) 84 22 16 4 2 4 2 81(1 ) 3 16 2 27(1 ) 3 64 27 64 11 0 g t t c ab c ab t c t c ab c ab c ab c ab ab c ab c ab c cabc ab c ab c ⎛⎞ = +- + + + + +- - + ⎜⎟ ⎝⎠ ⎛⎞ ≥ +- +++ ⎜⎟ ⎝⎠ + + +- -+ ⎛⎞ = + -+ ⎜⎟ ⎝⎠ = -+ -≥ ( ) ( ) 3 2 3 24 9 7 63 6 () ( ) 4 4 8 2 81(1 ) 11 4 8 2 81 1 ( 1) 4(64 81 96 33 1) 01 ñoàng bieán. gt g t f ab c ab c ab t t t ab tt t t tt t t ⇒ ⎛⎞ ⇒ ≥ = + -+ ⎜⎟ ⎝⎠ ⎛⎞ ⎛ ⎞ ⎛⎞ = + - + =≥ ⎜⎟ ⎜ ⎟ ⎜⎟ ⎜⎟ ⎝ ⎠ ⎝⎠ ⎝⎠ - + -+ = ≥ ∀≥ ( ) ()0 (**) 1 (,,) ,, ,, 0 ñuùng. (do (*)) gt f a b c f ab ab c f ab ab ab ⇒≥ ⇒ ⎛⎞ ⇒ ≥ =≥ ⎜⎟ ⎝⎠ Tìm tài liệu Toán ? Chuyện nh ỏ - www.toanmath.com219 ñpcm. ⇒ ii. Tröôùc heát xin ñöôïc nhaéc laïi khoâng chöùng minh keát quaû sau Cho caùc soá thöïc döông ,, abc . Khi ñoù, toàn taïi caùc soá thöïc 0 0 11 , ,, x y xy 011 ( , , 0) x xy ≥ sao cho 0 0 11 22 000 111 22 0 0 11 22 22 x y x y abc x x y x x y ab bc ca xy abc xy + = + = ++ + = + = ++ ≤≤ Ngoaøi ra 2 0 2 0 ( )4( )0 ( )4( )0 Neáu thì Neáu thì abc abbccay abc abbccay +++≥++≤ +++≤++≥ Trôû laïi baøi toaùn cuûa ta Ta coù baát ñaúng thöùc caàn chöùng minh töông ñöông vôùi 3 33 33 3 6 641 () cyc cyc a ab abc a b c ⎛⎞ + + + ≤ ++ ⎜⎟ ⎝⎠ ∑∑ 3336 642 () cyc cyc a ab abc ⎛⎞ ⇔ + + ≤ ++ ⎜⎟ ⎝⎠ ∑∑ 222 3336 642 () cyc cyc abc abc a ab abc ⎛⎞ ⇔ + + ≤ ++ ⎜⎟ ⎝⎠ ∑∑ Khoâng maát tính toång quaùt giaû söû 1 abc ++= . Ñaët , q ab bc ca r abc = + += . Khi ñoù, baát ñaúng thöùc caàn chöùng minh töông ñöông vôùi 2 32 1 (13 3) 33 64 2r r q r q qrr + - + + -+≤ 23 1 ()8(16)0 64 fr r qrq ⇔ = + - + -≤ Ta coù / // () 16 16 () 160 fr rq fr = +- => Tìm tài liệu Toán ? Chuyện nh ỏ - www.toanmath.com220 () laø haøm loõm. fr ⇒ * Tröôøng hôïp 1. 00 1 4 1 00 2 q yx ≥ ⇒ ≥⇒≤ ≤ { } 22 00 11 () ( ),() max f r f xy f xy ⇒≤ Ta coù 2 00 42 2 2 23 0 0 0 00 0 0 0 00 5 4 32 0 0 0 0 00 0 () 1 8 (16( 2)) ( 2) 64 (2 1)(1024 368 264 60 2 1) 1 0) 642 (do 0 f xy xy x xy xy x xy x x x x xx x = = +- + ++- - - + - ++ = ≤ ≤≤ 2 11 ( )0 Töông töï, ta coùf xy ≤ ( ) 0 (1) fr ⇒≤ * Tröôøng hôïp 2.41 q ≤ { } 2 11 () (0),() max f r f f xy ⇒≤ 2 11 3 3 ( )0 1 11 (0) 0 64 4 64 Theo treân, ta coù Ta laïi coù f xy fq ≤ ⎛⎞ = - ≤ -= ⎜⎟ ⎝⎠ ( ) 0 (2) fr ⇒≤ Töø (1) vaø (2) ta suy ra ñpcm. Baøi toaùn 6. (Greece 2002) Cho,,0 abc > thoûa 222 1 abc ++= . Chöùng minh raèng ( ) 2 2 22 3 . 1 1 14 a bc a a b b cc bca + + ≥ ++ + ++ Lôøi giaûi. AÙp duïng baát ñaúng Bunhiacopxki, ta coù Tìm tài liệu Toán ? Chuyện nh ỏ - www.toanmath.com221 ( ) ( ) ( ) 2 3 2 2 2 2 2 22 2 22 2 2 2 22 1 () 1 1 1.() 3 cyc cyc cyc cyc aabbcc aa b ab a a ab a a b b cc ab aabbcc abc ++ =≥ + ++ ++ = + ++ ≥ + ++ ∑∑ ∑ ∑ ( ) 2 1 1 3 aabbcc ++ = + ( ) 2 3 . 4 ñpcm. aabbcc = ++ ⇒ Ñaúng thöùc xaûy ra khi vaø chæ khi 1 . 3 abc = == Baøi toaùn 7. (Vasile Cirtoaje) a) Cho,,0 abc > . Chöùng minh raèng 1 1 19 () ( ) ( ) ( )2 ab bc ca aa b bb c cca ⎛⎞ ++ + +≥ ⎜⎟ + ++ ⎝⎠ b) Cho,,,0 abcd > . Chöùng minh raèng 111 1 ()8 ( ) ( ) ( ) () ab bc cd da aa b bb c cc d dda ⎛⎞ +++ + + +≥ ⎜⎟ + + ++ ⎝⎠ Lôøi giaûi. a) AÙp duïng baát ñaúng thöùc AM-GM, ta coù 3 1113 ( ) ( ) () ( )( )() aa b bb c cca abcabbcc a + +≥ + ++ + ++ Do ñoù, ta chæ caàn chöùng minh Tìm tài liệu Toán ? Chuyện nh ỏ - www.toanmath.com222 3 3 2( ) 3 ( )( )( ) ( ) ( ) ()3 ( ).( ).() ab bc ca abc a b b c c a ab c bc a ca b ab c bc a ca b + + ≥ + ++ ⇔ + + ++ +≥ + + + Ñieàu naøy hieån nhieân ñuùng theo bñt AM-GM. ñpcm. ⇒ b) AÙp duïng baát ñaúng thöùc AM-GM, ta coù 11 28 ()() ()() ( )() aab ccd acab cd aca bcd +≥≥ + + + + ++ ++ Töông töï, ta coù 1 18 ()()()() bbc dd a bd ab c d +≥ + + + + ++ Do ñoù 1 1 11 ( ) ( ) ( ) () 88 ()()()() 8 ( )() 8 aab bbc cc d dd a ac ab cd bd ab cd a cbd ab bc cd da + + +≥ + + ++ ≥+ + + ++ + + ++ = ++ = + ++ Suy ra 111 1 ()8 ( ) ( ) ( ) () (ñpcm) ab bc cd da aa b bb c cc d dda ⎛⎞ +++ + + +≥ ⎜⎟ + + ++ ⎝⎠ Baøi toaùn 8. (Vasile Cirtoaje) Cho,,0 abc > thoûa 1 abc ≥ . Chöùng minh raèng ..1 abc bca a bc ≥ Lôøi giaûi. Do ,,0 abc > vaø 1 abc ≥ neân ñaët / // ,, a ka b kb c kc = == vôùi /// 1,,,0 k abc ≥> vaø /// 1 abc = . Khi ñoù, baát ñaúngthöùc caàn chöùng minh töông ñöông vôùi Tìm tài liệu Toán ? Chuyện nh ỏ - www.toanmath.com223 /// / / / / / / /// / // . . .1 abc abc bca b ca k a bc ++ ≥ Do 1 k ≥ neân ta chæ caàn chöùng minh /// /// / // . .1 abc b ca abc ≥ Do ñoù, khoâng maát tính toång quaùt coù theå giaû söû 1 abc = . Khi ñoù, baát ñaúng thöùc caàn chöùng minh töông ñöông vôùi ln ln ln 0 a ab bc c b ca ++≥ Coù 2 tröôøng hôïp xaûy ra * Tröôøng hôïp 1. ln ln ln abc a b c ≥≥⇒ ≥≥ + Tröôøng hôïp 1.1.01 a bc b bca < ≤ ⇒ ≥≥ ⇒ Theo baát ñaúng thöùc Chebyshev, ta coù ln ln ln 1 . (ln ln ln ) 0 3 a ab bc c ab c abc b c a bca ⎛⎞ + + ≥ ++ + += ⎜⎟ ⎝⎠ + Tröôøng hôïp 1.2. 1 ln0 bb ≥⇒≥ Ta coù baát ñaúng thöùc caàn chöùng minh töông ñöông vôùi ln ln ln ln ln ln ( )ln ( )ln ( )ln 0 ln ln ln ln () ()0 ( )( ln ln ) ( )( ln ln ) 0 a ab bc c abc b ca ababc bcac b ca a c bc ab bc ba ca ab a ab c bc a b c c ab ac ++≥ + + - -- ⇔ + +≥ ⎛ ⎞ ⎛⎞ ⇔- - +- -≥ ⎜ ⎟ ⎜⎟ ⎝ ⎠ ⎝⎠ - - -- ⇔ +≥ Chuù yù raèng ln0 1 11 ln ln0 vaø neân vaø c abc abc c ab ab ≤ ⎧ ≥≥ = ≤ ≥≥⇒ ⎨ ≥≥ ⎩ ln ln0 ln ln0 aabc abcc -≥ ⎧ ⇒ ⎨ -≥ ⎩ Tìm tài liệu Toán ? Chuyện nh ỏ - www.toanmath.com224 ( )( ln ln ) ( )( ln ln ) 0 a ba a bc b cab cc ab ac - - -- ⇒ +≥ * Tröôøng hôïp 2. 10 abc c a ≤ ≤⇒≥ ≥ > Theo treân, ta coù baát ñaúng thöùc caàn chöùng minh töông ñöông vôùi ( )( ln ln ) ( )( ln ln ) 0 ( )( ln ln ) ( )( ln ln ) 0 ab a ab c bc a bc c ab ac bab ca a c bc ca b ab ac - - -- +≥ - - -- ⇔ +≥ Do ln ln ln ln 0. neân vaø abc abc c ≤≤ ≤≤≥ ln ln ln ln (ln ln ) 0 ln ln ln ln (ln ln ) 0 ( )( ln ln ) ( )( ln ln ) 0 b c aaac aaa c a cc abac aba c b b abc a a c bc c ab ab ac -≥ -= - ≥ ⎧ ⇒ ⎨ - ≥ - = -≥ ⎩ - - -- ⇒ +≥ Toùm laïi, ta luoân coù ln ln ln 0 a ab bc c b ca ++≥ (ñpcm) Ñaúng thöùc xaûy ra khi vaø chæ khi 1 abc = == . Baøi toaùn 9. (Phaïm Kim Huøng) Cho 12 , ,..., 0 n aaa > thoûa 12 ...1 n aaa = . chöùng minh raèng vôùi moïi 0 k > thì 12 1 11 ... min 1, (1 ) (1 ) (1 ) 2 k k kk n n a aa ⎧⎫ + ++≥ ⎨⎬ + ++ ⎩⎭ Lôøi giaûi. + Caùch 1. Ta coù Boå ñeàø sau Boå ñeà. 12 , ,..., n xxx laø n soá thöïc döông thoûa maõn 12 12 ... ( , ) 1, ... i) ii) iii) n i n xxx x in x x xC ≤ ≤≤ ∈ -∞+∞ ∀= + + += Tìm tài liệu Toán ? Chuyện nh ỏ - www.toanmath.com225 vaø f laø moät haøm treân( ,) -∞ +∞ thoûa maõn f loài treân( ,] c -∞ vaø loõm treân[,) c +∞ Ñaët 12 ( ) () ... () n F fx fx fx = + ++ Khi ñoù, 1 23 ... ñaït min khi n F xxxx ≤ = == . Chöùng minh. Giaû söû 12 , ,..., ( , ] i xxxc ∈ -∞ , do f loài treân( ,] c -∞ neân 1 2 12 ( ) ( ) ... ( ) ( 1) () ( ... ( 1)) ii fx fx fx i f c f x x x i c + ++ ≥ - + + ++ - - Maët khaùc do f loõm treân[,) c +∞ neân 12 12 ( 1) ... ( 1) ( ) ( ) ( ) ... ( ) ( 1) 1 i in i in icxxx i f c fx fx fx n f n ++ ++ - + + ++ ⎛⎞ - + + + + ≥- ⎜⎟ - ⎝⎠ Do ñoù 12 12 1 ( 1) ... ( ) ( 1) ( ... ( 1) ) 1 n i in ki k icxxx F fx n f fx x x i c n ++ = - + + ++ ⎛⎞ = ≥- + ++ + - - ⎜⎟ - ⎝⎠ ∑ Boå ñeà ñöôïc chöùng minh. Trôû laïi baøi toaùn cuûa ta Neáu 1 n = thì baát ñaúng thöùc ñaõ cho hieån nhieân ñuùng. Neáu 2 n = + Neáu01 k << thì ta coù 1 2 1 21 1 1 1 1 1 11 1 1 (1 )(1 )1 11 1 kk a a a aa a + ≥+ =+ = + + + ++ + + Neáu 1 k≥ thì ta coù 1 1 1 21 1 1 11 ( (1 ) (1 ) (1)2 theo baát ñaúng thöùc Holder) k k k kk a a aa - + + =≥ + ++ Xeùt 3 n ≥ Ta chöùng minh baát ñaúng thöùc ñuùng cho giaù trò tôùi haïn 1 2 2 log k n kn = ⇔= . Do 3 n ≥ neân 11 nk - >> . Khi ñoù Tìm tài liệu Toán ? Chuyện nh ỏ - www.toanmath.com226 , ta coù mk +∀≥ 11 11 1 1 11 .. (1 ) (1 ) 22 m m nn k k m m k m km ii ii kk nn aa nn -- == ⎛⎞ ⎛⎞ ≥ ≥= ⎜⎟ ⎜⎟ ++ ⎝⎠ ⎝⎠ ∑∑ , ta coù mk +∀≤ 1 11 1 11 11 (1) (1) (1 ) k n nn m m km i ii i ii a aa = == ⎛⎞ ≥ ≥⇒≥ ⎜⎟ + ++ ⎝⎠ ∑ ∑∑ Khoâng maát tính toång quaùt giaû söû 12 0 ... n aaa < ≤ ≤≤ . Ñaët 12 1 122 1 2 12 ... ln , ln ,..., ln ... 0 ... 1) thì (do n nn nn xxx x ax a xa x x x aaa ≤ ≤≤ ⎧ = == ⎨ +++== ⎩ Xeùt haøm soá 1 () ( 1) xk fx e = + Ta coù // 2 // .( 1) () ( 1) ( ) 0 ln xx xk ke ke fx e fx xk + - = + = ⇔ =- Töø ñoù, ta coù f loài treân( , ln] k -∞- vaø loõm treân[ln, ) k - +∞ ⇒ Theo Boå ñeà treân, ta coù 11 11 (1) (1) i nn kx k ii i P a e == == + + ∑∑ ñaït min khi 1 23 ... n xxxx ≤ = == ( 1) ( 1) 1 11 min min ( 0) ( 1) ( 1) 1 min ( 1) (1) ( 1) ( 1) t k ntk nk t k nk n Pt ee nx xe xx -- - - ⎧⎫ - ⇒≥ +≥ ⎨⎬ ++ ⎩⎭ ⎧⎫ - = + =≥ ⎨⎬ ++ ⎩⎭ Tieáp theo, ta seõ tìm min cuûa haøm soá ( 1) 1 1 ()1 ( 1) ( 1) vôùi nk k nk nx gxx xx - - - =+≥ ++ Tìm tài liệu Toán ? Chuyện nh ỏ - www.toanmath.com227 Ta coù (1)1 / 1 11 1 ( ) ( 1) ( 1) ( 1) nk n kk x gx nk xx -- - ++ ⎛⎞ =-- ⎜⎟ ++ ⎝⎠ / 1)1 1 11 ( ) 0 .( 1) ( 1) nk k nk gx x xx (-- + -+ =⇔ + =+ 1)1 1 1 .( 1) 1 (2) nk n k x xx (-- - + ⇔ + =+ Ñaët 1 1 1 k txt + = ⇒≥ . Khi ñoù, phöông trình (2) trô ûthaønh 1) 1 1 ( 1)( 1) (1)( 1) 1)1 .(1)1 10 n k k nk n k nk nk t tt t tt (- - + -+ - + (-- +=+ ⇔ - - += Xeùt haøm soá ( 1)( 1) 1) 1 ()1 n k nk nk ht t tt - + (-- = --+ vôùi 1 t ≥ Ta coù /(1)21 ( ) .(( 1)( 1) ( 1) 1) nk nk h t t n k t nkt n k --+ = - + - - -+ / ()0 ht =⇔ 1 ( 1)( 1) ( 1) 1 0 nk n k t nkt nk + - + - - - += Xeùt tieáp haøm soá 1 () ( 1)(1) ( 1)1 1 vôùi nk mtn ktnkt nkt + =- + - --+≥ Ta coù /1 ( ) ( 1) (( 1) ) k nk mtnk tntk -- = + -- Chuù yù raèng 1 nk -> neân / ( ) ( 1) (( 1) ) 0 k mtnk tnk ≥ + - -> () mt ⇒ laø haøm ñoàng bieán treân[1,) +∞ Ta laïi coù (1) ( 1)( 1) ( 1) 1 (1 ) 0, lim ( ) t m n k nk n k n k mt →+∞ =- + - - - += -< =+∞ Neân phöông trình ()0 mt = coù nghieäm duy nhaát 0 1 t > ⇒ Phöông trình / ()0 ht = coù nghieäm duy nhaát 0 1 t > Baûng bieán thieân cuûa () ht Caên cöù vaøo baûng bieán thieân, ta coù t / () ht 1 0 () ht _ + 0 t +∞ +∞ 0 Tìm tài liệu Toán ? Chuyện nh ỏ - www.toanmath.com228 10 ()01 coù 2 nghieäm phaân bieät laø 1 vaø ht tt = >> Do ñoù / ()0 gx = coù 2 nghieäm phaân bieät laø 1 vaø 1 1 1 k t + > . Baûng bieán thieân cuûa () gx Caên cöù vaøo baûng bieán thieân, ta suy ra { } ( ) min (1),lim ( ) 1 1 (3) x gx g gx x →∞ ≥ = ∀≥ Töø (1) vaø (3), ta suy ra ñpcm. + Caùch 2. Ñaët 1 () ( 1) k ft t = + Ta coù boå ñeà sau Boå ñeà. Neáu 0 vaø thì a b c d ad bc <≤≤≤= { } ( ) ( ) min ( ) ( ),1 fa f d f b fc + ≥+ Chöùng minh. Ñaët m ad bc == vaø()() (1)1 k k mm g t f mt f mt tt - - ⎛⎞ ⎛⎞ = + = + ++ ⎜⎟ ⎜⎟ ⎝⎠ ⎝⎠ vôùi moïi soá döông t . Ñaët 1 2 12 , . 1. Ta coù cd t t tt mm = = ≤≤ Ñeå chöùng minh Boå ñeà, ta caàn chöùng minh { } 21 ( ) min ( ),1 gt gt ≥ Deã thaáy lim ( ) 1. t gt →+∞ = Xeùt tính ñôn ñieäu cuûa haøm g treân khoaûng [ ) 1,+∞ , ta coù 1 1 x / () gx 1 0 () gx _ + 1 1 k t + +∞ Tìm tài liệu Toán ? Chuyện nh ỏ - www.toanmath.com229 1 1 2 1 () ( 1) .1 k k m g t mk mt t t -- / -- ⎛⎞ ⎛⎞ = -+ ++ ⎜⎟ ⎜⎟ ⎜⎟ ⎝⎠ ⎝⎠ 1 / 12 21 11 ()0 ( 1) .1 10 k k k kk m gt mtt t t mt mt + + - ++ ⎛⎞ >⇔ + >+ ⎜⎟ ⎝⎠ ⇔ - + -< Xeùt haøm soá 21 11 () 11 vôùi . k kk h t t mt mt t - ++ = -+ -≥ Ta coù 12 / 11 / 13 // 1 2 (1)0 21 ().. 11 21 (1) . 1 2(1) () . .() (1) kk kk k k h k h t t m mt kk k hm kk k h t t t mk k -- ++ + - + = - = -+ ++ ⎛⎞ =- ⎜⎟ + ⎝⎠ - =- + Tuøy thuoäc vaøo caùc giaù trò cuûa , vaø ta coù caùc tröôøng hôïp sau mk * Tröôøng hôïp 1. 1,1 km =≤ . Khi ñoù, ta coù()(1 )(1)01 ht mtt = - - ≥ ∀> , do ñoù 0 h ≥ treân khoaûng (1, ). +∞ * Tröôøng hôïp 2. 1,1 km => . Khi ñoù, ta coù()(1 )(1)01 ht mtt = - - < ∀> , do ñoù 0 h < treân khoaûng (1, ). +∞ * Tröôøng hôïp 3. 1 1, km k <≤ . Khi ñoù, ta coù /// 0 1, (1) 0 vì h th >∀>≥ vaø 1 t > , neân / 0 h > treân khoaûng (1, ) +∞ . Vì (1)0 h = neân 0 (1,) treân khoaûng h > +∞ . * Tröôøng hôïp 4. 1 1, km k <> . Khi ñoù, ta coù / // (1) 0 0 1, ) vaø treân ( h h mk << . Do ñoù suy ra / 0 1, ). treân ( h mk < Vì (1)0 h = neân ( ] 0 1, treân h mk < . Treân khoaûng ( ,) mk +∞ , ta coù // 0, ( ,) coù nghóa laø haøm loõm treân . h h mk > +∞ Vì ( )0 h mk < vaø Tìm tài liệu Toán ? Chuyện nh ỏ - www.toanmath.com230 lim () t ht →+∞ = +∞ neân toàn taïi duy nhaát moät soá thöïc 1 p > sao cho 0 h < treân (1, ) p vaø 0 h > treân ( , ). p +∞ Trong caùc tröôøng hôïp noùi treân + Neáu 2 ()0 ht ≥ thì 0 h ≥ treân 2 (,) t +∞ , töùc laø / 0. g ≤ Suy ra haøm g ñôn ñieäu giaûm treân khoaûng [ ) 2 , t +∞ vaø 2 ( ) lim()1. t gt gt →+∞ ≥= + Neáu 2 ()0 ht ≤ thì 0 h ≥ treân khoaûng 2 (1, ), t hay / 0. g ≥ Vaäy g laø haøm ñôn ñieäu taêng, suy ra 12 ( ) ( ). gt gt ≤ Boå ñeà ñöôïc chöùng minh hoaøn toaøn. Trôû laïi baøi toaùn cuûa ta Ta seõ chöùng minh baát ñaúng thöùc ñaõ cho baèng quy naïp theo n . Neáu 1 n = thì baát ñaúng thöùc ñaõ cho trôû thaønh ñaúng thöùc. Xeùt 2. n ≥ Goïi m laø trung bình nhaân cuûa 12 , ,..., n aaa thì ta coù 1 m = . Ta coù, baát ñaúng thöùc caàn chöùng minh töông ñöông vôùi { } 12 ( ) ( ) ... ( ) min ( ),1 n fa fa f a nfm + ++≥ Neáu 2 n = thì ta coù { } 1 1 min 2 ( ),1 min ,1 2 k fm - ⎧⎫ = ⎨⎬ ⎩⎭ + Neáu01 k << thì ta coù 1 2 1 21 1 1 1 11 11 1 1 (1 )(1 )1 1 1 1 kk a a a aa a + ≥+ =+ = + + + ++ + + Neáu 1 k≥ thì ta coù 1 1 1 21 1 1 11 ( (1 ) (1 ) ( 1) 2 theo baát ñaúng thöùc Holder) k kkkk a a aa - + + =≥ + ++ Vaäy khaúng ñònh ñuùng khi 2 n = . Tìm tài liệu Toán ? Chuyện nh ỏ - www.toanmath.com231 Giaû söû khaúng ñònh ñuùng cho soá caùc bieán beù hôn ( 2). nn ≥ Ta seõ chöùng minh khaúng ñònh ñuùng cho soá bieán baèng n. Deã thaáy raèng trong daõy 12 , ,..., n aaa luoân chöùa ít nhaát moät soá khoâng lôùn hôn m vaø ít nhaát moät soá khoâng nhoû hôn m . Khoâng maát tính toång quaùt, ta coù theå giaû söû 12 . a ma ≤≤ Kyù hieäu 12 12 12 min, , max, . aa aa x m xm mm ⎧ ⎫ ⎧⎫ == ⎨ ⎬ ⎨⎬ ⎩ ⎭ ⎩⎭ Khi ñoù, ta coù 1 12 2 1212 . vaø a x x a xx aa ≤≤≤= Töø ñaây, theo keát quaû cuûa boå ñeà treân, ta coù { } 12 1 2 12 ( ) ( ) min ( ) ( ),1 min ( ) ,1 aa fa fa f x f x fm f m ⎧⎫ ⎛⎞ + ≥ + =+ ⎨⎬ ⎜⎟ ⎝⎠ ⎩⎭ Trung bình nhaân cuûa 12 3 , ,..., n aa aa m cuõng baèng m vaø soá bieán laø 1 nn -< neân theo giaû thieát quy naïp, ta coù { } 12 3 ( ) ... ( ) min ( 1) ( ),1 n aa f fa f a n fm m ⎛⎞ + ++ ≥- ⎜⎟ ⎝⎠ Suy ra 12 12 3 ( ) () ... () min ( ) ,1 ( ) ... ( ) n n fa fa fa aa fm f fa fa m + ++≥ ⎧⎫ ⎛⎞ ≥ + + ++ ⎨⎬ ⎜⎟ ⎝⎠ ⎩⎭ { } { } { } { } 12 33 12 3 12 min ( ) ( ) ... ( ),1 ( ) ... ( ) min ( ) ( ) ... ( ),1 min ( ) min ( 1) ( ),1 ,1 min ( ),1 ( ) ( ) ... ( ) min ( ),1 khaúng ñònh ñuùng vôùi nn n n aa f m f fa f a fa fa m aa fm f fa fa m fm n fm nfm fa fa fa nfm ⎧⎫ ⎛⎞ = + +++ +++ ⎨⎬ ⎜⎟ ⎝⎠⎩⎭ ⎧⎫ ⎛⎞ ≥ + + ++ ⎨⎬ ⎜⎟ ⎝⎠ ⎩⎭ ≥ +- ≥ ⇒ + ++≥ ⇒ . soá bieán soá baèng n Tìm tài liệu Toán ? Chuyện nh ỏ - www.toanmath.com232 Theo nguyeân lyù quy naïp, ta suy ra khaúng ñònh ñuùng vôùi moïi . n Ñaây chính laø ñieàu ta caàn phaûi chöùng minh. Baøi toaùn 10. (Moldova 1999) Cho,,0 abc > . Chöùng minh raèng ( ) ( ) () ab bc ca a b c cc a aa b bbc c a ab bc + + ≥++ + + + + ++ Lôøi giaûi. Ñaët ,, a bc x yz b ca = == thì ta coù,,0 xyz > vaø 1 xyz = . Khi ñoù, ta coù ()1 1 11 1 1 cyc cyc cyc cyc cyc cyc b aby c c ccaz a a c caz a == ++ + == ++ + ∑ ∑∑ ∑∑∑ Do ñoù, baát ñaúng thöùc caàn chöùng minh töông ñöông vôùi 111 1 1 1 1 11 (1 )(1) 1 (1 )(1 )(1 ) 32 12 cyc cyc cyc cyc cyc cyc cyc x yz yz x x yz xy x y x yz x xy x y x xy + + ≥++ ++ + + ++ ++ ⇔≥ + + ++ ++ ⇔≥ + ++ ∑ ∑ ∑∑ ∑ ∑∑ Aùp duïng baát ñaúng thöùc Bunhiacopxki, ta coù 22 2 1 () cyc cyc cyc cyc cyc cyc cyc xx xx y x xy x xy x xy ⎛⎞ ⎛⎞ ⎜⎟ ⎜⎟ ⎝⎠ ⎝⎠ = ≥= + +++ ∑∑ ∑∑ ∑ ∑∑ Tìm tài liệu Toán ? Chuyện nh ỏ - www.toanmath.com233 Do ñoù, ñeå chöùng minh baát ñaúng thöùc ñaõ cho, ta chæ caàn chöùng minh 2 2 32 2 2 32 cyc cyc cyc cyc cyc cyc cyc cyc cyc cyc cyc cyc cyc cyc x x xy x xy x xy x x xy x xy x xy ⎛⎞ ++ ⎜⎟ ⎝⎠ ≥ + ++ ⎛⎞⎛ ⎞⎛ ⎞⎛⎞ ⇔ + + ≥ + ++ ⎜⎟⎜ ⎟⎜ ⎟⎜⎟ ⎝⎠⎝ ⎠⎝ ⎠⎝⎠ ∑ ∑∑ ∑ ∑ ∑∑ ∑ ∑∑ ∑∑ ∑∑ 322 3 33 cyc cyc cyc cyc cyc cyc cyc cyc x x xy x xy x xy xy ⎛ ⎞⎛ ⎞⎛⎞ ⎛ ⎞⎛⎞⎛⎞ ⇔+ ≥+++ ⎜ ⎟⎜ ⎟⎜⎟ ⎜ ⎟⎜⎟⎜⎟ ⎝ ⎠⎝ ⎠⎝⎠ ⎝ ⎠⎝⎠⎝⎠ ∑ ∑ ∑ ∑ ∑ ∑ ∑∑ Söû duïng baát ñaúng thöùc AM-GM vaø giaû thieát, ta deã daøng chöùng minh ñöôïc caùc baát ñaúng thöùc sau 2 3 3 3 cyc cyc cyc cyc x xy x xy ≥ ≥ ⎛ ⎞ ⎛⎞ ≥ ⎜ ⎟ ⎜⎟ ⎝ ⎠ ⎝⎠ ∑ ∑ ∑∑ Do ñoù, ta coù 32 3 2 2 3 3 9 (1) 9 (2) 9 cyc cyc cyc cyc cyc cyc cyc cyc cyc cyc cyc cyc cyc cyc cyc cyc x xx xxy x x xy x xy x x xy x x xy xy ⎛⎞ ⎛⎞⎛⎞ ⎛⎞⎛ ⎞ =≥ ⎜⎟ ⎜⎟⎜⎟ ⎜⎟⎜ ⎟ ⎝⎠ ⎝⎠⎝⎠ ⎝⎠⎝ ⎠ ⎛⎞⎛ ⎞⎛ ⎞ ⇒≥ ⎜⎟⎜ ⎟⎜ ⎟ ⎝⎠⎝ ⎠⎝ ⎠ ⎛⎞⎛ ⎞⎛⎞⎛⎞⎛ ⎞ =≥ ⎜⎟⎜ ⎟⎜⎟⎜⎟⎜ ⎟ ⎝⎠⎝ ⎠⎝⎠⎝⎠⎝ ⎠ ⎛ ⎞⎛⎞ ≥ ⎜ ⎟⎜⎟ ⎝ ⎠⎝⎠ ∑ ∑∑ ∑∑ ∑ ∑∑ ∑ ∑ ∑∑∑ ∑ ∑∑ 22 (3) 3 (4) cyc cyc cyc cyc x xy xy ⎛ ⎞⎛⎞ ⎛⎞ ≥ ⎜ ⎟⎜⎟ ⎜⎟ ⎝ ⎠⎝⎠ ⎝⎠ ∑ ∑ ∑∑ Tìm tài liệu Toán ? Chuyện nh ỏ - www.toanmath.com234 Coäng caùc baát ñaúng thöùc (1), (2), (3) vaø (4) veá theo veá, roài chia caû hai veá cho 3, ta thu ñöôïc 322 3 33 (ñpcm) cyc cyc cyc cyc cyc cyc cyc cyc x x xy x xy x xy xy ⎛ ⎞⎛ ⎞⎛⎞ ⎛ ⎞⎛⎞⎛⎞ + ≥+++ ⎜ ⎟⎜ ⎟⎜⎟ ⎜ ⎟⎜⎟⎜⎟ ⎝ ⎠⎝ ⎠⎝⎠ ⎝ ⎠⎝⎠⎝⎠ ∑ ∑ ∑ ∑ ∑ ∑ ∑∑ Ñaúng thöùc xaûy ra khi vaø chæ khi 1. x y z abc = == ⇔ == Baøi toaùn 11. Cho ,, abcR ∈ thoûa ( )( )( ) 0 a bb cc a - - -≠ . Chöùng minh raèng 2 2 2 2 22 2 22 (1 )(1 ) (1 )(1 ) (1 )(1 ) 1 ( ) ( ) () a bb cc a ab bc c a - - - - -- + + ≥- - -- Lôøi giaûi. Ta coù baát ñaúng thöùc caàn chöùng minh töông döông vôùi 22 2 2 (1 )(1) 12 () 1 2 (*) cyc cyc ab ab ab ab ⎛⎞ -- +≥ ⎜⎟ - ⎝⎠ - ⎛⎞ ⇔≥ ⎜⎟ - ⎝⎠ ∑ ∑ Ñaët 1 11 ,, ab bc ca x yz a b b c ca - -- = == - -- thì ta coù ( 1)( 1)( 1) ( 1)( 1)( 1) 1 x yz xyz xy yz zx + + + = - -- ⇒ ++=- Khi ñoù 2 22 2 (*) 2( ) ( )0 (ñuùng) ñpcm. x y z xy yz zx xyz ⇔ + + ≥- ++ ⇔ ++≥ ⇒ Tìm tài liệu Toán ? Chuyện nh ỏ - www.toanmath.com235 Baøi toaùn 12. Cho,,0 abc > . Chöùng minh raèng 3 3 3 3 33 0 2 22 c a ab bc ab bc ca - -- ++≥ + ++ Lôøi giaûi. Khoâng maát tính toång quaùt coù theå giaû söû { } ,, max a abc = Coù 2 tröôøng hôïp xaûy ra * Tröôøng hôïp 1.abc ≥≥ Khi ñoù, ta coù 2 20 2 20 ab bc a b ca + ≥ +> ⎧ ⎨ + ≥ +> ⎩ . Do ñoù 3 33 3 3 33 33 3 3 3 0 2 2 2 2 22 c a ab bc c a ab b c ab bc c a ab a b ab - -- - -- ++≥ ++= + + + + ++ * Tröôøng hôïp 2.acb ≥≥ Khi ñoù, ta coù 2 20 2 20 c a bc ab bc + ≥ +> ⎧ ⎨ + ≥ +> ⎩ . Do ñoù 3 33 3 3 33 33 3 3 3 0 2 2 2 2 22 c a ab bc c a ab b c ab bc c a bc bc bc - -- - -- ++≥ ++= ++ + +++ Toùm laïi, ta luoân coù 3 3 3 3 33 0 2 22 (ñpcm) c a ab bc ab bc ca - -- ++≥ + ++ Ñaúng thöùc xaûy ra khi vaø chæ khi . abc == * Nhaän xeùt. Baèng caùch laøm hoaøn toaøn töông töï, ta coù 0 ,,,0 2 22 n n n n nn c a a b bc abcn ab bc ca - -- + + ≥∀> + ++ Tìm tài liệu Toán ? Chuyện nh ỏ - www.toanmath.com236 Baøi toaùn 13. Cho,,0 abc > . Chöùng minh raèng 22 2 0 n nn cyc abc b bcc -- ≥ -+ ∑ 0 trong ñoù laø haèng soá cho tröôùc. n > Lôøi giaûi. Khoâng maát tính toång quaùt giaû söû 0 abc ≥≥> . Ta coù baát ñaúng thöùc caàn chöùng minh töông ñöông vôùi 2 22 22 2 2 22 222 2 11 () 1 12 ()0 nn nn ab b bc c a ac c a ab b bc b bc c a ac c a ab b ⎛⎞ - - -+ ⎜⎟ -+-+-+ ⎝⎠ ⎛⎞ +- + -≥ ⎜⎟ - + - + -+ ⎝⎠ Do 0 abc ≥≥> neân 2 222 2 222 0 0 a ab b b bc c a acc b bcc ⎧ - +≥ - +> ⎪ ⎨ -+ ≥ - +> ⎪ ⎩ 2 22 222 2 11 0 b bcc aac c aabb ⇒ - -≥ -+-+-+ Do ñoù, ta chæ caàn chöùng minh 2 22 222 2 22 22 222 2 2 22 1 12 0 1 1 11 ( )( )( ) ( )( )( ) b bc c a ac c a ab b b bc c a ab b a ab b a ac c ac a c b a ac c bc abcb bc c + -≥ - + - + -+ ⇔ - ≥- - + - + - + -+ ⇔ -+- - + ≥ --- - + + Neáuabc ≤+ thì ta coù ngay ñpcm. + Neáuabc >+ thì ta coù 2 222 0 0 0 ac bc a cb abc a acc b bcc ⎧ - ≥ -≥ ⎪ + - ≥ --> ⎨ ⎪ -+ ≥ - +> ⎩ 2 2 22 ( )( )( ) ( )( )( ) ñpcm. a c a c b a ac c bc abcb bc c ⇒- + - - + ≥ - - - -+ ⇒ Tìm tài liệu Toán ? Chuyện nh ỏ - www.toanmath.com237 Ñaúng thöùc xaûy ra khi vaø chæ khi . abc == Baøi toaùn 14. (Toaùn Hoïc Tuoåi Treû 2006) Cho 12 , ,..., n aaa laø caùc soá thöïc döông thoûa maõn 11 ( 1) 1, kk i ii a ii kn == ≤ + ∀= ∑∑ . Chöùng minh raèng 12 111 ... 1 n n a a an + + +≥ + Lôøi giaûi. Tröôùc heát, ta chöùng minh boå ñeà sau Boå ñeà. Cho 3, n nN ≥∈ . Khi ñoù, vôùi hai daõy soá thöïc ( ) ( ), vaø ta coù nn xy 1 1 1 1 11 ( ).. n n in ii i i j ni i i ji xy x x y xy - + = = == = -+ ∑ ∑ ∑∑ Chöùng minh. Ta coù 1 1 1 11 11 1 11 1 11 1 21 111 21 1 1 11 1 11 11 21 21 ( ).. . .. . . .. . .. n in i i j ni i ji n i n in i j i j ni ij i ji n i n i nn i j i j n i ni i j i j ii n i ni ij ij n i j ij x x y xy x y x y xy xy x y x y xy xy xy xy xy x y - + = == -- + == = == - -- + = = = = == - -- = = == - += = -+ = + - -+ =+-+ ∑ ∑∑ ∑∑ ∑∑∑ ∑∑ ∑∑ ∑∑ ∑ ∑ ∑∑ 1 11 11 11 2 11 1 11 2 1 . nn ii ii n ii i j j nn i jj n ii nn i n ii i y xy x y y xy xy xy xy xy - == -- = == - = = ⎛⎞ - ⎜⎟ ⎝⎠ ⎛⎞ = + -+ ⎜⎟ ⎝⎠ = ++ = ∑∑ ∑ ∑∑ ∑ ∑ Tìm tài liệu Toán ? Chuyện nh ỏ - www.toanmath.com238 1 1 1 1 11 ( ). .. Vaäy Boå ñeà ñöôïc chöùng minh. n n in ii i i j ni i i ji xy x x y xy - + = = == = -+ ∑ ∑ ∑∑ Trôû laïi baøi toaùn cuûa ta + Neáu 1 n = thì hieån nhieân ta coù 1 1 11 ( 2) 2 do giaû thieát a a ≥≤ (1) + Neáu 2 n = thì theo giaûthieát, ta coù 11 12 21 22 88 aa aa aa ≤≤ ⎧⎧ ⇒ ⎨⎨ + ≤ ≤- ⎩⎩ Do ñoù 1 2 11 1 1 11 8 aaaa + ≥+ - 11 2 11 2 11 8 (8) 8 8 8 (2) 44 8 4.24 aa aa aa = - = -+ = - - ++ ≥ + 2 3 = (2) + Neáu 3 n ≥ AÙp duïng baát ñaúng thöùc Bunhiacopxki, ta coù 2 2 2 1 2 11 2 2 22 22 11 1 1 ( 1) ( 1)11 . ( 1) ( 1) ( 1) ( 1) n nn i nn ii ii i i ii ii ii n aa an a ii ii ii = == == ⎛⎞ ⎛⎞ ⎜⎟ ⎜⎟ + + ⎝ ⎠⎝⎠ = ≥= + ⎛⎞ ⎜⎟ ++ + ⎝⎠ ∑ ∑∑ ∑∑ AÙp duïng boå ñeà vôùi 1 ; 1,2,..., , ( 1) ta coù i ii x yain ii = = ∀= + 22 1 ( 1) n i i a ii = = + ∑ Tìm tài liệu Toán ? Chuyện nh ỏ - www.toanmath.com239 1 2 2 2 2 22 1 11 1 2 2 2 2 22 1 11 1 11 .. ( 1) ( 1) ( 2) ( 1) 111 . ( 1) . ( 1) ( ( 1) ( 1) ( 2) ( 1) gt) n in ji i ji n in i ji aa ii i i nn j j ii i i i i nn - = == - = == ⎛⎞ = -+ ⎜⎟ ++++ ⎝⎠ ⎛⎞ ≤ - +++ ⎜⎟ ++++ ⎝⎠ ∑ ∑∑ ∑ ∑∑ 22 1 , ( 1) 1,2,..., , ( 1) Laïi aùp duïng boå ñeà vôùi ta coù ii x yii in ii = = + ∀= + 22 11 1 2 2 2 2 22 1 11 11 . ( 1) ( 1) ( 1) 111 . ( 1) . ( 1) ( 1) ( 1) ( 2) ( 1) nn ii n in i ji ii ii ii j j ii ii i i nn == - = == =+ ++ ⎛⎞ = - +++ ⎜⎟ ++++ ⎝⎠ ∑∑ ∑ ∑∑ 22 11 1 (1) (1) 1 Do ñoù nn i ii a n ii iin == ≤= + ++ ∑∑ Suy ra 22 22 1 22 1 1 11 .. ( 1) ( 1) 1 1 ( 1) n n i i i i n nn n a an nn n ii = = ≥ ≥= + ++ ⎛⎞ ⎜⎟ + + ⎝⎠ ∑ ∑ Vaäy 1 1 1 n i i n an = ≥ + ∑ (3) Töø (1), (2) vaø (3), ta suy ra ñpcm. ( 1) 1,2,..., . Ñaúng thöùc xaûy ra khi vaø chæ khi i aii in = + ∀= * Nhaän xeùt. Baèng caùch laøm hoaøn toaøn töông töï, ta coù keát quaû sau 12 11 ... ( ),() 1, laø hai daõy soá thöïc döông thoûa maõn . Khi ñoù, ta coù n kk nn ii ii bbb ab a b kn == ≤ ≤≤ ⎧ ⎪ ⎨ ≤ ∀= ⎪ ⎩ ∑∑ 11 11 nn ii ii ab == ≥ ∑∑ Tìm tài liệu Toán ? Chuyện nh ỏ - www.toanmath.com240 Baøi toaùn 15. (Phaïm Kim Huøng) Cho 4, n nN ≥∈ vaø 12 , ,..., 0 n aaa > thoûa 12 ... 1. n aaa = Chöùng minh raèng 1 2 12 11 13 ...3 ... nn n n aa aaaa + + + + ≥+ + ++ Lôøi giaûi. + Caùch 1. Ñaët 12 1 2 12 11 13 ( , ,..., ) ... ... n nn n faaa aa aaaa = ++ ++ + ++ . Ta caàn tìmmin. f Giaû söû vôùi boä soá 12 ( , ,..., ) n xxx thoaû 12 ...1 n xxx = thì 12 ( , ,..., ) min . n fxxxf = Khoâng maát tính toång quaùt giaû söû 12 0 ... n xxx < ≤ ≤≤ . Ta chöùng minh raèng ñeå 12 ( , ,..., ) min n fxxxf = thì 121 .... n xxx - = == Thaät vaäy, giaû söû 12 ... (1 3) i x x x in = = = ≤ ≤- . Ta chöùng minh 11 ... ii x xx + = == . Giaû söû ngöôïc laïi 1ii xx + > . Ta seõ chöùng minh ( ) ( ) 1 1 1 1 1 12 1 1 1 1 11 23 1 1 1 12 2 1 ( ,..., , ,..., ) ,..., , , , ,..., 1 11 1 3 11 ... ... ... ... ... 21113 ... ... 2 ... i i n i ii ii i n ii n ii ni i in ii i ii i n ii f x x x x f x x xx xx x x n xxx xx xx xxx n x xx xx x x xxxx xx x + - + ++ + +- ++ + - ++ + > ⇔ ++ + ++ + > + + + + + + ++ + + + + ++ ++ + + ++ - ⇔ ( ) ( ) ( ) ( ) 2 1 1 1 1 1 1 12 1 1 11121 12 121 3 ( ... ... ) ... 2 ... ( ... ... ) ... 2 ... 3 ( ... ) ( 1) 2 ... 3 ii ii ii n i ii in i i n i ii i n ii i i i n i ii i n ii n xx x xxx xxx xx x x x x x x x x xx x x nxx ix x x x i x xx x x nxx + + + - ++ + -+++ ++ +++ - > ++ + ++ ++ + + ++ ⇔ + + ++ + + ++ + + +> ⇔ ++ + + - + + + +> Ta coù 1 1 11 ... ... 0 13 nn i ii xx xxxx in - +- ≥ ≥ ≥> == = > ⎧ ⎨ ≤ ≤- ⎩ Tìm tài liệu Toán ? Chuyện nh ỏ - www.toanmath.com241 ( ) 1 2 1 11 12 12 121 ... ( ) ( )30 (1)2 ...0 ( ... ) ( 1) 2 ... 3 i i i n i i ii i ii i ni ii in i ii in ii ix x x x ix n ix n ix x i x xx x x nx ix x x x i x xx x x nxx + + + ++ ++ ++ +++ + + ++ ≥ +- > - ≥> ⎧ ⎪ ⇒ ⎨ - + + + + >> ⎪ ⎩ ⇒++ + + - + + + +> Vaäy ( ) 1 1 1 1 1 12 ( ,..., , ,..., ) ,..., , , , ,..., i i n i ii ii i n f x x x x f x x xx xx x x + - + ++ > . Ñieàu naøy voâ lyù vì 12 ( , ,..., ) min n fxxxf = . Vaäy ta phaûi coù 11 ... ii x xx + = == . Baèng laäp luaän töông töï, ta ñi ñeán keát quaû sau ñeå 12 ( , ,..., ) min n fxxxf = thì ta phaûi coù 2 31 ... . nn x xx -- = == Tieáp theo, ta seõ chöùng minh 1 21 .... nn xxx -- = == Giaû söû ngöôïc laïi 12 nn xx -- > Khi ñoù, ta seõ chöùng minh ( ) ( ) 1 2 2 1 1 2 3 21 21 2 1 2 21 21 ( , ,..., , , ) , ,..., , , , (( 2) ) ( 3) 2 3 n n n nnnnn n n n n n nn n nn fxx x x x fxx x xx xx x n x x x n x xx x nxx - - ----- - - - -- -- > ⇔ - + + - + +> Do 1 2 31 ...0 n n nn xxx xx - -- ≥ > = = => neân ( ) 21 12 2 21 12 2 1 2 21 1 212 ( 2) 2 ( 2) 0 (3) 2 (1)0 (( 2) ) ( 3) 2 (2( 2) )((1) ) nnnnn n nnnnn n n n n nnn n nnn nx x x x nx n x x x x x nx n x x x n x xxx x nxx n x -- -- - -- -- - - - -- - --- - + +≥ +-> ⎧ ⎪ ⎨ - + +> +-> ⎪ ⎩ ⇒ - + + - + +> > +- +- Do ñoù, ta chæ caàn chöùng minh 1 2 1 2 12 2 22 1122 (2( 2) )((1) )3 2 4 ( 32)0 (ñuùng) n n n n nn nnnn x nxx n x nxx x xx n nx - - - - -- ---- +- +- > ⇔ - + -+> Vaäy ta coù ( ) 1 2 2 1 1 2 3 21 21 ( , ,..., , , ) , ,..., , , , n n n n n n n nn fxx x x x fxx xxxxx x - - - - - -- > . Ñieàu naøy voâ lyù vì 12 ( , ,..., ) min n fxxxf = . Vaäy ta phaûi coù 1 21 .... nn xxx -- = == Nhö vaäy, ta ñi ñeán keát quaû Ñeå 12 ( , ,..., ) min n fxxxf = thì 1 21 .... n nn xxxx -- ≥ = == Tìm tài liệu Toán ? Chuyện nh ỏ - www.toanmath.com242 12 1 1 min ( , ,..., ) 111 min , ,..., , ( 1) 13 min ( 1) ( 1) (1) 1 n n nn faaa f xx xxx nx nxx x xn - - ⇒≥ ⎛⎞ ≥≥ ⎜⎟ ⎝⎠ ⎧⎫ = -++≥ ⎨⎬ +- ⎩⎭ Tieáp theo, ta seõ tìm min cuûa haøm soá 1 13 ()(1)1 1 vôùi nn nx gxnxx x xn - =-++≥ +- Ta coù 22 / 2 ( 1)( 1)( ( 2) ( 1) ) () ( 1) n nn nn n x x n xn gx xxn - - - + +- = +- Theo baát ñaúng thöùc AM-GM, ta coù 22 ( 1) 2( 1) ( 2) ( 4) do n nn x n n x n xn +-≥-≥+≥ [ ) / ()01 (). ñoàng bieán treân 1,+ gxx gx ⇒ ≥ ∀≥ ⇒∞ ( ) (1) 3 1 (2) gxg nx ⇒ ≥ = + ∀≥ Töø (1) vaø (2), ta suy ra 12 1 2 12 min ( , ,..., ) 3 11 13 ...3 ... (ñpcm) n nn faa an n n aa aaaa ≥+ ⇒ + + + + ≥+ + ++ Ñaúng thöùc xaûy ra khi vaø chæ khi 12 ... 1. n aaa = = == + Caùch 2. Ta seõ chöùng minh keát quaû maïnh hôn nhö sau Cho n soá thöïc döông 12 , ,..., n aaa thoûa 12 ... 1. n aaa = Khi ñoù, ta coù 1 2 12 111 ... ... nn kk n aa aaa an ++ ++ ≥ + + ++ Vôùi 4( 1) kn =- (noùi rieâng neáu 4 n ≥ thì 3 kn ≥ ) Ta seõ chöùng minh baèng doàn bieán. 1,2 nn == laø caùc tröôøng hôïp taàm thöôøng neân ôû ñaây ta seõ khoâng xeùt tôùi. Ta seõ xeùt tröôøng hôïp 3 n ≥ . Tìm tài liệu Toán ? Chuyện nh ỏ - www.toanmath.com243 Ñaët 12 1 2 12 111 ( , ,..., ) ... ... n nn kk faaan aa aaaan = ++ ++ - - + ++ Ta coù Nhaän xeùt sau (i) Neáu 12 12 1 axa aa ≤≤ ⎧ ⎨ ≤ ⎩ thì 12 12 ( , ,..., ) , ,..., nn aa faaafxa x ⎛⎞ ≥ ⎜⎟ ⎝⎠ (ii) Neáu 1 2 12 12 13 (1 )(1 ) 1 0 nn ii ii a a kaa a a aa == ⎡⎤ ⎛⎞⎛⎞ - - - + +≥ ⎢⎥ ⎜⎟⎜⎟ ⎝⎠⎝⎠ ⎣⎦ ∑∑ thì 1 2 12 ( , ,..., ) (1, ,..., ) nn faaa f aaa ≥ Chöùng minh Nhaän xeùt. Ñeå tieän vieäc trình baøy xin ñöôïc kyù hieäu 3 . n i i Aa = = ∑ (i) Ta coù 12 12 12 1 2 12 12 ( , ,..., ) , ,..., 1 11 nn aa faaafxa x x kk aa a a x aa a aA xA x ⎛⎞ -= ⎜⎟ ⎝⎠ =+--+- ++ ++ 12 1 2 1 2 12 2 1 2 1 2 12 ()( )() ( )() aa xa ax a a A x Akaa x aa a a A x Ax aa ⎡⎤ ⎛⎞ - - + + + +- ⎜⎟ ⎢⎥ ⎝⎠ ⎣⎦ = + + ++ AÙp duïng baát ñaúng thöùc AM-GM, ta coù 2 12 1 2 12 ( ) 4( 1) aa a a A x A n n k kaa x ⎛⎞ + + + + ≥ ≥ - =≥ ⎜⎟ ⎝⎠ Do ñoù 12 12 ( , ,..., ) , ,..., nn aa faaafxa x ⎛⎞ ≥ ⎜⎟ ⎝⎠ (i) ñöôïc chöùng minh. Tìm tài liệu Toán ? Chuyện nh ỏ - www.toanmath.com244 (ii) Khai trieån töông töï nhö treân, ta coù 1 2 123 1 2 1 2 1 2 12 12 1 2 12 1 2 123 ( , ,..., ) (1, , ,..., ) (1 )(1 )[( )(1 ) ] ( )(1) 0 ( , ,..., ) (1, , ,..., ) nn nn faa a f aaaa a a a a A aa A kaa aa a a A A aa faa a f aaaa -= - - + + + +- = + + ++ ≥ ⇒≥ (ii) ñöôïc chöùng minh. Nhaän xeùt ñöôïc chöùng minh hoaøn toaøn. Löu yù raèng caùc bieán 12 , ,..., n aaa bình ñaúng neân 12 , aa trong Nhaän xeùt coù theå thay baèng, () ij aa ij ≠ tuøy yù. Trôû laïi baøi toaùn cuûa ta Ta seõ chöùng minh raèng luoân coù theå ñöa veà tröôøng hôïp trong n bieán coù 1 n - bieán khoâng lôùn hôn 1. Thaät vaäy, giaû söû trong n bieán coù nhieàu hôn 1 bieán lôùn hôn 1, khoâng maát tính toång quaùt, ta coù theå giaû söû laø 12 ,. aa Xeùt 2 tröôøng hôïp * Tröôøng hôïp 1. 1 2 12 13 1 nn ii ii kaa a a aa == ⎛⎞⎛⎞ ≥ ++ ⎜⎟⎜⎟ ⎝⎠⎝⎠ ∑∑ . Khi ñoù, theo Nhaän xeùt (ii), ta coù 1 2 123 ( , ,..., ) (1, , ,..., ) nn faa a f aaaa ≥ Nhö vaäy, ta coù theå thay boä soá 12 ( , ,..., ) n aaa bôûi boä soá 123 (1, , ,..., ) n aaaa ñeå f khoâng taêng. Khi ñoù, soá bieán baèng 1 taêng leân ít nhaát laø 1. * Tröôøng hôïp 2. 12 12 13 1. nn ii ii kaa a a aa == ⎛⎞⎛⎞ ≤ ++ ⎜⎟⎜⎟ ⎝⎠⎝⎠ ∑∑ Khi ñoù, vôùi moïi 2 1 j aa <≤ ( j a luoân toàn taïi vì 12 ...1 n aaa = ) ta ñeàu coù 11 1 1, 1 n j i ij i i ij kaa a a aa = ≠≠ ⎛⎞ ⎛⎞ ≤ ++ ⎜⎟ ⎜⎟ ⎜⎟ ⎝⎠ ⎝⎠ ∑∑ Thaät vaäy, ta coù Tìm tài liệu Toán ? Chuyện nh ỏ - www.toanmath.com245 1 11 1, 1 11 1 1 n ij ijj i ij i jj a aa a aa aa kaa kaa ≠≠ = ++ + +-- = ∑ ∑ 1 1 1 12 1 12 1 1 1 1 n ij i j n i i a aa kaa a aa kaa = = +-- =+ +-- ≥+ ∑ ∑ 12 3 12 11 1 1, 1 1 n i i n j i ij i i ij a aa kaa kaa a a aa = = ≠≠ ++ = ⎛⎞ ⎛⎞ ⇒ ≤ ++ ⎜⎟ ⎜⎟ ⎜⎟ ⎝⎠ ⎝⎠ ∑ ∑∑ Do ñoù 111 1 1, (1 )(1 ) 1 0 n j j i ij i i ij a a kaa a a aa = ≠≠ ⎡⎤ ⎛⎞ ⎛⎞ - - - + +≥ ⎢⎥ ⎜⎟ ⎜⎟ ⎜⎟ ⎝⎠⎢⎥ ⎝⎠ ⎣⎦ ∑∑ . Söû duïng Nhaän xeùt (ii), ta coù theå thay boä soá 12 ( , ,..., ) n aaa bôûi boä soá 21 (1, ,..., ,..., ) jn aaaa ñeå f khoâng taêng. Khi ñoù soá bieán baèng 1 cuõng taêng leân ít nhaát laø 1. Toùm laïi, neáu vaãn coøn 2 bieán lôùn hôn 1 thì ta luoân coù theå thay boä soá ñang xeùt bôûi moät boä soá khaùc maø soá bieán baèng 1 taêng leân ít nhaát laø 1 ñeå f khoâng taêng. Vieäc thay theá naøy chæ coù theå ñöôïc thöïc hieän khoâng quaù n laàn (vì coù khoâng quaù n baèng 1). Do ñoù, sau moät soá böôùc höõu haïn (khoâng quaù n), ta seõ ñöa baøi toaùn veà tröôøng hôïp trong n bieán coù 1 n - bieán khoâng lôùn hôn 1. Tieáp theo, ta seõ chöùng minh coù theå thay 1 n - bieán khoâng lôùn hôn 1 bôûi trung bình nhaân cuûa chuùng. Thaät vaäy, khoâng maát tính toång quaùt, giaû söû 121 ...1 n aaa - ≤ ≤≤≤ . Ñaët 1 121 ... 1. n n x aaa - - =≤ Neáu 11 n a xax - =∨= thì 121 .... n aa ax - = === Neáu toàn taïi (1 1) j a jn < <- sao cho j ax ≠ thì ta coù 11 . n a xa - << Söû duïng Nhaän xeùt (i), ta Tìm tài liệu Toán ? Chuyện nh ỏ - www.toanmath.com246 coù theå thay boä soá 12 ( , ,..., ) n aaa bôûi boä soá 11 2 , ,..., , n n aa xaa x - ⎛⎞ ⎜⎟ ⎝⎠ ñeå f khoâng taêng. Khi ñoù, soá bieán baèng x taêng leân ít nhaát laø 1. Ta cuõng chuù yù raèng 11 1 1 n aa a xx - ≤≤ neân vieäc thay nhö treân vaãn ñaûm baûo trong n bieán coù 1 n - bieán khoâng lôùn hôn 1, ñieàu ñoù cho pheùp vieäc thay theá nhö treân coù theå thöïc hieän lieân tieáp. Tuy nhieân, vieäc thay theá naøy chæ coù theå ñöôïc thöïc hieän khoâng quaù n laàn (vì coù khoâng quaù n baèng x). Do ñoù, sau moät soá laàn thay (khoâng quaù n), ta ñöa ñöôïc veà tröôøng hôïp trong n bieán coù 1 n - bieán khoâng lôùn hôn 1 baèng nhau. Cuoái cuøng, ñeå chöùng minh baát ñaúng thöùc ñaõ cho, ta chæ caàn chöùng minh 1 1 1 1 , ,..., 0 (0 1) 1 ( ) 0 (0 1) 1 n n n fxxx x n kk gx x nx xn nx x - - - ⎛⎞ ≥ <≤ ⎜⎟ ⎝⎠ - ⇔ = + + - - ≥ <≤ + Ta coù 2 / 2 ( 1)( 1) ( 1) 1 ( ) . 0 (0,1] ( 1)1 ( ) (0,1]. ( ) (1) 0 (0,1] nghòch bieán treân ñpcm. nn n n x nx gxx x nx gx gxgx ⎛⎞ - - -- = ≤ ∀∈ ⎜⎟ -+ ⎝⎠ ⇒ ⇒ ≥ = ∀∈ ⇒ Ñaúng thöùc xaûy ra khi vaø chæ khi 12 ... 1. n aaa == == Tìm tài liệu Toán ? Chuyện nh ỏ - www.toanmath.com247 Baøi toaùn 16. Cho ,, abcR ∈ thoûa ( )( )( ) 0 a bb cc a - - -≠ . Chöùng minh raèng 2 2 2 2 22 2 22 ( 2) ( 2 ) ( 2 ) ( 2 ) ( 2 ) ( 2) 22 ( ) ( ) () ab ac bc ba ca c b b c c a ab - + - - + - - +- ++≥ - -- Lôøi giaûi. Ta coù 22 2 2 22 2 2 22 2 2 22 2 2 (2) (2) 22 () 24() 44 22 () 2( ) 22 11 () 2() () () 11 () 8 cyc cyc cyc cyc cyc a b ac bc a bca bc bc a bca bc bc a bca bc bc bc bca bc - +- ≥ - - + ++ ⇔≥ - - + ++ ⇔≥ - - + ++ +- ⇔≥ - +- ⎛⎞ ⇔≥ ⎜⎟ - ⎝⎠ ∑ ∑ ∑ ∑ ∑ Ñaët ,, bc a c ab abc xyz bc c a ab + - + - +- = == --- thì ta coù ( 2)( 2)( 2) ( 2)( 2)( 2) 4 xy z xyz xy yz zx -- - = ++ + ⇒ ++=- Ta coù baát ñaúng thöùc caàn chöùng minh töông ñöông vôùi 2 22 2 22 2 8 2() ( )0 (ñuùng) ñpcm. xyz x y z xy yz zx xyz + +≥ ⇔ + +≥- ++ ⇔ + +≥ ⇒ Tìm tài liệu Toán ? Chuyện nh ỏ - www.toanmath.com248 Baøi toaùn 17. (APMO 2005) Cho , , 0 8. thoûa Chöùng minh raèng a b c abc >= 222 3 3 3 3 33 4 3 (1 )(1 ) (1 )(1 ) (1 )(1 ) abc a b b c ca + +≥ + + + + ++ Lôøi giaûi. Ta coù 2 3 12 0 2 1 x x x ≥ ∀> + + (*) Thaät vaäy, ta coù 223 22 (*) (2 ) 4(1 ) ( 2)0 (ñuùng) xx xx ⇔ + ≥+ ⇔ -≥ Vaäy (*) ñuùng. Do ñoù 22 22 33 4 2(,,)2 36 (2 )(2 ) 36 ( , , ) (1 )(1 ) 1 ( ,,) cyc cyc a a S abc a b S abc ab S abc ≥ == + ++ ++ + ∑∑ trong ñoù 2 22 22 2222 (,,)2() Sabc abc ab bcca = + + + ++ Theo baát ñaúng thöùc AM-GM, ta coù 2222 3 2222224 3 3 ( ) 12 3 ( ) 48 (, ,) 72 a b c abc ab bc ca abc S abc ++≥= ++≥= ⇒≥ Do ñoù 2 33 2 24 36 36 3 (1 )(1 ) 11 (, ,) 72 . ñpcm cyc a ab S abc ≥ ≥= ++ ++ ⇒ ∑ Ñaúng thöùc xaûy ra khi vaø chæ khi 2. abc = == Tìm tài liệu Toán ? Chuyện nh ỏ - www.toanmath.com249 Baøi toaùn 18. (Vasile Cirtoaje) Cho , , 0. abc ≥ Chöùng minh raèng 2 22 2 2 2 2 22 2 3 3 23 3 23 3 2 3 a bc b c bc c a ca a b ab + +≥ + - + - +- Lôøi giaûi. AÙp duïng baát ñaúng thöùc Bunhiacopxki, ta coù 24 22 2 22 332 (332) cyc cyc aa b c bc a b c bc = +- +- ∑∑ 2 2 22 2 22 2 2 22 2 2 2 2 22 () (3 3 2) () 6( )2() cyc abc a b c bc abc ab bc ca abca b c ++ ≥ +- ++ = + + - ++ ∑ Do ñoù, ñeå chöùng minh baát ñaúng thöùc ñaõ cho, ta chæ caàn chöùng minh 2 2 22 2 2 2 2 22 2222 222222 444 222222 ( )2 6( ) 2 ( )3 3( ) 12( ) 4 ( ) 3 ()2( ) ()0 abc ab bc ca abca b c a b c ab bc ca abca b c a b c abc a b c a b b c c a abc a b c ++ ≥ + + - ++ ⇔ + + ≥ + + - ++ ⎡⎤ ⇔ + + + ++ - + + + ++ ≥ ⎣⎦ Theo baát ñaúng thöùc Schur thì 44 4 22 2 2 2 2 22 ( ) () 2() (theo AM-GM) cyc abc abca b c abab abbcca +++ ++≥+ ≥ ++ ∑ Vaäy ta coù 44 4 2222 22 3 ( ) 2( ) ( )0 a b c abc a b c a b b c c a abc a b c ⎡⎤ + + + ++ - + + + ++ ≥ ⎣⎦ ⇒ ñpcm. Ñaúng thöùc xaûy ra khi vaø chæ khi ( , , ) ( , ,0) ( 0). abc tt t => Tìm tài liệu Toán ? Chuyện nh ỏ - www.toanmath.com250 Baøi toaùn 18. (Vasile Cirtoaje) Cho , , 0. abc ≥ Chöùng minh raèng 2 22 2 22 222 2 a bc b bc c c ca a a ab b + +≥ - + - + -+ Lôøi giaûi. + Caùch 1. AÙp duïng baát ñaúng thöùc Bunhiacopxki, ta coù 24 2 2 222 2 2 22 222 2 2 22 2 2 2 2 22 () () () () 2( ) () cyc cyc cyc aa b bcc a b bcc abc a b bcc abc ab bc ca abca b c = -+ -+ ++ ≥ -+ ++ = + + - ++ ∑∑ ∑ Do ñoù, ñeå chöùng minh baát ñaúng thöùc ñaõ cho, ta chæ caàn chöùng minh 2 2 22 2 2 2 2 22 () 2 2( ) () abc ab bc ca abca b c ++ ≥ + + - ++ 2 222 22 2222 4 44 22 2222 ( )4( )2() ( ) 2( ) ( )0 a b c ab bc ca abcabc a b c abca b c ab bc ca abca b c ⇔ + + ≥ + + - ++ ⎡⎤ ⇔ + + + ++ - + + + ++ ≥ ⎣⎦ Theo baát ñaúng thöùc Schur thì 44 4 22 2 2 2 2 22 ( ) () 2() (theo AM-GM) cyc abc abca b c abab ab bc ca +++ ++≥+ ≥ ++ ∑ Vaäy ta coù 44 4 2222 22 ()2( ) ()0 a b c abcabc ab bc ca abcabc ⎡⎤ + + + ++ - + + + ++ ≥ ⎣⎦ ⇒ ñpcm. Ñaúng thöùc xaûy ra khi vaø chæ khi ( , , ) ( , ,0) ( 0). abc tt t => Tìm tài liệu Toán ? Chuyện nh ỏ - www.toanmath.com251 + Caùch 2. Khoâng maát tính toång quaùt coù theå giaû söû0. abc ≤ ≤≤ Khi ñoù, ta coù 2 2 22 2 2 22 00 00 a ac c ca a c a ab a ab b b ⎧⎧ -≤ ≤ - +≤ ⎪⎪ ⇒ ⎨⎨ - ≤ ≤-+ ≤ ⎪⎪ ⎩⎩ Do ñoù 2 2 2 2 22 222 22 22222 a b c a bc b bccc caaa abb b bcccb + + ≥ ++ - + -+ - + -+ 22 22 2 (theo AM-GM) bc cb ≥+≥ ñpcm. ⇒ Ñaúng thöùc xaûy ra khi vaø chæ khi ( , , ) ( , ,0) ( 0). abc tt t => Baøi toaùn 19. (VMEO 2004) Cho,,0 xyz > thoûa 1 xyz + += . Chöùng minh raèng 222 ( ) ( ) () 3 12 12 12 y z z x xy x yz - -- + ++ ++≤ Lôøi giaûi. Khoâng maát tính toång quaùt coù theå giaû söû 0 x yz ≥ ≥> 0 2 13 xyxyzz ⇒ ≤- ≤+ - = - Ta seõ chöùng minh 222 2( ) ( ) (*) x u yv x y uv + + + ≤ + ++ trong ñoù , 12 12 y z xz uv -- == Thaät vaäy 22 (*) 2 2()() x y uv x u yv ⇔++ ≥ + + 2 ( ) 4( )( )0 x y u v xv yu ⇔ - + - -≥ Tìm tài liệu Toán ? Chuyện nh ỏ - www.toanmath.com252 22 2 1 ( ) .( )( )0 3 ( )(3 )0 (ñuùng) xy xy x y z xy zxy ⇔ - - - + -≥ ⇔ - + - -≥ Vaäy (*) ñuùng. Do ñoù 2 22 22 ( ) ( 2) () 2() 12 12 12 ( 2 ) (1 3) 2()3 12 12 (ñpcm) cyc y z x y z xy x xyz xyzz xyz - +-- + ≤ + + ++ +-- ≤ ++ ++= ∑ Ñaúng thöùc xaûy ra khi vaø chæ khi 1 . 3 x yz = == Baøi toaùn 20. Cho,,0 xyz ≥ thoûa 1. xyz + += Tìm giaù trò lôùn nhaát cuûa bieåu thöùc ( , ,) n nn f xyz x y yz zx = ++ trong ñoù 2, n nR ≥∈ laø haèng soá cho tröôùc. Lôøi giaûi. Giaû söû vôùi boä soá 00 0 00 0 ( , ,) ( , ,) max . thì xy z fxy z f = Khoâng maát tính toång quaùt coù theå giaû söû { } 0 0 00 max ,, x xyz = . Khi ñoù, neáu 00 yz ≤ thì ta coù 00 0 0 00 0 0 0 0 0 0 00 00 0 ( , , ) (, , ) ( ) ( ) () n n n nn f x z y f x y z z y x y z x yz yz gx - = - + - +-= Ta coù / 11 0 0 0 0 00 0 0 0 00 0 0 ( ) ( ) ( ) ( 1)( )0 nn n nn n n n gx nz yx yz nz yz yz n z y -- = - + - ≥ - + - = - -≥ ⇒ 0 () gx ñoàng bieán. ⇒ 00 ( ) ( )0 gx gz ≥= ⇒ 00 0 0 00 (, , ) (, , ) fx z y fx y z ≥ Do ñoù, khoâng maát tính toång quaùt, ta chæ xeùt 0 x yz ≥ ≥≥ laø ñuû. Theo baát ñaúng thöùc Becnulli, ta coù Tìm tài liệu Toán ? Chuyện nh ỏ - www.toanmath.com253 1 ().1.10 n nnnnn z nz xzxxxnxz xx - ⎛ ⎞ ⎛⎞ + = + ≥ + =+≥ ⎜ ⎟ ⎜⎟ ⎝ ⎠ ⎝⎠ Do ñoù, ta coù 1 1 11 ( ,,0)() () ( 2 )(2 do ) n nn nn n nn fx zy x zy x nx zy x xzyn x y x yz x yz - - -- + =+ ≥+ ≥+≥ = ++ (0 do ) n nn xy yz zx x yz ≥ + + ≥ ≥≥ ( , ,) f xyz = Ta laïi coù 1 1 ( ,,0)() (1) 1 .( ).(1 ) 1 (1) . 1 ( 1) n n n n n n fx zy x zy yy nyy n nyny nn n n + + + =+ =- =- +- ⎛⎞ ≤ ⎜⎟ + ⎝⎠ = + Do ñoù 1 ( , ,) ( 1) n n n f x yz n + ≤ + Ñaúng thöùc xaûy ra chaúng haïn khi 1 , ,0 11 n xyz nn = == ++ . Keát luaän 1 max. ( 1) n n n f n + = + Tìm tài liệu Toán ? Chuyện nh ỏ - www.toanmath.com254 Baøi toaùn 21. (Voõ Quoác Baù Caån) Cho,,03 thoûa . Tìm giaù trò nhoû nhaát cuûa bieåu thöùc abc abc > ++= 2 22 a bc P b ab c bc a ca = ++ + ++ Lôøi giaûi. Ta coù 32 32 12 32 32 32 3 12 32 43 2 43 43 22 2 22 3 2 3 3 3. 23 3. 6 (theo bñt AM-GM) 1 3 1 (theo bñt AM-GM) 3 cyc cyc cyc cyc cyc cyc cyc aa P aa b ab b ab ab ab ab ab ab a ab ab ⎛⎞ = = -+ ⎜⎟ ++ ⎝⎠ =- + ≥- =- ++ ≥- ∑ ∑∑ ∑ ∑ ∑ ∑ 2 43 43 2 43 43 43 43 13 13 11 3 . ( 2 ). 186 11 3 .( ). 186 51 . 26 51 .. 26 5 1 ( 1) . 263 51 . (4) 2 18 51 .(4( )3) 2 18 cyc cyc cyc cyc cyc cyc cyc a ab ab abc a b ab abab abab abc ab bc ca abc =- +- =- ++- =- =- ++ ≥- =-- = - + +- ∑∑ ∑ ∑ ∑ ∑ ∑ Maët khaùc, aùp duïng baát ñaúng thöùc Schur, ta coù Tìm tài liệu Toán ? Chuyện nh ỏ - www.toanmath.com255 3 33 3 33 3 () 3 ( )6 4 ( )3 cyc cyc cyc a b c abc ab a b a b c ab a b abc ab a b abc +++ ≥+ ⇒ + ++ ++ ≥ ++ ∑ ∑∑ 3 ( ) 4 ( )3 cyc a b c ab a b abc ⇔ + + ≥ ++ ∑ 27 4 (3 ) 3 cyc ab c abc ⇔ ≥ -+ ∑ 27 12( ) 9 4( )39 5 1 51 2 .(4( ) 3 ) .9 2 2 18 2 18 1 ab bc ca abc ab bc ca abc P ab bc ca abc P ⇔ ≥ + +- ⇒ + + -≤ ⇒ ≥- + + - ≥-= ⇒≥ Ñaúng thöùc xaûy ra khi vaø chæ khi 1. abc === Vaäy min 1. P = Baøi toaùn 22. (Voõ Quoác Baù Caån) Cho , , 0. Chöùng minh raèng abc > 2 2 2 2 22 4 44 27 2 2 2 36 bc ca ab a b c a b c bc c a ab ⎛ ⎞⎛ ⎞⎛⎞ ⎛⎞ + + + + ≥ ++ ⎜ ⎟⎜ ⎟⎜⎟⎜⎟ + ++ ⎝⎠ ⎝ ⎠⎝ ⎠⎝⎠ Lôøi giaûi. Ta coù Boå ñeà sau Boå ñeà. , , 0. Khi ñoù, ta coù xyz > 2 22 ( 2)( 2)( 2) 9( ) x y z xy yz zx + + + ≥ ++ Chöùng minh. Theo nguyeân lyù Dirichlet, ta coù trong 3 soá 2 22 1, 11 vaø x yz - -- luoân toàn taïi ít nhaát 2 soá cuøng daáu. Khoâng maát tính toång quaùt coù theå giaû söû 22 11 vaø xy -- cuøng daáu. 22 2222 ( 1) 1) 0 1 ( xy xy xy ⇒ - -≥ ⇒ ≥+- Tìm tài liệu Toán ? Chuyện nh ỏ - www.toanmath.com256 22 2 2 22 2 2 22 2 2 4 3( 1) ( 2) 2) 3( 1) ( xy x y xy x y xy ⇒ + + + ≥ ++ ⇔ + + ≥ ++ Do ñoù 2 2 2 2 22 222 2 ( 2)( 2)( 2) 3( 1)( 2) 3( 1)(11) 3() 9() (theo bñt Bunhiacopxki) xyz xy z xyz x yz xy yz zx + + +≥ + ++ = + + ++ ≥ ++ ≥ ++ Boå ñeà ñöôïc chöùng minh hoaøn toaøn. Ñaúng thöùc xaûy ra khi vaø chæ khi 1. xyz = == Trôû laïi baøi toaùn cuûa ta AÙp duïng Boå ñeà treân vôùi 2 22 ,, bc ca ab x yz a bc = == , ta coù 22 22 22 333 4 4 4 2 2 2 2 22 9() 2 2 29. .. b c c a a b bc ca ca ab ab bc a b c a b c a b b c c a abc ⎛ ⎞⎛ ⎞⎛ ⎞ ++ ⎛⎞ + + +≥ + += ⎜ ⎟⎜ ⎟⎜ ⎟⎜⎟ ⎝⎠ ⎝ ⎠⎝ ⎠⎝ ⎠ Do ñoù, ta chæ caàn chöùng minh 3 33 3 33 9() 27 36 34 abc a b c abc bc c a ab abc a b c abc b cc aa b ++⎛⎞ + ≥ ++ ⎜⎟ + ++ ⎝⎠ ++⎛⎞ ⇔+ ≥ ++ ⎜⎟ + ++ ⎝⎠ Chuù yù raèng 2 3 33 ( ).() 3 2 cyc abc ab abc abc abc ++- ++ =+ ∑ 2 () 23 ( )() cyc a b c ab bc c a ab acbc - ⎛⎞ ++=+ ⎜⎟ + + + ++ ⎝⎠ ∑ Do ñoù 333 3 4. cyc abc a abc b c ++ + -= + ∑ Tìm tài liệu Toán ? Chuyện nh ỏ - www.toanmath.com257 2 2 2 22 2 ( ).() () 33 23 2 ( )() 1 ( ) (( )( )( ) 4 ) . 2 ( )() 1 ( )(( )( ) 4 ) . 2 ( )() 1 ( )(4 ( )4) . 2 ( )() (the cyc cyc cyc cyc cyc ab c ab ab abc a c b c a b a b c a c b c abc abc a c b c a b a c b c abc abcacbc a b ac b c abc abcacbc ++- ⎛⎞ - =++ -+ ⎜⎟ ++ ⎝⎠ - + + + +- = ++ - + +- ≥ ++ - +- ≥ ++ ∑ ∑ ∑ ∑ ∑ 2 () 2. ( )() 0 o bñt AM-GM) cyc cab ba cbc - = ++ ≥ ∑ 3 33 34 ñpcm. abc a b c abc b c c a ab ++ ⎛⎞ ⇒+ ≥ ++ ⎜⎟ + ++ ⎝⎠ ⇒ Ñaúng thöùc xaûy ra khi vaø chæ khi . abc == Baøi toaùn 23. (Voõ Quoác Baù Caån) 1 0 , , 1. 2 Cho thoûa Chöùng minh raèng xyz xyz ≤ ≤ + += 3 33 19 4 4 32 x y z xyz ≤++ + ≤ Lôøi giaûi. Tröôùc heát, ta chöùng minh 3 33 1 4 4 x y z xyz + ++≥ 3333 3 33 44416 () 10 ( ) (*) 3 cyc x y z xyz x yz x y z xyz xy x y ⇔ + + + ≥ ++ ⇔+++ ≥+ ∑ Theo bñt Schur thì 3 33 3 () cyc x y z xyz xy x y +++ ≥+ ∑ Tìm tài liệu Toán ? Chuyện nh ỏ - www.toanmath.com258 (*) ñuùng. ⇒ Tieáp theo, ta seõ chöùng minh 3 33 9 4 32 23 3( )7 32 23 3 ( ) (3 7) 32 x y z xyz xy yz zx xyz xyzyzx + ++≤ ⇔ + + -≥ ⇔ + + -≥ 11 32 Khoâng maát tính toång quaùt giaû söûxyzx ≥ ≥ ⇒ ≤≤ Coù 2 tröôøng hôïp xaûy ra * Tröôøng hôïp 1. 133 0 377 x zy ≤ ≤ ⇒ ≤ ≤≤ 33 0 77 93 93 12 3 ( ) (1) 497 497 497 yz yz yz xx ⎛ ⎞⎛⎞ ⇒ - -≥ ⎜ ⎟⎜⎟ ⎝ ⎠⎝⎠ ⇒ ≥-+ + =-+ - = - Do ñoù 12 3 36 23 3 ( ) (3 7) 3 (1 ) (3 7) 49 7 49 32 xy z xyz x x x x ⎛⎞ + +- ≥ - +- - = > ⎜⎟ ⎝⎠ * Tröôøng hôïp 2. 31 72 . Khi ñoù ta coù x ≤≤ ( ) 2 1 3 ( ) (3 7) 3 (1 ) (3 7) 4 (theo bñt AM-GM) xy z xyz x x y z x ++- ≥ -+ +- ( ) 2 1 3 (1 ) 1 (3 7) 4 xx xx = -+ -- 2 1 23 23 1 (1 2 )(28 6 1) ) 32 32 32 2 (do xxxx =- -++≥≤ Toùm laïi, ta luoân coù 3 33 19 4 4 32 x y z xyz ≤++ + ≤ (ñpcm) Tìm tài liệu Toán ? Chuyện nh ỏ - www.toanmath.com259 Baøi toaùn 24. (Jack Grafunkel) Tìm haèng soá k nhoû nhaát sao cho baát ñaúng thöùc sau ñuùng vôùi moïi,,0 xyz ≥ x yz k xyz x y y z zx ++≤ ++ + ++ Lôøi giaûi. Ta seõ chöùng minh baát ñaúng thöùc ñaõ cho ñuùng khi 5 4 k = . Ñaây chính laø haèng soá toát nhaát cuûa baát ñaúng thöùc ñaõ cho vì ta coù ñaúng thöùc xaûy ra khi 0, 3,1 x yz = == . Ñaët 2 22 , , ( , , 0) xy c yz a zx b abc += +=+=≥ ⇒ 222 ,, abc laø ñoä daøi ba caïnh cuûa moät tam giaùc (coù theå suy bieán). Baát ñaúng thöùc caàn chöùng minh trôû thaønh 2 2 2 2 22 2 22 2 22 5 . (1) 22 abc a bc abc abc c ab - ++ - + +- + + ≤ ++ Khoâng maát tính toång quaùt, ta coù theå giaû söû ,. a bc ≥ AÙp duïng baát ñaúng thöùc Bunhiacopxki, ta coù 22 222 2 a bc abc ++ ≤ ++ Do ñoù, ñeå chöùng minh (1), ta chæ caàn chöùng minh ( ) ( ) ( ) 2 2 2 2 22 2 22 22 22 22 5 . 4 ( )( )( )( ) 5 . 4 4 ( ) 4( )( )( )( ) 5 abc a bc abc a bc c ab ab c ab ac cb abc a b c abc abc a b c a b c a b a c c b abc a b c - ++ - + +- + + ≤ ++ ++-- - ⇔+++ ≤ ++ ⇔ ++ + ++ - - - ≤ + + Töø ñaây suy ra khoâng maát tính toång quaùt, ta chæ caàn xeùt tröôøng hôïpacb ≥≥ laø ñuû. Ta coù ( ) 22 4 ( ) 4( )( )( )( ) 5 abc a b c a b c a b a c c b abc a b c ++ + ++ - - - ≤ ++ Tìm tài liệu Toán ? Chuyện nh ỏ - www.toanmath.com260 ( ) 3 2 22 3 2 2 3 22 ()4( ) 4() 444450 f a a c b abc bcc b a b b c bc c bc b c ⇔ = - - + -+ + + + - - +≤ Do 222 ,, abc laø ñoä daøi ba caïnh cuûa moät tam giaùc (coù theå suy bieán) neân ta coù 22 a bc ≤+ . Do ñoù, ta caàn chöùng minh ()0 fa ≤ vôùi 22 (2) b c a bc ≤≤≤+ . + Neáubc = thì ( ) 2 () ( )5270 faababb ⎡⎤ =- -+ -≤ ⎣⎦ + Neáubc < thì () fa laø moät haøm ña thöùc baäc ba coù heä soá cao nhaát vaø thaáp nhaát döông. Ta coù lim () (0) 0 lim () a a fa f fa →-∞ →+∞ = -∞ > = +∞ Ta laïi coù ( ) 2 22 () 5 4 30 fcbc b cb c =- +-- < Vì 22 22 25( ) (3 4 ) (3 4 ) 0 b c c b bc + - + = -> Ngoaøi ra, ( ) ( ) ( ) 22 22 22 2 22 245 22 0 fbc bc bb c b c bcb cb + = + -- =- +- ≤ Do ñoù f coù ba nghieäm phaân bieät (1 nghieäm aâm, 1 nghieäm thuoäc (0, ) c vaø 1 nghieäm khoâng nhoû hôn 22 bc + ). Do 22 c a bc ≤≤+ neân ()0 fa ≤ . ⇒ ñpcm. Tìm tài liệu Toán ? Chuyện nh ỏ - www.toanmath.com261 Ñaúng thöùc xaûy ra khi vaø chæ khi 22 22 20 13 2 1 3 ax a bc by b cb z c ⎧ == ⎧ ⎧ =+ ⎪ ⎪⎪ ⇔ = ⇔= ⎨ ⎨⎨ ⎪ ⎪⎪ += ⎩ = = ⎩ ⎩ Vaäy min 5 . 4 k = Baøi toaùn 25. Cho,,01 thoûa . Chöùng minh raèng a b c abc >= 3 3 3 2 22 1 11 bc c a ab a bc b ca c ab a b c + ++ + + ≤ ++ + ++ Lôøi giaûi. Tröôùc heát, ta chöùng minh caùc Boå ñeà sau Boå ñeà 1. , , 0. Khi ñoù, ta coù xyz > 2 2 2 2 2 22 ( ( ) ( ) ( )) 4( )( ) xy x y yz y z zx z x xy yz zx x y y z z x + + ++ + ≥+ + ++ Chöùng minh. Ta coù 2 2 2 2 2 22 22 2 3 33 (()()())4( )() () 2 ( 3 ( ) () ( )) cyc xy x y yz y z zx z x xy yz zx x y y z z x x y x y xyz x y z xyz xy x y yz y z zx z x ++ ++ + - ++ + += = - + + + + - + - + -+ ∑ Do 222 3 33 ( )0 3 ( ) ( ) ( )0 (theo bñt Schur) cyc xy xy x y z xyz xy x y yz y z zx z x ⎧ -≥ ⎪ ⎨ ⎪ + ++ - + - + - +≥ ⎩ ∑ Neân 2 2 2 2 2 22 2 2 2 2 2 22 ( ( ) ( ) ( )) 4( )( ) 0 ( ( ) ( ) ( )) 4( )( ) xy x y yz y z zx z x xy yz zx x y y z z x xy x y yz y z zx z x xy yz zx x y y z z x ++ ++ + - ++ + +≥ ⇒ + + ++ + ≥+ + ++ Boå ñeà 1 ñöôïc chöùng minh hoaøn toaøn. Tìm tài liệu Toán ? Chuyện nh ỏ - www.toanmath.com262 Boå ñeà 2. , , 0. Khi ñoù, ta coù xyz > 2 2 2 2 22 2 22 () ( )( )2( ) 2 22 xy z yz xzx y xy z x yz x yz y zx z xy + + + ++ + +≤ ++ + ++ Chöùng minh. Ta coù 2 22 2 22 2 22 () 2() 22 ( ) 2() 22 () 2() 22 x yz yzyz yz xyz xyz y zx zxzx zx y zx y zx z xy xyxy xy z xy z xy ++ = +- ++ ++ =+- ++ ++ =+- ++ Do ñoù 2 22 () () 2() 22 cyc cyc x y z yzyz x yz x yz x yz ⎛⎞ ++ = + +- ⎜⎟ ⎜⎟ ++ ⎝⎠ ∑∑ 2 2 22 2 ()2() 2 cyc xy z xyz xyz x yz + ++ ⇒≤ ++ + ∑ 2 22 2 2 22 2 2 () () 2 () 2 ()2() 2 cyc cyc cyc yzy z x yz xyz xyz x yz yzy z x yz x yz x yz x yz yz y z xy yz zx x yz x yz + ++ ⇔++-≤ ++ + + ++ ⇔ ≥ + +- ++ + + ++ ⇔≥ ++ + ∑ ∑ ∑ AÙp duïng baát ñaúng thöùc Bunhiacopxki, ta coù 2 22 2 2 ( ) ( ( )) 2 ( 2)( ) ( ( ) ( ) ( )) ( 2)() cyc cyc cyc yzy z xyxy x yz z xyxyx y xy x y yz y z zx z x z xyxyx y ++ = + ++ + + +++ ≥ ++ ∑∑ ∑ Tìm tài liệu Toán ? Chuyện nh ỏ - www.toanmath.com263 2 2 2 2 2 22 ( ( ) ( ) ( )) 2()() 2() (theo Boå ñeà 1) xy x y yz y z zx z x x y z x y yz zx xy yz zx xyz + + + ++ = ++ ++ ++ ≥ ++ Boå ñeà 2 ñöôïc chöùng minh hoaøn toaøn. Boå ñeà 3. , , 0. Khi ñoù, ta coù abc > 2 22 333 1 11 3 33 . .. 1 11 1 11 1 11 bc c a ab abc a bc b ca c ab abc abc abc + ++ + + ≤ ++ + ++ ++ ++ ++ Chöùng minh. Ñaët 1 11 , , ,,0 x y z xyz a bc = = =⇒> . Khi ñoù, ta coù 3 3 3 22 3 3 3 2 2 ( )() 3 2() . 1 11 ( )() 24 () . 2 2 (theo bñt AM-GM) b c x y z x yz x x yz y z xyz a bc abc x y z x yz x xyz xyzx yz x yz + + ++ = + +++ + ++ + ++ ≤ + +++ = + Töông töï, ta coù 2 2 3 () . 3 2 2 . 1 11 c a x y zy zx y zx b ca abc + +++ ≤ + + ++ 2 2 3 () . 3 2 2 . 1 11 a b xy zz xy z xy c ab abc + +++ ≤ + + ++ Do ñoù 2 2 3 () . 3 2 2 . 1 11 cyc cyc b c x yz x yz x yz a bc abc + +++ ≤ + + ++ ∑∑ Tìm tài liệu Toán ? Chuyện nh ỏ - www.toanmath.com264 2 22 2 22 222 2() . 2 1 11 (theo Boå ñeà 2) x y zxy z x yz x yz abc + + ++ ≤ ++ = ++ = ++ Boå ñeà 3 ñöôïc chöùng minh hoaøn toaøn. Trôû laïi baøi toaùn cuûa ta AÙp duïng baát ñaúng thöùc AM-HM, ta coù 3 33 11 11 1 11 1 (do ) abc abc abc abc ≥ ⇒≥= ++ ++ Do ñoù 3 222 3 1 11 3 . 1 11 (theo Boå ñeà 3) ñpcm. cyc cyc bc bc a bc abc a bc abc ++ ≤ ≤ ++ + + ++ ⇒ ∑∑ Ñaúng thöùc xaûy ra khi vaø chæ khi 1. abc === Baøi toaùn 26. (Voõ Quoác Baù Caån) Cho ,, 0 ( )( )( )0 thoûa . Chöùng minh raèng abc abbc c a ≥ + + +> 222 2 2 2 ( ) () ( ) ( )( )() 2 214 ()( )() abc bca cab abcabbcca a bc b ca c ab a bc b ca c ab + + + + ++ + + ≥ ++ + + + +++ Lôøi giaûi. Ñaët 2 22 ( ) ( ) () ,, abc bc a cab x yz a bc b ca c ab + ++ = == + ++ . Khi ñoù, baát ñaúng thöùc caàn chöùng minh töông ñöông vôùi 2 214 x y z xyz ++≥ ++ Ta seõ chöùng minh 2 22 2 2 2 2 22 2 (1) 1 (2) x yz x y yz zx + +≥ ++≥ Tìm tài liệu Toán ? Chuyện nh ỏ - www.toanmath.com265 Khi ñoù, ta coù 2 2 22 2 2 2 2 22 ( ) 2() 22() 22 2() 22 4 2 x y z x y z xy yz zx xy yz zx x y y z z x xyz x y z x yz + + = + + + ++ ≥ + ++ = + + + + ++ ≥+ = ⇒ + +≥ Vaø do ñoù 2 2 2 2 2 22 2 2 2 2 22 ()22 2() 224 2 214 2 214 x y z x y yz zx xyzx y z xy yz zx xyz xyz x y z xyz ++ ≥ + + + + ++ ≥+ + ++ ≥++ ⇒++≥ ++ Ñaây chính laø ñieàu chuùng ta caàn phaûi chöùng minh. Vaäy nhieäm vuï cuûa chuùng ta baây giôø chæ laø chöùng minh tính ñuùng ñaén cuûa caùc baát ñaúng thöùc (1) vaø (2) nöõa thoâi. * Chöùng minh (1). 222 ( ) ( ) () (1)2 abc bc a cab a bc bcac ab + ++ ⇔ ++≥ +++ Ta coù 2 22 2 222 ( ) ( ) ( ) ( ) ( )( )( ) 20 ()( )() cyc abc ab bc ca abcabbcc a a bc a bc b ca c ab + - - - + + ++ -=≥ + +++ ∑ 2 () 2 cyc abc a bc + ⇒≥ + ∑ Vaäy (1) ñuùng. * Chöùng minh (2). 22 22 2 2 ( )( ) ( )( ) ( )( ) (2)1 ( )( )( )( )()() aba cbc bcb a c a cacb ab a bc b ca b ca c ab c ab a bc + + + + ++ ⇔+ +≥ + + + + ++ Tìm tài liệu Toán ? Chuyện nh ỏ - www.toanmath.com266 Ta coù 22 222 22 ()( ) 2 ( )( )() 10 ( )() ( )()( ) ( )() 1 ( )() cyc cyc abacbc abcabbcc a a bc b ca a bc b ca c ab abacbc a bc b ca + + + ++ -=≥ ++ ++ + ++ ⇒≥ ++ ∑ ∑ Vaäy (2) ñuùng. ⇒ ñpcm. Ñaúng thöùc xaûy ra khi vaø chæ khi ( , , ) ( , ,0) ( 0). abc tt t => Baøi toaùn 27. (Voõ Quoác Baù Caån) Cho,,0 1( )( )( ) 0 thoûa vaø . abc a bc abbcc a ≥ + + = + + +> Chöùng minh raèng 2 22 5 55 8 abbcca bc c a ab + ++ + +≥ + ++ Lôøi giaûi. Ta coù Boå ñeà sau Boå ñeà. ,, laø caùc soá thöïc thoûa xyz 0 0 xyz xy yz zx + +≥ ⎧ ⎨ ++≥ ⎩ . Khi ñoù, ta coù 2 22 ( )( ) () 0 ,, xb c yc a za b abc R - + - + - ≥ ∀∈ Boå ñeà treân chöùng minh raát ñôn giaûn (chæ caàn duøng tam thöùc baäc hai laø ñöôïc) neân ôû ñaây ta khoâng nhaéc laïi chöùng minh cuûa noù. Trôû laïi baøi toaùn cuûa ta Ta coù baát ñaúng thöùc caàn chöùng minh töông ñöông vôùi 2 22 1 5 30 cyc cyc aa b c ab ⎛ ⎞⎛⎞ - + -≥ ⎜ ⎟⎜⎟ ⎜ ⎟⎜⎟ ++ ⎝ ⎠⎝⎠ ∑∑ Chuù yù raèng 2 22 2 ( )( ) () 1 ( )() ( )() (theo gt) cyc cyc cyc a a b abc ab abc bc acbc a cbc -++- = + + + =+ + + + ++ ∑ ∑∑ Tìm tài liệu Toán ? Chuyện nh ỏ - www.toanmath.com267 22 3 3 2 () 1 ( )() () 2 3 3( )( )( ) () 2 3 3( )( )( ) cyc cyc cyc cyc cyc cyc a ab bc acbc ab a ab abb cca ab a a b a bb cc a - ⇒ -= + ++ - =- + + ++ - ⇒ - =- + + ++ ∑∑ ∑ ∑ ∑ ∑ Do ñoù, baát ñaúng thöùc caàn chöùng minh töông ñöông vôùi 3 2 2 5() () 0 ()( )3( )( )() ( )(4 )0 cyc cyc cyc ab ab acbc abbcca a b ba - - -≥ + + + ++ ⇔ - -≥ ∑ ∑ ∑ Ñaët 4 , 4 ,4 a bc S c bS a cS b a = - = - =- . Khi ñoù, baát ñaúng thöùc caàn chöùng minh töông ñöông vôùi 222 ( )( )( ) 0 a bc S b c Sc a Sa b - + -+ -≥ Coù 2 tröôøng hôïp xaûy ra * Tröôøng hôïp 1. 00 b abc S ≥ ≥≥⇒ ≥ . Khi ñoù, ta coù 4 30 340 ab bc S S abc S S a bc + = - +≥ += +- ≥ Chuù yù raèng 2 22 ( ) ( ) () a c ab bc -≥ -+ - . Do ñoù 2 2 2 22 ( ) ( ) ( ) ( )( ) ( )( ) 0 a b c ab bc S b c S c a S ab S S b c S S ab - + -+ - ≥ + - ++ -≥ * Tröôøng hôïp 2.0 ,0 ac abc S S ≤≤≤⇒≥ . Neáu 0 b S ≥ thì ta coù ngay ñpcm, do ñoù ta chæ caàn xeùt 0 b S ≤ laø ñuû. + Tröôøng hôïp 2.1.2. Khi ñoù, ta coù: bc ≥ 2 8 20 2 6 4 20 ab cb SSab c S S abc + = - +≥ + = +-≥ Tìm tài liệu Toán ? Chuyện nh ỏ - www.toanmath.com268 Maët khaùc, theo baát ñaúng thöùc Bunhiacopxki, ta coù 2 22 ( ) 2( ) 2( ) c a a b bc - ≤ - +- Do ñoù 2 2 2 22 () ( ) ( ) ( 2)() (2)( ) 0 a b c ab cb S b c S c a S ab S S bc S S ab - + -+ -≥ + - ++ -≥ + Tröôøng hôïp 2.2. 2 cb ≥ - Tröôøng hôïp 2.2.1. ( ) 22 31 3 3( ) () a c b b c ca + - ≥ ⇔ - ≥- . Khi ñoù, ta coù 3120 ab S S abc +=-+≥ Do ñoù 2 2 22 ( ) ( ) ( )( 3 )( )0 a b c ab S b c S c a S ab S S bc - + -+ -≥ + - ≥ - Tröôøng hôïp 2.2.2. ( ) 312 313. 5 3 c a cbbc - +-≤ ⇒≥≥ Khi ñoù, ta coù 2 22 22 2 2 2 3( )0 (4 )(4 ) (4 )(4 ) (4 )(4 ) 13( ) 4( ) 13 4( ) 2 13. . 4 52 5 0 abc ab bc ca S S S abc SS SS SS c b ac ac ba ba cb ab bc ca a b c bc bc cc cc c + + = + +≥ + + =- - + - - +- - = ++ - ++ ≥ -+ ⎛⎞ ⎛⎞ ≥ -+⎜⎟ ⎜⎟ ⎜⎟ ⎝⎠ ⎝⎠ = ≥ AÙp duïng boå ñeà treân vôùi ,, a bc x S y S zS = == , ta suy ra 222 ( )( )( ) 0 a bc S b c Sc a Sa b - + -+ -≥ Toùm laïi, trong moïi tröôøng hôïp, ta luoân coù 222 ( )( )( ) 0 a bc S b c Sc a Sa b - + -+ -≥ (ñpcm) Tìm tài liệu Toán ? Chuyện nh ỏ - www.toanmath.com269 Ñaúng thöùc xaûy ra khi vaø chæ khi 1 . 3 abc = == Baøi toaùn 28. (Phaïm Kim Huøng) Cho ,, 0 ( )( )( )0 thoûa . Chöùng minh raèng abc abbc c a ≥ + + +> 222 ( ) ( ) () 2 abc bc a cab a bc bcacab + ++ ++≥ +++ Lôøi giaûi. Ta coù 2 caùch giaûi * Caùch 1. (tham khaûo lôøi giaûi baøi toaùn 26) * Caùch 2. Khoâng maát tính toång quaùt coù theå giaû söû . abc ≥≥ Baát ñaúng thöùc caàn chöùng minh töông ñöông vôùi 2 22 ( )()( )() ( ) a bac b cac bc a a bc c ab b ca - - - -+ +≤ + ++ Ta coù 2 2 22 2 2 2 2 ( )()( )() () () ()(2) 2 abac bcac ab bc ac a bc c ab a bc c ab ab bc ac ab a a c ab b ac ab ab b ac ab - - - - -- ⎛⎞ + =-+ ⎜⎟ + + ++ ⎝⎠ -- ⎛⎞ ≤-+ ⎜⎟ ⎝⎠ - -- = -- ≤ Do ñoù, ñeå chöùng minh baát ñaúng thöùc ñaõ cho, ta chæ caàn chöùng minh 2 2 2 22 2 2 22 2 () (2 )( ) () ( ) 2() (ñuùng theo bñt AM-GM) ab b ac b c a ab b ca ab b ac b ca ab c a a b b a c a b bac --+ ≤ + ⇔ -- + ≤+ ⇔ - + ≥- ⇒ ñpcm. Tìm tài liệu Toán ? Chuyện nh ỏ - www.toanmath.com270 Ñaúng thöùc xaûy ra khi vaø chæ khi ( , , ) ( , ,0) ( 0). abc tt t => Baøi toaùn 29. (Phaïm Kim Huøng) Cho,,02 thoûa . Tìm giaù trò lôùn nhaát cuaû bieåu thöùc abc abc ≥ ++= 2 22 222 ( )( )() P a ab b b bc c c ca a = - + -+ -+ Lôøi giaûi. Khoâng maát tính toång quaùt giaû söû 0. Khi ñoù, ta coù abc ≥≥≥ 2 22 2 22 0 0 a acca b bccb ≤ -+≤ ≤ -+≤ Do ñoù 22 2 2 22 ( )(3), (trong ñoù ) P a b a ab b v u v u a b v ab ≤ - += - =+= AÙp duïng baátñaúng thöùc AM-GM, ta coù 3 2 6 6 68 22 33 3 4 4 4( ) 4( ) 2 22 ( 3). 9 3 243 243 243 243 vv uv u a b abc Pvuv ⎛⎞ + +- ⎜⎟ + ++ ≤-≤ == ≤= ⎜⎟ ⎜⎟ ⎝⎠ Ñaúng thöùc xaûy ra khi vaø chæ khi 42 , ,0 33 abc === vaø caùc hoaùn vò. Vaäy 8 2 max 243 P = Baøi toaùn 30. (Voõ Quoác Baù Caån) Cho,,04 x y z xy yz zx xyz > + + += thoûa . Chöùng minh raèng 1 1 11 5 15 15 1 2 xyz + +≥ + ++ Lôøi giaûi. Ta coù 111 2 22 x yz + += + ++ Tìm tài liệu Toán ? Chuyện nh ỏ - www.toanmath.com271 (2 )(2 ) (2 )(2 ) (2 )(2 ) (2 )(2 )(2 ) 12 4( ) ( ) 8 4( ) 2( ) 84( )( )4 8 4( ) 2( ) 84( )( )( ) 8 4( ) 2( x y yz zx x yz x y z xy yz zx x y z xy yz zx xyz x yz xy yzzx x y z xy yz zx xyz x y z xy yz zx xy yz zx xyz x y z xy + + + + + + ++ = + ++ + + + + ++ = + + + + + ++ + ++ + + ++ = + + + + + ++ + + + + ++ + ++ + = + ++++ ) 8 4( ) 2( ) 8 4( ) 2( ) 1 (theo gt) yz zx xyz x y z xy yz zx xyz x y z xy yz zx xyz ++ + + + + + ++ = + + + + + ++ = Vaäy 111 1 2 22 x yz ++= + ++ Ñaët 1 11 ,, 2 22 a bc x yz === +++ thì ta coù 1 0 ,, 2 1 1-2 1 2 12 ,, abc abc a bc x yz a bc ⎧ << ⎪ ⎪ ++= ⎨ ⎪ -- ⎪= == ⎩ Do ñoù 11 5(1 2) 5 1 59 1 a a xa a == - +- + Töông töï, ta coù 1 5 1 59 1 5 1 59 a yb c zc = +- = +- 111 5 1 5 1 5 1 5 9 59 59 abc xy z abc ⇒++ = + + + + + - -- 19 99 . 9 59 59 59 1 (9 5) 5 (9 5) 5 (9 5) 5 . 9 59 5 9 59 abc abc abc abc ⎛⎞ = ++ ⎜⎟ - -- ⎝⎠ -+ - + -+ ⎛⎞ = ++ ⎜⎟ - -- ⎝⎠ Tìm tài liệu Toán ? Chuyện nh ỏ - www.toanmath.com272 15 1 11 . 3 9 59 59 59 159 . 3 9(59) (59) (59) 15 3 15 9( ) 15 3 159 1 2 abc abc abc ⎛⎞ =-+ ++ ⎜⎟ - -- ⎝⎠ ≥-+ - + - +- =-+ - ++ =-+ - = Vaäy 1 1 11 5 15 15 1 2 xyz + +≥ + ++ (ñpcm) Ñaúng thöùc xaûy ra khi vaø chæ khi 1. xyz = == * Nhaän xeùt. Coù theå ñaây laø moät baøi toaùn khoâng khoù nhöng ñieàu ñaëc saéc cuûa noù chính laø ôû choã töø giaû thieát 4 xy yz zx xyz + + += ta coù theå suy ra ñöôïc ñaúng thöùc 111 1 2 22 x yz ++= + ++ Vaø chính nhôø ñaúng thöùc naøy maø baøi toaùn cuûa ta ñaõ trôû neân cöïc kyø ñôn giaûn. Baèng caùch söû duïng ñaúng thöùc naøy, ta coù theå deã daøng chöùng minh ñöôïc caùc keát quaû sau (1) 1 1 1 1 11 (2) 10 2 2 2 2 22 m mm n nn x y z xy yz zx mn x y z x yz + + ≥ ++ + + ≤ + + ∀ > >> + + + + ++ Tìm tài liệu Toán ? Chuyện nh ỏ - www.toanmath.com273 Baøi toaùn 31. (Voõ Quoác Baù Caån) Cho,,0. Chöùng minh raèng abc > 2 2 2 222 2 22 22 22 ( ) () () 3( ) 2() 2 ( ) 2 ( ) 2( ) bc a c ab abc a b c a bc b c a c ab abc + - + - + - ++ ++≥ + + + + + + ++ Lôøi giaûi. Do caû hai veá cuûa baát ñaúng thöùc ñaõ cho ñoàng baäc neân khoâng maát tính toång quaùt, ta coù theå giaû söû 3. abc ++= Khi ñoù, baát ñaúng thöùc caàn chöùng minh töông ñöông vôùi 2 22 2 22 2 2 2 2 22 2 22 2 22 2 22 (32) (32) (32) 1 .() 6 2 (3 ) 2 (3 ) 2 (3 ) 2(3 2 ) 2(3 2 ) 2(3 2 ) 2 3 2 3 23 abc abc a a b b cc abc abc a ab bc c - -- + + ≥ ++ +- +- +- - -- ⇔ + + ≥ ++ - + - + -+ Ta seõ chöùng minh 2 2 2 2(3 2 ) 6 6 (0,3) (*) 23 x x xx xx - ≥ - + ∀∈ -+ Thaät vaäy 222 2 (*) 2(3 2 ) ( 6 6)( 2 3) (1)(6)0 03 (ñuùng do ) x x x xx xxxx ⇔ - ≥ -+ -+ ⇔ - - ≥ << Vaäy (*) ñuùng. Do ñoù, ta coù 2 2 2 2 2 2 2 2 2 2(3 2 ) 66 23 2(3 2 ) 66 23 2(3 2 ) 66 23 a aa aa b bb bb c cc cc - ≥-+ -+ - ≥-+ -+ - ≥-+ -+ 2 22 2 22 2 22 2 22 2(3 2 ) 2(3 2 ) 2(3 2 ) 6( ) 18 2 3 2 3 23 abc a b c abc a ab bc c abc - -- ⇒ + + ≥ + + - + ++ - + - + -+ = ++ Tìm tài liệu Toán ? Chuyện nh ỏ - www.toanmath.com274 2 22 2 22 2 22 2(3 2 ) 2(3 2 ) 2(3 2 ) 2 3 2 3 23 (ñpcm) a bc abc a ab bc c - -- ⇒ + + ≥ ++ - + - + -+ Ñaúng thöùc xaûy ra khi vaø chæ khi . abc == Baøi toaùn 32. (Voõ Quoác Baù Caån) Cho,,0. Chöùng minh raèng abc > 222 3332 2 22 1 2() .4 3 a b c a b c abc ab bc ca abc abc ⎛⎞ + + + + ++ + +≥ ⎜⎟ ++ ++ ⎝⎠ Lôøi giaûi. Do hai veá cuûa baát ñaúng thöùc ñaõ cho ñoàng baäc neân khoâng maát tính toång quaùt, ta coù theå giaû söû 2 22 3. abc ++= Ñaët p ab bc ca = ++ thì ta coù 0 3. p <≤ Khi ñoù, ta coù 222 3332 2 22 1 2() . 3 a b c a b c abc ab bc ca abc abc ⎛⎞ + + + + ++ + += ⎜⎟ ++ ++ ⎝⎠ 2 22 33 3 3 1 2(3 2 ) . .(3 )3 33 311114 . .(3)5 33 3194 . .(3 ) 5 33 1244 93 8 14 4 93 8 14 4.2 . 3 93 4 1 . 3 (theo bñt AM-GM) abcp p p abc p p p ab bc ca p p pp p p p pp p p a b c abc ab bc ca +++ ⎛⎞ = + - ++ ⎜⎟ ⎝⎠ ⎛⎞ ⎛⎞ = + + + - ++ ⎜⎟ ⎜⎟ ⎝⎠ ⎝⎠ ⎛⎞ ≥ + - ++ ⎜⎟ ⎝⎠ = +- ⎛⎞ = + +- ⎜⎟ ⎝⎠ ≥+- = + + ++ ⇒+ ++ 2 2 22 2() 4 (ñpcm) abc abc abc ⎛⎞ ++ +≥ ⎜⎟ ++ ⎝⎠ Ñaúng thöùc xaûy ra khi vaø chæ khi . abc == Tìm tài liệu Toán ? Chuyện nh ỏ - www.toanmath.com275 Baøi toaùn 33. (Voõ Quoác Baù Caån) Cho , , 0. Chöùng minh raèng abc > 1 1 13 3 2 32 325 a a b b b c c c a abc + +≥ + ++ Lôøi giaûi. Ñaët 1 11 , , ,,0 thì ta coù . x y z xyz a bc ===> Khi ñoù, baát ñaúng thöùc caàn chöùng minh töông ñöông vôùi 3 3 2 3 2 3 25 3 5 5 .3 2 5 .3 2 5.3 2 xyz zx yz xy zx yz xy xyz z x y xyz y zx + +≥ + ++ ⇔ + +≥ + ++ AÙp duïng baát ñaúng thöùc AM-GM vaø baát ñaúng thöùc Bunhiacopxki, ta coù 5.3 2 5 .3 2 5.3 2 2 3 2 5 5 32 2 53 xyz z x y x yz y zx xyz x yz xy z x yz ++≥ + ++ ⎛⎞ ≥ ++ ⎜⎟ + + + + ++ ⎝⎠ 2 2 2 22 2 2 22 2 2 22 2 22 2 2 22 2() (32 5) (5 3 2)(25 3) 2() 3( )7() 2() 1 20 3( ).().() 33 2() 1 20 3( ) .( ) .( ) 33 3() 5(2 xyz xx yz yxy z z x yz x yz x y z xy yz zx x yz x y z xy yz zx xy yz zx x yz x y z x y z xy yz zx x yz x yz ++ ≥ + + + + + + ++ ++ = + + + ++ ++ = + + + ++ + ++ ++ ≥ + + + + + + ++ ++ = + ++ 3 5 2 2) xy yz zx = ++ 3 5 5 .3 2 5 .3 2 5.3 2 xyz zx y x y z yz x ⇒+ +≥ + ++ Tìm tài liệu Toán ? Chuyện nh ỏ - www.toanmath.com276 ñpcm. ⇒ Ñaúng thöùc xaûy ra khi vaø chæ khi . x y z abc = =⇔ == Baøi toaùn 34. (R. Stanojevic ù) Cho,,0 abc > thoûa 1. Chöùng minh raèng abc = 111 2 111111 222 acc cbb + +≥ ++++++ Lôøi giaûi. Do 1 abc = neân toàn taïi caùc soá,,0 xyz > sao cho ,, xzy a bc yxz = == , chaúng haïn 3 33 2 22 , ,. x ca y bc z ab = == Khi ñoù, ta coù 1 12 22 111 22 y x yz xz a c yy == ++ + + ++ Töông töï, ta coù 12 22 11 2 z x yz c b = ++ ++ 12 22 11 2 x x yz b a = ++ ++ Khi ñoù, baát ñaúng thöùc caàn chöùng minh trôû thaønh 1 2 2 2 2 22 xyz xy z y zx zxy ++≥ + + + + ++ AÙp duïng baát ñaúng thöùc AM-GM, ta coù 22 22 ( 22) 2 . 22 x x xx x y z x x y z xyz x x yz = ≥= + + + + + ++ ++ Töông töï, ta coù Tìm tài liệu Toán ? Chuyện nh ỏ - www.toanmath.com277 22 22 yy yzx x yz zz z x y x yz ≥ + + ++ ≥ + + ++ 2 2 2 2 22 1 xyz x yz yz x zx y xyz xy z xy z xy z ⇒ ++≥ + + + + ++ ≥+ += ++ ++ ++ ñpcm. ⇒ Baøi toaùn 35. (Taiwanese Mathematical Olympiad 1992) Cho 3, vaø n nN ≥∈ 1 2 12 , ,..., 0 ... 1. thoûa Chöùng minh raèng nn xx x xxx ≥ + + += 2 22 12231 4 ... 27 n xx xx xx + ++≤ Lôøi giaûi. Ñaët 2 22 12 12231 ( , ,..., ) ... nn f x x x xx xx xx = + ++ Khoâng maát tính toång quaùt, coù theå giaû söû 1 max ( 1,) i x xin == . Goïi max ( 2, ) ki x xin == . Khi ñoù, ta coù 1 2 11 2 0 20 2 1 2 11 2 1 12 11 2 22 12 23 11 (1 , ,0,0,...,0) ( ... ... , ,0,0,...,0) ( ... ... ) ( 2 ( ... ... )) ... max ( soá soá (do k k k k nk nn k k nk k k nk n ki f x x fx x x x xx x x x x xx x xx x x xx xx xx xx x x x -+ -- -+ -+ - = + + + + ++ = + + + + ++ ≥ + + + + ++ ≥ + + + ≥= 1 4 2 43 1 4 2 43 2, )) in = 12 ( , ,..., ) n fxxx = Ta laïi coù 3 22 20 2 2(1) 1 14 (1 , ,0,0,...,0) (1 ) .(2 ).(1 ) . 2 2 3 27 soá kk kk kk kk n xx f xx xx xx - +- ⎛⎞ - =-=-≤= ⎜⎟ ⎝⎠ 1 4 2 43 Tìm tài liệu Toán ? Chuyện nh ỏ - www.toanmath.com278 Vaäy 12 4 ( , ,..., ) 27 (ñpcm) n fxxx ≤ . Baøi toaùn 36. Cho,,01 thoûa . Chöùng minh raèng a b c a b c abc ≥ +++ = (2 )(12) 7 abc abc ab bc ca abc ++ + +≤ - Lôøi giaûi. Ñaët 3 1. 3 abc m m abc ++ = ⇒ += Khi ñoù, ta coù 33 22 3 2 (2 )(1 2 ) (3 3 )(3 6 ) 3(1 )(1 2 ) 7 632 3(1 )(1 2 ) 3(1 3 ) 3( ) 33 2 22 3() 3 2 abc abc m m m m A abc m m m m m m abc m Amm m mm abc m Am m + + - - -- = == - ++ -- --- ⇒-= -= = + ++ - ⇒=+ + Do ñoù, baát ñaúng thöùc caàn chöùng minh töông ñöông vôùi 3 2 3 2 3 2 23 3() 3 2 3() 3 2 ( ) 27 3( )9() 2 3(2 )(( ) 3( )) ( ) 27 abc m m ab bc ca m m abc m ab bc ca m a b c abc a b c ab bc ca m m a b c ab bc ca a b c abc - + ≥ ++ + - ⇔ - - -≥ + ++- ⇔ ++ - + +≥ + ⇔ + ++ - + + ≥ ++ - Do 1 7( )4() 3 1 27 3 33 neân abc abc m abc m m m ++ ++ += ≤⇒+≥=≥ . Do ñoù, ta chæ caàn chöùng minh 23 33 3 4( )(( ) 3( )) ( ) 27 4 3 3 ( ) 21 3 9 3 () cyc cyc cyc cyc cyc abc ab c ab bcca a bc abc a abc a ab a b abc a abc ab a b ++ ++ - + + ≥ ++ - ⎛⎞ ⇔ - ≥ + +- ⎜⎟ ⎜⎟ ⎝⎠ ⇔ + ≥+ ∑ ∑∑ ∑∑ Tìm tài liệu Toán ? Chuyện nh ỏ - www.toanmath.com279 3 3 () (ñuùng theo Schur) ñpcm. cyc cyc a abc ab a b ⇔ + ≥+ ⇒ ∑∑ Baøi toaùn 37. (Voõ Quoác Baù Caån) Cho , , 0 2 4. thoûa Chöùng minh raèng a b c abc ab bc ca > = + ++ 7 77 222 128 3 1 11 abc ab bc ca + +≥ + ++ Lôøi giaûi. AÙp duïng baát ñaúng thöùc Bunhiacopxki, ta coù 7 7 7 7 77 222 2 2 2 7 2 12 7 2 12 7 2 12 2 2 22 7 2 12 7 2 12 7 2 12 2 1 11 ( . . .) ( . . .) ( )(1) abc ac ba cb ab bc ca c ab c a abc b ca b ac b a cb a b c a bc ab c abc a c b a cb a b c abc + + = ++ + + + +++ ++ ≥ +++++ ++ = +++ AÙp duïng baát ñaúng thöùc AM-GM, ta coù 7 2 12 7 2 12 7 2 12 2 72 12 72 12 72 12 2 7 2 12 7 2 12 7 2 12 2 7 2 12 7 2 12 7 2 12 7 2 12 7 2 12 7 2 12 23 . 11 . 9 . 43 23 . 11 . 9 . 43 23 . 11 . 9 . 43 43( . . . ) 43 ( ) . . . () ac ba c b abc ba c b ac abc c b a c b a abc a c b a c b abc a b c a c b a c b abc a b c + +≥ + +≥ + +≥ ⇒ + + ≥ ++ ⇔ + + ≥ ++ Do ñoù 7 7 7 72 12 72 12 72 12 2 222 2 2 22 ( . . .) ( )(1) 1 11 ( ( )) ( )(1) () 1 a b c ac ba c b a b c abc ab bc ca abc a b c a b c abc abc abc abc ++ + +≥ +++ + ++ ++ ≥ +++ ++ = + Ta coù Tìm tài liệu Toán ? Chuyện nh ỏ - www.toanmath.com280 44 2 4 4 . . .4 42 8 abc ab bc ca abbcca abc abc = +++ ≥ = ⇒≥ Do ñoù 2 22 2 22 3 222 ( ) .3 11 6 1 abc a b c abc abc abc abc abc abc ++ ≥ ++ ≥ + 22 6.8 ( ) (0, )) 181 (do ñoàng bieán treân t ft t ≥ = +∞ ++ 128 3 = 7 7 7 222 222 ( ) 128 13 1 11 abc abcabc abc ab bc ca ++ ⇒++=≥ + + ++ (ñpcm) Ñaúng thöùc xaûy ra khi vaø chæ khi 2. abc = == Baøi toaùn 38. (Phaïm Kim Huøng) Cho,,01 thoûa . Chöùng minh raèng a b c abc >= 2 22 2 22 1 1 12 1 (1 )(1 )(1 ) (1 ) (1 ) (1 ) 3 33 3 (1 ) (1 ) (1 ) abc abc abc abc +++≥ + ++ + ++ + ++ ++≥ + ++ a) b) Lôøi giaûi. a) Ñaët 2 22 1, 1, 1 , , [ 1,1] 1 11 x y z x yz a bc = - = - = - ⇒ ∈- + ++ 2 22 (1 )(1 )(1 ) 2 2 2 1 11 8 (1 )(1 )(1 ) 8 (1 )(1 )(1 ) xyz abc abc abc abc ⎛ ⎞⎛ ⎞⎛ ⎞ ⇒---=- -- ⎜ ⎟⎜ ⎟⎜ ⎟ + ++ ⎝ ⎠⎝ ⎠⎝ ⎠ = + ++ = + ++ (1 )(1 )(1 ) x yz =+ ++ Tìm tài liệu Toán ? Chuyện nh ỏ - www.toanmath.com281 0 x y z xyz ⇒+++= Do ñoù, baát ñaúng thöùc caàn chöùng minh trôû thaønh 2 22 2 22 2 22 2 22 2222 2 2 2 2 22 ( 1) ( 1) ( 1) ( 1)( 1)( 1) 4 3( )0 20 2( ) 2( ) 4 0 ( )40 40 xy zx y z x y z xy yz zx x y z xyz x y z xy yz zx xyz x y z xy yz zx xyz x y z x y z xyz x y z x y z xyz ++ ++ ++ + + + ≥ ⇔++ + + + + + + + ≥ ⇔++ + + + - ≥ ⇔+ + + + + -≥ ⇔++ + + + - ≥ ⇔++ + - ≥ AÙp duïng baát ñaúng thöùc AM-GM, ta coù 2 2 2 2 22 2 2 2 2 22 4 2 2 2 2 22 4 ... 44 40 (ñpcm) x y z x y z x y z x y z xyz xyz x y z x y z xyz +++ ≥ =≥ ⇒ + + + -≥ b) Ñaët 2 22 1, 1, 1 , , [ 1,1] 1 11 x y z x yz a bc = - = - = - ⇒ ∈- + ++ 2 22 (1 )(1 )(1 ) 2 2 2 1 11 xyz abc ⎛ ⎞⎛ ⎞⎛ ⎞ ⇒---=- -- ⎜ ⎟⎜ ⎟⎜ ⎟ + ++ ⎝ ⎠⎝ ⎠⎝ ⎠ 8 (1 )(1 )(1 ) abc abc = + ++ 8 (1 )(1 )(1 ) (1 )(1 )(1 ) 0 abc x yz x y z xyz = + ++ =+ ++ ⇒+++= Baát ñaúng thöùc caàn chöùng minh töông ñöông vôùi 2 22 2 22 ( 1)( 2) ( 1)( 2) ( 1)( 2) 0 3() 3 x x y y zz x y z xyz x y z xyz + ++ + +++ + ≥ ⇔++ ≥- + + ⇔++ ≥ AÙp duïng baát ñaúng thöùc AM-GM, ta coù 2 2 2 2 22 3 2 22 3 3 , , [ 1,1]) 3 (do (ñpcm) x y z x yz xyz xyz x y z xyz + + ≥ ≥ ∈- ⇒ + +≥ Tìm tài liệu Toán ? Chuyện nh ỏ - www.toanmath.com282 Baøi toaùn 39. (Voõ Quoác Baù Caån) Cho , , 0. Chöùng minh raèng abc ≥ 2 2 2 2 22 ( ) ( ) () 2 abc bc a cab bc c a ab + ++ + +≥ + ++ Lôøi giaûi. Ta seõ chöùng minh 22 2 2 22 () 2 (1) ( )() 1 (2) ( )() cyc cyc abc bc aba cbc a c bc + ≥ + ++ ≥ ++ ∑ ∑ Khi ñoù, ta coù 2 22 2 2 2 2 2 2 22 22 22 22 2 2 22 () ( ) ( ) () ( )() 2 ( )() ( )() 22 ( )() ( )() 224 ( )() cyc cyc cyc cyc ab c bc a cab abc abacbc bc ca ab bc ac bc abacbc ac bc abacbc a c bc ⎛⎞ + + + + ++ + + =+ ⎜⎟ + + + + ++ ⎝⎠ ++ ≥+ ++ ++ ≥+≥ ++ ∑∑ ∑ ∑ 2 2 2 2 22 ( ) ( ) () 2 abcbcacab bccaab + ++ ⇒+ +≥ + ++ Ñaây chính laø ñieàu ta caàn phaûi chöùng minh, vaäy nhieäm vuï cuûa ta baây giôø chæ laø chöùng minh tính ñuùng ñaén cuûa caùc baát ñaúng thöùc (1) vaø (2) nöõa thoâi. * Chöùng minh (1). Ta coù 2 2 2 2 2 22 22 2 2 22 2 2 22 ( )( 2)8 () 20 ( )( )( ) () 2 (1) ñuùng. cyc cyc cyc aba b a b c abc abc bc ab bc ca abc bc - + ++ + -=≥ + + ++ + ⇒≥ + ⇒ ∑ ∑ ∑ Tìm tài liệu Toán ? Chuyện nh ỏ - www.toanmath.com283 * Chöùng minh (2). Ta coù 222 22 22 2 2 22 2 2 ( )( ) 2 (( )( ) ) 10 ( )( ) ( )( )( ) cyc ab a c b c abc a b c a b c abc a c bc ab bc ca + + ++ + +- -=≥ + + + ++ ∑ ⇒ (2) ñuùng ⇒ ñpcm. Ñaúng thöùc xaûy ra khi vaø chæ khi ( , , ) ( , ,0) ( 0). abc tt t => * Nhaän xeùt. Ngoaøi ra, ta coøn coù moät baát ñaúng thöùc maïnh hôn nhö sau Cho , , 0. Khi ñoù abc ≥ 2 2 22 2 2 2 2 2 2 22 ( ) () ( ) ( )( )() 2 214 ( )( )() abc bca cab abca bbcca b c ca a b a b b c ca + + + + ++ + + ≥ ++ + + + + ++ Baøi toaùn 40. (Voõ Quoác Baù Caån) Cho , , 0 1. thoûa Chöùng minh raèng abc abc > ++= ( ) ( ) ( ) 1 2 3 2 3 23 b c c a ab a c ab b a bc c b ca + +≥ + ++ Lôøi giaûi. Ta coù baát ñaúng thöùc caàn chöùng minh töông ñöông vôùi 1 32 3 2 32 bc ca ab abc ca ab bc a bc b ca + +≥ + ++ Ñaët ,, bc ca ab xyz abc = == thì ta coù ,,0 11 (do ) xyz xy yz zx abc > ⎧ ⎨ ++= + + = ⎩ Baát ñaúng thöùc caàn chöùng minh trôû thaønh 1 2 32 32 3 x yz y yz z zx x xy + +≥ + ++ Tìm tài liệu Toán ? Chuyện nh ỏ - www.toanmath.com284 AÙp duïng baát ñaúng thöùc Bunhiacopxki, ta coù 2 22 2 3 2 3 2 32 3 2 32 32 3 () 2( )33 3() 2( )33 3 233 3 2 3 3. 3 1 x y z x yz y yz z zx x xy xy xyz yz xyz zx xyz x yz xy yz zx xyz xy yz zx xy yz zx xyz xyz xy yz zx + + = ++ + + + + ++ ++ ≥ + ++ ++ ≥ + ++ = + ≥ ++ ⎛⎞ + ⎜⎟ ⎝⎠ = 1 2 32 32 3 ñpcm. x yz y yz z zx x xy ⇒ + +≥ +++ ⇒ Baøi toaùn 41. (Voõ Quoác Baù Caån) Cho ,, xyz laø ñoä daøi ba caïnh cuûa moät tam giaùc (coù theå suy bieán). Tìm haèng soá k lôùn nhaát sao cho 2 22 1 1 1 ( )( )( ) ( )6 ( )( )( ) x y z kx y yz zx x yz x y z y z zx xy xyzxy yz zx ⎛⎞⎛⎞ - -- ++ ++≥ + ++ ⎜⎟⎜⎟ + + + + ++ ⎝⎠⎝⎠ Lôøi giaûi. Ta coù baát ñaúng thöùc ñaõ cho töông ñöông vôùi 22 2 22 ( )( 2)() ( )( )( ) (1) cyc x y z xz yz xy z x y kx y y z zx - + +- +≥ ≥ - -- ∑ Do ,, xyz laø ñoä daøi ba caïnh cuûa moät tam giaùc (coù theå suy bieán) neân toàn taïi caùc soá khoâng aâm ,, abc sao cho ,, x bcy c az ab = + = + =+ . Thay vaøo (1), ta coù baát ñaúng thöùc (1) trôû thaønh Tìm tài liệu Toán ? Chuyện nh ỏ - www.toanmath.com285 2 2 22 222 ( ) ( )( )( 2 ) .( ) ( ) ( ) (2) 2 cyc a b a b c ababab c k ab bc c a - + - + + ++≥ ≥ - -- ∑ Cho 0 c + → , ta ñöôïc (2) trôû thaønh 3 2 2 3 22 22 2 2 222 2 4 22 2 2 222 4 2 2 2 22 2( ( )(2 ) ( )( 2 ) ( )( )()) . () 2 2(( )() ()( )( )) . () 2(( ) ( )( ) ) . bb a ab aa b ab k ab a b ab a b ab ab a b ab ab a b ab ab kab ab ab ababab kab - + + - ++ +- ++ +≥- ⇔ - ++- ++ +≥- ⇔ + + + + +≥ Cho 1, 56. ta suy ra ñöôïc: abk ==≤ Ta seõ chöùng minh max 56, töùc laø chöùng minh k = 22 22 2 22 ( )( )( )( 2) 28( )( )( ) cyc ab a b c ab ab ab c ab bc ca - +-+ + ++ ≥ - -- ∑ Khoâng maát tính toång quaùt giaû söû 0 abc ≥ ≥≥ . Ta coù baát ñaúng thöùc treân töông ñöông vôùi 2 2 2 2 22 2 22 2 2 22 22 (( ) ( )( )(2 ) ( ) ( )( )( 2 )) ( )( )( )( 2) (( )( ) ( )( 2 ) ( ) ( )( 2 )) ( )( ) ( )(2 ) b c b a bc a bc ac a b ac a bc ab abababab c acc a c ac a b c c a b abab c bcc b c bc abc - - + ++ + - - + + ++ + - + + + +++ + +-+ + +- - + ++ + + + - + + +≥ 2 22 28( ) ( ) ( ) ab bc ca ≥ - -- Do 0 abc ≥ ≥≥ neân 22 ( )( ) ( )(2 ) 0 bcc b c bc abc + - + + +≥ (3) 2 2 22 22 22 22 2 222 ( )( )( )( 2 ) ()()( 2) 2()( )( 2 ) ()( )( 2) ( ) (2( )( 2 ) ( )( 2 )) ( )( 2 2( 2 ) 2)0 acc ac aca bc cab abab c ca b a ca bc ca b abab c cab aca bc abab c c a b a ab b c a b c bc +-+ ++- - + ++ ≥ ≥ -+ ++- -+ ++ = - + + + - + ++ = - + -+ + +-≥ Tìm tài liệu Toán ? Chuyện nh ỏ - www.toanmath.com286 22 22 ( )( ) ( )( 2 ) ( ) ( )( 2 ) (4) acc ac a ca bc c ab a b ab c ⇒ +-+ + +≥ ≥ - + ++ Laïi do 0 abc ≥ ≥≥ neân .( )0 a ac bc b - ≥ -≥ 2 2 2 2 22 2 2 2 2 2 22 2 22222 2 ( ) ( )( )(2 ) ( ) ( )( )( 2 ) ( ) ( )( )(2 ) .( ) ( )( )( 2 ) ()( )(( )( 2) ()(2 )) b c b a bc a bc ac a b a c a bc a bc b a bc abc b c a b a c a bc b bc a b a a c a bc b bc abc b ⇒- -+ +++- -+ + +≥ ≥- -+ +++ - -+ + + - - + + + - + ++ = 2 22 44 22 3 3 2 22 2 ( )()(()2()2( )2( ) ()) b c a b a b aba b ca b abca b c a b b - - -+-+ -+-+ - = 2 22 4 4 22 2 224 2 2 2 22 2 2 22 ( )()(()2()) ( )( )() ( ) ( ) .16 16( ) ( ) (theo bñt AM-GM) b c a b ab abab b b ca ba b b bc ab ab b ab bc a - - -+- ≥ - -+ = -- ≥ = -- 2 22 16( ) ( ) ( ) ab b c ac ≥ --- (5) Ta laïi coù 2 22 2222 2 2 22 2 22 ( )( )( )( 2) ( )( )() ( ) (2 )4 12( ) 12( )( )() (theo bñt AM-GM) a b a b ababab c a b a b abab a b ab ab ab a b ab ab bc ca - + + + ++≥ ≥- +++ ≥-+ =- ≥ - -- Töø (3),(4),(5) vaø (6), ta suy ra 22 22 2 22 ( )( )( )( 2) 28( )( )( ) cyc ab a b c ab ab ab c ab bc ca - +-+ + ++ ≥ - -- ∑ Vaäy max 56. k = Tìm tài liệu Toán ? Chuyện nh ỏ - www.toanmath.com287 Baøi toaùn 42. (Poland 2005) Cho , , [0,1] abc ∈ . Chöùng minh raèng 2 1 11 a bc bc ca ab + +≤ + ++ Lôøi giaûi. Do , , [0,1] abc ∈ neân 1 10 11 aa bc abc bc abc +≥ +> ⇒ ≤ ++ Töông töï, ta coù 11 11 bb ca abc cc ab abc ≤ ++ ≤ ++ 1 1 11 a b c abc bc ca ab abc ++ ⇒ + +≤ + + ++ Do ñoù, ñeå chöùng minh baát ñaúng thöùc ñaõ cho, ta chæ caàn chöùng minh 2 1 2(1) abc abc a b c abc ++ ≤ + ⇔ ++≤ + Do , [0,1] ab ∈ neân(1)(1)01 a b a b ab - - ≥ ⇒ + ≤+ 1 abc abc ⇒ ++≤ + + Laïi do , , [0,1] abc ∈ neân (1 )(1 ) 0 1 ab c ab c abc - - ≥ ⇒ + ≤+ 1 2 2(1) ñpcm. a b c ab c abc abc ⇒ ++≤ + +≤ + ≤ + ⇒ Baøi toaùn 43. (China 2006) Cho , , 0 1. thoûa Chöùng minh raèng xyz x yz > + += 2 2 xy yz zx xy yz yz zx zx xy + +≤ + ++ Lôøi giaûi. Ta coù baát ñaúng thöùc caàn chöùng minh töông ñöông vôùi Tìm tài liệu Toán ? Chuyện nh ỏ - www.toanmath.com288 2 2 xy y z zx z x x y yz + +≤ + ++ AÙp duïng baát ñaúng thöùc Bunhiacopxki, ta coù ( ) 1 .0 2 2. zx zx xy xy zx zx +≥ +> ⇒≤ ++ Töôïng töï, ta coù 2. 2. yz yz xy xy zx zx yz yz ≤ ++ ≤ ++ 2. xy xy yz z x yz zx zx xy yz zx x y yz ⎛⎞ ⇒ ++≤ + + ⎜⎟ ⎜⎟ + + + + ++ ⎝⎠ Do ñoù, ñeå chöùng minh baát ñaúng thöùc ñaõ cho, ta chæ caàn chöùng minh 1 2 1 2 1 2 12. cyc cyc cyc cyc xy y z zx z x x y yz xy xy xy zx xy xyz zx ++≤ + ++ ⎛⎞ ⇔ - +≤ ⎜⎟ ⎜⎟ + ⎝⎠ ⇔ ≤+ + ∑∑ ∑∑ 1 2 2. cyc cyc cyc xy x xyz zx ⇔ ≤+ + ∑ ∑∑ (*) Ñaët ,, axb ycz = == thì ta coù,,0 abc > . Khi ñoù, baát ñaúng thöùc (*) trôû thaønh 222 111 2 2() a b c abc ab bc ca ab bc ca ⎛⎞ + + + + + ≥ ++ ⎜⎟ + ++ ⎝⎠ (**) Tìm tài liệu Toán ? Chuyện nh ỏ - www.toanmath.com289 Do caû hai veá cuûa baát ñaúng thöùc treân ñoàng baäc neân khoâng maát tính toång quaùt, coù theå giaû söû 1. abc + += Ñaët 1 ,0 3 q ab bc ca r abc q = + + = ⇒ ≤≤ . Khi ñoù, ta coù 222 12 111 1 abcq q ab bc ca qr + + =- + + += + + +- Do ñoù 2(1) (**) 1 4 0 (1 4 )( ) 2 (1 ) 0 (1 4) (1 6) 0 rq q qr qqr rq q qrq + ⇔-+≥ - ⇔ - - + +≥ ⇔ - + +≥ Coù 2 tröôøng hôïp xaûy ra * Tröôøng hôïp 1. 1 0 4 q ≤≤ . Trong tröôøng hôïp naøy, baát ñaúng thöùc treân hieån nhieân ñuùng. * Tröôøng hôïp 2. 11 . 43 q ≤≤ AÙp duïng baát ñaúng thöùc Schur, ta coù 41 0 9 q r - ≥≥ . Do ñoù (4 1)(1 6 ) (4 1)(1 3 ) (1 4) (1 6 ) (1 4) 0 99 q q qq qq r q qq -+ -- -+ + ≥ -+ = ≥ Toùm laïi, ta luoân coù (1 4 ) (1 6 ) 0 ñpcm. q qrq - + +≥ ⇒ Ñaúng thöùc xaûy ra khi vaø chæ khi 1 . 3 x yz = == Tìm tài liệu Toán ? Chuyện nh ỏ - www.toanmath.com290 Baøi toaùn 44. (Phaïm Kim Huøng) Cho , , , 0. Tìm giaù trò nhoû nhaát cuûa bieåu thöùc abcd ≥ 2 2 22 a bcd P a bc bc d c d a d ab ⎛ ⎞⎛ ⎞⎛ ⎞⎛ ⎞ = + ++ ⎜ ⎟⎜ ⎟⎜ ⎟⎜ ⎟ + + + + + + ++ ⎝ ⎠⎝ ⎠⎝ ⎠⎝ ⎠ Lôøi giaûi. Ta seõ chöùng minh raèng 4 min. 9 P = Trong caùc soá ,,,, abcd goïi p laø soá lôùn nhaát, , soá lôùn nhaát trong 3 soá coøn laïi laøq soá lôùn nhaát trong 2 soá coøn laïi laø vaø laø soá nhoû nhaát. rs Khi ñoù, ta coù 1 1 11 pqr s pqr pq s pr s q r s ≥ ≥≥ ⎧ ⎪ ⎨ ≤ ≤≤ ⎪ ++ ++ + + + + ⎩ Do ñoù, theo baát ñaúng thöùc saép xeáp laïi, ta coù 2 2 22 2 2 22 a bcd P a bc bc d c d a d ab p qrs p q r pqs pr s qr s ⎛ ⎞⎛ ⎞⎛ ⎞⎛ ⎞ = + ++ ⎜ ⎟⎜ ⎟⎜ ⎟⎜ ⎟ + + + + + + ++ ⎝ ⎠⎝ ⎠⎝ ⎠⎝ ⎠ ⎛ ⎞⎛ ⎞⎛ ⎞⎛ ⎞ ≥ + ++ ⎜ ⎟⎜ ⎟⎜ ⎟⎜ ⎟ + + + + ++ ++ ⎝ ⎠⎝ ⎠⎝ ⎠⎝ ⎠ Khoâng maát tính toång quaùt, coù theå giaû söû 1. p q rs + + += Khi ñoù, ñeå chöùng minh 4 9 P ≥ , ta chæ caàn chöùng minh 2 2 22 2 2 22 4 9 (1 ) (1 ) (1 ) (1 ) p q rs s r qp + + +≥ --- - Ñaët 22 22 ,, (1 ) (1) ps m p sn pst sp =+= =+ -- thì ta coù01 m ≤≤ vaø 2 3 4 22 2 22 6 42 (1) m n m mnm mnn t mn - - + + -+ = -+ 2 22 (2 )2(1)(21 )(1)( ) 0 n t nm m t m mt ⇒ -- - - -+ - - = (*) Tìm tài liệu Toán ? Chuyện nh ỏ - www.toanmath.com291 + Neáu 2 t ≥ thì hieån nhieân ta coù 2 2 22 2 2 22 4 9 (1 ) (1 ) (1 ) (1 ) p q rs s r qp + + +≥ --- - + Neáu 1,0 2 mn t == ⎡ ⎢ = ⎣ thì ta coù 1 t ≥ vì 2 2 1 (1) p s = - . Do ñoù 2 2 22 2 2 22 4 9 (1 ) (1 ) (1 ) (1 ) p q rs s r qp + + +≥ --- - + Neáu 2,1 tm << . Xem (*) laø phöông trình baäc hai ñoái vôùi n. Do n luoân toàn taïi neân ta phaûi coù / 2 2 22 22 2 2 ( 1) (2 1 ) (2 )( 1) ( ) 0 (2 1) (2)() 2 41 (2) m m t tm mt m t tmt mm t m Δ= - - - - - - -≥ ⇔ -- ≥-- - +- ⇒≥ - Töông töï, ñaët 1 lqr lm = + ⇒ =- . Baèng laäp luaän töông töï nhö treân, roõ raøng ta chæ caàn xeùt tröôøng hôïp 1 l < vaø 22 22 2 (1 ) (1) qr rq +< -- laø ñuû. Khi ñoù, ta coù 2 2 22 2 2 22 2 4 1 12 (1 ) (1 ) (2 ) (1 ) q r llm r q lm - +-- + ≥= - - -+ Do ñoù 2222 22 2 2 2 2 22 22 22 2 4 1 12 (1 ) (1 ) (1 ) (1 ) (2 ) (1 ) (2 1) (11 10 10 ) 4 9 9(2 ) ( 1) 4 9 p q r s mmm s r q p mm m mm mm - + -- + + + ≥+ --- - - + - +- =+ -+ ≥ 4 9 P ⇒≥ Ñaúng thöùc xaûy ra khi vaø chæ khi . abcd = == Tìm tài liệu Toán ? Chuyện nh ỏ - www.toanmath.com292 Vaäy 4 min. 9 P = Baøi toaùn 45. (Phaïm Kim Huøng) Cho,,03 thoûa vaø laø moät haèng soá cho tröôùc. abc abc k > ++=∈R Tìm haèng soá k C nhoû nhaát sao cho () k kk k C a b c ab bc ca + + ≥ ++ Lôøi giaûi. Ta coù Boå ñeà sau Boå ñeà.,,03 thoûa abc abc > ++= . Khi ñoù, ta coù 2 2 3 ( ) ( ) ( ) max 3, 2 k kkk k ab bc ca k R ⎧⎫ + + ≤ ∀∈ ⎨⎬ ⎩⎭ Chöùng minh. Ta chöùng minh baát ñaúng thöùc ñuùng cho giaù trò tôùi haïn 2 2 3 ln3 3 2ln3 2ln2 2 k k k = ⇔= - Khi ñoù + mk ∀≥ , ta coù 22 22 33 ( ) ( ) () (( ) () ()) 22 m m km k m m m kkk k km ab bc ca ab bc ca ⎛⎞ + + ≤ + + ≤= ⎜⎟ ⎝⎠ 2 2 3 ( ) ( ) ( ) max 3, 2 m mmm m ab bc ca ⎧⎫ ⇒ + +≤ ⎨⎬ ⎩⎭ + , mk ∀< ta coù 11 2 2 (( ) () ()) 3 (( ) () ())3 .3 3 ( ) () () 3 3 ( ) ( ) ( ) max 3, 2 k k kk m m m kkk m m mm m mm m m mm m ab bc ca ab bc ca ab bc ca ab bc ca -- + + ≤ + + ≤= ⇒ + +≤ ⎧⎫ ⇒ + +≤ ⎨⎬ ⎩⎭ Tìm tài liệu Toán ? Chuyện nh ỏ - www.toanmath.com293 Khoâng maát tính toång quaùt, coù theå giaû söûabc ≤≤ . Ta chöùng minh veá traùi ñaït max khi . bc = Thaät vaäy, ñaët 2, 20 bc zc b t z t a zt + = - = ⇒ ≥ ≥∧ ≤- . Khi ñoù, ta coù ( ) 22 ( ) ( ) ( ) () k k kk VT a z t z t z t ft = + + - + -= Ta coù / 1 1 2 21 () (( ) ( ))2() kk kk f t ka z t z t tk z t - -- = + -- -- 1 ()0 Xeùt haøm soá vôùi k gxxx - =≥ . Ta coù /2 ( ) ( 1) k gxkx - =- //3 ()(1)(2)0 k gx k kx - =- -≤ / () ()( )() 0 theo ñònh lyù Larange, ta coùgx gy x yg y yx ⇒ - ≤ - ∀ ≤≤ , AÙp duïng cho , ta ñöôïc y z tx zt =- =+ 12 ( ) () 2( 1)() k kk zt zt tk zt -- + - - -≤ -- Do ñoù / 21 111 ()2() (( 1)()()) 2()((1)()) ) (do kkk kkk f t tkzt a k zt zt tkz t a k zt a zt -- --- ≤ - --+- ≤ - - - + ≤- 111 2()(())0) (do kkk tkzt a z t a zt zt --- ≤ - - + ≤ ≤- ≤+ [ ) 2 () ( ) (0) 2 (3 2 ) laø haøm nghòch bieán treân 0, k kk ft ft f b bb ⇒ +∞ ⇒≤ = -+ Baây giôø ta coøn phaûi chöùng minh 3 () 2 (32) 2 31 2 k kk gbb bbb = - - ≤ ∀≤< Ta coù 1 / 21 3 2 32 ()2 21 kk k bb g b kb bb - - ⎡⎤ -- ⎛ ⎞ ⎛⎞ = -+ ⎢⎥ ⎜ ⎟ ⎜⎟ ⎝ ⎠ ⎝⎠ ⎢⎥ ⎣⎦ Tìm tài liệu Toán ? Chuyện nh ỏ - www.toanmath.com294 Ñaët 32b x b - = (*) thì01 x <≤ vaø roõ raøng öùng vôùi moãi (0,1] x ∈ thì ta coù duy nhaát 3 1, 2 b ⎡⎞ ∈ ⎟ ⎢ ⎣⎠ thoûa maõn (*). Xeùt haøm soá 1 ( ) 2 1 (0,1] vôùi kk hxxxx - =- +∈ Ta coù /2 / ( ) ( 2( 1)) () () coù toái ña 1 nghieäm coù toái ña 2 nghieäm (theo ñònh lyù Rolle) k hx x kxk hx hx - = -- ⇒ ⇒ Ta laïi coù 1 8 15 1 23 (1) 0, 0, 0 82 82 kk kk hhh -- ⎛ ⎞ ⎛⎞ = = > =< ⎜ ⎟ ⎜⎟ ⎝ ⎠ ⎝⎠ 0 / 0 0 11 () , ,1 82 3 () ,1 2 hx xx gb bb x ⎛⎞ ⇒ ∈= ⎜⎟ ⎝⎠ ⇒ == + coù ñuùng 2 nghieäm laø coù ñuùng 2 nghieäm laø Baûng bieán thieân cuûa () gb Caên cöù vaøo baûng bieán thieân, ta thaáy 33 ( ) max (1), 3 1, 22 gb ggb ⎧⎫ ⎛ ⎞ ⎡⎞ ≤ = ∀∈ ⎨⎬ ⎜⎟⎟ ⎢ ⎝ ⎠ ⎣⎠ ⎩⎭ Boå ñeà ñöôïc chöùng minh hoaøn toaøn. Trôû laïi baøi toaùn cuûa ta b / () gb 1 3 2 0 0 () gb _ + 0 3 2 x + Tìm tài liệu Toán ? Chuyện nh ỏ - www.toanmath.com295 * Neáu 10 kk ≥ ∨≤ thì ta coù 3 kkk a b c ab bc ca ++≥ ≥ ++ vaø daáu baèng ñaït taïi 1 abc = == neân hieån nhieân 1. k C = * Xeùt (0,1) k ∈ Cho 1 abc = == ta suy ra 1 k C ≥ . Cho 3 , 0, 2 abc =→→ ta ñöôïc 2 3 3 2 k k k C - - ≥ . Ngöôïc laïi, ta seõ chöùng minh 2 3 3 max 1, 2 k k k C - - ⎧⎫ = ⎨⎬ ⎩⎭ thoûa maõn ñieàu kieän cuûa ñeà baøi, nghóa laø ta phaûi chöùng minh 22 ( )( )3() k k k kk k C a b c ab c ab bcca -- + + + + ≥ ++ (1) AÙp duïng baát ñaúng thöùc Holder, ta coù 3 2 22 2 333 ( )() k kkkk k kk a b cabc abc - - --- ⎛⎞ + + ++ ≥ ++ ⎜⎟ ⎜⎟ ⎝⎠ Do ñoù, (1) laø heä quaû cuûa 3 222 2 3 33 3() k k k kk k C a b c ab bc ca - - - -- ⎛⎞ + + ≥ ++ ⎜⎟ ⎜⎟ ⎝⎠ (2) Ñaët 222 333 3 ,, 2 vaø kkk k A a B b Cc --- - === λ= thì (2) töông ñöông vôùi 2 ( )() ( ) 33 k A B C AB BC CA C λ λ λλ ++ ++ ⎛⎞ ≥ ⎜⎟ ⎝⎠ (3) Do caû hai veá cuûa (3) ñoàng baäc neân khoâng maát tính toång quaùt, coù theå giaû söû 3 ABC + += . Khi ñoù, (3) trôû thaønh ( ) ()( ) 3 k AB BC CAC λ λλ + +≤ 2 3 ( ) ( ) ( ) max 3, 2 AB BC CA λ λ λλ ⎧⎫ ⎪⎪ ⎛⎞ ⇔ + +≤ ⎨⎬ ⎜⎟ ⎝⎠ ⎪⎪ ⎩⎭ (4) Tìm tài liệu Toán ? Chuyện nh ỏ - www.toanmath.com296 AÙp duïng keát quaû cuûa Boå ñeà treân, ta suy ra (4) ñuùng. ⇒ ñpcm. Keát luaän + 1 01 k k kC ≥∨ ≤ ⇒= + 2 3 3 0 1 max1,. 2 k k k kC - - ⎧⎫ < <⇒= ⎨⎬ ⎩⎭ Baøi toaùn 46. Cho,,.abcRn ∈ Tìm taát caû caùc soá nguyeân döông sao cho ( )( )( ) 0 nnn aab bbc cca + + +++ ≥ Lôøi giaûi. Nhaän xeùt raèng n phaûi leû. Neáu 6 n ≥ thì cho 19 , , 1. 44 a bc = =-= Khi ñoù, ta coù 2 2 5 ( ) ( ) ( ) 13. 2 0 6 4 n nnn n aab bbc cc a n - - ⎛⎞ + + + + + = - < ∀≥ ⎜⎟ ⎝⎠ Do ñoù 5 n ≤ maø 1 3 5. leû neân n n nn = ∨= ∨= Ta seõ chöùng minh ñoù laø taát caû nhöõng giaù trò caàn tìm, töùc laø chöùng minh 333 555 ( ) ( ) ( ) 0 (1) ( ) ( ) ( ) 0 (2) ( ) ( ) ( ) 0 (3) aab bbc cca aa b bb c cca aab bbc cca + + + + +≥ + + + + +≥ + + + + +≥ * Chöùng minh (1). Ta coù 222 1 (1) .(( )( )( )) 0 2 a b b c ca ⇔ + + + + +≥ (ñuùng) * Chöùng minh (2). Tìm tài liệu Toán ? Chuyện nh ỏ - www.toanmath.com297 Ñaët 2 2 2 z a b a x yz y c a b xyz x b c c x yz =+ =- ++ ⎧⎧ ⎪⎪ = + ⇔ =-+ ⎨⎨ ⎪⎪ = + =+- ⎩⎩ Ta coù 4 4 4 3 33 3 33 2 2 2 22 (2) 8( ) 0 4( )0 cyc cyc x y z xy yz zx x y y z z x x y xy xy ⇔ + + +++ - - - ≥ ⎛⎞ ⇔--+≥ ⎜⎟ ⎜⎟ ⎝⎠ ∑∑ (ñuùng) * Chöùng minh (3). Ñaët 2 2 2 z a b a x yz y c a b xyz x b c c x yz =+ =- ++ ⎧⎧ ⎪⎪ = + ⇔ =-+ ⎨⎨ ⎪⎪ = + =+- ⎩⎩ Ta coù 6 66 5 6 55 5 5 2 2 2 22 (3) 32( ) 0 16 ( )( ) 0 cyc xyzxyyzzx xy yz zx x y x y xy ⇔ + + + + + - - -≥ ⇔ + - -≥ ∑ (ñuùng) Vaäy taát caû caùc giaù trò n caàn tìm laø 1, 3, 5. n nn = == Baøi toaùn 47. Cho , , , 0 4. thoûa Chöùng minh raèng abcd abcd > + + += 2 2 22 2 2 22 1 3 3 33 a bcd bcd a + + +≥ + + ++ Lôøi giaûi. AÙp duïng baát ñaúng thöùc Bunhiacopxki, ta coù 2 2 22 2 2 22 3 3 33 a bcd b c da + ++≥ + + ++ 4 4 44 22 2 22 2 22 2 222 2 2 2 22 2 2 22 22 22 2222 3 3 33 () 3() a b cd aba bc b cd c dad a b cd a b c d ab bc cd d a = + ++ + + ++ + ++ ≥ + + + + + ++ Tìm tài liệu Toán ? Chuyện nh ỏ - www.toanmath.com298 AÙp duïng baát ñaúng thöùc AM-GM, ta laïi coù 22 2222 22 22 2 2 2 22 22 1 ()().() 4 ab bccd da ac bd a bcd + + + = + + ≤ + ++ Do ñoù, ñeå chöùng minh baát ñaúng thöùc ñaõ cho, ta chæ caàn chöùng minh 2 22 22 2 22 2 2 22 22 2 2 22 22222 1 ()3().() 4 4 1 .() 4 a b cd a b cd a b cd a b cd a b c d abcd + ++ ≥ + ++ + + ++ ⇔ + + +≥ ⇔ + + + ≥ + ++ ⇒ (ñuùng theo bñt Bunhiacopxki) ñpcm. Baøi toaùn 48. Cho 12 , ,..., n aaa laø n soá thöïc döông thoûa 1 1 1. 1 n i i a = = + ∑ Chöùng minh raèng 11 1 ( 1) nn i ii i an a == ≥- ∑∑ Lôøi giaûi. Ñaët 1 ( 1,) 1 i i x in a == + thì ta coù 1 1 0 ( 1, ), 1 ( 1, ). n i i ii i i x x i n x a in x = - > = = == ∑ vaø Khi ñoù, ta coù baát ñaúng thöùc caàn chöùng minh töông ñöông vôùi ( ) 11 1 12 1 1 ( 1) 1 1 0 (1) ... 0 (1) 11 ()0 (1 ) (1 ) ( ) (1 ) (1 ) 0 (1 )(1 ) nn ii ii ii n i i ii n ni i ii ij ij j j ii i j i i jj ij ij ij xx n xx nx xx x x x nx xx xx x x xx xx x x xx xx xx == = = ≠ ≠ - ≥- - - ⇔≥ - + + +- ⇔≥ - ⎛⎞ ⎜⎟ ⇔- -≥ ⎜⎟ -- ⎝⎠ - - -- ⇔≥ -- ∑∑ ∑ ∑ ∑ ∑ Tìm tài liệu Toán ? Chuyện nh ỏ - www.toanmath.com299 ( ) 2 ( )(1) 0 (1 ) (1 ) (1 )(1 ) (ñuùng) ñpcm. ij ij ij i i j j ij ij xx xx x x xxxx x x ≠ - -- ⇔≥ - + - -- ⇒ ∑ Baøi toaùn 49. (Poland 1990) Cho 3 n ≥ vaø 12 , ,..., 0. n xxx > Chöùng minh raèng 2 2 1 12 1 n i i i ii x n x xx = ++ ≤- + ∑ trong ñoù 1 1 22 . nn x x xx ++ == vaø Lôøi giaûi. Ta chöùng minh baèng quy naïp. + 3 n = Khi ñoù, baát ñaúng thöùc caàn chöùng minh töông ñöông vôùi 2 2 2 2 1 cyc cyc x x yz yz x yz ≤ + ⇔≥ + ∑ ∑ AÙp duïng baát ñaúng thöùc Bunhiacopxki, ta coù 22 2 2 22 2 22 2222 2 22 2 2 2 22 () () () cyc cyc yz yz x yz xyz yz xy yz zx x y yz zx x yz xyz xyz xy yz zx xy yz zx x yz xy z xyz = ++ ++ ≥ + + +++ ++ = + + --- ∑∑ Nhöng ta laïi coù 2 2 2 22 () 1 () xy yz zx xy yz zx x yz xy z xyz ++ ≥ + + --- , do ñoù 2 1 cyc yz x yz ≥ + ∑ Vaäy khaúng ñònh ñuùng khi 3. n = + Giaû söû khaúng ñònh ñuùng cho n bieán soá, ta seõ chöùng minh noù cuõng ñuùng cho 1 n + bieán soá. Tìm tài liệu Toán ? Chuyện nh ỏ - www.toanmath.com300 Khoâng maát tính toång quaùt, ta coù theå giaû söû { } 1 121 max , ,..., nn x xxx ++ = . Ta caàn phaûi chöùng minh 2 1 2 1 12 n i i i ii x n x xx + = ++ ≤ + ∑ (*) Theo giaû thieát quy naïp, ta coù 2 2 1 12 1 n i i i ii x n x xx = ++ ≤- + ∑ . Do ñoù, ñeåchöùng minh (*), ta chæ caàn chöùng minh 2 2 2 22 1 11 2 2 2 22 1 12 11 1 1 12 11 1 n n n nn n n n nnn n nn x x x xx x xx x x x x xx x xx x xx + -- + +-+- + + - -≤ + + + ++ 2 2 1 2 22 1 12 12 1 1 2 1 22 111 1 22 1 1 2 1 11 12 2 2 2 22 1121211 1 11 1 11 1. 11 .0 ( ) () 0 ( )( )( )() n n n n nn n nn nnn n n n nn n n n n nn nnn x x x xx xxx xx x x xxx xxx xxxx xxxx xx xxx x xxx xx x xxxxx + ++ - - -+ + -+ + + - -+ ⎛ ⎞⎛⎞ ⇔- + -+ ⎜ ⎟⎜⎟ + ++ ⎝ ⎠⎝⎠ ⎛⎞ + -≥ ⎜⎟ ++ ⎝⎠ -- ⇔+ +≥ + + + ++ (ñuùng) Vaäy khaúng ñònh ñuùng cho 1 n+ bieán soá. Theo nguyeân lyù quy naïp, khaúng ñònh ñuùng cho moïi 3. n ≥ . ⇒ ñpcm Baøi toaùn 50. Cho , ,, abcd laø caùc soá thöïc thoûa maõn 2 2 22 1 a bcd + + +≤ . Tìm giaù trò lôùn nhaát cuûa bieåu thöùc 4 4 444 4 ( )( )( )( )( )( ) P a b ac ad bc bc cd = + + + + + + ++ ++ + Lôøi giaûi. Ta coù 4 4 4 4 4 22 () ( ) () 2( 6 ) a b ab ab a b ab +≤ - + += + + Töôïng töï, ta coù Tìm tài liệu Toán ? Chuyện nh ỏ - www.toanmath.com301 4 4 4 22 4 4 4 22 4 4 4 22 4 4 4 22 4 4 4 22 ( ) 2( 6) ( ) 2( 6) ( ) 2( 6) ( ) 2( 6) ( ) 2( 6) a c a c ac a d a d ad b c b c bc b d b d bd c d c d cd + ≤ ++ + ≤ ++ + ≤ ++ + ≤ ++ + ≤ ++ Do ñoù 4 4 4 4222222 22 22 22 2 2 2 22 6( 2 2 2 2 2 2) 6() 6 P a b c d ab ac ad bc bd cd a bcd ≤ ++++ + + + ++ = +++ ≤ Ñaúng thöùc xaûy ra khi vaø chæ khi 1 . 2 abcd = = = =± Vaäy max 6. P = Baøi toaùn 51. (Phaïm Kim Huøng) Cho , , 0. Chöùng minh raèng abc > 2 22 111 4 ( ,,) 444 f abc abc a bc b ca c ab =+ +≥ ++ +++ Lôøi giaûi. Khoâng maát tính toång quaùt, coù theå giaû söû 0. abc ≥≥> Ta seõ chöùng minh ( , ,) (,,), . 2 trong ñoù ab f abc f ttc t c + ≥ =≥ Thaät vaäy, aùp duïng baát ñaúng thöùc AM-GM, ta coù 2 2 22 4 1 12 4 4 (4 )(4 ) a bc b ca a bc b ca +≥ + + ++ Maët khaùc, ta coù 2 2 22 2 2 22 () 4. (4 )(4 ) 22 () 6 3300 4 (do ) ab abc a bc b ca c a b a b ab ac bc a b c ⎛⎞ ++ ⎛⎞ + - + += ⎜⎟ ⎜⎟ ⎜⎟ ⎝⎠ ⎝⎠ ⎛⎞ = - + + + - - ≥ ≥≥> ⎜⎟ ⎝⎠ Tìm tài liệu Toán ? Chuyện nh ỏ - www.toanmath.com302 2 2 2 22 4 1 1 22 4 4 (4 )(4 )4 a bc b ca a bc b ca t tc ⇒+≥≥ + + + ++ Cuõng theo baát ñaúng thöùc AM-GM, ta coù 2 2 22 2 1 11 44 4 2 c ab ct ab c ≥= ++ + ⎛⎞ + ⎜⎟ ⎝⎠ Do ñoù 222 2 22 111 ( ,,) 4 44 21 44 (,,) f abc a bc b ca c ab t tc ct f ttc = ++ +++ ≥+ ++ = Vaäy ñeå chöùng minh 4 ( ,,) f abc abc ≥ ++ , ta chæ caàn chöùng minh 2 22 4 (,,) 2 2 14 2 44 f ttc tc tc t tc ct ≥ + ⇔ +≥ + ++ 2 22 2 1 1 1 42 2 44 t t t ct t tc ct ⎛ ⎞⎛⎞ ⎛⎞ ⇔ -+ -≥- ⎜ ⎟⎜⎟ ⎜⎟ + ⎝⎠ ++ ⎝ ⎠⎝⎠ ( ) ( ) ( ) ( ) 2 2 2 2 22 2 2 2 2 22 2 14 .0 (2) 4 .2 4 4 .4 2 14 0 (2) 4 .2 4 4 .4 c c t tc t tc t t tc t ct t ct c t tc t tc t t tc t ct t ct ⎛⎞ ⎜⎟ ⇔- -≥ ⎜⎟ + + + + + ++ ⎜⎟ ⎝⎠ ⇔- -≥ + + + + + ++ ( ) 22 11 3(2) 4 .24 t tc t tc t t tc ⎛⎞ ⎜⎟ ⇔-+ ⎜⎟ + + ++ ⎜⎟ ⎝⎠ Tìm tài liệu Toán ? Chuyện nh ỏ - www.toanmath.com303 ( ) 2 2 22 54 0 3(2) 4 .4 c t tc t ct t ct ⎛⎞ ⎜⎟ +-≥ ⎜⎟ + + ++ ⎜⎟ ⎝⎠ Nhö vaäy, ñeå chöùng minh 4 (,,), 2 f ttc tc ≥ + ta chæ caàn chöùng minh ( ) ( ) 22 22 22 11 0 (1) 3(2) 4 .24 54 0 (2) 3(2) 4 .4 t tc ttc t ttc c t tc t c t t ct -≥ + + ++ -≥ + + ++ * Chöùng minh (1). Ta coù ( ) ( ) ( ) 22 2 2 11 0 3(2) 4 .24 11 0 3(2) 4 .24 4 .2 4 3(2) 24 4 63 4 (ñuùng do ) ttc t tc t t tc tc tc t tc t c t tc tc t tctct c t tc tc tc -≥ + + ++ ⇔-≥ + + ++ ⇔ + + +≥+ ⇔ + + +≥+ ⇔+≥+≥ (1) ñuùng. ⇒ * Chöùng minh (2). Ta coù ( ) 2 2 22 54 0 3(2) 4 .4 c t tc t ct t ct -≥ + + ++ ( ) 22 22 54 0 3(2) 4 .4 c tc ct t ct ⇔-≥ + + ++ ( ) 2 2 22 54 . 4 12(2) ct t ct ct c ⇔ + + + ≥+ Tìm tài liệu Toán ? Chuyện nh ỏ - www.toanmath.com304 22 22 2 2 22 222 2 22 3 2 23 5 4 5(4 ) 12 (2 ) 5 4 8 5 24 25 (4 ) (8 5 24 ) 4 (60 139 96 16 ) 0 (ñuùng do ) (2) ñuùng. t c t c t c tc t c t c t tc t c t c t tc c t tc tc c tc ⇔ ++ +≥+ ⇔ + + +≥ ⇔ + ≥ +- ⇔ -+-≥≥ ⇒ . ñpcm ⇒ Ñaúng thöùc xaûy ra khi vaø chæ khi ,0 vaø caùc hoaùn vò töông öùng. a bc == Baøi toaùn 52. (Vasile Cirtoaje) Cho , , 0. Chöùng minh raèng xyz > 2 22 2 22 1 1 1 111 ( ) 11() x y z x yz x yz xyz ⎛⎞ ⎛⎞ + + + + ≥+ + + + ++ ⎜⎟ ⎜⎟ ⎝⎠ ⎝⎠ Lôøi giaûi. Ñaët 2 22 , , ( , , 0) a x y z b xy yz zx c xyz a b c =++=++ => thì ta coù 3 0. abc ≥> Khi ñoù, ta coù 2 2 2 2 2 42 4 22 2224 111 2 1112 b x yz c x y z ab b ac xyzc + += + +=- - + += Do ñoù, baát ñaúng thöùc caàn chöùng minh töông ñöông vôùi 4 2 4 22 4 2 4 2 4 22 2 2 4 2 4 22 ( 2)(2) 11 ( 2)(2) ( ) ( 2)( 2) ab a b b ac c c ab c c a b b ac ab c c a b b ac -- ≥++ ⇔-≥ + -- ⇔-≥+ -- 2 2 4 2 4 22 2 ( 2)(2) ababc abbac ⇔ - ≥ -- 2 2 2 4 2 4 22 ( 2 ) ( 2)( 2) ababc a bbac ⇔ - ≥-- Tìm tài liệu Toán ? Chuyện nh ỏ - www.toanmath.com305 3 32 2( )0 (ñuùng) ñpcm. b ac ⇔ -≥ ⇒ Ñaúng thöùc xaûy ra khi vaø chæ khi . xyz == Baøi toaùn 53. (Mildorf) cho,, 0,. Chöùng minh raèng abck >∈R 22 max( , ).( ) 2 ( )( ) min( , ).( ) kk k kk cyc cyc cyc a b a b a ab a c a b ab -≥ --≥- ∑ ∑∑ Lôøi giaûi. Khoâng maát tính toång quaùt, coù theå giaû söû 0. abc ≥≥> Coù 2 tröôøng hôïp xaûy ra * Tröôøng hôïp 1. 0 0. k kk k abc ≥ ⇒ ≥ ≥> Tröôùc heát, ta chöùng minh 2 2 22 max( , ).( ) 2 ( )( ) () () ( ) 2 ()() kkk cyc cyc k k kk cyc a b ab a abac a ab a ac b bc a ab ac - ≥ -- ⇔ -+ - + -≥ - - ∑∑ ∑ Chuù yù raèng 2 22 ( ) ( ) ( ) 2( )( ) ab ac bc aba c - +- =- + - - , neân baát ñaúng thöùc treân töông ñöông vôùi 22 22 ( ) ( ) 2()( )2 ()( ) () () 2( )()2( )() ( ) ( )2 ( )2( ) 0 ( 2 )( ) 2( )( ) 0 (ñuùng) k k kk cyc k k kk k k kk k k k kk a bc b bc a ab ac a ab ac a b c bb c bb a b c cc a c b a bc b bc b a b c ac a b c bc b c ab - + - + -- ≥ -- ⇔ - + - ≥ - - + -- ⇔ -+ -+ - - - ≥ ⇔ + - - + - -≥ ∑ Tieáp theo, ta seõ chöùng minh 2 2 22 2 ( )( ) min( , ).( ) 2 ( )() ( ) () ( ) k kk cyc cyc k kkk cyc a ab ac a b ab a ab ac b ab c a c c bc --≥- ⇔ - -≥ - + - +- ∑∑ ∑ Tìm tài liệu Toán ? Chuyện nh ỏ - www.toanmath.com306 Chuù yù raèng 2 22 ( ) () ( ) 2( )() ac bc a b cacb - +- = - + -- , neân baát ñaúng thöùc treân töông vôùi 22 22 2 ( )() () () 2()( ) 2()( )2()( ) () () 2( )2()( ) () () 2( )( ) (2 )( ) 0 (ñuùng) k k kk cyc k k kk k k kk kk kk k a ab ac b ab c ab c c a cb a ab ac b ba bc b ab c ab a a c b a bbc b ab c ab ab ab abc bc - - ≥ - + - + -- ⇔ -- + -- ≥ - + - ⇔ - - - -≥ -+ - ⇔ - - + - - -≥ ∑ Vaäy trong tröôøng hôïp naøy, ta coù 22 max( , ).( ) 2 ( )( ) min( , ).( ) kk k kk cyc cyc cyc a b a b a ab a c a b ab -≥ --≥- ∑ ∑∑ * Tröôøng hôïp 2.0. k kk k abc < ⇒ ≤≤ Laäp luaän töông töï tröôøng hôïp 1, ta cuõng coù 22 max( , ).( ) 2 ( )( ) min( , ).( ) kk k kk cyc cyc cyc a b a b a ab a c a b ab -≥ --≥- ∑ ∑∑ Toùm laïi, trong moïi tröôøng hôïp, ta luoân coù 22 max( , ).( ) 2 ( )( ) min( , ).( ) ñpcm kk k kk cyc cyc cyc a b a b a ab a c a b ab -≥ --≥- ⇒ ∑ ∑∑ Baøi toaùn 54. (Vasile Cirtoaje) Cho . Chöùng minh raèng ABC Δ 111 2 2 cos 2 cos 2 cos ABC + +≥ --- Lôøi giaûi. Ta coù baát ñaúng thöùc caàn chöùng minh töông ñöông vôùi 22 (2 cos )(2 cos ) 2(2 cos )(2 cos )(2 cos ) 4 cos 2 cos .cos 3cos .cos .cos 4 0 4cos 8sin . 6sin .cos . 3 3cos 222 cyc cyc cyc A B ABC A A B ABC CCC C t Ctt - - ≥--- ⇔ - + -≥ ⇔ + - -++ ∑ ∑∑ Tìm tài liệu Toán ? Chuyện nh ỏ - www.toanmath.com307 22 2cos . 2cos .cos 4 0 (*) 2 C CtC + - -≥ trong ñoù - cos 1 sin 0 2 2 2 22 do A B C AB AB tt π -+ ⎛⎞ = ⇒≥≥ ≤ ≤ < ⎜⎟ ⎝⎠ Ñaët (*) () VT ft = Ta coù // ( ) 2(2cos 3) 0 ( ) sin ,1 . 2 ( ) min sin , (1) 2 laø haøm loài treân ftC C ft C ft ff = -< ⎡⎤ ⇒ ⎢⎥ ⎣⎦ ⎧⎫ ⎛⎞ ⇒≥ ⎨⎬ ⎜⎟ ⎝⎠ ⎩⎭ Ta coù 2 22 sin 4cos 8sin 6sin .cos 3sin 2 2 22 C C CC f CC ⎛⎞ = + - -+ ⎜⎟ ⎝⎠ 2 22 2 2 22 22 2 3cos 2cos .sin 2cos .cos 4 2 22 4cos 5sin 3cos 4sin .cos 2cos .cos 4 2 2 22 4cos 2sin 3 2sin .cos 2cos 4 22 2cos 2sin .(1 cos ) 1 2 C CC CC C C CC C CC CC C CC C CC + + -- =+ + - -- = + +- -- = + -- 2 2 2cos (1 cos ) 1 cos 0 CC C = +-- = ≥ Ta laïi coù 2 2 22 (1) 4cos 8sin 6sin .cos 3 3cos 222 2cos 2cos .cos 4 2 6cos 8sin 3sin 6sin .cos 2cos .cos 4 2 222 CCC fCC C CC C CCC C CC = + - -++ +-- =+- - -- Tìm tài liệu Toán ? Chuyện nh ỏ - www.toanmath.com308 2 sin . 2sin 1 2 sin 0 2 22 CCC ⎛ ⎞⎛⎞ = - -≥ ⎜ ⎟⎜⎟ ⎝ ⎠⎝⎠ Vaäy ta coù sin 0 (1) 0 min sin (1) 0 22 vaø , CC f f ff ⎧⎫ ⎛⎞ ⎛⎞ ≥≥⇒≥ ⎨⎬ ⎜⎟ ⎜⎟ ⎝⎠ ⎝⎠⎩⎭ ()minsin,(1)0 2 ñpcm. C ftff ⎧⎫ ⎛⎞ ⇒≥≥ ⎨⎬ ⎜⎟ ⎝⎠⎩⎭ ⇒ Ñaúng thöùc xaûy ra khi vaø chæ khi ,0 32 hoaëc ABC AB C ππ === =→→ vaø caùc hoaùn vò töông öùng. Baøi toaùn 55. Cho , , 0. Chöùng minh raèng abc > 2 22 ( ) 33() abc abc a b c bca ⎛⎞ + + + + ≥ ++ ⎜⎟ ⎝⎠ Lôøi giaûi. Ta coù ( ) 2 22 2 222 2 2 22 2 22 222 ( ) 33() 33() 2 33() 3() ( ) 1 () . 2 3() cyc cyc cyc cyc cyc cyc cyc abc abc a b c bca a ab abc a b c bc a ab b a abc a b c abc bc ab ab cab b ab a b c abc ⎛⎞ + + + + ≥ ++ ⎜⎟ ⎝⎠ ⇔ + + + + ≥ ++ ⎛⎞ ⎛⎞ ⇔ + - + - -- ≥ + + - -- ⎜⎟ ⎜⎟ ⎜⎟ ⎝⎠ ⎝⎠ - -- ⇔ +≥ + + + ++ ∑∑ ∑∑ ∑ ∑∑ AÙp duïng baát ñaúng thöùc Bunhiacopxki, ta coù 22 2 22 3( ) () 3 . 2 3() cyc cyc ab ab abc a b c abc -- ≤ ++ + + + ++ ∑∑ Tìm tài liệu Toán ? Chuyện nh ỏ - www.toanmath.com309 Do ñoù, ñeå chöùng minh baát ñaúng thöùc ñaõ cho, ta chæ caàn chöùng minh 2 22 2 22 22 2 22 () () 1 () 3 .. 22 () ( ) () 23 111 2() ()0 ()()1 2 . ( )( () () cyc cyc cyc cyc cyc cyc cyc cyc cyc ab a b cab b ab abc ab ab cab b ab abc c a b ab b a bc ab abc a b ac a b c ac bc babc abcabc - -- +≥ ++ - -- ⇔ +≥ ++ ⎛⎞⎛ ⎞ ⇔ - - + - -≥ ⎜⎟⎜ ⎟ ++ ++ ⎝⎠⎝ ⎠ -+ ⇔ + - ++ ++ ++ ∑ ∑∑ ∑ ∑∑ ∑∑ ∑ )0 cyc ab -≥ ∑ Deã thaáy 2 ( )() 20 () cyc a b ac babc -+ ≥ ++ ∑ . Do ñoù, ta chæ caàn chöùng minh 22 ( )( )0 cyc a b c ac bc ab - + + -≥ ∑ Khoâng maát tính toång quaùt, giaû söû 0 0. abc a c a b ≥≥> ⇒- ≥ - ≥ Khi ñoù, ta coù 22 2 2 22 22 ()() ( )( ) ( )( ) ()() cyc a b c ac bc ab b c a ab ac bc a c b ab bc ac a b c ac bc ab - + + -= = - + + - + - + + -+ + - + +- ∑ 2 2 22 ( )( ) ( )( ) a c b ab bc ac a b c ac bc ab ≥ - + +- + - + +- 2 2 22 22 ( )( ) ( )( ) ( )() 0 a b b ab bc ac a b c ac bc ab a b bc ≥ - + + - + - ++- =-+ ≥ Vaäy 22 ( )( )0 cyc a b c ac bc ab - + + -≥ ∑ . ñpcm ⇒ Ñaúng thöùc xaûy ra khi vaø chæ khi . abc == Tìm tài liệu Toán ? Chuyện nh ỏ - www.toanmath.com310 Baøi toaùn 56. (Leâ Trung Kieân) Cho , , 0. Chöùng minh raèng abc > 2 22 2 2 2 222 ( ) ( ) ( ) 2( )( )( ) 2 2 2 2 ( 2 )( 2 )( 2 ) abc bc a cab ab bc ca a bc b ca c ab a bc b ca c ab + + + - -- + + ≤+ +++ + + + Lôøi giaûi. Ta coù baát ñaúng thöùc caàn chöùng minh töông ñöông vôùi 2 22 2 222 2 22 2 222 2 2 22 2 222 3( ) 6( )( )() 6 2 ( 2 )( 2 )( 2) 3( ) 6( )( )() 20 2 ( 2 )( 2 )( 2 ) 2 3( )4 6( )( )() 2 ( 2 )( 2 )( cyc cyc cyc abc ab bc ca a bc a bc b ca c ab abc ab bc ca a bc a bc b ca c ab a abc bc ab bc ca a bc a bc b ca c + - -- ≤+ + + ++ + - -- ⎛⎞ ⇔-+≥ ⎜⎟ + +++ ⎝⎠ - ++ - -- ⇔+ + ++ ∑ ∑ ∑ 2 22 2 2 22 2 22 2 2 222 2 0 2) ( 2 )( ) ( 2 )( ) 6( ) ( ) ( ) 0 2 ( 2 )( 2 )( 2 ) ( 2 )( ) ( 2 )( ) 6( ) ( ) ( ) 0 2 2 ( 2)( 2)( 2 ) ( 2)( ) (2 2 cyc cyc cyc ab a c a b a bc a ab bc ca a bc a bc b ca c ab a c a b a bca a b b c ca a bc a bc a bc b ca c ab acabb a bc ≥ + - --- - - -- ⇔ +≥ + +++ - - - - - -- ⇔ - +≥ + + +++ - -- ⇔- + ∑ ∑∑ 2 22 2 222 2 2 2 22 22 222 )() 6()( )( ) 0 2 ( 2 )( 2 )( 2) ( )( 4 4( ) ) 6( )( ) ( ) 0 ( 2 )( 2 ) ( 2 )( 2 )( 2 ) cyc cyc cyc c ab ab bc c a b ca a bc b ca c ab ab c cab ab ab bc c a a bc b ca a bc b ca c ab - - -- +≥ + + ++ - - + +- - -- ⇔ +≥ ++ +++ ∑∑ ∑ 2 2 2 2 22 22 222 2 22 ( )(4 4( ))( 2)6( )( )() 0 3 ( )( 2) ( )(4 4( ) 4 )( 2) 6( )( ) )0 1 ( cyc cyc cyc ab c cab abc ab ab b c ca ab a b c ab a b c c a b ab c ab ab bc ca ⇔- - + +- + + -- - ≥ ⇔ - + + - - + +- ++ + - - -≥ ∑ ∑∑ 222 2 22 3 ( ) ( 2 ) 4( )( )( ). ( )( 2 ) 6( )( )( )0 1 cyc cyc abab c ab abbcc a abc ab ab bc ca ⇔ - + + - - - - ++ + - - -≥ ∑∑ 22 2 22 2 22 3 ( )( 2 ) 12( )( ) ( ) 6( )( )( )0 cyc aba b c ab ab bc c a ab bc c a ⇔ - + - - - -+ + - - -≥ ∑ Tìm tài liệu Toán ? Chuyện nh ỏ - www.toanmath.com311 22 2 22 ( )( 2 ) 2( )( ) ( ) 0 cyc abab c ab ab bc c a ⇔ - + - - - -≥ ∑ Khoâng maát tính toång quaùt, giaû söû 0. abc ≥≥> Khi ñoù, ta coù 22 22 222 2 22 ( )( 2) ( )( 2) 2 () 2( )( )() cyc ab a b c ab ab a b c ab ab ab ab bc ca -+ ≥ -+ ≥- ≥ - -- ∑ Vaäy 22 2 22 2 22 222 2 2 2 ( )( 2 ) 2( )( ) ( ) 0 ( ) () ( ) 2( )( )() 2 222 (2)(2)(2) (ñpcm) cyc abab c ab ab bc c a abc bc a cab ab bc ca a bc b ca c ab a bc b ca c ab - + - - - -≥ + + + - -- ⇒+ +≤+ + + + + ++ ∑ Ñaúng thöùc xaûy ra khi vaø chæ khi ,0 hoaëc vaø caùc hoaùn vò. abc abc == == Baøi toaùn 57. Cho , , 0. Chöùng minh raèng abc > 2 22 2(1 )(1 )(1 ) (1 )(1 )(1 ) 2(1 ) a b c a b c abc + + + ≥ + + + -+ Lôøi giaûi. * Caùch 1. Ta coù baát ñaúng thöùc caàn chöùng minh töông ñöông vôùi 2 22 2222 2(1 )(1 )(1 ) 1 2(1 )(1 )(1 ) ( 1 ) a b c a b c ab bc ca abc a b c a b c ab bc ca abc + + + ≥-+ + + + ++- ⇔ + + + ≥-+ + + + ++- Chuù yù raèng 222 22 ( 1 )(1 )(1 ) ( 1 ) ( ) a b c ab bc ca a b c abc + + + = - + + + + + +- Do ñoù, baát ñaúng thöùc caàn chöùng minh töông ñöông vôùi 2 22 2 2(1 ) 2( ) (1 ) (1 )0 (ñuùng) ñpcm. ab bc ca a b c abc a b c ab bc ca abc ab bc ca a b c abc - + + + + + +- ≥ - + + ++ + + - ⇔-+ ++- - - + ≥ ⇒ * Caùch 2. Ta coù baát ñaúng thöùc caàn chöùng minh töông ñöông vôùi Tìm tài liệu Toán ? Chuyện nh ỏ - www.toanmath.com312 222 2(1 )(1 )(1 ) 1 a b c a b c ab bc ca abc + + + ≥-+ + + + + +- 2222 2(1 )(1 )(1 ) ( 1 ) a b c a b c ab bc ca abc ⇔ + + + ≥-+ + + + ++- Ñaët , , ,, 2 a Ab Bc C ABC π ⎛⎞ = = = << ⎜⎟ ⎝⎠ tg tg tg 0 . Khi ñoù, baát ñaúng thöùc caàn chöùng minh töông ñöông vôùi 2 222 2 2 sin sin .sin sin .sin .sin 1 cos cos .cos cos .cos .cos cos .cos .cos 2 sin .cos .cos sin .sin .sin sin .sin .cos cos .cos .cos cyc cyc cyc cyc A AB ABC A AB ABC ABC A B C ABC A BC ABC ⎛⎞ ≥-+ +- ⎜⎟ ⎜⎟ ⎝⎠ ⎛ ⇔≥ -+ ⎜ ⎜ ⎝ ⎞ +- ⎟ ⎟ ⎠ ∑∑ ∑ ∑ Chuù yù raèng sin( ) sin .cos .cos sin .sin .sin cos( ) cos .cos .cos sin .sin .cos cyc cyc ABC A B C A B C ABC A BC A BC ++=- ++=- ∑ ∑ Do ñoù, baát ñaúng thöùc caàn chöùng minh töông ñöông vôùi 2 2 (sin( ) cos( )) (hieån nhieân ñuùng) ñpcm. ABC A BC ≥ ++ - ++ ⇒ Baøi toaùn 58. (France 2004) Cho , , , , , 0. thoûa Chöùng minh raèng abcdef R abc d e f ∈ + + + + += 2 2 2 2 22 1 .() 2 ab bc cd de ef fa a b c d e f + ++ + +≤ + + ++ + Lôøi giaûi. * Caùch 1. Ta coù 2 ( )( ) ( )0 acebd f ace ++ + + =- ++ ≤ Maët khaùc, ta coù Tìm tài liệu Toán ? Chuyện nh ỏ - www.toanmath.com313 ( )( )( )() a c e b d f ab bc cd de ef fa ad be fc + + + + = ++ + ++ + ++ Do ñoù 22 2 22 2 2 2 2 2 22 222 1 .() 2 ñpcm. ab bc cd de ef fa ad be fc a d b e cf a b c d ef ++ + ++ ≤-- - + ++ ≤ ++ = + + + ++ ⇒ * Caùch 2. Ñaët 2 22 22 2 Aa b c d ef B ab bc cd de ef fa C ac bd ce df ea fb D ad be cf = + + + ++ = + + +++ = + + + ++ = ++ Khi ñoù, ñeå chöùng minh baát ñaúng thöùc ñaõ cho, ta caàn chöùng minh 2 AB ≥ Theo giaû thieát, ta coù 2 ( ) 2 2 20 abc d e f A B C D +++ ++ = + + += Ta laïi coù 2 22 22 () ( ) () 2 ( )( )2 ad be c f AD ac e bd f A C + + + + + =+ + + + + + =+ Vì toång caùc bình phöông luoân khoâng aâm neân ta coù 2 2 2 22 ( )( )( )( )( ) 2 2 2 0 a d b e c f a ce bd f A C D + + ++ + + +++ + + =+ + ≥ Theo treân, ta coù 2 2 20 ABC D + + += Do ñoù 22 2 22 2 2 ñpcm. A C D ABC D AB ++ ≥ + ++ ⇒≥ ⇒ Tìm tài liệu Toán ? Chuyện nh ỏ - www.toanmath.com314 Baøi toaùn 59. (Voõ Quoác Baù Caån) Cho , 0. Chöùng minh raèng xy > 2 22 2 2 2 22 2 2 22 2 2 22 4(3)(3)83 4( ) 3 3( 3 )( 3) y x y y x xyxy xx y yx x y yx + + + ++ + + + ≤ ++ Lôøi giaûi. Ta coù Boå ñeà sau Boå ñeà. 0, 2 x π ⎛⎞ ∀∈ ⎜⎟ ⎝⎠ thì 1 33 sin .sin 2 24 xx +≤ Chöùng minh. Xeùt haøm soá 1 ( ) sin .sin 2 2 fx xx =+ vôùi 0, 2 x π ⎛⎞ ∈ ⎜⎟ ⎝⎠ . Ta coù /2 ( ) cos cos 2 2cos cos 1 (1 cos )(2cos 1) fx x x x x xx =+= +-=+- / ()0. 3 fxx π = ⇔= Qua 3 π thì / () fx ñoåi daáu töø döông sang aâm, neân 33 ( ) 0, 342 fxfx ππ ⎛⎞ ⎛⎞ ≤ = ∀∈ ⎜⎟ ⎜⎟ ⎝⎠ ⎝⎠ Boå ñeà ñöôïc chöùng minh hoaøn toaøn. Trôû laïi baøi toaùn cuûa ta Ta coù baát ñaúng thöùc caàn chöùng minh töông ñöông vôùi 2 2 22 2 2 22 22 2 2 2 2 22 3 2 3 ( ) 3 33 4 (3)( 3) ( 3) 3 (3) 3 y xy xxy xy yx yx xy xy yx + + +≤ + + + + ++ Chuù yù raèng Tìm tài liệu Toán ? Chuyện nh ỏ - www.toanmath.com315 22 22 2 2 22 22 22 22 22 2 2 2 2 22 2 2 22 2 2 22 22 2 2 22 3 1. 3 (3)(3)3 232 1. 3 ( 3) 3 ( 3 )( 3) ()34 1. ( 3 )( 3) (3)33 y xy xy x y y x yx x y y xy yx yx x y x y yx xxy xyx x y yx x y y x xy =- + +++ =- + + + ++ + =- ++ +++ Maët khaùc 2 2 22 2 2 22 2 0 , , 1. 3 3 ( 3 )( 3) x y xy x yyx xy yx << + + ++ Do ñoù, ta coù theå ñaët 2 2 22 2 2 22 2 cos ,cos ,cos 3 3 ( 3 )( 3) x y xy ABC x y y x x y yx === + + ++ trong ñoù0 ,,. 2 ABC π << Khi ñoù, baát ñaúng thöùc caàn chöùng minh töông ñöông vôùi 33 sin .cos sin .cos sin .cos 4 A B B C CA + +≤ Maët khaùc, töø caùch ñaët, ta coù 222 cos cos cos 2cos .cos .cos 1 . A B C ABC A BC π +++= ⇒ + += , , 0 (sin ,sin ,sin ) (cos ,cos ,cos ) 2 Do neân vaø ABC ABC ABC π >> laø 2 daõy ñôn ñieäu ngöôïc chieàu. Theo baát ñaúng thöùc saép xeáp laïi, ta coù ⇒ sin .cos sin .cos sin .cos sin .cos sin .cos sin .cos 1 sin .sin 2 2 A B B C C A A C B B CA BB + +≤+ + =+ 33 4 (theo Boå ñeà treân) ≤ ñpcm. ⇒ Tìm tài liệu Toán ? Chuyện nh ỏ - www.toanmath.com316 Ñaúng thöùc xaûy ra khi vaø chæ khi . 3 A B C xy π = = = ⇔= Baøi toaùn 60. (Phaïm Kim Huøng) Cho , , 0. Tìm haèng soá nhoû nhaát sao cho baát ñaúng thöùc sau ñuùng abck > 3 2 k kk k a bc b c c a ab ⎛ ⎞⎛ ⎞⎛ ⎞ + +≥ ⎜ ⎟⎜ ⎟⎜ ⎟ + ++ ⎝ ⎠⎝ ⎠⎝ ⎠ Lôøi giaûi. Cho 1,0 bca + ==→ , ta suy ra 3 1. 2 ln ln kn ≥ -= Ta seõ chöùng minh min , kn = töùc laø chöùng minh 3 2 n nn n a bc bccaab ⎛ ⎞ ⎛ ⎞ ⎛⎞ + +≥ ⎜ ⎟ ⎜ ⎟ ⎜⎟ + ++ ⎝ ⎠ ⎝ ⎠ ⎝⎠ Khoâng maát tính toång quaùt, coù theå giaû söû 0 1 0 1 3 abc a abc < ≤≤ ⎧ ⇒ <≤ ⎨ ++= ⎩ . Ñaët 2 2 0 btm cbt ctm cbm tm =- ⎧ += ⎧ ⎪ ⇒ =+ ⎨⎨ -= ⎩ ⎪ >≥ ⎩ Xeùt haøm soá ( ) () () ( )() nn nn t m tm fm t m a t ma +- =+ - + ++ vôùi 0. m ≥ Ta coù 11 / 11 ( ) () () (2) () () nn nn t m tm f m n ta t ma t ma -- ++ ⎛⎞ +- =+- ⎜⎟ - + ++ ⎝⎠ Ta seõ chöùng minh / ()00 . fmm ≥ ∀≥ Thaät vaäy / ( )0 (1 )(ln( ) ln( )) ( 1)(ln( ) ln( )) fm n t m t m n t ma t ma ≥ ⇔ - + - - ≤ + + + - -+ Do 1 2 1 n n + > - neân ta chæ caàn chöùng minh Tìm tài liệu Toán ? Chuyện nh ỏ - www.toanmath.com317 ln( ) ln( ) 2(ln( ) ln( )) t m t m t ma t ma + -- ≤ + + -- + Xeùt haøm soá ( ) ln( ) ln( ) 2ln( ) 2ln( ) gm t m t m t ma t ma = +- -- + + + - + Ta coù / 1 1 22 ()0 gm m t t ma t ma t m = - --≤ + - ++ +- (do ) abc ≤≤ / () () (0)00 ln( ) ln( ) 2(ln( ) ln( )) ( )0 laø haøm nghòch bieán treân [0,+ ). gm gmgm t m t m t ma t ma fm ⇒∞ ⇒ ≤ = ∀≥ ⇒ + -- ≤ + + -- + ⇒≥ () laø haøm ñoàng bieán treân [0,+ ). fm⇒∞ 1- ()(0)2 20 1 nn ta fmfm taa ⎛ ⎞ ⎛⎞ ⇒ ≥ = = ∀≥ ⎜ ⎟ ⎜⎟ ++ ⎝ ⎠ ⎝⎠ Do ñoù ( ) () 1 ( )() 1 2 11 () nnnn nn nn nn a b c tm tm a bccaab a t m a t ma aa aa ha +- ⎛ ⎞ ⎛ ⎞ ⎛ ⎞ ⎛⎞ + + = ++ ⎜ ⎟ ⎜ ⎟ ⎜ ⎟ ⎜⎟ +++- - + ++ ⎝ ⎠ ⎝ ⎠ ⎝ ⎠ ⎝⎠ - ⎛ ⎞ ⎛⎞ ≥+ ⎜ ⎟ ⎜⎟ +- ⎝ ⎠ ⎝⎠ = Ta coù 11 / 11 4(1) () (1 ) (1 ) nn nn aa han aa -- ++ ⎛⎞ - =- ⎜⎟ -+ ⎝⎠ / ()0 ln 4 ( 1)ln(1 ) 2 ln(1 ) ( 1)ln ha n a n a na = ⇔ = + + - -+- Ñaët ( ) ( 1)ln(1 ) 2ln(1 ) ( 1)ln a n a a na φ = + + - -+- Ta coù / 2 12 1 (31)1 () 11 (1) n n n n an a a aa aa φ + - + +- = + -= +- - / 0 11 () 3 13 coù 1 nghieäm döông duy nhaát laø n aa n φ - ⇒ =< + ln31 1 ln23 do n ⎛⎞ = -> ⎜⎟ ⎝⎠ Tìm tài liệu Toán ? Chuyện nh ỏ - www.toanmath.com318 Qua 0 a thì / () a φ ñoåi daáu töø aâm sang döông vaø 0 1 lim ( ) , ln 4 3 a aφφ → ⎛⎞ =+∞= ⎜⎟ ⎝⎠ neân phöông trình ( ) ln4 a φ= coù 2 nghieäm döông phaân bieät laø 1 11 0. 33 vaø a << / ()0 phöông trìnhha ⇒= coù 2 nghieäm döông phaân bieät laø 1 1 . 3 vaøa Qua 1 a thì / () ha ñoåi daáu töø döông sang aâm, qua 1 3 thì / () ha ñoåi daáu töø aâm sang döông neân ta coù 13 ( ) min (0), 32 n ha hh ⎧⎫ ⎛⎞ ≥= ⎨⎬ ⎜⎟ ⎝⎠ ⎩⎭ 3 2 n nn n a bc bc c a ab ⎛ ⎞⎛ ⎞⎛ ⎞ ⇒ + +≥ ⎜ ⎟⎜ ⎟⎜ ⎟ + ++ ⎝ ⎠⎝ ⎠⎝ ⎠ Vaäy min . kn = Baøi toaùn 61. (Traàn Nam Duõng) Cho 0. x > Tìm haèng soá s döông nhoû nhaát sao cho 11 2 13 s s xx x x ⎛ ⎞ ⎛⎞ + + ≥+ ⎜ ⎟ ⎜⎟ ⎝ ⎠ ⎝⎠ Lôøi giaûi. Roõ raøng vôùi 1 x = thì baát ñaúng thöùc ñaõ cho trôû thaønh ñaúng thöùc. Do ñoù, khoâng maát tính toång quaùt, ta chæ caàn xeùt 1 x > laø ñuû. Khi ñoù, ta coù 22 1 2 11 2 13 2( 1 ) 3( 1) 13 . 12 s s s s s s xx x x xx x x x x x - ⎛ ⎞ ⎛⎞ + + ≥+ ⎜ ⎟ ⎜⎟ ⎝ ⎠ ⎝⎠ -- ⇔≥ - ⇔≥ - Theo ñònh lyù Lagrange, toàn taïi (1,) yx ∈ sao cho Tìm tài liệu Toán ? Chuyện nh ỏ - www.toanmath.com319 /1 1 () 1 s ss x y sy x - - == - Do ñoù 1 2 13 . 12 s s x x x - - ≥ - 1 1 2 3 . 2 s s syx - - ⇔≥ Cho 1 x + → thì 1 y + → , ta suy ra ñöôïc 3 2 s ≥ . Ta seõ chöùng minh ñaây laø giaù trò caàn tìm. Ñeå coù ñieàu naøy, ta chæ caàn chöùng minh 1 2 1 . ,1 1 s s x sx xs x - - ≥ ∀> - laø ñuû. Ta coù 1 2 11 22 1 . 1 ()10 s s ss s x sx x f x x sx sx - +- - ≥ - ⇔ = -- +≥ Ta coù 31 / 22 ( 1 )( 1) () .1 2 ss sx fxsxx -+ ⎛⎞ ⎛⎞ +- = --⎜⎟ ⎜⎟ ⎜⎟⎜⎟ ⎝⎠⎝⎠ Theo ñònh lyù Lagrange, toàn taïi (1,) zx ∈ sao cho / 1 11 2 22 ( 1)( 1) ( 1)( 1) 1 .( 1) . ,1 22 (do ) s ss s x sx x z x z sz + +- ⎛⎞ + - +- -=-= >> ⎜⎟ ⎜⎟ ⎝⎠ / ()01 fxx ⇒ > ∀> () fx ⇒ laø haøm ñoàng bieán treân (1, ) +∞ . 1 ( ) lim ( ) 0 1 x fxfxx → ⇒ ≥ = ∀> Vaäy min 3 . 2 s = Tìm tài liệu Toán ? Chuyện nh ỏ - www.toanmath.com320 Baøi toaùn 62. (Bulgaria 2003) Cho ,,0 abc > thoûa 3. Chöùng minh raèng abc + += 2 22 3 1 1 12 a bc b ca + +≥ + ++ Lôøi giaûi. * Caùch 1. Baèng caùch quy ñoàng maãu soá vaø thu goïn, ta coù baát ñaúng thöùc caàn chöùng minh töông ñöông vôùi 2323 23 33 3 2 2 2 222 2222 22 22 2 23 2323 33 3 2 2 2 222 2222 22 22 2 2() 3() 2( 3) 3() ab bc ca a b c ab bc ca a b c abc abbcca abc ab bc ca a b c ab bc ca abc abbcca abc + + + + ++ + + + + +≥ ≥ + + + + ++ ⇔ + + + + + + + + +≥ ≥ + + + + ++ AÙp duïng baát ñaúng thöùc AM-GM, ta coù 2 3 2 22 2 3 2 22 23 2 22 2 3 2 22 3 .( )3 2 3 .( )3 2 3 .( )3 2 3 . 3 (1) 2 cyc cyc cyc ab ab ab bc bc bc ca ca ca ab ab ab +≥ +≥ +≥ ⎛ ⎞⎛⎞ ⇒ +≥ ⎜ ⎟⎜⎟ ⎜ ⎟⎜⎟ ⎝ ⎠⎝⎠ ∑ ∑∑ Laïi aùp duïng baát daúng thöùc AM-GM, ta coù 3 3 3 333 333 2 13 2 13 2 13 2( ) 3 3( )9 2( ) 6 (2) aa bb cc abc a bc abc +≥ +≥ +≥ ⇒ + + +≥ ++ = ⇒ + +≥ Tieáp tuïc aùp duïng baát ñaúng thöùc AM-GM, ta coù Tìm tài liệu Toán ? Chuyện nh ỏ - www.toanmath.com321 2 3 2 3 2 3 2 2 2 43 43 43 2 22 3 2 22 1 .( )3 2 3 ab bc ca ab bc ca a b c abc abc + + + + +≥ = 2 22 2 3 3 abc abc ≥ ++ ⎛⎞ ⎜⎟ ⎝⎠ 222 3abc = 2 3 23 2 3 2 2 2 2 22 1 .( ) 3 (3) 2 ab bc ca ab bc ca abc ⇒ + ++ + +≥ Coäng caùc baát ñaúng thöùc (1),(2) vaø (3) veá theo veá, ta ñöôïc 2323 23 33 3 2 2 2 222 2222 22 22 2 2( 3) 3() ab bc ca a b c ab bc ca abc abbcca abc + + + + + + + + +≥ ≥ + + + + ++ ñpcm. ⇒ * Caùch 2. Ta coù baát ñaúng thöùc caàn chöùng minh töông ñöông vôùi 2 22 2 22 2 22 3 1 1 12 3 1 1 12 abc a b c abc b ca ab bc ca b ca ⎛ ⎞⎛ ⎞⎛ ⎞ - + -+ -≥ + + + ⎜ ⎟⎜ ⎟⎜ ⎟ +++ ⎝ ⎠⎝ ⎠⎝ ⎠ ⇔ + +≤ + ++ AÙp duïng baát ñaúng thöùc AM-GM, ta coù 22 2 1 . 1 22 ab ab ab bb ≤= + Töông töï, ta coù 2 2 2 2 1 . 12 1 . 12 bc ab c ca ab a ≤ + ≤ + Do ñoù Tìm tài liệu Toán ? Chuyện nh ỏ - www.toanmath.com322 2 22 2 22 13 .() 11122 ab bc ca ab bc ca b ca + +≤++ ≤ + ++ ñpcm. ⇒ Ñaúng thöùc xaûy ra khi vaø chæ khi 1. abc === Baøi toaùn 63. Cho 4, n nN ≥∈ vaø 12 , ,..., 0 n aaa ≥ thoûa maõn 12 ... 2. n aaa + + += Tìm giaù trò nhoû nhaát cuûa bieåu thöùc 12 2 22 2 31 ... 1 11 n a aa P a aa = + ++ + ++ Lôøi giaûi. Ta coù Boå ñeà sau Boå ñeà. 12 4, , ,..., 0. vaø n n nN aaa ≥∈≥ Khi ñoù, ta coù 2 12 23 1 12 4( ... ) ( ... ) nn aaaa aa aaa + + + ≤ + ++ Chöùng minh. Ñaët 2 1 2 12 23 1 12 ( , ,..., ) 4( ... ) ( ... ) n n nn faa a aaaa aa aaa = + + + - + ++ . Ñeå chöùng minh Boå ñeà treân, ta seõ chöùng minh baèng quy naïp theo n raèng 12 ( ,,..., )0 nn faaa ≤ + 4, n = ta coù 2 4 1 2 3 4 1223 3441 1 23 4 2 1 3 2 4 1234 2 1 234 4 1234 ( , , ,)4( ) ( ) 4()()() () 0 ( ,, , )4 faaaa aa aaaa aa a a aa a a aa a aa a a a aa f a a aa = + + + - + ++ = + + - + ++ =- -+- ≤ ⇒≤ Vaäy khaúng ñònh ñuùng khi 4. n = Giaû söû khaúng ñònh ñuùng cho 1 n - bieán soá ( 5) n ≥ , ta seõ chöùng minh khaúng ñònh ñuùng cho n bieán soá. Tìm tài liệu Toán ? Chuyện nh ỏ - www.toanmath.com323 Khoâng maát tính toång, quaùt coù theå giaû söû 1 12 max{ , ,..., }. n a aaa = Khi ñoù, ta coù 12 1 12 3 2 1 21 1 1 2 1 11 1 2 11 ( , ,..., ) ( , ,..., , , ) 4( ()()) 4() 0 n n n n n nn n n nn n n nn nn n nn nn faa a faa aa a a aa aa aaa aa aaa aa a a aa - - -- --- --- - -- - += = + + - + -+ = -- ≤ 1 2 11 2 21 ( , ,..., ) ( , ,..., , ) n n n n nn f aa a f aa a a a - -- ⇒ ≤+ Theo giaû thieát quy naïp, ta coù 11 2 21 ( ,,..., , )0 n n nn f aa a aa - -- +≤ Do ñoù 12 ( ,,..., )0 nn faaa ≤ Vaäy khaúng ñònh ñuùng cho n bieán soá. Theo nguyeân lyù quy naïp, khaúng ñònh ñuùng vôùi moïi 4. n ≥ Boå ñeà ñöôïc chöùng minh hoaøn toaøn. Trôû laïi baøi toaùn cuûa ta Ta coù 12 2 22 2 31 2 11 1 2 1 2 1 1 2 1 1 1 1 1 2 12 ... 1 11 1 2 1 2 2 1 .2 2 1 .( ... ) 2 8 3 2 (theo bñt AM-GM) (theo Boå ñeà treân) n nn i ii ii i n ii i i n ii i i n ii i n a aa P a aa a aa a aa a aa a aa aaa == + + = + + = + + = = + ++ + ++ ⎛⎞ = -+ ⎜⎟ + ⎝⎠ =-+ + ≥-+ =-+ ≥- + +++ = ∑∑ ∑ ∑ ∑ Tìm tài liệu Toán ? Chuyện nh ỏ - www.toanmath.com324 Ñaúng thöùc xaûy ra chaúng haïn khi 1 2 34 1, ... 0. n aa aaa == == == Vaäy 3 min 2 P = Baøi toaùn 64. Cho , ab laø caùc soá thöïc thoûa 0 ab +≠ vaø,1 xy > laø caùc haèng soá döông cho tröôùc. Tìm giaù trò nhoû nhaát cuûa bieåu thöùc 22 2 ( 1 ) ( 1) ( ,) () xy ab f ab ab ++ = + Lôøi giaûi. AÙp duïng baát ñaúng thöùc AM-GM môû roäng, ta coù 22 1 2 11 111 2 (1).. 1 ( 1)( 1) (2)1 .. ( 1) ( 1) 1 x xx x x xx x xy a xa x xy x xy x xy xa x y x xy - -- ⎡⎤ ⎛⎞ - +- += ++ ⎢⎥ ⎜⎟ +- +- - ⎝⎠⎣⎦ ⎛⎞ +- ≥+ ⎜⎟ +- - +- ⎝⎠ Töông töï, ta coù 1 22 11 (2)1 (1).. ( 1) ( 1) 1 y yy yy xy byb xy y xy - -- ⎛⎞ +- +≥+ ⎜⎟ +- - +- ⎝⎠ Do ñoù 2 22 2 11 22 ( 2) ( 1) ( 1) . ( 1 ) ( 1) ( 1) 11 11 x x xy x y xy xyxy xy a b xy xy xy ab xy xy +- +--- +- + +≥ +- -- ⎛ ⎞⎛⎞ ++ ⎜ ⎟⎜⎟ +- +- ⎝ ⎠⎝⎠ AÙp duïng baát ñaúng thöùc Bunhiacopxki, ta coù 2 222 1 1 1 11 . . .() 111 11 a b a b ab xy xy xy xy xy ⎛⎞ ⎛ ⎞⎛⎞ + + ≥ + =+ ⎜⎟ ⎜ ⎟⎜⎟ ⎜⎟ +- + - +- +- +- ⎝ ⎠⎝⎠ ⎝⎠ Do ñoù Tìm tài liệu Toán ? Chuyện nh ỏ - www.toanmath.com325 2 222 1 11 2 1 11 ( 2) ( 1) ( 1) .( ) ( 1) ( 1) ( 1) ( 2) ( ,) ( 1) ( 1) ( 1) x y xy xy xy xy x y xy xy xy xy xy a b ab xy xy xy xy f ab xy xy +- +- -- +- +- -- +- ++≥+ +- -- +- ⇒≥ +- -- Ñaúng thöùc xaûy ra khi vaø chæ khi 1 ( 1)( 1) 1 ( 1)( 1) 0 y a xyx x b xyy ab ⎧ - = ⎪ +-- ⎪ ⎪ - ⎪ = ⎨ +-- ⎪ ⎪ > ⎪ ⎪ ⎩ Vaäy 2 1 11 ( 2) min ( ,) ( 1) ( 1) ( 1) x y xy xy xy xy xy f ab xy xy +- +- -- +- = +- -- * Ghi chuù. Ñeå coù ñöôïc moät lôøi giaûi ngaén goïn nhö treân, ta phaûi traûi qua moät böôùc choïn ñieåm rôi nhö sau Giaû söû 00 (,) Mab laø ñieåm cöïc trò cuûa haøm soá ( ,) f ab thì 00 ( ,) ab laø nghieäm cuûa heä phöông trình / / 2 2 2 22 0 0 ( 1) 10 ( 1) 10 ( 1) 10 ( 1) ( ) ( 1) 0 a b f f x a xab y b yab x a xab xa xyab yb ⎧ = ⎪ ⎨ = ⎪ ⎩ ⎧ - + -= ⎪ ⇔ ⎨ - + -= ⎪ ⎩ ⎧ - + -= ⎪ ⇔ ⎨ -+ - - -= ⎪ ⎩ 2 2 ( 1) 10 ( )(( 1) ( 1) ) 0 ( 1) 10 ( 1) ( 1) x a xab ab x a yb x a xab x a yb ⎧ - + -= ⇔ ⎨ + - - -= ⎩ ⎧ - + -= ⇔ ⎨ - =- ⎩ Tìm tài liệu Toán ? Chuyện nh ỏ - www.toanmath.com326 1 ( 1)( 1) 1 ( 1 )( 1) 0 y a xyx x b xyy ab ⎧ - = ⎪ +-- ⎪ ⎪ - ⎪ ⇔= ⎨ +-- ⎪ ⎪ > ⎪ ⎪ ⎩ Töø ñaây, ta ñi ñeán moät lôøi giaûi hôi “choaùng” nhö treân. Baøi toaùn 65. (Vasile Cirtoaje) Cho 2 21 3, ,0 ( 1) n n nNk n - ≥ ∈ <≤ - vaø 12 , ,..., 0 n aaa > thoûa 12 ...1 n aaa = . Chöùng minh raèng 12 1 11 ... 1 111 n n ka ka ka k + ++≤ + + ++ Lôøi giaûi. Ta coù boå ñeàø sau Boå ñeà. 12 , ,..., n aaa laø n soá thöïc döông thoûa maõn 12 12 ... ( , ) 1, ... i) ii) iii) n i n aaa a in a a aC ≤ ≤≤ ∈ -∞+∞ ∀= + + += vaø f laø moät haøm treân( ,) -∞ +∞ thoûa maõn f loài treân( ,] c -∞ vaø loõm treân[,) c +∞ Ñaët 12 ( ) ( ) ... () n F fa f a fa = + ++ Khi ñoù, 121 .... ñaït max khi nn F a a aa - = = =≤ Chöùng minh. Giaû söû 1 , ,..., [ , ) iin aa ac + ∈ +∞ , do f loõm treân[,) c +∞ neân 11 ( ) ( ) ... ( ) ( 1) () ( ... ( 1)) i i n iin fa f a fa i fc fa a a i c ++ + + + ≤- + + ++ - - Maët khaùc do f loài treân( ,] c -∞ neân Tìm tài liệu Toán ? Chuyện nh ỏ - www.toanmath.com327 121 1 21 ( 1) ... ( 1) ( ) ( ) ( ) ... ( ) ( 1) 1 i i icaaa i fc fa fa fa n f n - - - + + ++ ⎛⎞ - + + + + ≤- ⎜⎟ - ⎝⎠ Do ñoù 121 1 1 ( 1) ... ( ) ( 1) 1 ( ... ( 1)) n i k k iin icaaa F fa nf n fa a a ic - = + - + + ++ ⎛⎞ =≤-+ ⎜⎟ - ⎝⎠ + + + + -- ∑ Boå ñeà ñöôïc chöùng minh. Trôû laïi baøi toaùn cuûa ta Ñaët 12 ( 1, ) ... n iin y kai n yy yk = =⇒= vôùi 12 2 21 ... ( 1) n n n k yyy n - =≤ - . Khi ñoù, baát ñaúng thöùc caàn chöùng minh trôû thaønh 12 1 11 ... 1 111 n n y y yk + ++≤ + + ++ Khoâng maát tính toång quaùt giaû söû 12 0 ... n yyy < ≤ ≤≤ . Ñaët 12 1 122 1 2 12 ... ln , ln ,..., ln ... ln ... 1) thì (do n nn nn xxx x yxy xy x x x n k aaa ≤ ≤≤ ⎧ = == ⎨ +++== ⎩ Xeùt haøm soá 12 1 () ( 1) x fx e = + Ta coù // // 52 .( 2) ( ) , ( ) 0 ln2 4( 1) xx x ee fx fxx e - = = ⇔= + Töø ñoù, ta coù f loài treân ( ,ln 2] -∞ vaø loõm treân [ln 2, ) +∞ ⇒ Theo boå ñeà treân, ta coù 12 12 11 11 (1) (1) i nn x ii i P y e == == + + ∑∑ ñaït max khi 121 .... nn x x xx - = = =≤ ln ( 1) 11 max max ( ln ) 11 t nkn t n P tk ee -- ⎧⎫ - ⇒≤ +≤ ⎨⎬ ++ ⎩⎭ Tìm tài liệu Toán ? Chuyện nh ỏ - www.toanmath.com328 1 2 1 1 max ( ) (1) 1 n t nn nx x ek x xk - - ⎧⎫ -⎪⎪ = + =≤ ⎨⎬ + + ⎪⎪ ⎩⎭ Tieáp theo, ta seõ tìm max cuûa haøm soá 1 2 1 1 () 1 vôùi n nn nx gx xk x xk - - - =+≤ + + Ta coù 3 2 / 33 1 22 11 (). 2 ( ) ( 1) n n nn n kx gx x kx - - ⎛⎞ - ⎜⎟ =- ⎜⎟ ⎜⎟ ++ ⎝⎠ 3 33 /1 2 22 23 1 33 ()0 .(1)() .( 1) (2) n n nn nn nn gx kx x xk kx x xk - - - - =⇔ + =+ ⇔ +=+ Ñaët 22 33 t x tk = ⇒≤ . Khi ñoù, phöông trình (2) trô ûthaønh 2 3 3 3( 1) 3222 22 3(1)3 33 2 22 .1 0 nnn n nn n nn n kt t tk t kt kt k -- -- ⎛⎞ +=+ ⎜⎟ ⎝⎠ ⇔ - - += Xeùt haøm soá 22 3(1)3 33 2 22 () nn n nn n ht t kt ktk -- = - -+ vôùi 2 3 tk ≤ Ta coù 5 22 3 / 3 33 2 1 ( ) . . 3( 1) ( 3) 2 n nn n ht t n t nkt nk - ⎛⎞ = - - -- ⎜⎟ ⎜⎟ ⎝⎠ / ()0 ht =⇔ 22 3 33 2 3( 1) ( 3)0 nn n n t nkt nk - - --= Xeùt tieáp haøm soá 2 22 3 3 33 2 ( ) 3( 1) ( 3) vôùi nn n mt n t nk t n k tk =-- --≤ Ta coù 2 1 23 / 3 22 3 ( ) . 2( 1) 2 n n n mt t n tk - ⎛⎞ = -- ⎜⎟ ⎜⎟ ⎝⎠ 2 2 23 3 / 0 ()0 2( 1) n n k mtt n - ⎛⎞ ⎜⎟ = ⇔= ⎜⎟ - ⎜⎟ ⎝⎠ Tìm tài liệu Toán ? Chuyện nh ỏ - www.toanmath.com329 Do 2 21 0 ( 1) n k n - <≤ - neân 2 3 0 tk < . Qua 0 t thì / () mt ñoåi daáu töø aâm sang döông neân () mt nghòch bieán treân 0 (0, ] t vaø ñoàng bieán treân 2 3 0 , tk ⎡⎞ ⎟ ⎢ ⎟ ⎣⎠ . Ta laïi coù 22 33 2 2 -1 (0)3 0, (2 )02 ( -1) do n n mnmknkkk n ⎛⎞ ⎛⎞ =-≤ = -> >≥ ⎜⎟ ⎜⎟ ⎜⎟ ⎝⎠ ⎝⎠ Neân phöông trình ()0 mt = coù nghieäm duy nhaát 2 3 1 0 tk << . ⇒ Phöông trình / ()0 ht = coù nghieäm duy nhaát 2 3 1 0 tk << Baûng bieán thieân cuûa () ht Caên cöù vaøo baûng bieán thieân, ta coù 23 21 ()0 coù 2 nghieäm döông phaân bieät laø vaø ht k tt =< . Do ñoù / ()0 gx = coù 2 nghieäm döông phaân bieät laø k vaø 32 32 21 t tk << . Baûng bieán thieân cuûa () gx Caên cöù vaøo baûng bieán thieân, ta suy ra t / () ht 0 0 () ht _ + 1 t 23 k 0 n k () gx x / () gx 0 0 _ + 32 2 t k 0 Tìm tài liệu Toán ? Chuyện nh ỏ - www.toanmath.com330 { } ( ) max (0), ( ) (3) 1 n gx g gk xk k ≤ = ∀≤ + Töø (1) vaø (3), ta suy ra ñpcm. Baøi toaùn 66. Cho ,, abc laø ñoä daøi ba caïnh cuûa moät tam giaùc. Chöùng minh raèng 0 ( )()() ab bc ca bbca cc a b aab c --- + +≥ + - + - +- Lôøi giaûi. Do ,, abc laø ñoä daøi ba caïnh cuûa moät tam giaùc neân toàn taïi caùc soâ thöïc döông ,, xyz sao cho ,, a yzb zxc xy =+=+=+ . Khi ñoù, baát ñaúng thöùc caàn chöùng minh töông ñöông vôùi 0 ( ) ( ) () y x x z zy xz x zy z yxy - -- + +≥ +++ Ñeán ñaây, ta coù hai caùch chöùng minh cho baát ñaúng thöùc treân * Caùch 1. Coù 2 tröôøng hôïp xaûy ra + Tröôøng hôïp 1. 0 x yz ≥ ≥> . Khi ñoù, ta coù 0 ( ) ( ) () 1 1 11 () ()0 () ( ) () () y x x z zy xzx zyz yxy x y yz z y z xz x z y z yx y - -- + +≥ +++ ⎛ ⎞⎛⎞ ⇔- - +- - ≥ ⎜ ⎟⎜⎟ + + ++ ⎝ ⎠⎝⎠ Baát ñaúng thöùc naøy ñuùng do 0. xyz ≥ ≥> + Tröôøng hôïp 2. 0 z yx ≥ ≥> . Khi ñoù, ta coù 0 ( ) ( ) () 1 1 11 () ()0 ( )( ) ( )( ) y x x z zy xzx zyz yxy y x zy xzx zy z yx y zyz - -- + +≥ +++ ⎛ ⎞⎛⎞ ⇔- - +- -≥ ⎜ ⎟⎜⎟ + + ++ ⎝ ⎠⎝⎠ Tìm tài liệu Toán ? Chuyện nh ỏ - www.toanmath.com331 Baát ñaúng thöùc naøy ñuùng do 0 z yx ≥ ≥> . Toùm laïi, trong moïi tröôøng hôïp ta luoân coù 0 ( ) ( ) () ñpcm. y x x z zy xz x zy z yxy - -- + +≥ +++ ⇒ * Caùch 2. Ta coù 0 ( ) ( ) () 1 1 1 1 11 . . . . .. y x x z zy xzx zyz yxy x yz xy z y zz z x x x y y y z x z x y x yz - -- + +≥ +++ ⇔ + + ≥ ++ + + + + ++ Ñieàu naøy ñuùng theo baát ñaúng thöùc saép xeáp laïi do ,, xyz y zz xxy ⎛⎞ ⎜⎟ + ++ ⎝⎠ vaø 111 ,, x yz ⎛⎞ ⎜⎟ ⎝⎠ laø hai daõy ngöôïc chieàu nhau. Vaäy ta coù ñpcm. Ñaúng thöùc xaûy ra khi vaø khi . x y z abc = =⇔ == Baøi toaùn 67. Cho , , , 0. k abcd ≥> Chöùng minh raèng 4444 4 4 44 (2 ) (2 ) (2 ) (2 ) (2 )(2 )(2 )(2 ) a b c d ka k b k c kd abcd ka k b kc kd + + + - +- +- +- ≥ - --- Lôøi giaûi. Khoâng maát tính toång quaùt, coù theå giaû söû 0. k abcd ≥ ≥ ≥ ≥> Khi ñoù, baát ñaúng thöùc caàn chöùng minh töông ñöông vôùi 2 22 2 22 2 2 2 2 2 22 ( ) ( ) 2( ) ((2 ) (2 )) (2 )(2 )(2 )(2 ) a b c d ab cd k a kb abcd abcd abcd k a k b k c k d - - + - -- ++≥+ - - -- 2 22 2 2 22 ((2 ) (2 )) 2((2 )(2 ) (2 )(2 )) (2 )(2 )(2 )(2 ) (2 )(2 )(2 )(2 ) kc kd k a k b kc kd ka k b kc kd k a k b kc k d -- - - -+ -- ++ - -- - - -- - Tìm tài liệu Toán ? Chuyện nh ỏ - www.toanmath.com332 Do ñoù, ñeå chöùng minh baát ñaúng thöùc ñaõ cho, ta chæ caàn chöùng minh 2 22 2 22 2 2 2 2 22 2222 2 2 22 ( ) ((2 ) (2 )) (1) (2 )(2 )(2 )(2 ) ( ) ((2 ) (2 ) ) (2) (2 )(2 )(2 )(2 ) 2( ) 2((2 )(2 ) (2 )(2 )) (3) (2 )(2 )(2 )(2 ) a b k a kb abcd k a k b k c k d c d k c kd abcd k a k b k c k d ab cd ka k b k c kd abcd k ak bk ck d - - -- ≥ - - -- - - -- ≥ - - -- + - -+ -- ≥ - - -- Ta coù 2 22 2 22 2 2 22 22 ( ) ((2 ) (2 )) (2 )(2 )(2 )(2 ) ( ) ( ) (2 )(2 )(2 )(2 ) ( ) (4 ) ( ) (2 )(2 )(2 )(2 ) (4 ) a b k a kb abcd k a k b k c k d ab a b k a k b k c k d ab k ab abcd a b k a k b k c k d k a b abcd - - -- ≥ - - -- ⇔ - + - - - - ≥ - -- ⇔ + - - - - ≥ -- Do 0 (2 )(2 ) 0. k c d kc kd cd ≥ ≥ > - - ≥> neân Do ñoù, ñeå chöùng minh (1), ta chæ caàn chöùng minh 22 ( ) (2 )(2 ) (4 ) a b k a k b k a b ab + - - ≥ -- 2 22 2 22 2 22 22 2 2 2 2 22 2 00 2 22 (ñuùng do ) ab ab ab ab ab ab kk ab ab ab k kab ⎛ ⎞ ⎛⎞ + +- + +- ⎛⎞⎛ ⎞⎛⎞⎛ ⎞⎛⎞⎛⎞ ⇔ --≥-- ⎜ ⎟ ⎜⎟ ⎜⎟⎜ ⎟⎜⎟⎜ ⎟⎜⎟⎜⎟ ⎜ ⎟ ⎜⎟ ⎝⎠⎝ ⎠⎝⎠⎝ ⎠⎝⎠⎝⎠ ⎝ ⎠ ⎝⎠ ⎛⎞ - ++ ⎛ ⎞⎛ ⎞⎛⎞ ⇔ - - ≥ ≥≥> ⎜⎟ ⎜ ⎟⎜ ⎟⎜⎟ ⎜⎟ ⎝ ⎠⎝ ⎠⎝⎠ ⎝⎠ Vaäy (1) ñuùng. Chöùng minh töông töï, ta coù (2) ñuùng. Tieáp theo, ta seõ chöùng minh (3) ñuùng. Ta seõ chöùng minh (2 )(2 ) (2 )(2) ab k c kd cd k a kb -- ≥ -- Thaät vaäy (2 )(2 ) (2 )(2) ab k c kd cd k a kb -- ≥ -- Tìm tài liệu Toán ? Chuyện nh ỏ - www.toanmath.com333 2 222 2 222 (2 ) (2 ) .1 (2 ) (2 ) ( ) () . 10 ( ) () (ñuùng do ) ak a bk b c k cdkd k k a k kb k abc kkc kkd -- ⇔≥ -- - - -- ⇔ ≥ ≥ ≥≥> -- -- Vaäy (2 )(2 ) 1 (2 )(2) ab k c kd cd k a kb -- ≥≥ -- Do haøm soá 1 () fxx x =+ ñoàng bieán treân[1,) +∞ neân ta coù (2 )(2) (2 )(2) ab kc kd ff cd k a kb ⎛⎞ -- ⎛⎞ ≥ ⎜⎟⎜⎟ -- ⎝⎠ ⎝⎠ Maët khaùc, ta coù 2 2 22 2 2 22 (2 )(2 ) (2 ) (2 ) (2 ) (2 ) (2 )(2 ) (2 )(2 )(2 )(2 ) ab ab cd f cd abcd k c kd k a k b k c kd f ka k b k a k b k c kd + ⎛⎞ = ⎜⎟ ⎝⎠ ⎛⎞ - - - -+ -- = ⎜⎟ -- -- - - ⎝⎠ Do ñoù 2222 2 2 22 2( ) 2((2 )(2 ) (2 )(2 )) (2 )(2 )(2 )(2 ) ab cd k a k b kc kd abcd ka k b k c kd + - - + -- ≥ - - -- Vaäy (3) ñuùng. Töø ñaây, ta suy ra ñieàu phaûi chöùng minh. Ñaúng thöùc xaûy ra khi vaø chæ khi . abcd = == Baøi toaùn 68. ,,01 Cho thoûa . Chöùng minh raèng xyz x yz > + += 2 2 2 2 22 23 x xyz y xyz z xyz x y z xy yz zx xyz + + + + + ≥ + ++ + ++ Lôøi giaûi. Ta coù baát ñaúng thöùc caàn chöùng minh töông ñöông vôùi Tìm tài liệu Toán ? Chuyện nh ỏ - www.toanmath.com334 ( ) 2 2 2 2 2 22 23 2 ()()2 ()()2 ()( ) ( 3 ) 23 x xyz y xyz z xyz x y z xy yz zx xyz xy x yz y zx yz y zx z xy zx z xy x yz xy yz zx xyz xyz + + + + + ≥ + ++ + ++ ⇔+ +++ ++ + +≥ ≥ ++-+ Ta coù ( ) 22 2( )( )2(() )(()) 2( ) ( )( ) 2() ( )3 ( )3 (theo bñt Bunhiacopxki) xyxyz y zx xyxxy z yz yxy z zx x y xyz xzy x y x y xyz x y xy xyz xy xyz x y xyz + + = + ++ + ++ = + ++ =++ ≥++ = - ++ Töông töï, ta coù 2 ( )( ) ( )3 2 ( )( ) ( )3 2 ()( )2 ( )( )2 ( )() ( 3 ) 23 yz y zx z xy yz xyz y z xyz zx z xy x yz zx xyz z x xyz xy x yz y zx yz y zx z xy zx z xy x yz xy yz zx xyz xyz + + ≥ - ++ + + ≥ - ++ ⇒ + + + + + + + +≥ ≥ ++-+ ñpcm. ⇒ Ñaúng thöùc xaûy ra khi vaø chæ khi 1 . 3 x yz = == Baøi toaùn 69. (USAMO 1999) Cho caùc soá thöïc 12 , ,..., ( 3) n aa an > thoûa 12 2 2 22 12 ... . ... n n a a an a a an + + +≥ ⎧ ⎪ ⎨ + + +≥ ⎪ ⎩ Chöùng minh raèng 12 max{ , ,... } 2. n aaa ≥ Lôøi giaûi. * Caùch 1. Giaû söû ngöôïc laïi 2 1, i a in < ∀= . Khi ñoù, ta coù 12 ... 2( 1) 2 1, i a a a i in + + + - - < ∀= Do 2 1, i a in < ∀= neân Tìm tài liệu Toán ? Chuyện nh ỏ - www.toanmath.com335 2 2 2 22 2 2 22 (2 )(2 ) 0 42( )0 84()20 (24()())2 ( 2)2 ij i j ij i j ij ij ij i j i j ij aa a a aa a a aa aa aa a a a a aa - -> ⇒- + +> ⇔- + +> ⇔ - + + + + >+ ⇔ + - +>+ Do ñoù 2 2 22 1 2 12 2222 12 3 3 12 2222 12 121 ( 2)2 ( 2.2) 2 ( 2) ........ ( ... 2( 1)) 2 ( ... 2( 2)) n nn a a aa a a a a aa aaa n a aaa n - + - +>+ ++ - + > + +- ++ +- - + > + ++ + - - Coäng 1 n - baát ñaúng thöùc treân laïi veá theo veá, ta ñöôïc 2 222 1 2 12 ( ... 2( 1)) 4( 1) ... nn aa a n n aaa + ++ - - + - > + ++ Do 2 1, i a in < ∀= vaø 12 ... n a a an + + +≥ neân 12 2 ... 2( 1) 2 n aa ann > ++ +- - ≥ - Do 4 - 2 2. neân Do ñoù nn ≥≥ 12 22 12 22 12 2 ... 2( 1) 2 ( 2) ( ... 2( 1)) ( 2) 4( 1) ( ... 2( 1)) 4( 1) n n n n aa ann n aa an n n aa ann - > + + + - - ≥- ⇒- > ++ + - - ⇒- + - > ++ +- - + - Do ñoù 2 2 222 12 ( 2) 4( 1) ... n n n n aaa = - + - > + ++ Ñieàu naøy traùi vôùi giaû thieát 2 2 22 12 ... n a a an + + +≥ . Vaäy ta phaûi coù 12 max{ , ,..., } 2. n aaa ≥ * Caùch 2. Giaû söû ngöôïc laïi 2 1, i a in < ∀= . Ñaët 2 11 2(1,),. vaø nn i i ii ii b ai nS b Tb == =-= == ∑∑ Theá thì, töø giaû thieát, ta coù Tìm tài liệu Toán ? Chuyện nh ỏ - www.toanmath.com336 12 2 2 22 12 2 (2 )(2)...(2) (2 ) (2) ...(2) 44 n n b b bn b b bn Sn T n nS - +- + +- ≥ ⎧ ⎪ ⎨ - +- + +- ≥ ⎪ ⎩ ≤ ⎧ ⇔ ⎨ ≥ -+ ⎩ Töø ñaây, ta coù 2 44 T n nS ≥ -+ ( 4)4 n nS = -+ (4)44 (1) (do ) Sn Sn nS ≥-+≥ = Do 2 1, 0 1, 1 , i ii a in b i n b n in Sn < ∀= > ∀= ⇒ < ∀= ≤ neân (do ). Do ñoù 2 11 (2) nn ii ii T b nb nS == = <= ∑∑ Töø (1) vaø (2), ta suy ra maâu thuaãn. Vaäy ta phaûi coù 12 max{ , ,..., } 2 (ñpcm) n aaa ≥ Baøi toaùn 70. (Toaùn Hoïc Tuoåi Treû 2006) Cho caùc soá thöïc 1111 , , , , , ( 0) a b c a b c aa ≠ thoûa 2 1 1 11 1 11 .0 c c b b bc bc a a a a aa ⎛⎞⎛⎞ - -+-< ⎜⎟⎜⎟ ⎝⎠⎝⎠ Chöùng minh raèng hai phöông trình 2 0 ax bxc + += vaø 2 1 11 0 ax bxc + += ñeàu coù hai nghieäm phaân bieät vaø caùc nghieäm naøy naèm xen keõ nhau khi bieåu dieãn treân truïc soá. Lôøi giaûi. Khoâng maát tính toång quaùt, ta chæ caàn xeùt 1 1 aa == laø ñuû. Khi ñoù, baøi toaùn chuyeån veà Tìm tài liệu Toán ? Chuyện nh ỏ - www.toanmath.com337 “Caùc soá thöïc 11 ,,, bcbc thoûa 2 1 1 11 ( ) ( )( )0 c c bbbc bc -+ - -< . Khi ñoù, caùc phöông trình 2 ()0 fx x bxc = + += vaø 2 11 ()0 gx x bxc = + += ñeàu coù hai nghieäm phaân bieät vaø caùc nghieäm naøy naèm xen keõ nhau khi bieåu dieãn treân truïc soá.” Ñeå chöùng minh hai phöông trình naøy coù hai nghieäm phaân bieät, ta caàn phaûi chöùng minh 2 2 11 40 40 f g bc bc ⎧Δ= -> ⎪ ⎨ Δ= -> ⎪ ⎩ Tröôùc heát, ta chöùng minh 22 11 (4)(4)0 bcbc - -> Giaû söû ngöôïc laïi 22 2 2 22 1 11 11 11 (4)(4)0 2 2 4 bb b c b c bc bc cc cc - - ≤⇒ + - ≥+ . Do ñoù 2 2 2 22 1 1 1 1 1 1 1 1 11 22 22 1 1 1 11 2 11 ( ) ( )( )( 2 ) () 2 () 4 1 .( 2( )) 4 0 c c b b bc bc b c b c cc c c bb c c bb cc c c bb c c bb cc - +- - = + - ++-+ ≥ + ++-+ = -+ ≥ Ñieàu naøy traùi vôùi giaû thieát. Vaäy ta phaûi coù 22 11 (4)(4)0 bcbc - -> (*) Tieáp theo, ta seõ chöùng minh 2 2 11 40 40 bc bc ⎧ -> ⎪ ⎨ -> ⎪ ⎩ . Giaû söû ñieàu naøy khoâng ñuùng. Khi ñoù, töø (*), ta coù 2 2 11 40 40 bc bc ⎧ -< ⎪ ⎨ -< ⎪ ⎩ . Do ñoù 2 2 22 1 1 1 1 1 1 1 11 2 1 1 1 11 ( ) ()( )( ) () ( ) 2 () cc b bbc bc cc bc bc bbc c c c bb cc bb c c -+ - - = -+ + - + ≥- + -+ ( ) ( ) ( ) 2 2 1 11 22 1 11 () cc bb cc c c c c bb =- -- ⎛⎞ = - +- ⎜⎟ ⎝⎠ Tìm tài liệu Toán ? Chuyện nh ỏ - www.toanmath.com338 ( )( ) 2 1 11 4 c c cc bb ≥-- ( ) 22 2 1 11 4. 44 0 bb c c bb ⎛⎞ ≥-- ⎜⎟ ⎜⎟ ⎝⎠ = Ñieàu naøy traùi vôùi giaû thieát. Vaäy ta phaûi coù 2 2 11 40 40 bc bc ⎧ -> ⎪ ⎨ -> ⎪ ⎩ . Töùc laø caùc phöông trình 2 ()0 fx x bxc = + += vaø 2 11 ()0 gx x bxc = + += ñeàu coù hai nghieäm phaân bieät. Goïi 12 , xx laø caùc nghieäm cuûa phöông trình 2 ()0 fx x bxc = + += thì theo ñònh lyù Viet, ta coù 1 2 12 . vaø x x b xxc +=-= Ñeå chöùng minh ( ) () vaø f x gx coù caùc nghieäm naèm xen keõ nhau khi bieåu dieãn treân truïc soá, ta chæ caàn chöùng minh 12 ( ).( )0 gx gx < Ta coù 22 1 1 1 1 1 1 11 22 2 1 21 2 2 22 0 ()() ()() 0 x bxc x bxc gx b bxcc gx b bx cc x bx c x bx c ⎧⎧ + + = =- - = - +- ⎧ ⎪⎪ ⇒⇒ ⎨ ⎨⎨ = - +- + += =-- ⎪ ⎪⎩ ⎩⎩ Do ñoù 1 2 1 1 1 1 21 22 1 1 1 1 2 1 12 22 1 1 11 2 1 1 11 2 1 1 11 ( ). ( ) (( ) )(( ) ) ( ) ( )()( )() ( ) ( )() () ( ) ( )( ( ) ( )) ( ) ( )( )0 (theo gt) gx gx b bxcc b bx cc cc ccb bx x b b xx c c bc cbb cbb c c b bcb b bcc c c bbbc bc = - +- - +- = - +- - + +- = - - - - +- = - + - - -- = - + - -< 12 ( ).( )0 gx gx ⇒< Vaäy ( ) () vaø f x gx coù caùc nghieäm naèm xen keõ nhau khi bieåu dieãn treân truïc soá. Töø caùc chöùng minh treân, ta suy ra ñpcm. Tìm tài liệu Toán ? Chuyện nh ỏ - www.toanmath.com339 Baøi 71. Cho caùc soá döông ,, abc thoûa 21 2 8 12. ab bc ca ++≤ Tìm giaù trò nhoû nhaát cuûa bieåu thöùc 1 23 P abc = ++ Lôøi giaûi. + Caùch 1. Ñaët 1 23 , ,, x yz a bc = == baøi toaùn chuyeån veà ,,0 xyz > thoûa2 4 7 2. x y z xyz + +≤ Tìm giaù trò nhoû nhaát cuûa bieåu thöùc P x yz =++ Khoâng maát tính toång quaùt, ta chæ caàn xeùt tröôøng hôïp2 4 72 x y z xyz + += laø ñuû (taïi sao?). Ñaët 7 27 7, .,. 27 x my nzp = == thì ta coùm npmnp + += . Do ñoù, toàn taïi tam giaùc nhoïn ABC sao cho , ,. tg tg tg mAnBp C = == Khi ñoù, ta coù 7 .(14 7 4 ) 14 tg tg tg P A BC = ++ Xeùt haøm soá () tg fxx = vôùi 0,. 2 x π ⎛⎞ ∈ ⎜⎟ ⎝⎠ Ta coù /2 //2 ()1 () 2 ( 1)0 tg tg tg fxx f x xx =+ = +> ⇒ () 0,. 2 laø haøm loõm treân fx π ⎛⎞ ⎜⎟ ⎝⎠ Do ñoù, theo tính chaát haøm loõm, ta coù / 37 37 37 () 7 77 arctg arctg arctg fAf fA ⎛ ⎞⎛ ⎞⎛⎞ ≥ +- ⎜ ⎟⎜ ⎟⎜⎟ ⎝ ⎠⎝ ⎠⎝⎠ Tìm tài liệu Toán ? Chuyện nh ỏ - www.toanmath.com340 37 16 37 . 777 37 14 ( ) 6 7 32 7 arctg arctg A fAA ⎛⎞ = +- ⎜⎟ ⎝⎠ ⎛⎞ ⇒ ≥ +- ⎜⎟ ⎝⎠ Töông töï, ta coù / 57 57 57 () 7 77 5 7 32 57 . 777 57 7 ( )5 732 7 arctg arctg arctg arctg arctg fBf fB B fBB ⎛⎞⎛⎞⎛ ⎞ ≥ +- ⎜⎟⎜⎟⎜ ⎟ ⎝⎠⎝⎠⎝ ⎠ ⎛⎞ = +- ⎜⎟ ⎝⎠ ⎛⎞ ⇒ ≥ +- ⎜⎟ ⎝⎠ ( ) ( )( ) ( ) ( ) / () 7 77 787 4()47327 arctg arctg arctg arctg arctg fCf fC C fCC ≥ +- = +- ⇒ ≥ +- Do ñoù 7 .(14( ) 7( ) 4()) 14 7 37 57 .157327 14 77 arctg arctg arctg P f A f B fC A BC = ++ ⎛⎞ ⎛⎞ ≥ + ++--- ⎜⎟ ⎜⎟ ⎜⎟ ⎝⎠ ⎝⎠ 15 3 7 57 ( 7) 2 77 vì arctg arctg arctg ABC π =++= ++= Ñaúng thöùc xaûy ra khi vaø chæ khi 37 1 3 7 3 57 54 . 7 25 2 3 7 2 arctg arctg arctg A a m B nb p C c ⎧ ⎧ = ⎪ = ⎪ = ⎧ ⎪ ⎪ ⎪ ⎪ ⎪ ⎪⎪ = ⇔ = ⇔= ⎨ ⎨⎨ ⎪ ⎪⎪ = ⎪ ⎪⎪ = ⎩ = ⎪⎪ ⎩ ⎪ ⎩ Vaäy Tìm tài liệu Toán ? Chuyện nh ỏ - www.toanmath.com341 15 min. 2 P = + Caùch 2. t 1 43 ,, 3 52 a bc xyz = == , bài toán chuyn v “,,0 xyz > và 3 5 7 15 x y z xyz + +≤ . Tìm giá tr nh nht c a biu th c 1 .(6 5 4 ). 2 P x yz = ++ ” Áp dng bt ng AM-GM cho 15 s d ng, ta có 3 57 15 12 10 8 15 6 54 15 15 3 5 7 15 1 1 xyz x y z x y z xyz x yz ≥ ++≥ ⇒≥ ⇔≥ i áp dng bt ng th c AM-GM cho 15 s d ng, ta có 6 54 15 1 15 15 .(6 5 4). 2 22 P x y z xyz =++≥≥ ng th c xy ra khi và ch khi 1 3 4 1. 15 3 57 5 3 2 a x yz xyzb xyz x yz c ⎧ = ⎪ ⎪ == ⎧ ⎪ ⇔ = = =⇔= ⎨⎨ = ++ ⎩ ⎪ ⎪ = ⎪ ⎩ t lu n 15 min. 2 P = Tìm tài liệu Toán ? Chuyện nh ỏ - www.toanmath.com342 Baøi toaùn 72. Cho ,,0 abc > . Chöùng minh raèng 21 ( )( )() cyc a abc bc abb cca ≥+ + + ++ ∑ Lôøi giaûi. Ta coù baát ñaúng thöùc caàn chöùng minh töông ñöông vôùi 3 33 ()()2( )() ( )( ) 2 ( ) ( )( ) 4( )( ) 2 () ( )( )3 ()9 cyc cyc cyc cyc cyc a a b a c a b c ab bc ca aab ac ab aba cbc ab cab bcca a b c a b ab a c b c ab a b abc + + ≥ + + ++ ⇔ + + + + + + ≥ + + ++ ⇔ + + + + + + ≥ ++ ∑ ∑∑ ∑∑ AÙp duïng baát ñaúng thöùc Bunhiacopxki, ta coù ( ) 2 ( )() a cb c abc + +≥+ Do ñoù, ñeå chöùng minh baát ñaúng thöùc ñaõ cho, ta chæ caàn chöùng minh ( ) 3 33 3 33 2 () 3 ()9 2 . ( ) ( )9 cyc cyc cyc cyc a b c a b c ab ab ab a b abc a b c abc a b c ab a b abc + + + + + ≥ ++ ⇔ + + + + ≥ ++ ∑∑ ∑∑ AÙp duïng baát ñaúng thöùc AM-GM, ta coù 2 . ( ) 12 cyc abc a b c abc +≥ ∑ Do ñoù, theo baát ñaúng thöùc Schur, ta coù 3 33 3 33 3 33 3 33 2 . ( ) 12 ( 3 )9 ( )9 2 . ( ) ( )9 cyc cyc cyc cyc a b c abc a b c a b c abc a b c abc abc ab a b abc a b c abc a b c ab a b abc + ++ + ≥ + ++ = ++++ ≥ ++ ⇒ + + + + ≥ ++ ⇒ ∑ ∑ ∑∑ ñpcm. Ñaúng thöùc xaûy ra khi vaø chæ khi ,0 abc abc == =→ hoaëc vaø caùc hoaùn vò. Tìm tài liệu Toán ? Chuyện nh ỏ - www.toanmath.com343 Baøi toaùn 73. (Phaïm Kim Huøng) Cho ,,0 abc ≥ thoûa 1. abc + += Chöùng minh raèng 22 2222 2 22 8( )( 16 ) ab bc ca a b b c c a a b c abc + + ≥ + + +++ Lôøi giaûi. Ñaët ,0 , 1 3 qr q ab bc ca r abc q ≥ ⎧ ⎪ = + + =⇒ ⎨ ≤ ⎪ ⎩ theá thì theo baát ñaúng thöùc Schur, ta coù 41 9 q r - ≥ . Töø caùch ñaët, ta coùù 22 22 222 222 2 12 ab bc ca q r abcq + + =- + + =- Do ñoù, baát ñaúng thöùc caàn chöùng minh trôû thaønh 2 2 8( 2 )(16 1 2 ) ( ) 8(2 )(16 1 2 ) 0 q q r rq fr r q r qq ≥ - +- ⇔ = - + - +≥ Ta coù / ( ) 6(32 (4 1)(2 1)) fr r qq = - -+ Coù 2 tröôøng hôïp xaûy ra * Tröôøng hôïp 1. / 1 4 ()0 q fr ≥ ⇒≥ ⇒ () fr laø haøm ñoàng bieán 0. r ∀≥ * Tröôøng hôïp 2. 41 410 9 q qr - ≥⇒≥≥ . Do ñoù / 32(4 1) ( ) 6(32 (4 1)(2 1)) 6 (4 1)(2 1) 9 2(4 1)(23 18 ) 3 0 q fr r qq qq qq -⎛⎞ = - -+ ≥ - -+ ⎜⎟ ⎝⎠ -- = ≥ ⇒ () fr laø haøm ñoàng bieán 0. r ∀≥ Toùm laïi, trong moïi tröôøng hôïp, ta luoân coù () fr laø haøm ñoàng bieán 0. r ∀≥ Do ñoù 2 ( ) (0) (4 1) 0 ñpcm. fr f qq ≥=- ≥ ⇒ Tìm tài liệu Toán ? Chuyện nh ỏ - www.toanmath.com344 * Nhaän xeùt. Coù theå deã daøng chöùng minh ñöôïc 16 laø haèng soá toát nhaát cho baát ñaúng thöùc 22 2222 2 22 8( )( ) ab bc ca a b b c c a a b c kabc + + ≥ + + +++ Baøi toaùn 74. (Voõ Quoác Baù Caån) Cho,,01 thoûa . Chöùng minh raèng a b c abc >= 3 33 3 2 1 11 a bc c ab ++≥ + ++ Lôøi giaûi. Ta coù baát ñaúng thöùc caàn chöùng minh töông ñöông vôùi 3 3 333 43 4 3 333 43 333 4 33 32 2 ( 1)( 1) 3( 1)( 1)( 1) 2 2 2 2 3 36 2 3 2 3 2 30 2 ( 1) (2 1) cyc cyc cyc cyc cyc cyc cyc cyc cyc cyc cyc cyc cyc cyc cyc aab abc ab a ab a aba ab abab a aba a abaa + +≥ + ++ ⇔ + + + ≥ ++ ⎛ ⎞⎛ ⎞⎛⎞ ⇔ +- + +- + -≥ ⎜ ⎟⎜ ⎟⎜⎟ ⎜ ⎟⎜ ⎟⎜⎟ ⎝ ⎠⎝ ⎠⎝⎠ ⇔ -+ ∑ ∑ ∑ ∑ ∑ ∑∑ ∑ ∑ ∑ ∑ ∑ ∑∑ ∑ 4 33 2 3 2 30 cyc cyc cyc cyc a ab aa ⎛ ⎞⎛⎞ + + - + -≥ ⎜ ⎟⎜⎟ ⎜ ⎟⎜⎟ ⎝ ⎠⎝⎠ ∑ ∑ ∑∑ Do ñoù, ñeå chöùng minh baát ñaúng thöùc ñaõ cho, ta chæ caàn chöùng minh 4 33 2 3 0 (1) 3 (2) cyc cyc cyc cyc a aba a +-≥ ≥ ∑ ∑∑ ∑ * Chöùng minh (1). Ta coù 4 3 3 4 33 3 433 44 43 3 3 2 3 2 3. 2 ( ). (theo AM-GM) cyc cyc cyc cyc cyc cyc cyc cyc cyc a ab a a ab abc a a ab abca ab cabbcca +-= +- ≥ + - ++ = + + - -- ∑∑ ∑ ∑∑ ∑ ∑∑∑ 4 43 1 . (3 3) 4 cyc a b ab = +- ∑ Tìm tài liệu Toán ? Chuyện nh ỏ - www.toanmath.com345 222 1 .( )(3 2) 4 0 (1) ñuùng. cyc ab a abb =-+ + ≥ ⇒ ∑ * Chöùng minh (2). Ta coù ( ) ( ) 2 3 3 3 3 33 1 3 3 . .0 2 cyc cyc cyc a a abc a bc ab -= -=++ - ≥ ∑∑∑ (2) ñuùng. ⇒ Töø (1) vaø (2), ta suy ra ñpcm. Ñaúng thöùc xaûy ra khi vaø chæ khi 1. abc === Baøi toaùn 75. (Vasile Cirtoaje) Cho,,03 thoûa . Chöùng minh raèng abc abc > + += 222 1 11 1 2 12 121 ab bc ca + +≥ +++ Lôøi giaûi. Ta coù 222 222 2 2 2 3 24 32 1 11 2 12 121 111 1113 2 1 21 21 3 2. 21 3 2. 3 2 3. 3 22 3. 33 (theo bñt AM-GM) (theo bñt AM-GM) cyc cyc cyc cyc ab bc ca ab bc ca ab ab ab ab ab ab + += +++ ⎛ ⎞⎛⎞⎛⎞ = -+ -+ -+ ⎜ ⎟⎜⎟⎜⎟ +++ ⎝ ⎠⎝⎠⎝⎠ =- + ≥- =- + ≥- ∑ ∑ ∑ ∑ Tìm tài liệu Toán ? Chuyện nh ỏ - www.toanmath.com346 222 2 3. 3 2 3 .3 3 1 111 1 2 12 121 (ñpcm) cyc a ab bc ca =- =- = ⇒ + +≥ +++ ∑ Ñaúng thöùc xaûy ra khi vaø chæ khi 1. abc === Baøi toaùn 76. (Vasile Cirtoaje) Cho , , 0. Chöùng minh raèng abc ≥ 222 ( ) ( ) ( )0 a bc b c b ca c a c ab a b - + + - ++ - +≥ Lôøi giaûi. Ta coù baát ñaúng thöùc caàn chöùng minh töông ñöông vôùi 2 (22)0 (( )( ) ( )( )) 0 cyc cyc a bc bc ac ab abca bc - +≥ ⇔ + - - + - +≥ ∑ ∑ ( ) 2 ( )( ) ( )( ) 0 ()() ( )( ) 0 ()()().0 () ()( ) 0 (ñuùng) ñpcm. cyc cyc cyc cyc cyc cyc a ca b b c abc a bc a c ab bc bc ab a c a b acbc a c bc a b acbc a c bc ⇔ + - +-+ -+≥ ⇔ +- +- +- +≥ ⇔ - + + + - +≥ - ++ ⇔≥ + ++ ⇒ ∑∑ ∑∑ ∑ ∑ Ñaúng thöùc xaûy ra khi vaø chæ khi . abc == Tìm tài liệu Toán ? Chuyện nh ỏ - www.toanmath.com347 Baøi toaùn 77. Cho ,,0 vaø abck > laø haèng soá döông cho tröôùc. Tìm giaù trò nhoû nhaát cuûa bieåu thöùc ( ,,) ( )( )() b c c a a b kabc f abc a b c abbcca + ++ =++ + + ++ Lôøi giaûi. Ñaët 2 22 22 2 ,6 x ab bc ca ac cb ba y abc = + + + + += . Khi ñoù, ta coù 6 (,,)6 6 2 62 xkyk f abc t yxyt = + =+ ++ trong ñoù 1 x t y =≥ . Ñaët ()6 62 k gtt t =+ + vôùi 1 t ≥ , ta caàn tìm giaù trò nhoû nhaát cuûa () gt . Coù 2 tröôøng hôïp xaûy ra * Tröôøng hôïp 1. 64 k ≤ . Khi ñoù, ta coù 3( 1)(48 16) ()60 8 8(3 1) ()6 8 (, ,) 6 8 k t tk gt t k gt k f abc - -+ --=≥ + ⇒ ≥+ ⇒ ≥+ Ñaúng thöùc xaûy ra khi vaø chæ khi1. t abc = ⇔ == Vaäy trong tröôøng hôïp naøy, ta coù min (, ,) 6 8 k f abc =+ * Tröôøng hôïp 2. 64 k ≥ . Khi ñoù, ta coù ( )( ) / 2 66 2 62 () (6 2) t ktk gt t +- ++ = + Tìm tài liệu Toán ? Chuyện nh ỏ - www.toanmath.com348 / 0 2 ()0 6 k gt tt - = ⇔ == Qua 0 t thì / () gt ñoåi daáu töø aâm sang döông neân ta coù 0 () ()2 21 (, ,) 2 2 gt gt k t fabck ≥ = - ∀≥ ⇒ ≥- Ñaúng thöùc xaûy ra khi vaø chæ khi ( ) 2 22 22 2 2 2 6 k t ab bc ca ac cb ba k abc - = ⇔ ++ + ++ = - (*) Ta chæ caàn chöùng minh raèng toàn taïi boä soá döông( ,,) abc thoûa maõn heä thöùc (*) laø baøi toaùn ñöôïc giaûi quyeát hoaøn toaøn. Thaät vaäy, cho 1 bc == thì heä thöùc (*) trôû thaønh 2 () 2 10 2 k faaa ⎛⎞ = - - += ⎜⎟ ⎝⎠ Ta coù 0 64 lim ( ) 1 0, (1) 4 4 0 22 a k faf + → = > =- ≤- = . Do ñoù, toàn taïi (0,1] a∈ sao cho ()0 fa = . Vaäy toàn taïi boä soá( ,,) abc thoûa maõn heä thöùc (*). Do ñoù, trong tröôøng hôïp naøy, ta coù min(,,)22 f abc k =- Keát luaän 64 min ( , , ) 6 8 64 min ( , , ) 2 2 Neáu thì Neáu thì k k fabc k fabc k + ≤ =+ + ≥ =- Tìm tài liệu Toán ? Chuyện nh ỏ - www.toanmath.com349 Baøi toaùn 78. (Voõ Quoác Baù Caån) Cho,,01 thoûa . Chöùng minh raèng a b c abc >= 3 33 1 8 1 8 181 a bc c ab ++≥ +++ Lôøi giaûi. Ta coù nhaän xeùt sau 2 3 11 0 (*) 21 81 x x x ≥ ∀> + + Thaät vaäy 23 (*) 2 1 81 xx ⇔ +≥+ 2 23 22 (2 1) 81 4 ( 1)0 (ñuùng) xx xx ⇔ + ≥+ ⇔ -≥ Vaäy (*) ñuùng. Do ñoù 2 22 3 33 2 12 12 1 8 18 18 1 a b c a bc cab c ab + + ≥++ + ++ + ++ Nhö vaäy, ñeå chöùng minh baát ñaúng thöùc ñaõ cho, ta chæ caàn chöùng minh 2 22 22 22 2 3 2 3 2 2 22 32 2 22 3 2 32 2 1 2 12 12 1 (2 1)(2 1) (2 1)(2 1)(2 1) 4 2 2 924 2 2 2 29 2( cyc cyc cyc cyc cyc cyc cyc cyc cyc cyc cyc cyc cyc cyc a bc cab aa b abc abaab a a ab abab ab a a aba ab ++≥ + ++ ⇔ ++≥ ++ + ⇔ + + + ≥++ ⎛ ⎞⎛⎞ ⇔ + - + - + +≥ ⎜ ⎟⎜⎟ ⎜ ⎟⎜⎟ ⎝ ⎠⎝⎠ ⇔ ∑ ∑ ∑ ∑ ∑ ∑∑ ∑∑ ∑ ∑ ∑ ∑∑ 2 3 2 32 1) 2 2 9 (**) cyc cyc cyc cyc cyc a a a aba ⎛⎞ - + - + +≥ ⎜⎟ ⎜⎟ ⎝⎠ ∑ ∑ ∑ ∑∑ Laïi coù theo baát ñaúng thöùc AM-GM thì Tìm tài liệu Toán ? Chuyện nh ỏ - www.toanmath.com350 3 3 2 5 55 3 2 6 39 cyc cyc a b a a b c abc + ≥ += ∑∑ Do ñoù, ñeå chöùng minh baát ñaúng thöùc (**), ta chæ caàn chöùng minh 32 0 (***) cyc cyc aa -≥ ∑∑ Nhöng ñieàu naøy hieån nhieân ñuùng vì 32 32 3 32 2 1 .3 3. 3 1 .3 .( 3 1 . ( )() 3 0 theo bñt AM-GM) cyc cyc cyc cyc cyc cyc cyc cyc a a a abc a a aa a b ab ⎛⎞ - =- ⎜⎟ ⎜⎟ ⎝⎠ ⎛⎞ ⎛⎞ ≥-⎜⎟ ⎜⎟ ⎜⎟ ⎜⎟ ⎝⎠ ⎝⎠ = -+ ≥ ∑∑ ∑∑ ∑ ∑∑ ∑ Vaäy (***) ñuùng. Töø ñaây, ta suy ra ñpcm. Ñaúng thöùc xaûy ra khi vaø chæ khi 1. abc === * Ghi chuù. Ngoaøi ra, ta coøn coù moät caùch khaùc ñeå chöùng minh baât ñaúng thöùc 2 22 1 2 12 12 1 a bc cab ++≥ + ++ Cuï theå nhö sau Do,, 0,1 a b c abc >= neân toàn taïi caùc soá thöïc döông ,, xyz sao cho ,, xy ab yz == z c x = . Khi ñoù, baát ñaúng thöùc treân trôû thaønh 3 33 2 2 2 2 22 1 (2 ) (2 ) (2 ) xyz yz x z x y x yz + +≥ + ++ AÙp duïng baát ñaúng thöùc Bunhiacopxki, ta coù Tìm tài liệu Toán ? Chuyện nh ỏ - www.toanmath.com351 3 3 3 4 44 22 22 22 222222 2 2 22 3 33 (2 ) (2 ) (2 ) (2 ) (2 ) (2 ) () 2() x y z x yz yzx z x y x y z xyzx yzx y zx yz x yz xy yz zx xyzx y z + + = ++ + + + + ++ ++ ≥ + + + ++ Do ñoù, ñeå chöùng minh baát ñaúng thöùc ñaõ cho, ta chæ caàn chöùng minh 2 2 22 3 33 4 3 222 22 2 22 () 1 2() 20 1 . ()(3 2 ) () 0 4 (ñuùng) cyc cyc cyc cyc cyc cyc x yz xy yz zx xyzx y z x xy xy xyz xy x xy y z xy ++ ≥ + + + ++ ⎛ ⎞⎛⎞ ⇔ - + -≥ ⎜ ⎟⎜⎟ ⎜ ⎟⎜⎟ ⎝ ⎠⎝⎠ ⇔ - + ++ -≥ ∑ ∑ ∑∑ ∑∑ ñpcm. ⇒ Baøi toaùn 79. (Ñinh Ngoïc An) Cho ,,0 thoûa abc ≥ a b c ab bc ca + + = ++ . Tìm giaù trò nhoû nhaát vaø giaù trò lôùn nhaát cuûa bieåu thöùc 111 S ab bc ca = ++ + ++ Lôøi giaûi. Ta seõ chöùng minh raèng 3 max min 2 vaø khoâng coù . SS = Thaät vaäy, töø giaû thieát, ta coù 1111 . 1. ab bc ca ab bc ca S a bc ab b c c a abc ab bc c a ++⎛ ⎞ ⎛⎞ = ++ =+ ++ ⎜ ⎟ ⎜⎟ ++ + ++ ++ + ++ ⎝ ⎠ ⎝⎠ Do ñoù 1 S > . Cho 0, ( 1) 1 a cba a ==> - thì ta cuõng coù a b c ab bc ca + + = ++ . Khi ñoù, ta coù 1 1 1 lim 1. a a S a S + → - =+ ⇒= Tìm tài liệu Toán ? Chuyện nh ỏ - www.toanmath.com352 Vaäy khoâng toàn taïimin. S Maët khaùc, aùp duïng baát ñaúng thöùc AM-GM, ta coù 2 22 1 1. 1 11 .( ) .( ) .( ) 1 4 44 1. 3 2 ab bc ca S abc ab bc c a ab bc ca a bc ab bc c a ⎛⎞ =+ ++ ⎜⎟ ++ + ++ ⎝⎠ ⎛⎞ + ++ ⎜⎟ ≤+ ++ ⎜⎟ ++ + ++ ⎜⎟ ⎝⎠ = Ñaúng thöùc xaûy ra chaúng haïn khi 1. abc === Vaäy 3 max. 2 S = Baøi toaùn 80. (Ñinh Ngoïc An) Cho,,,0 abcd ≥ thoûa 1 ab bc cd da + + += . Tìm giaù trò nhoû nhaát cuûa bieåu thöùc 2 2 22 (,,,)2 f abcd a b c d abcd = + + ++ Lôøi giaûi. Ta coù 1 ( )( )1 ab bc cd da a c b d ++ + = ⇔+ + = Neáu 1 ac ≥ thì deã thaáy (,,, )2 f abcd ≥ . Neáu 1 ac ≤ . Khi ñoù, ta coù 2 ( )(1) (, ,, ) , , , 0 2 22 (, ,, ) , ,, 22 b d bd bd ac f abcd f a c bd bd f abcd f a c + + -- ⎛⎞ - =≥ ⎜⎟ ⎝⎠ ++ ⎛⎞ ⇒≥ ⎜⎟ ⎝⎠ Baây giôø, neáu 2 () 1 4 bd + ≥ thì deã thaáy , ,,2 22 bd bd fac ++ ⎛⎞ ≥ ⎜⎟ ⎝⎠ . Do ñoù (,,, )2 f abcd ≥ . Neáu 2 () 1 4 bd + ≤ thì baèng laäp luaän töông töï nhö treân, ta coù Tìm tài liệu Toán ? Chuyện nh ỏ - www.toanmath.com353 , ,, , ,, 2 2 2222 bd bd acbdacb d fa cf + + ++ ++ ⎛ ⎞⎛⎞ ≥ ⎜ ⎟⎜⎟ ⎝ ⎠⎝⎠ Do ñoù 2 2 22 (, ,, ) , ,, 22 , ,, 2222 ( ) () ( )() 2 28 1 1 8 9 8 b d bd f abcd f a c acbd acbd f a c bd ac bd ++ ⎛⎞ ≥ ⎜⎟ ⎝⎠ ++ ++ ⎛⎞ ≥ ⎜⎟ ⎝⎠ + + ++ = ++ ≥+ = Toùm laïi, trong moïi tröôøng hôïp, ta luoân coù 9 (,,,). 8 f abcd ≥ Ñaúng thöùc xaûy ra khi vaø chæ khi 1 . 2 abcd = = == Vaäy 9 min ( , , ,) 8 f abcd = . Baøi toaùn 81. (Ñinh Ngoïc An) Cho ,,,0 abcd ≥ thoûa 1 ab bc cd da ac bd + + + + += . Tìm giaù trò nhoû nhaát cuûa bieåûu thöùc 2 2 22 (,,,)2 f abcd a b c d abcd = + + ++ Lôøi giaûi. Xeùt soá thöïc khoâng aâm y thoûa maõn 22 2( ) 1 ( )( 2) y ab y a b a b c d y y cd + + + =⇔ + + - =- (*) (chuù yù raèng y luoân luoân toàn taïi) Khi ñoù, ta phaûi coù 2 cd y + ≤ . Thaät vaäy, giaû söû ngöôïc laïi 2 cd y + > . Khi ñoù, ta coù Tìm tài liệu Toán ? Chuyện nh ỏ - www.toanmath.com354 2 2 2( ) ( )() 2 ( )() 1 cd y ab y a b ab a b c d cd ab a b c d ab bc cd da ac bd + ⎛⎞ + + +> + ++ + ⎜⎟ ⎝⎠ ≥ + + ++ = + + + ++ = Ñieàu naøy maâu thuaãn vì 2 2 ( )1 y ab yab + + += . Vaäy, ta phaûi coù 2 cd y + ≤ . Do ñoù, töø (*), ta suy ra ñöôïc 2 . y cd ≥ Khi ñoù, ta coù 22 ( ,, , ) ( ,, , ) ( ) 2(1 )( ) 0 ( , , , ) ( , , , ) f abcd f ab y y c d ab y cd f abcd f ab y y - = - + - -≥ ⇒≥ Baèng laäp luaän töông töï, ta daãn ñeán 2 2 22 (,,,)(,,,)2() f abcd f xx y y x y x y ≥ = ++ Vôùi,0 xy ≥ thoûa maõn 22 4 1. x y xy + += Neân 2 2 22 13 (, ,, ) ( , , , ) 2( ) 18 f abcd f xx y y x y x y ≥ = ++≥ Ñaúng thöùc xaûy ra khi vaø chæ khi 1 . 6 abcd = = == Vaäy 13 min(,,,). 18 f abcd = Baøi toaùn 82. Cho ,,0 xyz > thoûa 4 xy yz zx xyz + + += . Tìm haèng soá k toát nhaát cho baát ñaúng thöùc 2 22 3(1)() x y z k k x yz ++ +≥ + + + Lôøi giaûi. Cho 2, 21 xyz = = =- , ta suy ra ñöôïc 2 21 k≤+ . Ta seõ chöùng minh ñaây laø giaù trò caàn tìm, töùc laø chöùng minh Tìm tài liệu Toán ? Chuyện nh ỏ - www.toanmath.com355 ( ) ( ) 2 22 32 21 2 21( ) x y z x yz + + + + ≥ + ++ (*) Töø giaû thieát 4 xy yz zx xyz + + += , ta suy ra ñöôïc 111 1 2 22 x yz + += + ++ Do ñoù, ta coù theå ñaët 1 1 11 , , ,, 1 2 22 0 ,, 2 m np m n p mnp x yz mnp + += ⎧ ⎪ = = =⇒⇒ ⎨ + ++ << ⎪ ⎩ laø ñoä daøi ba caïnh cuûa moät tam giaùc. Do ñoù, toàn taïi caùc soá thöïc döông ,, abc sao cho , ,. m bcn c ap ab = + = + =+ Khi ñoù, töø caùch ñaët, ta coù 122 122 122 mnpma x m m bc n pmnb y n n ca pmnpc z p p ab - +- = == + - +- = == + - +- = == + Baát ñaúng thöùc (*) trôû thaønh ( ) ( ) ( ) ( ) ( ) ( ) 2 2 2 2 22 2 42 32 21 2 21. () 4 21 4 2210 () 44 21() 221() 0 () cyc cyc cyc cyc aa bc bc a a bc bc a ab c bc bc + +≥+ + + ⎛⎞ + ⎜⎟ ⇔ - + +≥ ⎜⎟ + + ⎝⎠ - + + + ++ ⇔≥ + ∑∑ ∑ ∑ ( ) ( ) ( ) ( ) ( ) ( ) 2 2 2 2 21 2 21 ( ) () 2 2 21 2 21 ( ) 0 () cyc cyc a b c ab bc a b c ca bc -+ -+ - ⇔- + - + - +- -≥ + ∑ ∑ Tìm tài liệu Toán ? Chuyện nh ỏ - www.toanmath.com356 ( ) ( ) ( ) ( ) ( ) ( ) ( ) ( ) ( ) ( ) 2 2 222222 2 2 21 2 21 ( ) () 2 2 21 2 21 ( ) 0 () ( ) 2 2 1 22 1 2 2 3 22 ( ) 0 cyc cyc cyc a b c ab bc b c aab ac a b a b c ab cab -+ -+ - ⇔- + -+ -+ - -≥ + ⇔ - + +- +- + - + ≥ ∑ ∑ ∑ Ñaët ( ) ( ) ( ) ( ) ( ) ( ) ( ) ( ) ( ) 2 2 2 222 222 222 2 2 1 22 1 22 3 22 ( ) 2 2 1 22 1 22 3 22 ( ) 2 2 1 22 1 22 3 22 ( ) a b c Sbcabc abc S c a b ca bca S a b c ab cab = + +- +- + - + =++- +- +-+ = + +- +- + - + Khi ñoù, baát ñaúng thöùc caàn chöùng minh trôû thaønh 2 22 2 22 2 2 2 2 22 ( ) ( ) ( )0 a bc S b c S ca S ab - + - + -≥ Khoâng maát tính toång quaùt, ta coù theå giaû söû 0. abc ≥≥> Khi ñoù, ta coù ( ) ( ) ( ) ( ) ( ) ( ) ( ) ( ) 2 222 222 2 22 2 2 1 2 2 1 22 3 22 ( ) 2 2 1 22 1 22 3 22 ( ) 2 21 4 21 2 b S c a b ca bca c b b cb bcb b bcc =++- +- +-+ ≥ + +- +- + - + = - - -+ ( ) ( ) 2 2210 bc = - -≥ ( ) ( ) ( ) ( ) ( ) ( ) ( ) ( ) ( ) ( ) ( ) ( ) ( ) ( ) ( ) 2 222 2 2 2 22 22 2 2 2 2 1 22 1 22 3 22 ( ) 4 1 22 1 22 3 22 ( ) 5 22 1 2 2 3 22 ( ) 5 22 1 22 23 22 6 4 2 23 22 4 21 0 c S a b c ab cab ab c ab c a b ab c cab c cc cc c = + +- +- + - + ≥ +- +- + - + =- +- +-+ ≥ - + - +- = - +- =- ≥ Tìm tài liệu Toán ? Chuyện nh ỏ - www.toanmath.com357 ( ) ( ) ( ) 22 2 222 21 ( ) 2 21 4 21( ) 4 ab S S a b ab cabc + = - + + - - - ++ ( ) ( ) ( ) ( ) 2 22 2 21() 4 21()4 2 1( )2 0 ab cab c abc = - + - - ++ = - +- ≥ Do ñoù 2 2 2 22 2 2 2 2 2 2 2 22 2 22 ( ) ( ) ( )( )( )0 (*) ñuùng. a b c ab S b c S c a S ab S S bc - + -+ -≥ + - ≥ ⇒ V aäy max 2 21 k =+ . Baøi toaùn 83. (Voõ Quoác Baù Caån) Cho tam giaùc ABC . Chöùng minh raèng 1 sin 1 cos 1 sin 1 cos 1 sin 1 cos 22 22 2 2 23 sin . 1 sin sin . 1 sin sin . 1 sin 2 2 2 2 22 AA BB C C A AB BC C ⎛ ⎞⎛ ⎞ ⎛ ⎞⎛ ⎞ ⎛ ⎞⎛ ⎞ - +- +- + ⎜ ⎟⎜ ⎟ ⎜ ⎟⎜ ⎟ ⎜ ⎟⎜ ⎟ ⎝ ⎠⎝ ⎠ ⎝ ⎠⎝ ⎠ ⎝ ⎠⎝ ⎠ + + ≥+ ⎛⎞ ⎛⎞ ⎛⎞ +++ ⎜⎟ ⎜⎟ ⎜⎟ ⎝⎠ ⎝⎠ ⎝⎠ Lôøi giaûi. AÙp duïng baát ñaúng thöùc Bunhiacopxki, ta coù 2 1sin1 1sin1 22 22 sin 1 sin sin 1 sin 1 1 sin 2 2 2 2 22 cos cos cos cyc cyc cyc AA AA A A A A AA ⎛⎞ ⎛ ⎞⎛⎞ ⎛ ⎞⎛⎞ -+ ⎜⎟ -+ ⎜ ⎟⎜⎟ ⎜ ⎟⎜⎟ ⎝ ⎠⎝⎠ ⎝⎠⎝⎠⎝⎠ ≥ ⎛⎞ ⎛ ⎞⎛ ⎞⎛⎞ + - ++ ⎜⎟ ⎜ ⎟⎜ ⎟⎜⎟ ⎝⎠ ⎝ ⎠⎝ ⎠⎝⎠ ∑ ∑ ∑ AÙp duïng baát ñaúng thöùc AM-GM, ta coù 2 sin 1 sin 1 22 sin 1 sin 2 2 24 AA AA ⎛⎞ +- ⎜⎟ ⎛⎞ -≤= ⎜⎟ ⎜⎟ ⎝⎠ ⎜⎟ ⎝⎠ Töông töï, ta coù Tìm tài liệu Toán ? Chuyện nh ỏ - www.toanmath.com358 1 sin 1 sin 2 24 1 sin 1 sin 2 24 BB CC ⎛⎞ -≤ ⎜⎟ ⎝⎠ ⎛⎞ -≤ ⎜⎟ ⎝⎠ sin 1 sin 1 1 sin 2 2 22 1 . 1 1 sin 4 22 11 . 3 sin cos . sin 4 2 22 cos cos cyc cyc cyc cyc cyc A A AA AA AA A ⎛ ⎞⎛ ⎞⎛ ⎞ ⇒ - + +≤ ⎜ ⎟⎜ ⎟⎜ ⎟ ⎝ ⎠⎝ ⎠⎝ ⎠ ⎛ ⎞⎛⎞ ≤ ++ ⎜ ⎟⎜⎟ ⎝ ⎠⎝⎠ ⎛⎞ = + ++ ⎜⎟ ⎝⎠ ∑ ∑ ∑ ∑∑ 3 33 33 sin , cos , sin 22 222 Chuù yù raèng cyc cyc cyc AA A ≤ ≤≤ ∑ ∑∑ ( ) 1 3 33 33 sin 1 sin 1 1 sin . 3 2 2 2 2 4 2 24 923 16 cos cyc A A AA ⎛⎞ ⎛ ⎞⎛ ⎞⎛ ⎞ ⇒ - + + ≤ +++ ⎜⎟ ⎜ ⎟⎜ ⎟⎜ ⎟ ⎝ ⎠⎝ ⎠⎝ ⎠ ⎝⎠ + = ∑ ( ) 1 16 923 sin 1 sin 1 1 sin 2 2 22 cos cyc A A AA ⇒≥ ⎛ ⎞⎛ ⎞⎛ ⎞ + - ++ ⎜ ⎟⎜ ⎟⎜ ⎟ ⎝ ⎠⎝ ⎠⎝ ⎠ ∑ ( ) 2 16 1 sin 1 1 sin 1 22 22 923 sin 1 sin 22 cos cos cyc cyc AA AA AA ⎛⎞ ⎛ ⎞⎛⎞ ⎛ ⎞⎛⎞ -+ ⎜⎟ -+ ⎜ ⎟⎜⎟ ⎜ ⎟⎜⎟ ⎝ ⎠⎝⎠ ⎝⎠⎝⎠⎝⎠ ⇒≥ ⎛⎞ + + ⎜⎟ ⎝⎠ ∑ ∑ Do ñoù, ñeå chöùng minh baát ñaúng thöùc ñaõ cho, ta chæ caàn chöùng minh ( ) 2 16 1 sin 1 22 23 923 cos cyc AA ⎛⎞ ⎛ ⎞⎛⎞ -+ ⎜⎟ ⎜ ⎟⎜⎟ ⎝ ⎠⎝⎠ ⎝⎠ ≥+ + ∑ Tìm tài liệu Toán ? Chuyện nh ỏ - www.toanmath.com359 3 33 1 sin 1 2 2 24 1 3 33 sin .sin 2 2 2 24 cos cos cyc cyc cyc AA AA A ⎛ ⎞⎛⎞ ⇔ - + ≥+ ⎜ ⎟⎜⎟ ⎝ ⎠⎝⎠ ⎛⎞ ⇔- + - ≥-+ ⎜⎟ ⎝⎠ ∑ ∑∑ Do 3 sin 22 neân cyc A ≤ ∑ 1 31 sin .sin .sin 2 2 2 2 22 cos cos cyc cyc cyc AAA AA ⎛⎞ ⎛⎞ - +- ≥-+- ⎜⎟ ⎜⎟ ⎝⎠ ⎝⎠ ∑∑∑ Do ñoù, ta chæ caàn chöùng minh 1 33 .sin 224 cos cyc A A ⎛⎞ -≥ ⎜⎟ ⎝⎠ ∑ Xeùt haøm soá 13 ( ) cos .sin 2 0; 2 642 vôùi fxx xxx ⎛⎞ = - +--∈ ⎜⎟ ⎝⎠ ππ Ta coù // 0 ( ) sin 1 2 sin .(2sin 1), ( ) 0 6 cos fx x x x x fx x x =- +- = - = ⇔ == π / 0 () Qua thì ñoåi daáu töø aâm sang döông neân x fx ( ) 0 0, 62 fxfx ππ ⎛⎞ ⎛⎞ ≥ = ∀∈ ⎜⎟ ⎜⎟ ⎝⎠ ⎝⎠ , , 0, 222 2 Do neân ABC π ⎛⎞ ∈ ⎜⎟ ⎝⎠ 0 2 A f ⎛⎞ ≥ ⎜⎟ ⎝⎠ 13 .sin 0 2 2 2 64 cos AA A π ⇒ - + - -≥ 13 .sin 2 2 6 24 cos AA A π⎛⎞ ⇒ - ≥ -+ ⎜⎟ ⎝⎠ Töông töï, ta coù Tìm tài liệu Toán ? Chuyện nh ỏ - www.toanmath.com360 13 .sin 2 2 6 24 13 .sin 2 2 6 24 cos cos BB B CC C ⎛⎞ - ≥-+ ⎜⎟ ⎝⎠ ⎛⎞ - ≥-+ ⎜⎟ ⎝⎠ π π Do ñoù 1 3 33 .sin 22 62 44 cos cyc cyc AA A ⎛⎞ ⎛ ⎞ ⎛⎞ - ≥ - += ⎜⎟ ⎜ ⎟ ⎜⎟ ⎝ ⎠ ⎝⎠ ⎝⎠ ∑∑ π ⇒ ñpcm. Ñaúng thöùc xaûy ra khi vaø chæ khi . 3 ABC π = == Baøi toaùn 84. Cho,,01 thoûa . Chöùng minh raèng abc abc > + += 2 22 2 2 22 11 18( ) 222 (1 )( 1 )(1 ) abc a b c abc ++ ⎛ ⎞⎛ ⎞⎛ ⎞ -+ -+ - ≥ ⎜ ⎟⎜ ⎟⎜ ⎟ --- ⎝ ⎠⎝ ⎠⎝ ⎠ Lôøi giaûi. Ñaët 2 22 x abc = ++ . Khi ñoù, deã thaáy 1 1. 3 x ≤< Do ñoù 2 ( 1)(3 1) 0 4 13 xx xx - -≤ ⇒ -≥ Ta laïi coù 2 2 2 2 22 () a b b c c a abc a b c abc + + ≥ + += Do ñoù 2222222 (4 1)( )3 x a b b c c a abcx - + +≥ Maët khaùc, ta laïi coù 222 41()()() x bc a c ab ab c -=+ - + + - ++ - AÙp duïng baát ñaúng thöùc Chebyshev, ta coù 222222222 3(( ) () () ) bca bc c a b ca ab c ab +-++-++- ≥ Tìm tài liệu Toán ? Chuyện nh ỏ - www.toanmath.com361 222222222 (( ) ( ) ( ) )( ) bc a c ab abc bc ca ab ≥ +- ++- + +- ++ Do ñoù, ta coù 222222222 222 222 222 222 2 22 (( ) () () ) ( ) ((12 ) (12) (12) ) ( ) b c a bc c a b ca a b c ab abca b c a b c b c a c a b abc a b c +-++-++- ≥ ++ ⇔ - +- +- ≥ + + AÙp duïng baát ñaúng thöùc AM-GM, ta coù ( )( )( ) 8 (1 )( 1 )(1 ) 8 a b b c c a abc a b c abc + + +≥ ⇔ - - -≥ Do ñoù 2 2 2 222 2 2 2 222 2 22 2 22 2 2 22 ((1 2 ) (1 2 ) (1 2 ) )(1 )(1 )(1 ) 8 () 11 18( ) 2 22 ( 1 )(1 )(1 ) abc bca cab abc abc a b c abc a b c abc - +- +- - - - ≥ ≥ ++ ++ ⎛ ⎞⎛ ⎞⎛ ⎞ ⇔ -+ -+ - ≥ ⎜ ⎟⎜ ⎟⎜ ⎟ - -- ⎝ ⎠⎝ ⎠⎝ ⎠ ñpcm. ⇒ Ñaúng thöùc xaûy ra khi vaø chæ khi 1 . 3 abc = == Baøi toaùn 85. (Voõ Quoác Baù Caån) Chöùng minh raèng vôùi moïi soá döông ,, abc ta ñeàu coù 2 22 333 24() 3( ) 36 () cyc abc a b c ab bc ca ab a b abc ++ ++- +≥++++ ++ ∑ Lôøi giaûi. Tröôùc heát xin ñöôïc nhaéc laïi khoâng chöùng minh keát quaû quen thuoäc sau Xeùt ba daõy ( ),( ),( ) nnn abc ñöôïc xaùc ñònh bôûi 0 00 22 21 2 2 1 21 21 21 22 2 1 2 2 22 ,, ,, 2 ,, 2 nn n n nn nn nn nn a ab bc c ab ccab nN ac a b b c nN + ++ ++ ++ ++ = == + = = = ∀∈ + = = = ∀∈ Khi ñoù, ta coù Tìm tài liệu Toán ? Chuyện nh ỏ - www.toanmath.com362 lim lim lim 3 nnn n nn abc abct →∞ →∞ →∞ ++ === = Trôû laïi baøi toaùn cuûa ta Ñaët 2 22 3 33 24( ) ( , , ) 3( ) ( ) 36 cyc abc f a b c a b c ab bc ca ab a b abc ++ = + + - - - - - ++ ++ ∑ Ta seõ chöùng minh ( , ,) min , , , (0, ,) 22 abab f abc f c f a bc ⎧⎫ ++ ⎛⎞ ≥+ ⎨⎬ ⎜⎟ ⎝⎠ ⎩⎭ Thaät vaäy, giaû söû ngöôïc laïi ( , ,) min , , , (0, ,) 22 abab f abc f c f a bc ⎧⎫ ++ ⎛⎞ <+ ⎨⎬ ⎜⎟ ⎝⎠ ⎩⎭ . Khi ñoù, ta coù (,,) ,, 22 ( , ,) (0, ,) abab f abc f c f abc f a bc ⎧ ++ ⎛⎞ < ⎪ ⎜⎟ ⎝⎠ ⎨ ⎪ <+ ⎩ Ta coù 2 (,,) ,, 22 9( ) 12 1 ()0 4 42 abab f abc f c ab abc ab abc ++ ⎛⎞ < ⎜⎟ ⎝⎠ + ++ ⎛⎞ ⇔ - - + -< ⎜⎟ ++ ⎝⎠ 48 10 10 2 1 0 (1) a bc abc ⇒+-+-< ++ ( , ,) (0, ,) 48 10 10 2 1 0 48 10 10 2 1 0 (2) f abc f a bc ab a bc abc a bc abc <+ ⎛⎞ ⇔--+-+< ⎜⎟ ++ ⎝⎠ ⇔+-+-> ++ Töø (1) vaø (2), ta suy ra maâu thuaãn. Vaäy ta phaûi coù ( , ,) min , , , (0, ,) 22 abab f abc f c f a bc ⎧⎫ ++ ⎛⎞ ≥+ ⎨⎬ ⎜⎟ ⎝⎠ ⎩⎭ Tìm tài liệu Toán ? Chuyện nh ỏ - www.toanmath.com363 Tieáp theo, ta seõ chöùng minh min , , , (0, , ) 0 22 abab f c f a bc ⎧⎫ ++ ⎛⎞ +≥ ⎨⎬ ⎜⎟ ⎝⎠ ⎩⎭ Tröôùc heát, ta chöùng minh (0, , ) 0 (3) f a bc +≥ Baèng laäp luaän töông töï nhö treân, ta coù (0, , ) min 0, , , (0,0, ) 22 ab cabc f abc f f abc ⎧⎫ ++ ++ ⎛⎞ + ≥ ++ ⎨⎬ ⎜⎟ ⎝⎠ ⎩⎭ Ta laïi coù 32 3 0, , 4 24 36 0 2 2 2 22 (0,0, ) 6(( ) 4( ) 6) 0 ab cabc abc abc abc f f abc abc abc ++ ++ ++ ++ ++ ⎛ ⎞⎛⎞⎛⎞ ⎛⎞ = - - +≥ ⎜ ⎟⎜⎟⎜⎟ ⎜⎟ ⎝ ⎠⎝⎠⎝⎠ ⎝⎠ ++ = ++ - ++ + ≥ Do ñoù (0, , ) 0 f a bc +≥ Vaäy (3) ñuùng. Baây giôø, ta seõ chöùng minh , , 0 (4) 22 abab fc ++ ⎛⎞ ≥ ⎜⎟ ⎝⎠ Töø (3) vaø keát quaû treân, ta suy ra ñöôïc 111 222 , , (, , ) 22 min{0, ( , , )} ... min{0, ( , , )} nnn a bab f c fabc fa bc fabc nN ++ ⎛⎞ = ⎜⎟ ⎝⎠ ≥ ≥ ≥ ∀∈ Do ( ,,) f abc lieân tuïc neân { } , , min 0, lim ( , , ) min{0, ( , , )} 22 nnn n a bab f c f a b c f ttt →∞ ++ ⎛⎞ ≥= ⎜⎟ ⎝⎠ Tìm tài liệu Toán ? Chuyện nh ỏ - www.toanmath.com364 Trong ñoù . 3 abc t ++ = Ta laïi coù 322 ( , , ) 3 3 24 36 3( 2) ( 3) 0 f ttt t t t t t = - - + = - +≥ Do ñoù , ,0 22 abab fc ++ ⎛⎞ ≥ ⎜⎟ ⎝⎠ Vaäy (4) ñuùng. Töø ñaây, ta suy ra ñpcm. Ñaúng thöùc xaûy ra khi vaø chæ khi 2. abc = == Baøi toaùn 86. Cho , , 0. Chöùng minh raèng abc > 2 222 23 a b c acb bc ca ab c b a ⎛⎞ ⎛⎞ + + +≥ ++ ⎜⎟ ⎜⎟ ⎝⎠⎝⎠ Lôøi giaûi. * Caùch 1. AÙp duïng baát ñaúng thöùc Bunhiacopxki, ta coù 2 3 a c b acb c b a cba ⎛⎞ ⎛⎞ + + ≤ ++ ⎜⎟ ⎜⎟ ⎝⎠ ⎝⎠ Do ñoù, ñeå chöùng minh baát ñaúng thöùc ñaõ cho, ta chæ caàn chöùng minh 2 22 3 3 3 2 22 2 33 2( )33() a b c acb bc ca ab c b a a b c abc a b b c c a ⎛⎞ ⎛⎞ + + + ≥ ++ ⎜⎟ ⎜⎟ ⎝⎠ ⎝⎠ ⇔ + + + ≥ ++ AÙp duïng baát ñaúng thöùc AM-GM, ta coù 3 22 3 22 2 22 2 2 2 a ab ab b bc bc c ca ca +≥ +≥ +≥ Tìm tài liệu Toán ? Chuyện nh ỏ - www.toanmath.com365 3 3 3 2 2 2 2 22 2( ) (1) a b c ab bc ca a b b c c a ⇒ + + + + + ≥ ++ Maët khaùc, theo baát ñaúng thöùc Schur thì 3 3 3 2 2 2 222 3 (2) a b c abc a b b c c a ab bc ca + + + ≥ + + + ++ Töø (1) vaø (2), ta suy ra ñöôïc 3 3 3 222 2 2 2 222 2( ) 33() a b c ab bc ca abc a b b c c a ab bc ca + + + ++ + ≥ + + + ++ 3 3 3 2 22 2( )33() ñpcm. a b c abc a b b c c a ⇔ + + + ≥ ++ ⇒ Ñaúng thöùc xaûy ra khi vaø chæ khi . abc == * Caùch 2. AÙp duïng baát ñaúng thöùc AM-GM, ta coù 22 22 22 2. 36 2. 36 2. 36 aca bc ab b bab ca bc c cbc ab ca a + +≥ + +≥ + +≥ 2 22 3 96 a b c abc bc ca ab b c a ⎛⎞ ⎛⎞ ⇒ + + +≥ ++ ⎜⎟ ⎜⎟ ⎝⎠⎝⎠ 2 22 3 2 (3) a b c abc bc ca ab b c a ⎛⎞ ⎛⎞ ⇔ + + +≥ ++ ⎜⎟ ⎜⎟ ⎝⎠⎝⎠ Laïi aùp duïng baát ñaúng thöùc AM-GM, ta coù 22 22 22 2. 3. 2. 3. 2. 3. a ba bc ca c b cb ca ab a c ac ab bc b +≥ +≥ +≥ Tìm tài liệu Toán ? Chuyện nh ỏ - www.toanmath.com366 222 222 33 (4) a b c acb bccaab cb a a b c acb bc ca ab c b a ⎛⎞ ⎛⎞ ⇒ + + ≥ ++ ⎜⎟⎜⎟ ⎝⎠ ⎝⎠ ⇔ + + ≥ ++ Töø (3) vaø (4), ta suy ra ñöôïc 222 2 2 22 2 32 2 3 () ñpcm a b c acb a b c bccaab cb a b c a a b c acb bc ca ab c b a ⎛⎞ ⎛⎞ + + +≥ + + + ++ ⎜⎟ ⎜⎟ ⎝⎠ ⎝⎠ ⎛⎞ ⎛⎞ ⇔ + + +≥ ++ ⎜⎟ ⎜⎟ ⎝⎠⎝⎠ Ñaúng thöùc xaûy ra khi vaø chæ khi . abc == Baøi toaùn 87. (Phaïm Kim Huøng) Cho ,, abc laø ñoä daøi ba caïnh cuûa moät tam giaùc. Chöùng minh raèng 2 22 22 2 2 abc a cb abc b ca cb a ⎛⎞ + + ≥ + ++ ++ ⎜⎟ ⎝⎠ Lôøi giaûi. Ta coù 2 Boå ñeà sau Boå ñeà 1. (IMO 1983) ,, abc laø ñoä daøi ba caïnh cuûa moät tam giaùc. Khi ñoù, ta coù 222 a b c ab bc ca b ca c ab + +≥ ++ Chöùng minh. Ta coù ,, abc laø ñoä daøi ba caïnh cuûa moät tam giaùc neân toàn taïi caùc soá döông ,, xyz sao cho , ,. a y zbz xcx y = + = + =+ Khi ñoù, ta coù 222 3 3 3 2 2 2 2 22 3 3 3 2 22 a b c ab bc ca b c a c ab ac cb ba ab bc ca x y y z z x x yz xy z xyz + +≥ ++ ⇔+ +≥ + + ⇔ + + ≥ ++ Tìm tài liệu Toán ? Chuyện nh ỏ - www.toanmath.com367 2 22 2 22 ( )( )( ) 0() ñuùng x zy xyz z yx x y yz zx x yz ⇔ + + ≥++ - -- ⇔ ++≥ Boå ñeà 1 ñöôïc chöùng minh hoaøn toaøn. Ñaúng thöùc xaûy ra khi vaø chæ khi . x y z abc = =⇔ == Boå ñeà 2. ,, abc laø ñoä daøi ba caïnh cuûa moät tam giaùc. Khi ñoù, ta coù 3 23 a b c acb bc a cba ⎛ ⎞⎛⎞ ++ ≥ ++ + ⎜ ⎟⎜⎟ ⎝ ⎠⎝⎠ Chöùng minh. Ta coù 2 3 23 3 323 (5 5 3)( )0 cyc a bc acb b ca cba a bc acb b c a cba abcab ⎛⎞⎛⎞ + + ≥ + ++ ⎜⎟⎜⎟ ⎝⎠⎝⎠ ⎛⎞⎛⎞ ⇔ + + - ≥ ++- ⎜⎟⎜⎟ ⎝⎠⎝⎠ ⇔ - + -≥ ∑ Ñaët 553, 5 5 3, 5 5 3 abc S bcaS cabS ab c = - + =- + = -+ . Baát ñaúng thöùc caàn chöùng minh töông ñöông vôùi 222 ( )( )( ) 0 a bc S b c Sc a Sa b - + -+ -≥ + Tröôøng hôïp 1.abc ≤≤ . Khi ñoù, ta coù 0 b S ≥ vaø 8 2 0(do ) 8 20 ab cb S S b a ba S S cb +=->≥ + = -> Do ñoù 2 2 2 22 ( ) ( ) ( ) ( )( ) ( )( ) 0 a a c ab cb S b c S c a S ab S S bc S S ab - + -+ -≥ + - ++ -≥ ⇒ ñpcm. Tìm tài liệu Toán ? Chuyện nh ỏ - www.toanmath.com368 + Tröôøng hôïp 2.abc ≥≥ . Khi ñoù ta coù ,0 ac SS ≥ . Do ñoù neáu 0 b S ≥ thì ta coù ngay ñpcm, vì vaäy ta chæ caàn xeùt tröôøng hôïp 0 b S ≤ laø ñuû. + Tröôøng hôïp 2.1. ( ) 31 3 3() a c b a c bc + - ≤ ⇔-≤- Ta coù 3 14 10 12 12( ) 0 ab S S bca bca += + -≥ +- > Do ñoù 222 2 ( ) ( ) ( )( 3 )( )0 a a c ab S b c S c a S ab S S bc - + -+ -≥ + - ≥ ⇒ ñpcm. + Tröôøng hôïp 2.2. ( ) ( ) 31 3 31( ) a c b a b bc + - ≥ ⇔-≥-- + Tröôøng hôïp 2.2.1. 3 2 b a ≥ Ta coù 211 5 7 8()7 0 3 2 13 5 13( ) 5 8 0 2 ab cb S S b c a bca b S S b c ab ab aa + = + - ≥ + -> ⎛⎞ + =+ - >+ - - =-≥ ⎜⎟ ⎝⎠ Do ñoù 2 2 2 22 ( ) ( ) ( )( 2 )( ) ( 2 )( )0 a a c ab cb S bc S c a S ab S S bc S S ab - + -+ -≥ + - + + -≥ ⇒ ñpcm. + Tröôøng hôïp 2.2.2. 3 2 b a ≤ + Tröôøng hôïp 2.2.2.1. 2 3 a ac bc + ≥ ⇒≥ Ta coù 211 5 7 8()7 0 2 2 13 5 13. 5 0 33 ab cb S S b c a bca aa SSbcaa + = + - ≥ + -> + = + - ≥ + -= Do ñoù Tìm tài liệu Toán ? Chuyện nh ỏ - www.toanmath.com369 2 2 2 22 ( ) ( ) ( )( 2 )( ) ( 2 )( )0 a a c ab cb S bc S c a S ab S S bc S S ab - + -+ -≥ + - + + -≥ ⇒ ñpcm. + Tröôøng hôïp 2.2.2.2. 2 2() a c b a c bc +≤ ⇔-≤- Ta coù ( ) ( ) 22 4 3 1 (5 5 3 ) 3 1 17 15 17 abc S S S a b c b ca + + - = - + - + +- Do ( ) 313 a cb + -≥ neân ( ) 31 33 c a b - ≤+ Suy ra ( ) ( ) ( ) ( ) 5 31 5 55353 33 5 3 1 5 23 33 5 31 3 c a abcac ac a - -+≥ - - + -- =+ - > Do ñoù ( ) ( ) ( ) ( ) ( ) 3 2 3 3 5 31 4 3 1 17 15 17 3 5 31 16 17 3 5 31 0 3 abc a S S S b ca a bca a a - + + - > + +- - ≥ + +- - > -> Do ñoù ( ) 2 2222 () ( ) ( ) 4 31 () 0 a b c abc S bc S c a S a b S S S bc ⎛⎞ - + - + - ≥ + + - -≥ ⎜⎟ ⎝⎠ ⇒ ñpcm. Boå ñeà 2 ñöôïc chöùng minh hoaøn toaøn. Tìm tài liệu Toán ? Chuyện nh ỏ - www.toanmath.com370 Ñaúng thöùc xaûy ra khi vaø chæ khiabc == . Trôû laïi baøi toaùn cuûa ta Theo keát quaû Boå ñeà 2, ta coù 3 23 a b c acb bc a cba ⎛ ⎞⎛⎞ ++ ≥ ++ + ⎜ ⎟⎜⎟ ⎝ ⎠⎝⎠ 222 222 3( ) 2() 3() 3 2 2() a b c acb abc abc abc b ca cba abc ab bc ca a cb abc b ca c ab c ba ⎛⎞ ⎛⎞ ⇒ ++ + + ≥ + + + + + ++ ⎜⎟ ⎜⎟ ⎝⎠ ⎝⎠ ⎛ ⎞ ⎛⎞ ⇔ + + + + + ≥ + + + ++ ⎜ ⎟ ⎜⎟ ⎝ ⎠ ⎝⎠ Maët khaùc, theo Boå ñeà 1 thì 222 a b c ab bc ca b ca c ab + +≥ ++ Do ñoù 2 22 2 22 2 22 222 222 43 2 2() 2 () ñpcm abc ab c ab bc ca b ca b ca c ab acb abc c ba abc acb abc b c a c ba ⎛⎞⎛⎞ + + ≥ ++ + ++ ⎜⎟⎜⎟ ⎝⎠⎝⎠ ⎛⎞ ≥ + + + ++ ⎜⎟ ⎝⎠ ⎛⎞ ⇒ + + ≥ + ++ ++ ⎜⎟ ⎝⎠ Ñaúng thöùc xaûy ra khi vaø chæ khi . abc == Baøi toaùn 88. (Voõ Quoác Baù Caån) Cho 2 soá khoâng aâm ,. ab Chöùng minh raèng ( )( 1) ( )( 1) ( 1 )(1 ) 0 aa ba bb ab a b - -+ - -+ - - ≥ Lôøi giaûi. Ta coù baát ñaúng thöùc caàn chöùng minh töông ñöông vôùi 2 ( ) ( 1) ( 1)( 1) 0 abab ab - + - + - -≥ Khoâng maát tính toång quaùt, ta coù theå giaû söû 0. ab ≥≥ Tìm tài liệu Toán ? Chuyện nh ỏ - www.toanmath.com371 Coù 3 tröôøng hôïp xaûy ra + Tröôøng hôïp 1. 1. ab ≥≥ Khi ñoù, hieån nhieân ta coù 2 ( ) ( 1) ( 1)( 1) 0 abab ab - + - + - -≥ + Tröôøng hôïp 2. 1 0. ab ≥ ≥≥ + Tröôøng hôïp 2.1. 1. ab +≥ Khi ñoù, hieån nhieân ta coù 2 ( ) ( 1) ( 1)( 1) 0 abab ab - + - + - -≥ + Tröôøng hôïp 2.2. 1 0. ab ≥ +≥ Khi ñoù, ta coù 22 2 ( ) ( 1) ( 1)( 1) ( ) ( 1) 1 (1 )(1 ( )) 0 ( 1 0, 1) do ab ab a b ab ab abab a b a b ab a b ab - + - + - - = - +- + - -+ =-- - -+ ≥ ≥ ≥ ≥ +≤ + Tröôøng hôïp 3. 1 0. ab ≥ ≥≥ Xeùt haøm soá 2 ( ) ( ) ( 1) ( 1)( 1) 1. vôùi faabab aba =- +-+- -≥ Ta coù / 22 // () 3 2 1 32 () 6 2 20 f a a a b b ab fa ab = - -- +- = - -> ⇒ / () fa laø haøm ñoàng bieán treân[1,) +∞ . ⇒ // ( ) ( 1) (1 ) 0 1 faf bba ≥ = - ≥ ∀≥ ⇒ () fa laø haøm ñoàng bieán treân[1,) +∞ . ⇒ 2 ( ) ( 1) (1 ) 0 1. faf bb a ≥ = - ≥ ∀≥ Do ñoù 2 ( ) ( 1) ( 1)( 1) 0 abab ab - + - + - -≥ Toùm laïi, trong moïi tröôøng hôïp, ta luoân coù 2 ( ) ( 1) ( 1)( 1) 0 abab ab - + - + - -≥ (ñpcm) Ñaúng thöùc xaûy ra khi vaø chæ khi ( , ) (1,0),(1,1). ab = Tìm tài liệu Toán ? Chuyện nh ỏ - www.toanmath.com372 Baøi toaùn 89. (Voõ Quoác Baù Caån) Cho caùc soá döông , ,. abc Chöùng minh raèng 3 3 3 2 22 2(1 )(1 )(1 ) (1 )(1 )(1 )(1 ) a b c abc abc + + + ≥+ + ++ Lôøi giaûi. Ta coù baát ñaúng thöùc caàn chöùng minh töông ñöông vôùi 3 33 3 3 3 3 3 23 2 3 23 2 (1 ) (1 ) (1 ) (1 ) (1 ) (1 ) (1 ) a b c abc abc ++ + ≥+ + ++ AÙp duïng baát ñaúng thöùc AM-GM, ta coù 3 3 3 3 3 3 33 33 3 3 333 2 22 33 3 3 (1 )( 1 )(1 ) 1 ( ) ( ) 133 (1) a b c a b c ab bc ca abc abcabcabc abc + + + =+ ++ + + ++ ≥+ ++ =+ Do ñoù, ñeå chöùng minh baát ñaúng thöùc ñaõ cho, ta chæ caàn chöùng minh 3 32 32 32 23 23 23 2 (1 ) (1 ) (1 ) ( 1 ) ( 1 ) (1 ) abc abc + ++ ≥+ ++ AÙp duïng baát ñaúng thöùc Chebyshev, ta coù 32 33 32 2 3 2 2 23 2 2 23 23 32 23 2(1) 2(1)(1) (1 )(1 )( 1 ) (1 )((1 )(1 ) (1 ) ) (1 ) (1 ) ( 1) (1 ) (1) 2(1 ) (1 ) 0 a aa a aa a a a aa aaaa a aa + = ++ ≥+ ++ = + + + - + ++ = + - ++ ≥+ ⇒ + ≥+> Töông töï, ta coù 32 23 32 23 2(1 )(1 )0 2(1 ) (1 )0 bb cc + ≥+> +≥+> Do ñoù 3 32 32 32 23 23 23 2 (1 ) (1 ) (1 ) ( 1 ) ( 1 ) (1 ) abc abc + ++ ≥+ ++ ⇒ ñpcm. Ñaúng thöùc xaûy ra khi vaø chæ khi 1. abc === Tìm tài liệu Toán ? Chuyện nh ỏ - www.toanmath.com373 Baøi toaùn 90. (Voõ Quoác Baù Caån) Cho caùc soá khoâng aâm ,, abc thoûa 1 abc ++= . Tìm haèng soá 0 k > lôùn nhaát sao cho baát ñaúng thöùc sau ñuùng 2 22 31 2 akb bkc cka k bc c a ab + + ++ + +≥ + ++ Lôøi giaûi. Ta coù baát ñaúng thöùc ñaõ cho töông ñöông vôùi 2 2 2 3 2 22 1 30 ( )( ) ( )( )() 0 ( )( ) ( )( )( ) 3( ) 3 ( )( )( ) 0 ()( ) ( )( )() () 3() ( )( )( cyc cyc cyc cyc cyc cyc aa k b c ab ab abc kabbcca acb c a bb cca a b ka bb cc a acb c abbcca k ab ab a cb c ab ⎛ ⎞⎛⎞ - + -≥ ⎜ ⎟⎜⎟ ⎜ ⎟⎜⎟ ++ ⎝ ⎠⎝⎠ - ++ - -- ⇔ -≥ + + + ++ - - -- ⇔ -≥ + + + ++ - - ⇔- + ++ ∑∑ ∑ ∑ ∑ ∑ 2 0 )( )() ( ) ((3 ) (3 ) ) 0 (*) cyc b cca a b ka kb ≥ ++ ⇔ - - ++≥ ∑ Khoâng maát tính toång quaùt, ta coù coù theå giaû söû min{ , , } a abc = . Ñaët , ( , 0) ba xca y xy =+ =+ ≥ . Khi ñoù, ta coù 22 2 2 2 3 2 23 3 2 23 32 32 (*) ((3 ) (3 )( )) ( ) ((3 )( ) (3 )( )) ((3 )( ) (3 ) ) 0 12( ) 6 3( 1) 3( 1) 6 0,,0 2 ( 1)( 1)2 0 , 0 2 (1) ( 1)200 2 x ka ka x x y ka x ka y y k a y ka x xyya x k xykxy y axy x kxy kxy y xy tktktt tt ⇔ - +++ + - -+ +++ + + - + ++≥ ⇔ -+ + + -- + + ≥∀≥ ⇔ + - - + + ≥ ∀≥ ⇔ + - - + + ≥ ∀≥ ⇔- 32 32 2 (1)0 2 2 (1 ) (0,1) 22 ( ) (0,1) (1) t kt tt t t t kt tt t tt k ftt tt - + ≥ - ∀≥ ⇔ - - + ≥ - ∀∈ - -+ ⇔ ≤ = ∀∈ - Ta coù Tìm tài liệu Toán ? Chuyện nh ỏ - www.toanmath.com374 4 32 / 22 2( 2 2 1) () (1) tt tt ft tt - + -+ =- - 1 / 2 1 2 2 21 2 ()0 1 2 2 21 () 2 (nhaän) loaïi t ft t ⎡ + -- ⎢ = ⎢ =⇔ ⎢ + +- ⎢ = ⎢ ⎣ Qua 1 t thì / () ft ñoåi daáu töø aâm sang döông neân 1 2 22 1 4 2 222 1 ( ) 2 1 (0,1) 2 2 2 11 ftft ⎛⎞ +- - -- ⎜⎟ ≥ =- - + ∀∈ ⎜⎟ -- ⎝⎠ Do ñoù 42 222 1 21 2 2 11 k -- ≤- -+ -- Qua caùc laäp luaän treân, ta suy ra ñöôïc max 42 2 22 1 21. 2 2 11 k -- =- -+ -- Baøi toaùn 91. (Traàn Tuaán Anh) Cho caùc soá khoâng aâm,,1 thoûa . abc abc ++= Tìm giaù trò lôùn nhaát vaø giaù trò nhoû nhaát cuûa bieåu thöùc 333 (,,) ( ) ( ) () Pabc abc bc a cab = - + - +- Lôøi giaûi. + Caùch 1. Khoâng maát tính toång quaùt, ta coù theå giaû söû b laø soá haïng naèm giöõa a vaø c . Ñeå tìm giaù trò lôùn nhaát vaø giaù trò nhoû nhaát cuûa P, tröôùc heát ta seõ tìm giaù trò nhoû nhaát cuûa haøm soá 3 3 32 () (1) (1) 2 3 fb bb b b b bb = - - - =- +- vôùi01 b ≤≤ . Ta coù Tìm tài liệu Toán ? Chuyện nh ỏ - www.toanmath.com375 /2 1 / 2 ( ) (6 6 1) 3 6 ()0 3 6 fb bb b fb b =- -+ ⎡ 3- = ⎢ ⎢ =⇔ ⎢ 3+ = ⎢ ⎣ Baûng bieán thieân cuûa () fb Caên cöù vaøo baûng bieán thieân, ta suy ra ñöôïc 01 33 min ( ) min (1), 6 b fb ff ≤≤ ⎧⎫ ⎛⎞ - ⎪⎪ = ⎜⎟ ⎨⎬ ⎪⎪ ⎝⎠ ⎩⎭ Ta laïi coù 333 (1) 0, 6 18 ff ⎛⎞ - = =- ⎜⎟ ⎝⎠ . Do ñoù 01 333 min () 6 18 b fbf ≤≤ ⎛⎞ - = =- ⎜⎟ ⎝⎠ Tieáp theo, ta seõ chöùng minh ( ,,0)(,,)( ,0,) Pa cb Pabc Pa c b + ≤ ≤+ Coù 2 tröôøng hôïp xaûy ra * Tröôøng hôïp 1. 1 0. abc ≥ ≥≥≥ Xeùt haøm soá 33333 () ( ) ( ) () () () ga abc bca cabbacbac = -+ - + - + +- + Ta coù / 3 2 2 23 3 32 ( ) ( ) 3( ) 3 ( ) 3( ) 12 ( ) 3() g a b c bac ca b bac b abc b bc cab = - - -+ - + +- = - +- +- 33 6 - 33 6 + 0 1 b 0 0 + - - / () fb () fb Tìm tài liệu Toán ? Chuyện nh ỏ - www.toanmath.com376 2 33 2 23 12 () 93 0 bcb bc bcbcc ≥ - +- =+- ≥ () ga ⇒ laø haøm ñoàng bieán. 33 () () () () ()(2)0 ga gb bb c b b c bcbc bc ⇒ ≥ = + - + = + +≥ 3 3 333 ( ) ( ) ( ) () () ( ,, ) ( ,,0) ab c bca ca b b ac bac Pabc Pa cb ⇒ - + -+ -≥ + - + ⇒ ≥+ Xeùt tieáp haøm soá 33 333 () () ( ) ( ) () ( ) ha ba c b a c ab c bc a ca b = + - + - - - - -- Ta coù / 23 2 32 ()3() 3() ( ) 3( ) 0 h a ba c b bac bc cab = + - + - - - - -≥ () laø haøm ñoàng bieán. ha ⇒ 33 23 2 32 33 333 () () () () ()(2 )0 3() 3( ) ( ) 3() 0 ( ) () ( ) ( ) () ( ,0, ) (,,) ha hb bbc b bc bcbc bc ba c b ba c bc cab ba c b ac abc bca cab Pa c b Pabc ⇒ ≥ = + - + = + +≥ ⇒ +- + -- - - - ≥ ⇒ + - + ≥ - + - +- ⇒ +≥ Vaäy trong tröôøng hôïp naøy, ta coù ( ,,0)(,,)( ,0,) Pa cb Pabc Pa c b + ≤ ≤+ * Tröôøng hôïp 2. 1 0. cba ≥ ≥ ≥≥ Xeùt haøm soá 333 33 () ( ) ( ) ( ) () () kc ab c bc a ca b ba c b a c = - + - + - + + -+ Ta coù / 2 2 233 33 33333 3333 3 () 3( )3( )3( ) ( ) 0 () ( ) () ( ) ( ) ( )( 2) 0 ( ) () ( ) ( ) ()0 ( ) ( ) ( ) () () (, laø haøm ñoàng bieán. k c bca bc a ac b b ba kc kc kb bb a b ba abab a b abc bca cabbacbac ab c bc a ca b b a c ba c Pa = -+ + - - - - -≥ ⇒ ⇒ ≥ = + - + = + +≥ ⇒ -+ - + - + +- + ≥ ⇒ - + -+ -≥ + - + ⇒ , ) ( , ,0) bc Pa cb ≥+ Xeùt tieáp haøm soá 33 333 () () ( ) ( ) ( ) () mc ba c b a c abc bc a cab = +- + - -- - - - Tìm tài liệu Toán ? Chuyện nh ỏ - www.toanmath.com377 Ta coù / 2223 3 () 3( ) 3 ( ) 3( ) ( ) mc bc a ac b bca b ba =+ +- - - - +- 233 12 3( ) () abc a c b b b a = + - - +- 233 2 23 33 33 333 33 333 12 () 39 0 () () () () ( ) ()( 2)0 () () ( ) ( ) ( ) 0 () () ( ) () () ( ,0, ) ( ,, laø haøm ñoàng bieán. ab b ba ab aba mc mc mb bab b ab ababa b ba c b a c abc bc a cab ba c b a c ab c bc a ca b Pa c b Pab ≥ -+- = +- ≥ ⇒ ⇒ ≥ = + - + = + +≥ ⇒ + - + - - - -- -≥ ⇒+ - + ≥- + - + - ⇒ +≥ ) c Vaäy trong tröôøng hôïp naøy, ta cuõng coù ( ,,0)(,,)( ,0,) Pa cb Pabc Pa c b + ≤ ≤+ Toùm laïi, trong moïi tröôøng hôïp, ta luoân coù ( ,,0)(,,)( ,0,) Pa cb Pabc Pa c b + ≤ ≤+ Maët khaùc, ta laïi coù 3 333 ( ,,0)()() (1 )(1 ) () Pacb b ac ac b b b b b fb + = +- += ---= 33 33 (,0,)() ()(1 ) (1 ) () Pa c b a c bb a c b bb b fb + = +- + =-- - =- Do ñoù, theo keát quaû treân, ta coù 3 ( , ,0) 18 3 ( ,0,) 18 Pa cb Pa cb + ≥- +≤ Nhö vaäy, ta coù 3 ( , , ) (1) 18 3 ( , , ) (2) 18 Pabc Pabc ≥- ≤ Tìm tài liệu Toán ? Chuyện nh ỏ - www.toanmath.com378 Ñaúng thöùc ôû (1) xaûy ra chaúng haïn khi 33 33 , , 0. 66 abc +- = == Ñaúng thöùc ôû (2) xaûy ra chaúng haïn khi 3 3 33 ,0,. 66 a bc +- = == Vaäy 3 max ( , ,) 18 3 min (,,) 18 Pabc Pabc = =- + Caùch 2. Khoâng maát tính toång quaùt, ta coù theå giaû söû b laø soá haïng naèm giöõa a vaø c . Ta coù 333 3 3 3 3 33 (,,) ( ) ( ) () ( )( )( )( ) ( )( )( ) Pabc abc bc a cab ab bc ca a b b c c a abbcca ab c abbcca = - + - +- = ++ -- - = - - - ++ = - -- Coù 2 tröôøng hôïp xaûy ra * Tröôøng hôïp 1.10 abc ≥ ≥ ≥≥ . Khi ñoù, ta coù (, ,) 0 Pabc ≤ . AÙp duïng baát ñaúng thöùc AM-GM, ta coù (,,) ( )( )() 4. .. 2 3 1 31 Pabc abbc ac a b b c ac =---- - -- =- -+ 3 3 2 31 31 4 3 ( )3 3 2 4 3 ab bc ac ab c - -- ⎛⎞ ++ ⎜⎟ -+ ≥-⎜⎟ ⎜⎟ ⎜⎟ ⎝⎠ ⎛⎞ + - ⎜⎟ ⎜⎟ =- ⎜⎟ ⎜⎟ ⎝⎠ Tìm tài liệu Toán ? Chuyện nh ỏ - www.toanmath.com379 3 3 .(3 1) 18 3 18 c =- ≥- Ñaúng thöùc xaûy ra khi vaø chæ khi 33 1 6 33 0 26 3 1 31 0 0 a abc ab bc ac b c c ⎧ + = ⎪ ++= ⎧ ⎪ ⎪ ⎪ ---- ⎪⎪ = = ≥ ⇔= ⎨⎨ -+ ⎪⎪ = ⎪⎪ = ⎩ ⎪ ⎪ ⎩ * Tröôøng hôïp 2. 1 0. cba ≥ ≥ ≥≥ Khi ñoù, deã thaáy ( , , ) 0. Pabc ≥ AÙp duïng baát ñaúng thöùc AM-GM, ta coù ( , , ) ( )( )( ) Pabc c b b a c a = --- 3 3 3 4. .. 2 31 31 2 3 1 31 4 3 ( )3 3 2 4 3 3 .(1 3) 18 3 18 c b b a ca cb ba ca cb a a - -- = -+ - -- ⎛⎞ ++ ⎜⎟ -+ ≤⎜⎟ ⎜⎟ ⎜⎟ ⎝⎠ ⎛⎞ + - ⎜⎟ ⎜⎟ = ⎜⎟ ⎜⎟ ⎝⎠ =- ≤ Tìm tài liệu Toán ? Chuyện nh ỏ - www.toanmath.com380 Ñaúng thöùc xaûy ra khi vaø chæ khi 33 1 6 33 0 26 3 1 31 0 0 c abc cb ba ca b a a ⎧ + = ⎪ ++= ⎧ ⎪ ⎪ ⎪ ---- ⎪⎪ ==≥⇔= ⎨⎨ -+ ⎪⎪ = ⎪⎪ = ⎩ ⎪ ⎪ ⎩ Töø caùc chöùng minh treân, ta suy ra ñöôïc 3 max ( , ,) 18 3 min (,,) 18 Pabc Pabc = =- Baøi toaùn 92. (Phaïm Vaên Thuaän) Cho caùc soá khoâng aâm,,1 thoûa . abc abc ++= Tuøy theo giaù trò cuûa , nN ∈ tìm giaù trò lôùn nhaát vaø giaù trò nhoû nhaát cuûa bieåu thöùc (,,) ()( ) ( ) nnn Pabc ab c bc a ca b = - +-+ - Lôøi giaûi. 0 (, ,) 1 1 (,,)0 n Pabc n Pabc +=⇒= +=⇒= + Xeùt 2 n ≥ a) n leû ⇒ 3 n ≥ . Ta seõ chöùng minh ( ,,0)(,,)( ,0,) Pa cb Pabc Pa c b + ≤ ≤+ Coù 2 tröôøng hôïp xaûy ra * Tröôøng hôïp 1. 1 0. abc ≥ ≥≥≥ Xeùt haøm soá()() () ( ) () () n nnnn ga ac b a cb abc bca cab = + - + - - - --- Ta coù / 1 11 1 11 () () ( ) () () ( ) ( )(() ( )) nn n nn nn n nn g a nba c b bc nbac ncab nba c b bc nba c cab - -- - -- = +- -- + --- = + - - - + - -- Tìm tài liệu Toán ? Chuyện nh ỏ - www.toanmath.com381 1 ( ) () 0 () () ()() () 0 () () ( ) () () ( ,0, ) (, ,) (1) laø haøm ñoàng bieán. n nn nn n n nnn nba c b bc ga ga gb bc b bcb ac b a cb abc bca cab Pa c b Pabc - ≥ + - -- ≥ ⇒ ⇒≥=+ -+ ≥ ⇒ + - + ≥ - + -+- ⇒ +≥ Xeùt tieáp haøm soá()() () ( ) () () n nnnn ha a c b a cb ab c bc a ca b = + - + + - + -+- Ta coù / 1 11 111 11 31 22 21 2 2 21 21 21 21 2 2 2 1 0 01 () () ( ) () () (() ()) ( ) ( ) (( ) ()) () 2 nn n nn n nn nn n n nn n nn l nl l l nl l l n l l n nn l ll h a nba c b bc nbac ncab nb a c ac b bc ncab nbbcbcb bc nbCb cCbcCbc - -- --- -- - -- + -- + + -- + - - = == = + - +- - - +- = + -- - +- +- ≥ + -- - +- = -+ ∑∑ 1 2 31 22 21 21 21 21 2 2 2 1 01 21 21 1 (2) 0 2 0) (do nn nl l l l lnlln n nn ll ll nn b c nC C Cb cc nC C ln -- --+ ++- - == ++ - = -+- ≥ - ≥ ∀< ∑ ∑∑ () () ()( ) ( )0 ( ) ( ) ( ) () () ( , , ) ( , ,0) (2) laø haøm ñoàng bieán. nn nnn n n ha ha hb b c b b cb ab c bca ca b a cb ac b Pabc Pa cb ⇒ ⇒ ≥= + -+≥ ⇒ - +-+ -≥ + - + ⇒ ≥+ Töø (1) vaø (2), ta suy ra trong tröôøng hôïp naøy, ta coù ( ,,0)(,,)( ,0,) Pa cb Pabc Pa c b + ≤ ≤+ * Tröôøng hôïp 2. 1 0. cba ≥ ≥ ≥≥ Xeùt haøm soá()() () ( ) () () n nnnn kc ac b acb abc bca cab = + - + - - - --- Ta coù / 1 11 111 () () ( ) () () (() ()) ( ) ( ) nn n nn n nn nn k c nbc a b nac b nbca ba nb c a c a b b a na c b - -- --- = + - +-- - +- = + -- - +- +- Tìm tài liệu Toán ? Chuyện nh ỏ - www.toanmath.com382 11 3 11 2 22 21 2 2 21 21 21 21 2 2 2 1 0 01 31 22 21 21 21 21 2 2 2 1 01 212 1 (( )()) () 2 (2) 02 (do n n nn n nn l n l l l nl l l nl l n nn l ll nn nl l l l l nl l n n nn ll ll nn nbb a ba b ba nbCb a C b a Cb a b a nC C Cb aa nCC -- - -- + -- + + -- + - - = == -- --+ ++- - == ++ - ≥ + - - - +- = -+ = -+- ≥- ∑ ∑∑ ∑∑ 1 0) () () ()( ) ( )0 () () ( ) () () ( ,0, ) ( , , ) (3) laø haøm ñoàng bieán. nn n n nnn ln kc kc kb ba b bab a c b a cb abc bc a cab Pa c b Pabc ≥ ∀< ⇒ ⇒ ≥ = + -+≥ ⇒ + - + ≥ - + -+- ⇒ +≥ Xeùt tieáp haøm soá ()() () ( ) () () n nnnn mc a c b a cb ab c bc a ca b = + - + + - +-+ - Ta coù / 1 11 1 11 () () ( ) ()() (( ) ( )) (( ) ( )) 0 () () ()() ()0 ()( ) ( ) ( )( ) ( ,, laø haøm ñoàng bieán. nn n nn nn n nn nn nnn n n mc nbc a b nac b nbca ba nbc a b b a nbc a ac b mc mc mb b a b b ab ab c bc a ca b a c b a cb Pabc - -- - -- = +- - - + --- = + - - - + - -- ≥ ⇒ ⇒ ≥ = + -+≥ ⇒ - +-+ -≥- + + + ⇒ ) ( , ,0) (4) Pa cb ≥+ Töø (3) vaø (4), ta suy ra trong tröôøng hôïp naøy, ta cuõng coù ( ,,0)(,,)( ,0,) Pa cb Pabc Pa c b + ≤ ≤+ Toùm laïi, trong moïi tröôøng hôïp, ta luoân coù ( ,,0)(,,)( ,0,) Pa cb Pabc Pa c b + ≤ ≤+ Xeùt haøm soá 1 ()0 ( 1) vôùi . n n tt ftt t + - => + Ta coù 1 / 2 1 () ( 1) nn n t nt nt ft t - + - + +- = + Tìm tài liệu Toán ? Chuyện nh ỏ - www.toanmath.com383 /1 () 0 10 nn f t t nt nt - = ⇔- + + -= Deã thaáy neáu 0 0 t > laø moät nghieäm cuûa phöông trình / ()0 ft = thì 0 1 t cuõng laø nghieäm cuûa phöông trình / ()0 ft = . Do ñoù, ta chæ caàn tìm nghieäm cuûa phöông trình / ()0 ft = treân (0,1] laø ñuû. Xeùt haøm soá 1 ( ) 1 (0,1]. vôùi nn t t nt nt t φ - =-+ +-∈ Ta coù / 12 ( ) (1 ) ( 1) 0 (0,1] ( ) (0,1]. laø haøm ñoàng bieán treân nn t nt nnt t t φ φ -- = - + - > ∀∈ ⇒ Ta laïi coù 0 lim ( ) 1 0, (1) 2( 1) 0 t tn φφ + → =-< = -> neân toàn taïi duy nhaát 1 (0,1] t ∈ sao cho 1 ( ) 0. t φ= Do ñoù, phöông trình / ()0 ft = chæ coù 2 nghieäm phaân bieät laø 1 t vaø 1 1 t . Qua 1 t thì / () ft ñoåi daáu töø aâm sang döông, qua 1 1 t thì / () ft ñoåi daáu töø döông sang aâm neân 0 1 1 ( ) max lim ( ), 0 t ft ftft t + → ⎧⎫ ⎛⎞ ⎪⎪ ≤ ∀> ⎨⎬ ⎜⎟ ⎪⎪ ⎝⎠ ⎩⎭ Ta laïi coù neáu 0 b > thì 11 () () (,0,)() () () 1 n nn nn nn a c ac ac b acb bb Pac b ac b a cb abc ac b ac f b ++ ++ ⎛⎞ ⎛⎞ - ⎜⎟ ⎜⎟ + -+ ⎝⎠ ⎝⎠ +=+-+== ++ +⎛⎞ + ⎜⎟ ⎝⎠ + ⎛⎞ = ⎜⎟ ⎝⎠ (,,0) ( ) () nn Pacb a c b acb + =- + ++ Tìm tài liệu Toán ? Chuyện nh ỏ - www.toanmath.com384 1 () () () nn n a c b a cb abc + + -+ =- ++ 1 1 n n a c ac bb ac b ac f b + ++ ⎛⎞ ⎛⎞ - ⎜⎟ ⎜⎟ ⎝⎠ ⎝⎠ =- +⎛⎞ + ⎜⎟ ⎝⎠ + ⎛⎞ =- ⎜⎟ ⎝⎠ Neân theo treân, ta coù 0 11 0 11 11 ( ,0, ) max lim ( ), max 0, 11 ( ,0, ) max lim ( ), max 0, t t Pac b ftf f tt Pac b ftf f tt + + → → ⎧ ⎫ ⎧⎫ ⎛⎞ ⎛⎞ ⎪ ⎪ ⎪⎪ +≤= ⎨ ⎬ ⎨⎬ ⎜⎟ ⎜⎟ ⎪ ⎪ ⎪⎪ ⎝⎠ ⎝⎠ ⎩ ⎭ ⎩⎭ ⎧ ⎫⎧⎫ ⎛⎞ ⎛⎞ ⎪ ⎪⎪⎪ + ≥- =- ⎨ ⎬⎨⎬ ⎜⎟ ⎜⎟ ⎪ ⎪⎪⎪ ⎝⎠ ⎝⎠ ⎩ ⎭⎩⎭ Ngoaøi ra, 0 ( , ,0) ( ,0, ) 0 neáu thì ta coù neân ta luoân coù b Pacb Pac b = + = += 0 11 0 11 11 ( ,0, ) max lim ( ), max 0, 11 ( ,0, ) max lim ( ), max 0, t t Pac b ftf f tt Pac b ftf f tt + + → → ⎧ ⎫ ⎧⎫ ⎛⎞ ⎛⎞ ⎪ ⎪ ⎪⎪ +≤= ⎨ ⎬ ⎨⎬ ⎜⎟ ⎜⎟ ⎪ ⎪ ⎪⎪ ⎝⎠ ⎝⎠ ⎩ ⎭ ⎩⎭ ⎧ ⎫⎧⎫ ⎛⎞ ⎛⎞ ⎪ ⎪⎪⎪ + ≥- =- ⎨ ⎬⎨⎬ ⎜⎟ ⎜⎟ ⎪ ⎪⎪⎪ ⎝⎠ ⎝⎠ ⎩ ⎭⎩⎭ Do ñoù, ta coù 11 11 max 0, ( , , ) max 0, f Pabc f tt ⎧⎫ ⎧⎫ ⎛⎞ ⎛⎞ ⎪⎪ ⎪⎪ - ≤≤ ⎨⎬ ⎨⎬ ⎜⎟ ⎜⎟ ⎪⎪ ⎪⎪ ⎝⎠ ⎝⎠ ⎩⎭ ⎩⎭ Deã thaáy raèng ñaúng thöùc luoân xaûy ra neân ta coù 1 1 1 min ( , , ) max 0, 1 max ( , , ) max 0, Pabcf t Pabc f t ⎧⎫ ⎛⎞ ⎪⎪ =- ⎨⎬ ⎜⎟ ⎪⎪ ⎝⎠ ⎩⎭ ⎧⎫ ⎛⎞ ⎪⎪ = ⎨⎬ ⎜⎟ ⎪⎪ ⎝⎠ ⎩⎭ Tìm tài liệu Toán ? Chuyện nh ỏ - www.toanmath.com385 b) 2. chaün nn ⇒≥ Khi ñoù, deã thaáy min (, ,) 0 Pabc = vaø ( ,,) Pabc laø moät bieåu thöùc ñoái xöùng vôùi , ab vaø c neân khoâng maát tính toång quaùt, ta coù theå giaû söû 1 0. abc ≥ ≥≥≥ Ta seõ chöùng minh ( , , ) ( ,0,) Pabc Pa c b ≤+ Xeùt haøm soá()() () ( ) () () n nnnn ua a c b acb ab c bca ca b = + - + - - - --- Ta coù / 1 11 1 11 1 () () ( ) ( ) () ( ) ( )(() ( )) ( ) () 0 () () ()() () 0 ( ) ( ) ()( laø haøm ñoàng bieán. nn n nn nn n nn n nn nn n nn u a nba c b bc nbac ncab nba c b bc nba c cab nba c b bc ua ua ub b c b b cb ac b a cb ab c bca - -- - -- - = +- -- + --- = + - - - + - -- ≥ + - -- ≥ ⇒ ⇒ ≥ = + -+≥ ⇒ + - + ≥ - +-) () ( ,0, ) (,,) nn cab Pa c b Pabc +- ⇒ +≥ Theo treân, ta laïi coù 0 11 11 ( ,0, ) max lim ( ), max 0, t Pac b ftf f tt + → ⎧ ⎫ ⎧⎫ ⎛⎞ ⎛⎞ ⎪ ⎪ ⎪⎪ +≤= ⎨ ⎬ ⎨⎬ ⎜⎟ ⎜⎟ ⎪ ⎪ ⎪⎪ ⎝⎠ ⎝⎠ ⎩ ⎭ ⎩⎭ Do ñoù 1 1 ( , , ) max 0, Pabc f t ⎧⎫ ⎛⎞ ⎪⎪ ≤ ⎨⎬ ⎜⎟ ⎪⎪ ⎝⎠ ⎩⎭ Ngoaøi ra, ta cuõng deã thaáy ñaúng thöùc luoân xaûy ra neân 1 1 max ( , , ) max 0, Pabc f t ⎧⎫ ⎛⎞ ⎪⎪ = ⎨⎬ ⎜⎟ ⎪⎪ ⎝⎠ ⎩⎭ Keát luaän 0 (, ,) 1 1 (, ,) 0 n Pabc n Pabc +=⇒= +=⇒= Tìm tài liệu Toán ? Chuyện nh ỏ - www.toanmath.com386 1 1 2 max ( , , ) max 0, 0 n Pabc f t ⎧⎫ ⎛⎞ ⎪⎪ +≥⇒= ⎨⎬ ⎜⎟ ⎪⎪ ⎝⎠ ⎩⎭ 1 1 * min ( , , ) max 0, * min (, ,) leû chaün n Pabcf t n Pabc ⎧⎫ ⎛⎞ ⎪⎪ ⇒ =- ⎨⎬ ⎜⎟ ⎪⎪ ⎝⎠ ⎩⎭ ⇒= trong ñoù 1 () ( 1) n n tt ft t + - = + vaø 1 t laø nghieäm döông duy nhaát thuoäc (0,1] cuûa phöông trình 1 1 0. nn t nt nt - - + + -= Baøi toaùn 93. (Vietnam TST 1996) Cho caùc soá thöïc ,, abc baát kyø. Chöùng minh raèng 4 4 4 4 44 4 (,,) ( )( )( ) .( ) 0 7 Fabc ab bc c a abc = + + + + + - + +≥ Lôøi giaûi. Ta coù ( ) 4 4 4 4 44 44 44 4 4 4 4 44 3 3 3 2 22 (,,) ,, 22 4 ()()().() 7 4 2 ( ) .2 2 72 4 ( )( )2 2 78 (4 4 ())3(22 () bcbc F abc F a ab bc c a a b c bc bc a bca bc bc ab c a a bc a b c bc a b c bc ++ ⎛⎞ -= ⎜⎟ ⎝⎠ = + + ++ + - + +- ⎛⎞ ++ ⎛ ⎞ ⎛⎞ - + -+ ++⎜⎟ ⎜ ⎟ ⎜⎟ ⎜⎟ ⎝ ⎠ ⎝⎠ ⎝⎠ ⎛⎞ + + ⎛⎞ ⎜⎟ = + + + - + + -- ⎜⎟ ⎜⎟ ⎝⎠ ⎝⎠ = + -+ + + -+ 4 2 44 2 2 2 2 22 2 2 22 3 () ). 78 3 3 ( )( ) 3 ( ) .( ) (7 7 10 ) 56 3 3 ( )( ) .( ) (7 7 10 ) 56 bc bc ab cbc a bc bc b c bc aa b cbc bc b c bc ⎛⎞ + + +- ⎜⎟ ⎝⎠ = + -+ -+ - + + = ++ - + - ++ Tìm tài liệu Toán ? Chuyện nh ỏ - www.toanmath.com387 Soá haïng cuoái cuøng luoân luoân khoâng aâm. Neáu ,, abc cuøng daáy thì baát ñaúng thöùc caàn chöùng minh laø hieån nhieân. Neáu ,, abc khoâng cuøng daáu thì phaûi coù ít nhaát moät trong ba soá ,, abc cuøng daáu vôùiabc ++ . Khoâng maát tính toång quaùt, giaû söû ñoù laø a . Töø ñaúng thöùc treân ta suy ra (, ,) , , 22 b cbc Fabc Fa ++ ⎛⎞ ≥ ⎜⎟ ⎝⎠ . Nhö vaäy, ta chæ caàn chöùng minh 4 4 44 (,,) 0, 4 2( ) (2 ) .( 2 ) 0 7 F abb ab ab b ab ≥ ∀∈ ⇔ + + - +≥ R Neáu 0 b = thì baát ñaúng thöùc laø hieån nhieân. Neáu 0 b ≠ , chia hai veá cua baát ñaúng thöùc cho 4 b roài ñaët a x b = thì ta ñöôïc baát ñaúng thöùc töông ñöông 44 4 2( 1) 16 .( 2) 0 7 xx + + - +≥ Baát ñaúng thöùc cuoái cuøng coù theå chöùng minh nhö sau Xeùt haøm soá 44 4 ( ) 2( 1) 16 .( 2) 7 fxxx = + +-+ Ta coù / 33 16 ()8( 1). 7 fxxx = +- / 3 2 ( ) 0 1 . 2.9294. 7 fx x xx = ⇔ + = ⇔ =- min ( 2.9294) 0.4924 0 ff=- => (Caùc phaàn tính toaùn cuoái ñöôïc tính vôùi ñoä chính xaùc tôùi 4 chöõ soá sau daáu phaåy. Do min f tính ñöôïc laø 0.4924 neân neáu tính caû sai soá tuyeät ñoái thì giaûtò chính xaùc cuûa min f vaãn laø moät soá döông. Vì ñaây laø moät baát ñaúng thöùc raát chaët neân khoâng theå Tìm tài liệu Toán ? Chuyện nh ỏ - www.toanmath.com388 traùnh ñöôïc caùc tính tính toaùn vôùi soá leû treân ñaây. Chaúng haïn neáu thay 4 7 baèng 16 27 ñeå min 3 x =- thì * min f coù giaù trò aâm! ÔÛ ñaây * 44 4 ( ) 2( 1) 16 .( 2) 7 fxxx = + +-+ .) * Chuù yù. Ta coù theå ñöa baøi toaùn veà chöùng minh (,,) 0, F abb ab ≥ ∀∈R baèng caùch söû duïng Boå ñeà sau Boå ñeà. ,, abc ∀∈R thì toàn taïi caùc soá thöïc 0 011 , ,, x y xy sao cho 0 0 11 22 0 0 0 1 11 22 00 11 22 22 p a b c x y xy q ab bc ca x x y x x y xy r abc xy =++= + =+ = + +=+ =+ ≤=≤ Ngoaøi ra, neáu ,,0 abc ≥ thì 011 ,,0 x xy ≥ . Trong ñoù 2 0 2 0 40 40 Neáu thì Neáu thì p qy p qy + ≥≤ + ≤≥ Baøi toaùn 94. (Phaïm Kim Huøng) Cho caùc soá khoâng aâm,,3 thoûa .Tìm giaù trò lôùn nhaát cuûa bieåu thöùc abc abc ++= (,,) ( ) ( ) () kkk f abc a b c b c a c a b = ++ + ++ trong ñoù 0 k > laø haèng soá cho tröôùc. Lôøi giaûi. Khoâng maát tính toång quaùt, ta coù theå giaû söû 0. abc ≥≥≥ Coù 3 tröôøng hôïp xaûy ra * Tröôøng hôïp 1. 1 0. k >> Khi ñoù, aùp duïng baát ñaúng thöùc Holder, ta coù 1 11 1 1 (,,) ()( ) ( ) (3 ) (3 ) (3 ) 3( )() ()() 3.6 33 kkk k kk k k k k kk kk kk fabc a bc b c a c ab a ab bc c a b c abc abc abc + ++ + - = ++ + ++ = - + - +- = + + - ++ ++ ++ ≤ -= Tìm tài liệu Toán ? Chuyện nh ỏ - www.toanmath.com389 Ñaúng thöùc xaûy ra khi vaø chæ khi 1. abc === * Tröôøng hôïp 2. 2 k > . Khi ñoù, ta seõ chöùng minh (, , ) (, ,0) () ( ) ( ) ()() ( ) ( )() (()) kkk k k k kk k k k kk f abc f ab c ab c bc a c a b ab c b ca b c a ca b b ca b c b c a b c bc ≤+ ⇔++ ++ +≤+++ ⇔ ++ +≤+ ⇔ + - - ≥+ Do 2 k > neân 1 11 11 11 () ()() ( )() (())()0 (, ,) ( , ,0) (do ) k kk k kk k k kk kk k k k k k kk b c b c bc bc b c b c b c bc b c bc b c b c a b c bc abc bc abc f abc f ab c - -- -- -- + -- = + + -- ≥ + + -- =+ ⇒ + - - ≥ + ≥ + ≥≥≥ ⇒ ≤+ Tieáp theo, ta seõ tìm giaù trò lôùn nhaát cuûa bieåu thöùc ( , ,0) kk f ab ab ba =+ trong ñoù,03 thoûa . ab ab ≥ += Khoâng maát tính toång quaùt, ta coù theå giaû söû 0 0. aba ≥ ≥ ⇒> Khi ñoù, ta coù 11 1 ( , ,0) 3 . 3 () () kk kkkk k ab ba f ab ab ba gt ab ++ + + =+== + trong ñoù 1 ( ) 1. ( 1) vaø k k ttb gtt a t + + = =≤ + Ta coù 1 / 2 /1 1 () ( 1) ( ) 0 1 0 (*) kk k kk t kt kt gt t g t t kt kt - + - - + -+ = + = ⇔- + - += Deã thaáy 1 laø moät nghieäm cuûa phöông trình (*). Goïi 01 , ,..., m ααα laø taát caû caùc nghieäm thuoäc [0,1) neáu coù phöông trình (*). Khi ñoù, deã thaáy ( ) max{ (1), ( )} [0,1] i gt ggt α ≤ ∀∈ Tìm tài liệu Toán ? Chuyện nh ỏ - www.toanmath.com390 1 ( , ,0) 3 max{ (1), ( )} k i fab gg α + ⇒≤ Ngoaøi ra, deã thaáy ñaúng thöùc luoân xaûy ra. * Tröôøng hôïp 3. 2 1. k ≥≥ Ñaët 2, 2 0,0 a b ta b u tu tc + = -= ⇒ ≥ ≥ ≥≥ . Khi ñoù, ta coù (, , ) ( ) ( ) ( ) ( ) 2 () k kk fabc tu tuc t u t uc ct hu = + - ++ - + ++ = Ta coù /11 11 () ( ) ( )( ) () ( )() ( ) (( 1) ( 1) ) ( ) (( 1) ( 1) ) k kkk kk hu ktu tuc tu ktu tu c tu t u k t k u kc t u k t k u kc -- -- = + - + -+ - - + + +- = + - - ++ - - - + ++ Neáu (1)(1)0 tu k t k u kc = ⎡ ⎢ - - + +≤ ⎣ thì ta coù / ()0 hu ≤ . Neáu ( 1) ( 1) 0 tu k t k u kc > ⎧ ⎨ -- + +> ⎩ thì do21 k ≥≥ neân 12 () ()() kk t u t utu -- + ≤ +- . Do ñoù, ta coù /2 1 2 2 ( ) ( )( ) (( 1) ( 1) ) ( ) (( 1) ( 1) ) ( ) (( )(( 1) ( 1) ) ( )(( 1) ( 1) )) 2( )(2) 0 k k k k hu tut u k t k ukc t u k t k u kc tu tu k t k u kc tu k t k u kc u t u t kc - - - - ≤ + - - - + +- - - - + ++ = - + - - ++ - - - + ++ =--- ≤ Toùm laïi, ta luoân coù / () 0 () laø haøm nghòch bieán treân [0,+ ). h u hu≤⇒∞ Do ñoù ( , ,) ( ) (0) (, ,) f abc hu h f ttc =≤= Baây giôø, ta seõ tìm giaù trò lôùn nhaát cuûa bieåu thöùc (,,) 2 ( )2 kk f t t c t t c tc = ++ trong ñoù 0 23 thoûa . t c tc ≥ ≥ += Ta coù 11 1 () ( , , ) 2 ( ) 2 2.3 . 2.3 ( ) (2) kk k kkk k t t c tc fttcttctcx tc φ ++ + ++ =++== + Tìm tài liệu Toán ? Chuyện nh ỏ - www.toanmath.com391 trong ñoù 1 1 ( ) 1. ( 2) vaø k k xxc xx t x φ + ++ = =≤ + Ta coù 1 / 2 /1 21 () ( 2) ( ) 0 2 1 0 (**) kk k kk x kx kx k x x x x kx kxk φ φ - + - - + - +- = + =⇔- + - + -= Deã thaáy 1 laø moät nghieäm cuûa phöông trình (**). Goïi 01 , ,..., n βββ laø taát caû caùc nghieäm thuoäc [0,1) neáu coù phöông trình (**). Khi ñoù, deã thaáy 1 ( ) max{ (0), (1), ( )} [0,1] ( , , ) 2.3 max{ (0), (1), ( )} i k i xx f ttc φ φ φ φβ φ φ φβ + ≤ ∀∈ ⇒≤ Ngoaøi ra, deã thaáy ñaúng thöùc luoân xaûy ra. Keát luaän 1 1 1 0 max (,,)6 2 1 max ( , , ) 2.3 max{ (0), (1), ( )} 2 max ( , , ) 3 max{ (1), ( )} k i k i k f abc k f abc k f abc g g φ φ φβ α + + +>>⇒= +≥≥⇒= +>⇒= trong ñoù + 1 1 () ( 2) k k xx x x φ + ++ = + vaø 01 , ,..., n βββ laø taát caû caùc nghieäm thuoäc [0,1) neáu coù phöông trình 1 2 10 kk x kx kxk - - + - + -= . + 1 () ( 1) k k tt gt t + + = + vaø 01 , ,..., m ααα laø taát caû caùc nghieäm thuoäc [0,1) neáu coù phöông trình 1 2 10 kk x kx kxk - - + - + -= . Tìm tài liệu Toán ? Chuyện nh ỏ - www.toanmath.com392 Baøi toaùn 95. (Voõ Quoác Baù Caån) Chöùng minh raèng vôùi moïi soá döông ,, xyz thoûa 1 xy yz zx ++= ta luoân coù baát ñaúng thöùc 2 22 2 22 2( ) 32 x yz x yz yzx + + - + + ≥- Lôøi giaûi. Ta coù Boå ñeà sau Boå ñeà. ,, laø caùc soá thöïc thoûa xyz 0 0 xyz xy yz zx + +≥ ⎧ ⎨ ++≥ ⎩ . Khi ñoù, ta coù 2 22 ( )( ) () 0 ,, xb c yc a za b abc R - + - + - ≥ ∀∈ Boå ñeà treân chöùng minh raát ñôn giaûn (chæ caàn duøng tam thöùc baäc hai laø ñöôïc) neân ôû ñaây ta khoâng nhaéc laïi chöùng minh cuûa noù. Ta coù baát ñaúng thöùc caàn chöùng minh töông ñöông vôùi ( ) 2 22 2 22 22 2 2 3 2( 1) ( ) 1 () . () 2 3 11 ( ) 10 23 cyc cyc cyc cyc xyz x y z x y z xyz y zx xy xy xy y x yz xy y x yz ⎛⎞ + + -- - ++ + - ≥ + + - ⎜⎟ ⎝⎠ -- ⇔ + ≥- + ++ ⎛⎞ ⎜⎟ ⇔ - + -≥ ⎜⎟ + ++ ⎝⎠ ∑ ∑∑ ∑ Ñaët ( ) ( ) ( ) 11 1 23 11 1 23 11 1 23 x y z S z x yz S x x yz S y x yz =+- + ++ =+- + ++ =+- + ++ Khi ñoù, baát ñaúng thöùc caàn chöùng minh töông ñöông vôùi Tìm tài liệu Toán ? Chuyện nh ỏ - www.toanmath.com393 222 ( ) ( ) ( )0 x yz S yz S zx S x y - + - + -≥ Ta coù ( ) 1113 3 23 xyz SSS x yz x yz + + = + + -+ + ++ ( ) 3 3 23 xy yz zx xyz xyz ++ = -+ + ++ ( ) ( ) ( ) 3 2 13 3 23 333 3 23 () 3 3 33 23 0 xyz x yz x yz xy yz zx xyz = -+ + ++ ≥ -+ + ++ ++ = -+ + ++ > Ñaët ( ) 1 23 t x yz = + ++ . Khi ñoù, ta coù 2 1 1 11 1 1 11 11 11 111 1 1 1 111 32 3 23 1 1 1 1 11 23 32 xy y z zx SSSSSSt t tt x y yz tt zx tt x yz xy yzzx x yz xy yzzx x yz x y z xyz xyz ⎛⎞⎛⎞ ⎛ ⎞ ⎛⎞ + + = + - + -+ + - + -+ ⎜ ⎟⎜ ⎟ ⎜ ⎟⎜ ⎟ ⎝ ⎠ ⎝⎠ ⎝⎠⎝⎠ ⎛ ⎞⎛⎞ + + - +- ⎜ ⎟⎜⎟ ⎝ ⎠⎝⎠ ⎛ ⎞ ⎛⎞ =+ + + - +++ - + ++ ⎜ ⎟ ⎜⎟ ⎝ ⎠ ⎝⎠ ⎛⎞ > ++ - + ++ ⎜⎟ ⎝⎠ +++- = Ta chöùng minh 32 0 3 2 0 (*) x y z xyz xyz x y z xyz +++- ≥ ⇔++ + - ≥ Tìm tài liệu Toán ? Chuyện nh ỏ - www.toanmath.com394 Neáu 2 x yz + +≥ thì (*) hieån nhieân ñuùng. Neáu 2 x yz + +≤ , ñaët 2 3. pxyzp = + + ⇒ ≥≥ Theá thì theo baát ñaúng thöùc Schur, ta coù 3 4 9 pp xyz - ≥ Do ñoù 33 4 76 (2 )( 1)(3) 3220 3 33 (*) ñuùng. pp p p ppp pxyzp - - + - - -+ +-≥-+= =≥ ⇒ Vaäy ta coù 0 0 xyz xyyzzx S SS SS SS SS ++> ⎧ ⎪ ⎨ + +> ⎪ ⎩ neân theo Boå ñeà treân, ta coù 222 ( ) ( ) ( )0 x yz S yz S zx S x y - + - + -≥ (ñpcm) Ñaúng thöùc xaûy ra khi vaø chæ khi 1 . 3 xyz = == Baøi toaùn 96. Cho,,,0 abcd ≥ thoûa 2 2 22 1. abc d + + += Tìm giaù trò nhoû nhaát cuûa bieåu thöùc 11 11 ab cd P bcd cda dab abc = + ++ ++ ++ Lôøi giaûi. AÙp duïng baát ñaúng thöùc Bunhiacopxki, ta coù 2 2 22 2 () 4 12() 4 a bcd P a abcd b abcd c abcd d abcd a b cd a b c d abcd ab ac ad bc bd cd a b c d abcd = + ++ + + ++ + ++ ≥ + + ++ + + + + ++ = + + ++ Ta laïi coù 12( )( 4) ab ac ad bc bd cd a b c d abcd + +++++-++++= Tìm tài liệu Toán ? Chuyện nh ỏ - www.toanmath.com395 (1 )(1 )(1 )( 1 ) ( ) ( )5 ( ) ( )5 0 12( )4 12() 1 4 1 a b c d ab ac ad bc bd cd abc bcd cda dab abcd ab ac ad bc bd cd abc bcd cda dab abcd ab ac ad bc bd cd a b c d abcd ab ac ad bc bd cd a b c d abcd P = - - - - + + + + + ++ + + + +- ≥ + + + + + + + + +- ≥ ⇒ + + + + + + ≥ + + ++ + + + + ++ ⇒≥ + + ++ ⇒≥ Ñaúng thöùc xaûy ra khi vaø chæ khi ( , , , ) (1,0,0,0). abcd = Vaäy min 1. P = Baøi toaùn 97. (Vasile Cirtoaje) Chöùng minh raèng vôùi moïi soá thöïc ,, abc ta luoân coù baát ñaúng thöùc 2 2 22 3 33 ( )3() a b c ab bc ca + + ≥ ++ Lôøi giaûi. * Caùch 1. Khoâng maát tính toång quaùt, ta coù theå giaû söû min{ , , }. a abc = Ñaët , ( , 0) ba pca q pq =+ =+ ≥ . Khi ñoù, ta coù 2 2 22 3 33 2 2 2 3 2 23 4 3 224 ( )3() ()( ) ( 5 4) ( 3 2 )0 a b c ab bc ca fa p pq qa p pq pqq p pq pqq + + ≥ ++ ⇔ = - + - - + ++ + - + +≥ Ta coù 3 2 2 32 3( 2 )0 ()0 ñpcm. f p pq pqq fa Δ=- - - +≤ ⇒≥ ⇒ * Caùch 2. Ta coù Tìm tài liệu Toán ? Chuyện nh ỏ - www.toanmath.com396 2 2 223 3 3 2 22 2 2 22 3 33 1 ( ) 3( ) . ( 2 )0 2 ( ) 3( )() ñpcm cyc a b c a b b c c a a ab bc c ca a b c ab bc ca + + - + + = - + - +≥ ⇒ + + ≥ ++ ∑ * Caùch 3. Ta coù 2222333 2222 2 2 22 3 33 1 ( ) 3( ) . (2 3 3 )0 6 ( ) 3( )() ñpcm cyc a b c a b b c c a a b c bc ab a b c ab bc ca ++ - + + = --+ -≥ ⇒ + + ≥ ++ ∑ Ñaúng thöùc xaûy ra khi vaø chæ khi 222 42 : : sin : sin : sin 7 77 abc abc π ππ == ⎡ ⎢ ⎢ = ⎣ * Nhaän xeùt. Ñaây laø moät baát ñaúng thöùc maïnh vaø coù nhieàu öùng duïng. Sau ñaây laø moät soá öùng duïng cuûa noù + ÖÙng duïng 1. (Vasile Cirtoaje) Cho,,03 thoûa . Chöùng minh raèng abc abc > + += 3 1 1 12 abc ab bc ca + +≥ +++ Lôøi giaûi. Ta coù 11 cyc cyc aa a abc ab ab ⎛⎞ = - + ++ ⎜⎟ ++ ⎝⎠ ∑∑ 2 3 1 3 1 cyc cyc a a ab ab ab ⎛⎞ =+- ⎜⎟ + ⎝⎠ =- + ∑ ∑ 2 3 () 2 theo bñt AM-GM cyc ab ab ≥- ∑ Tìm tài liệu Toán ? Chuyện nh ỏ - www.toanmath.com397 3 2 12 1 3. 2 cyc ab =- ∑ Theo treân, ta coù 32122 1 .( )3 3 cyc a b abc ≤ ++= ∑ Do ñoù 32 12 1 13 3 . 3 .3 1 2 22 . ñpcm cyc cyc a ab ab ≥- ≥-= + ⇒ ∑∑ Ñaúng thöùc xaûy ra khi vaø chæ khi 1. abc === + ÖÙng duïng 2. Cho caùc soá khoâng aâm , ,, abcx thoûa 222 1 abc ++= . Chöùng minh raèng 222 3 1 1 13 abc xab xbc xca x ++≥ + + ++ Lôøi giaûi. AÙp duïng baát ñaúng thöùc AM-GM, ta coù 2 22 2 2 22 2 2 22 2 22 2 22 3 33 36 (1) 1 33 36 (1) 1 33 36 (1) 1 33 6 33 () 1 1 1 3 33 aa a xab xab x x bb b xbc xbc x x cc c xca xca xx a bc x ab bc ca xab xbc xca x x x ⎛⎞ ++≥ ⎜⎟ + ++ ⎝⎠ ⎛⎞ ++≥ ⎜⎟ + ++ ⎝⎠ ⎛⎞ ++≥ ⎜⎟ + ++ ⎝⎠ ⎛ ⎞ ⎛⎞ ⇒ + + ≥ - - ++ ⎜ ⎟ ⎜⎟ + + + + ++ ⎝ ⎠ ⎝⎠ Theo treân, ta coù 3 3 3 2 2 22 11 .() 33 ab bc ca a b c + + ≤ + += Do ñoù Tìm tài liệu Toán ? Chuyện nh ỏ - www.toanmath.com398 22 2 22 3 33 22 2 22 6 33 () 1 1 1 3 33 6 3 31 . 3 3 33 3 3 3 () 1 1 13 ñpcm abc x ab bc ca xab xbc xca x x x x x xx x abc xab xbc xca x ⎛ ⎞ ⎛⎞ + + ≥ - - ++ ⎜ ⎟ ⎜⎟ + + + + ++ ⎝ ⎠ ⎝⎠ ⎛⎞ ⎛⎞ ≥ -- ⎜⎟ ⎜⎟ + ++ ⎝⎠ ⎝⎠ = + ⇒ + +≥ + + ++ Ñaúng thöùc xaûy ra khi vaø chæ khi 1 . 3 abc = == Baøi toaùn 98. (Komal) Cho caùc soá döông,,1 thoûa . Chöùng minh raèng a b c abc = 2 2 2 2 22 111 3 1112 . a b c abc a b c abc ⎛⎞ + + - ≥ ++ ⎜⎟ ++ ++ ⎝⎠ Lôøi giaûi. Ta coù baát ñaúng thöùc caàn chöùng minh töông ñöông vôùi 2 2 2 2 22 222 32() abc a b b c c a ab bc ca abc abc ++ ++-≥ ++ ++ Do caû hai veá cuûa baát ñaúng thöùc naøy ñoàng baäc neân khoâng maát tính toång quaùt, ta coù theå giaû söû 1 abc ++= . Ñaët 1 , 0, 0. 3 q ab bc ca r abc q r = + + = ⇒ ≥≥≥ Khi ñoù, ta coù baát ñaúng thöùc caàn chöùng minh töông ñöông vôùi 2 2( 2) 3 12 (6 1) (1 4 ) 0 qr qr q rq qq - -≥ - ⇔ + + -≥ Neáu14q ≥ thì baát ñaúng thöùc treân hieån nhieân ñuùng. Neáu41 q ≥ thì theo baát ñaúng thöùc Schur, ta coù 41 0. 9 q r - ≥≥ Do ñoù Tìm tài liệu Toán ? Chuyện nh ỏ - www.toanmath.com399 (4 1)(6 1) (4 1)(1 3 ) (6 1) (1 4 ) (1 4 ) 0 99 ñpcm. q q qq rq qq qq - + -- + +- ≥ +- = ≥ ⇒ Baøi toaùn 99. (Nguyeãn Anh Cöôøng) Cho caùc soá döông ,, xyz thoûa 1. xyz + += Chöùng minh raèng 2 ( )( ) ()( ) ( )() y xz x y x z yz y z y x zx z x z y xy ++≥ + + + + + + + ++ Lôøi giaûi. Ta coù baát ñaúng thöùc caàn chöùng töông ñöông vôùi 2 ( )() 2 () 1 2 1 cyc cyc cyc x x y x z yz x x x y z yz yz yz yz xx ≥ + ++ ⇔≥ + + ++ ⇔≥ ++ ∑ ∑ ∑ Ñaët ,, yz zx xy m np x yz = == thì ta coù,,01 vaø . m n p mn np pm > + += Khi ñoù, ta coù baát ñaúng thöùc caàn chöùng minh töông ñöông vôùi 2 2 1 2 1 12 cyc cyc mm mm ≥ ++ ⇔ + -≥ ∑ ∑ Ñaët 2 22 1, 1,1 a mmb nnc p p = +- = +- = +- thì ta coù1 ,,0 abc >> vaø 2 2 1 2 1 2 a m a b n b - = - = Tìm tài liệu Toán ? Chuyện nh ỏ - www.toanmath.com400 2 22 22 1 2 (1 )(1) 1 4 (1 )(1 )4 ( ) ( ) ( )4 ( )(1 ) 0 (*) cyc cyc cyc c p c ab mn np pm ab c a b abc a b c ab a b abc ab bc ca abc a b c abc ab bc ca - = -- ⇒ = + += ⇔ - -= ⇔ ++ - + + + += ⇔ ++- - - -= ∑ ∑ ∑ Do1 ,,0 abc >> neân 0 a b c abc ++-> . Do ñoù (*)1 ab bc ca ⇔ + += Do ñoù, ñeå chöùng minh baát ñaúng thöùc ñaõ cho, ta chæ caàn chöùng minh 2 (**) abc ++≥ trong ñoù ,,0 abc > thoûa 1 ab bc ca + += . Ta coù ( ) 4 (**) 16( ) (***) a b c ab bc ca ⇔ + + ≥ ++ Do caû 2 veá cuûa baát ñaúng thöùc treân ñoàng baäc neân khoâng maát tính toång quaùt, ta coù theå giaû söû 1. abc + += Ñaët 1 , 0, 0. 3 q ab bc ca r abc q r = + + = ⇒ ≥≥≥ Khi ñoù, ta coù 2 (***) 1 16( 2 ) 32 (14)(14 ) 0 qr r qq ⇔≥- ⇔ +- +≥ Neáu14q ≥ thì baát ñaúng thöùc treân hieån nhieân ñuùng. Neáu41 q ≥ thì theo baát ñaúng thöùc Schur, ta coù 41 0. 9 q r - ≥≥ Do ñoù 32(4 1) (4 1)(23 36 ) 32 (1 4)(1 4) (1 4 )(1 4) 0 99 (***) ñuùng. q qq r q q qq - -- +- +≥ +- += ≥ ⇒ ⇒ ñpcm. Tìm tài liệu Toán ? Chuyện nh ỏ - www.toanmath.com401 Baøi toaùn 100. (Phaïm Kim Huøng, Voõ Quoác Baù Caån) Cho,,01 thoûa . abc abc ≥ ++= Tìm ñieàu kieän caàn vaø ñuû vôùi ,, abc ñeå baát ñaúng thöùc sau ñuùng 2 2 2 2 2 2 2 22 ( )(8 8 8 19 ) a b c a b b c c a abc ab bc ca ++ + + + ≥ ++ Lôøi giaûi. Ta seõ chöùng minh raèng ñieàu kieän caàn vaø ñuû ñeå baát ñaúng thöùc treân ñuùng laø , ab vaø c laø ñoä daøi ba caïnh cuûa moät tam giaùc (coù theå suy bieán). + Ñieàu kieän caàn. Giaû söû ,, abc khoâng laø ñoä daøi ba caïnh cuûa moät tam giaùc (coù theå suy bieán). Khi ñoù, cho 0,,0 c ab => thì baát ñaúng thöùc treân trôû thaønh 2 2 22 22 8() 8( ) 1 (*) a b a b ab a b ab +≥ ⇔ +≥ Cho 1,0 ab + =→ thì ta coù 22 0 lim8( )0 1 b abab + → + =< neân (*) khoâng ñuùng. Vaäy ta phaûi coù ,, abc laø ñoä daøi ba caïnh cuûa moät tam giaùc (coù theå suy bieán). + Ñieàu kieän ñuû. Giaû söû ,, abc laø ñoä daøi ba caïnh cuûa moät tam giaùc (coù theå suy bieán). Khi ñoù, ta seõ chöùng minh 2 2 2 2 2 2 2 22 ( )(8 8 8 19 ) a b c a b b c c a abc ab bc ca ++ + + + ≥ ++ Ñaët 1 , 0, 0. 3 q ab bc ca r abc q r = + + = ⇒ ≥≥≥ Do ,, abc laø ñoä daøi ba caïnh cuûa moät tam giaùc (coù theå suy bieán) neân ( )( )( )( ) 410 11 34 q abc abc bca c ab q -= + + + - + - + -> ⇒ ≥≥ Tìm tài liệu Toán ? Chuyện nh ỏ - www.toanmath.com402 Do ñoù, theo baát ñaúng thöùc Schur, ta coù 41 0. 9 q r - ≥≥ Baát ñaúng thöùc caàn chöùng minh töông ñöông vôùi 2 (1 2 )(19 8( 2 )) q r q rq - + -≥ 2 (1 2 )(3 8 ) (**) q r qq ⇔- +≥ Ta coù 22 41 (1 2)(3 8 ) (1 2) 8 3 (4 1)(1 3 )(4 1) 3 0 q qr q q q qq q qq -⎛⎞ - + -≥- +- ⎜⎟ ⎝⎠ - -+ = ≥ Do ñoù (**) ñuùng. Vaäy ñieàu kieän caàn vaø ñuû ñeå baát ñaúng thöùc ñaõ cho ñuùng laø ,, abc laø ñoä daøi ba caïnh cuûa moät tam giaùc (coù theå suy bieán). Baøi toaùn 101. (Titu Andreescu) Cho caùc soá thöïc , ab thoûa3( )21 a b ab + ≥+ . Chöùng minh raèng 3 3 33 9( )1 a b ab + ≥+ Lôøi giaûi. Ñaët , S a b P ab =+= thì töø giaû thieát, ta coù3 21 SP ≥+ (*). Baát ñaúng thöùc caàn chöùng minh trôû thaønh 22 9 ( 3 ) 1 ( 1) SS P P PP - ≥ + -+ Coù 2 tröôøng hôïp xaûy ra * Tröôøng hôïp 1. 7 3 5 7 35 . 22 PP -+ ≤ ∨≥ Khi ñoù, töø (*), ta coù 2 2 2 2 1 (4 19 4) 4( 1) 9(3)613 93 P PP P SSPPP ⎛⎞ + -+ + - ≥ + -= ⎜⎟ ⎝⎠ Tìm tài liệu Toán ? Chuyện nh ỏ - www.toanmath.com403 Do ñoù, ñeå chöùng minh baát ñaúng thöùc ñaõ cho, ta chæ caàn chöùng minh 22 2(4 19 4) 3( 1) 7 3 5 7 35 50 22 (ñuùng) P P PP PP - + ≥ -+ ⎛ ⎞⎛⎞ -+ ⇔- -≥ ⎜ ⎟⎜⎟ ⎝ ⎠⎝⎠ * Tröôøng hôïp 2. 7 35 7 35 22 P -+ ≤≤ . Töø ñaây, ta coù , ab cuøng daáu, töø ñoù töø giaû thieát, ta suy ra ñöôïc , 0. ab > Khi ñoù, (*) trôû thaønh 32 32(1)0. 2 S SPP - ≥ + ⇒ <≤ Baát ñaúng thöùc caàn chöùng minh trôû thaønh 23 33 9( 3)1 9 27 10 SS PP S PSP - ≥+ ⇔ - - -≥ + Tröôøng hôïp 2.1. 2 2 32 6 4 0. 42 SS SS - ≥ ⇔ - +≥ Khi ñoù, ta coù 3 3 33 2 27 (3 2) (3 2) 927191 28 45 ( 6 4) 8 0 SSS S PSPS SSS -- - --≥- -- -+ = ≥ + Tröôøng hôïp 2.2. ( ) ( ) 2 33 3 32 3 5 3 5. 42 SS S - ≤ ⇔ - ≤ ≤+ Khi ñoù, ta coù 36 3 33 27 927 191 4 64 SS S PSPS - - -≥ - -- ( ) ( ) 33 33 3 5 35 64 0 SS ⎛ ⎞⎛⎞ + - -- ⎜ ⎟⎜⎟ ⎝ ⎠⎝⎠ = ≥ Toùm laïi, trong moïi tröôøng hôïp, ta luoân coù 23 9( 3)1 SS PP - ≥+ (ñpcm) Ñaúng thöùc xaûy ra khi vaø chæ khi 3 535 3535 (,) , , ,. 22 22 ab ⎛ ⎞⎛⎞ - - ++ = ⎜ ⎟⎜⎟ ⎝ ⎠⎝⎠ Tìm tài liệu Toán ? Chuyện nh ỏ - www.toanmath.com404 Baøi toaùn 102. (Phaïm Kim Huøng) Cho caùc soá khoâng aâm , ,. abc Chöùng minh raèng 333 2 2 2 2 22 3 2 22 a b c abc ab bc c a ++ + +≥ + ++ Lôøi giaûi. Ta coù baát ñaúng thöùc caàn chöùng minh töông ñöông vôùi 2 22 2 2 2 2 22 2 22 .( ) .( ) .( ) 0 2 22 ba c b ac ab bc ca ab bc c a - -- - + - + -≥ + ++ Coù 2 tröôøng hôïp xaûy ra + Tröôøng hôïp 1. 0 abc ≥ ≥≥ . Khi ñoù, ta coù 2 2 22 2 2 22 2 2 22 4 0 22 22 0 22 4 22 0 22 bc a b ca aa a b ca b a ac a b ca -≥ ++ - +≥ ++ -- ⇒ +≥ ++ 22 2 2 22 422 .( ) .( ) 0 (1) 22 b a ac a b ca a b ca -- ⇒ - + -≥ ++ 22 2 2 22 (4 2) (2) 0 22 c bb a ca b c ca -- +≥ ++ 22 2 2 22 422 .( ) .( ) 0 (2) 22 c b ac b c ca b c ca -- ⇒ - + -≥ ++ Coäng caùc baát ñaúng thöùc (1) vaø (2) veá theo veá roâi chia caû hai veá cho 2, ta ñöôïc 2 22 2 2 2 2 22 2 22 .( ) .( ) .( ) 0 2 22 ba c b ac ab bc ca ab bc c a - -- - + - + -≥ + ++ + Tröôøng hôïp 2. 0. c ba ≥ ≥≥ + Tröôøng hôïp 2.1.2. b ca ≥+ Khi ñoù, ta seõ chöùng minh 2 2 22 2 4(2) 0 22 ba ac a b ca -- +≥ ++ Tìm tài liệu Toán ? Chuyện nh ỏ - www.toanmath.com405 Thaät vaäy, deã thaáy veá traùi laø haøm taêng cuûa c neân ta chæ caàn chöùng minh khi , cb = töùc laø chöùng minh 2 2 22 3 2 2 32 23 22 2 4(2 ) 0 22 4 2 2 16 8 8 40 3(5 2 2 )0 (ñuùng) b a ab a b ba bab ab a ab abb aa abb -- +≥ ++ ⇔ + - +-+ - ≥ ⇔ - +≥ Do ñoù 2 2 22 2 4(2) 0 22 ba ac a b ca -- +≥ ++ V aäy 2 22 2 2 2 2 22 22 2 2 22 2 22 .( ) .( ) .( ) 2 22 22 .( ) .() 4(2 )2 0 ba c b ac ab bc c a a b b c ca b a ac ca ca a b ca - -- -+-+-≥ + ++ -- ≥ -+- ++ ≥ + Tröôøng hôïp 2.2.2. b ca ≤+ Khi ñoù, ta seõ chöùng minh 2 2 22 2 63 0 (3) 22 ba ac a b ca -- +≥ ++ Thaät vaäy, deã thaáy veá traùi laø haøm taêng cuûa c neân chæ caàn chöùng khi 2 c ba =- . Baát ñaúng thöùc (3) trôû thaønh 2 2 22 3 2 23 2 96 0 2 838 10 15 2 15 0 (ñuùng) ba ab a b b a ab b ba aba -- +≥ + +- ⇔- + +≥ Tieáp theo, ta seõ chöùng minh 2 2 22 2 32 . 0 (4) 2 22 cb ac b c ca -- +≥ ++ Thaät vaäy, vì veá traùi laø haøm giaûm theo a neân ta chæ caàn chöùng minh khiab = , baát ñaúng thöùc trôû thaønh 2 2 22 3 2 23 42 63 0 22 5 2 2 100 (ñuùng) c b bc b c cb c cb bc b -- +≥ ++ ⇔ +-+≥ Tìm tài liệu Toán ? Chuyện nh ỏ - www.toanmath.com406 Neáu 2 ca ≤ thì ta coù baát ñaúng thöùc caàn chöùng minh ñuùng. Neáu 2 ca ≥ thì töø 2 baát ñaúng thöùc treân, vôùi chuù yù raèng 2 22 3 ( ) 3( ) .( ), 2 ca ba cb - ≤ - +- ta coù 2 22 2 2 2 2 22 22 2 2 2 2 2 2 22 2 22 .( ) .( ) .( ) 2 22 2 3(2 ) 2 32 .( ) . .() 2 2 2 22 0 b a c b ac ab bc ca ab bc ca ba a c cb ac b a cb ab ca bc c a --- - + - + -≥ + ++ - - -- ⎛ ⎞⎛⎞ ≥ + -+ +- ⎜ ⎟⎜⎟ + + ++ ⎝ ⎠⎝⎠ ≥ Toùm laïi, ta luoân coù 2 22 2 2 2 2 22 2 22 .( ) .( ) .( ) 0 2 22 (ñpcm) ba c b ac a b b c ca ab bc c a - -- -+-+-≥ + ++ Ñaúng thöùc xaûy ra khi vaø chæ khi . abc == Baøi toaùn 103. (Voõ Quoác Baù Caån) Cho n soá thöïc 12 , ,..., 0 n aaa > thoûa 11 1 . nn i ii i a a == = ∑∑ Chöùng minh raèng 2 11 38 nn ii ii aa == ≥+ ∑∑ Lôøi giaûi. Ta coù 11 2 1 1 1 0 nn i ii i n i i i a a a a == = = - ⇔= ∑∑ ∑ Baát ñaúng thöùc caàn chöùng minh töông ñöông vôùi 2 2 1 8( 1) 0 38 n i i ii a aa = - ≥ ++ ∑ Tìm tài liệu Toán ? Chuyện nh ỏ - www.toanmath.com407 2 1 2 1 0 8 31 i n i i i a a a = - ⇔≥ ++ ∑ Khoâng maát tính toång quaùt, ta coù theå giaû söû 12 ... 0. n aaa ≥ ≥ ≥> Khi ñoù, deã thaáy 2 22 12 12 2 22 12 1 11 ... 1 11 ... 8 88 31 31 31 n n n a aa a aa a aa ⎧ - -- ≥ ≥≥ ⎪ ⎪ ⎪ ⎨ ≥ ≥≥ ⎪ ⎪ ++ ++ ++ ⎪ ⎩ neân theo baát ñaúng thöùc Chebyshev, ta coù 2 2 1 11 22 1 1 11 .0 88 31 31 ñpcm. i n nn ii i ii i ii a aa na aa = == ⎛⎞ - ⎜⎟ ⎛⎞ - ⎜⎟ ≥= ⎜⎟ ⎜⎟ ⎝⎠ ++ ++ ⎜⎟ ⎜⎟ ⎝⎠ ⇒ ∑ ∑∑ Ñaúng thöùc xaûy ra khi vaø chæ khi 12 ... 1. n aaa == == Baøi toaùn 104. Cho caùc soá khoâng aâm , ,. abc Chöùng minh raèng 3 3 33 32 2 bc a b c abc a + ⎛⎞ ++- ≥- ⎜⎟ ⎝⎠ Lôøi giaûi. Ñaët 3 333 (,,) 32 2 bc f a b c a b c abc a + ⎛⎞ = + + - +- ⎜⎟ ⎝⎠ Khi ñoù, ta caàn chöùng minh (,,)0 f abc ≥ Tröôùc heát, ta chöùng minh Tìm tài liệu Toán ? Chuyện nh ỏ - www.toanmath.com408 ( , , ) , , (*) 22 b cbc f abc f a ++ ⎛⎞ ≥ ⎜⎟ ⎝⎠ Thaät vaäy 3 3 33 23 3 33 32 22 (*) 32 2 () 3.2 4 22 4 4 () 3(() 4) 0 44 3()( )3() 0 44 (ñuùng) bc a b c abc a b c bc bc a aa b c bc abc bc bc bc abc 3 + ⎛⎞ ⇔++- + -≥ ⎜⎟ ⎝⎠ + ++ ⎛ ⎞⎛⎞ ≥ + - +- ⎜ ⎟⎜⎟ ⎝ ⎠⎝⎠ + - + +- ⇔ +≥ - +- ⇔ +≥ Vaäy (*) ñuùng. Tieáp theo, ta seõ chöùng minh ( , , ) 0 (**) f att ≥ trong ñoù . 2 bc t + = Ta coù 33 23 23 2 (**) 2 3 2( ) 0 ()( 2)2() 0 3( )0 (ñuùng) a t at at a t a t at aat ⇔ + - + -≥ ⇔ - + + -≥ ⇔ -≥ Töø (*) vaø (**), ta suy ra ñieàu phaûi chöùng minh. Ñaúng thöùc xaûy ra khi vaø chæ khiabc == hoaëc 0,. a bc == Tìm tài liệu Toán ? Chuyện nh ỏ - www.toanmath.com409 Baøi toaùn 105. Chöùng minh raèng vôùi moïi soá döông ,, abc thì 3 33 22 2 2 22 2 3 a b c abc ab bc ca a b c abc a ab b b bc c c ca a ++ + +≥ + ++ ++ + +≥ + + ++ ++ a b) ) Lôøi giaûi. a) Ñaët 2 22 , , ( , , 0) axb y cz xyz == => . Baát ñaúng thöùc caàn chöùng minh trôû thaønh 2 22 2 2 2 2 22 2 2 2 22 2 2 2 2 22 4 22 2 22 2 2 22 2 2 24 () ( )() cyc cyc x y z x yz x y yz zx x y z x yz x y yz zx x xy xyz xy x y yz ++ + +≥ + ++ ⎛⎞ ++ ⎛⎞ ⎜⎟ ⇔ + +≥ ⎜⎟ ⎜⎟ ⎝⎠ + ++ ⎝⎠ ⇔ + ≥ ++ + ++ ∑∑ Löu yù raèng 44 2 2 22 0 cyc cyc xy xy xy -= ++ ∑∑ Do ñoù, baát ñaúng thöùc caàn chöùng minh töông ñöông vôùi 4 4 22 2 22 2 2 22 4 () ( )() cyc cyc x y xy x yz xy x y yz + + ≥ ++ + ++ ∑∑ Deã thaáy 2 2 2 2 22 2 2 2 2 22 ,, x y yz zx xy y z zx ⎛⎞ ⎜⎟ ⎜⎟ + ++ ⎝⎠ vaø 2 2 2 2 22 1 11 ,, xy y z zx ⎛⎞ ⎜⎟ ⎜⎟ + ++ ⎝⎠ laø 2 daõy ñôn ñieäu ngöôïc chieàu nhau neân theo baát ñaúng thöùc saép xeáp laïi, ta coù 22 22 22 2 2 22 ( )() cyc cyc xy xy xy x y yz ≥ + ++ ∑∑ Do ñoù, ñeå chöùng minh baát ñaúng thöùc ñaõ cho, ta chæ caàn chöùng minh Tìm tài liệu Toán ? Chuyện nh ỏ - www.toanmath.com410 4 4 22 2 2 2 22 22 2 22 4 () 2 2() cyc cyc cyc cyc x y xy xyz xy xy xy x xy yz zx xy + + ≥ ++ ++ ⇔ + ≥ ++ + ∑∑ ∑∑ 2 2 22 1 () . () 2 cyc cyc xyxy xy xy - ⇔ -≥ + ∑∑ 4 22 () 0 (ñuùng) cyc xy xy - ⇔≥ + ∑ ñpcm. ⇒ Ñaúng thöùc xaûy ra khi vaø chæ khi . abc == b) Ñaët 2 22 , , ( , , 0) axb y cz xyz == => . Baát ñaúng thöùc caàn chöùng minh trôû thaønh 3 33 422 4 4 22 4 4 224 2 2 3 33 422 4 4 224 4224 6 33 4 224 4 224 422 4 2 22 3 3 2 ( )() 222 3 cyc cyc x y z x yz xxy y y yz z z zxx x y z x yz xxy y y yz z z zxx x xy x xyy x xyy yyz z x y z xy yz zx ++ + +≥ + + + + ++ ⎛⎞ ++ ⎛⎞ ⎜⎟ ⇔ + +≥ ⎜⎟ ⎜⎟ ⎝⎠ + + + + ++ ⎝⎠ ⇔+≥ ++ + + ++ + + +++ ≥ ∑∑ Löu yù raèng 66 4 224 4 224 0 cyc cyc xy x xyy x xyy -= ++ ++ ∑∑ Do ñoù, baát ñaúng thöùc caàn chöùng minh töông ñöông vôùi 6 6 33 4 224 4 224 422 4 2 22 1 .2 2 ( )() 222 3 cyc cyc x y xy x xyy x xyy yyz z x y z xy yz zx + +≥ ++ + + ++ + + +++ ≥ ∑∑ Tìm tài liệu Toán ? Chuyện nh ỏ - www.toanmath.com411 33 66 22 4 224 4 224 422 4 1 3() 6 .4 2 ( )() cyc cyc xy xy x y xy x xyy x xyy yyz z ⎛⎞ + ⇔ ≥ + +- ⎜⎟ ++ + + ++⎝⎠ ∑∑ 33 33 4 2 4 224 4 224 422 4 6 6 ( )() ( )() cyc cyc xy xy x y xy x xyy x xyy yyz z --+ ⇔≥ ++ + + ++ ∑∑ Maët khaùc, deã thaáy 3 3 3 3 33 4 22 4 4 22 4 4224 ,, ( )( )( ) x y yz zx x xy y y yz z z zx z ⎛⎞ ⎜⎟ ⎜⎟ + + + + ++ ⎝⎠ vaø 4 22 4 4 22 4 4224 1 11 ,, ( )( )( ) x xy y y yz z z zx z ⎛⎞ ⎜⎟ ⎜⎟ + + + + ++ ⎝⎠ laø hai daõy ñôn ñieäu ngöôïc chieàu nhau neân theo baát ñaúng thöùc saép xeáp laïi, ta coù 3 3 33 4 224 4 224 422 4 ( )() cyc cyc xy xy x xyy x xyy yyz z ≥ ++ + + ++ ∑∑ Töø ñaây, ta suy ra ñieàu phaûi chöùng minh. Ñaúng thöùc xaûy ra khi vaø chæ khi . abc == Baøi toaùn 106. (Phan Thaønh Vieät) Cho caùc soá khoâng aâm , ,. abc Chöùng minh raèng 4 4 4 3 33 222 222 a b c abc abc a ab b b bc c c ca a ++ + +≥ ++ + + ++ ++ Lôøi giaûi. Ta coù 2 333 2 ()3 3 cyc cyc cyc cyc a b c a ab abc a b c abc a ab abc abc abc ⎛⎞ ++ -+ ⎜⎟ ⎜⎟ ++ ⎝⎠ = = +- ++ ++ ++ ∑∑ ∑∑ Do ñoù, baát ñaúng thöùc caàn chöùng minh töông ñöông vôùi 4 2 22 3 cyc cyc cyc a abc a ab abc a abb - +≥ ++ ++ ∑ ∑∑ Tìm tài liệu Toán ? Chuyện nh ỏ - www.toanmath.com412 4 2 22 3 22 3 3 cyc cyc a abc a ab abc a abb ab abc abc a abb ⎛⎞ ⇔ - +≥ ⎜⎟ ++ ++ ⎝⎠ ⇔≥ ++ ++ ∑ ∑ AÙp duïng baát ñaúng thöùc Bunhiacopxki, ta coù 3 22 2 22 () cyc cyc ab a ab b abc ab a abb ⎛⎞⎛⎞ ++ ≥ ++ ⎜⎟⎜⎟ ⎜⎟⎜⎟ ++ ⎝⎠⎝⎠ ∑∑ 32 2 2 22 () 3 cyc cyc ab abc a abb ab ab ++ ⇒≥ +++ + ∑ ∑ Do ñoù, ñeå chöùng minh baát ñaúng thöùc ñaõ cho, ta chæ caàn chöùng minh 2 22 3 22 3 33 ( )3 3 ( )9 3 () 3 (ñuùng theo bñt AM-GM) ñpcm. cyc cyc a b c abc abc ab ab abc abc cab a b c abc ++ ≥ ++ + + ⇔++≥ ++ ⇔ ++≥ ⇒ ∑ ∑ Ñaúng thöùc xaûy ra khi vaø chæ khi . abc == Baøi toaùn 107. Chöùng minh raèng vôùi moïi soá döông ,, abc thoûa 1 abc = ta coù baát ñaúng thöùc 2 22 1 (2 )( 1 ) (2 )(1 ) (2 )(1 ) 2 abc a b ab b c bc c a ca + +≥ + + + + ++ Lôøi giaûi. Do ,,0 1 abc abc > ⎧ ⎨ = ⎩ neân toàn taïi caùc soá döông ,, xyz sao cho , xy ab yz == vaø . z c x = Khi ñoù, baát ñaúng thöùc caàn chöùng minh trôû thaønh Tìm tài liệu Toán ? Chuyện nh ỏ - www.toanmath.com413 22 2 1 2 ( 2)() cyc xy zz xy xy ≥ ++ ∑ AÙp duïng baát ñaúng thöùc Bunhiacopxki, ta coù 22 44 22 2 2 2 2 2 22 2 2 2 2 2 2 22 22 2 2 2 2 2 22 2 2 2 2 22 2 2 22 1 . ( 2)( ) ( 2)( ) 1() . ( 2)() 1 () . 2 ( )2 () () 2( )() cyc cyc cyc cyc xy xy xyz zz xy xy xyz xy xy x y yz zx xyz xy z xy x y x y yz zx xyz xyz xy yz zx x y x y x y yz zx xyzx y yz zx x y z x y yz = ++ ++ ++ ≥ ++ ++ = ++++ ++ = + + ++ + = ∑∑ ∑ ∑ 22 2() 1 2 ñpcm. zx xyzxyz + ++ ≥ ⇒ Baøi toaùn 108. (Vasile Cirtoaje) Cho caùc soá thöïc , ,. abc Chöùng minh raèng 2 2 2 2 22 3(1 )(1 )(1 ) 1 a a b b cc abcabc -+ -+ -+ ≥++ Lôøi giaûi. Söû duïng ñaúng thöùc 2 2 22 2 22 2(1 )(1 ) 1 ( ) (1 ) (1 ) aabb ab ab ab -+-+ =+ +- +-- Ta coù 2 2 22 2(1 )(1 ) 1 a a b b ab - + - + ≥+ Do ñoù, ñeå chöùng minh baát ñaúng thöùc ñaõ cho, ta chæ caàn chöùng minh 22 2 22 2 22 2 22 22 3(1 )(1 ) 2(1 ) ( ) (3 ) (3 2 3 ) 1 3 0 ab c c abc abc f c ab c ab ab c ab + -+ ≥ ++ ⇔ =+ -+ + ++≥ Tìm tài liệu Toán ? Chuyện nh ỏ - www.toanmath.com414 Ta coù 4 3(1 )0 f ab Δ=- -≤ Do ñoù ()0 fc ≥ Ñaúng thöùc xaûy ra khi vaø chæ khi 1. abc === Baøi toaùn 109. (Vasile Cirtoaje) Chöùng minh raèng vôùi moïi soá khoâng aâm , ,, abcd thoûa 2 222 a ab b c cdd -+ = -+ ta coù baát ñaúng thöùc ( )( ) 2( ) a b c d ab cd + + ≥+ Lôøi giaûi. Ñaët ( , , , ) ( )( ) 2( ) f a b c d a b c d ab cd = + + -+ Khoâng maát tính toång quaùt, ta coù theå giaû söû 0 c d ab + ≥ +≥ . Khi ñoù, ta coù ( ) ( ) 2222 2 22 2 22 ( ,, , ) ,, , ( ) 2 2() 3() ( )2 2 00 (do ) fabcd fabccdd ccdd abcd ccdd cd ab cd cd ccdd cdab - -+ -+ = =+ +- -++- ⎛⎞ + =-- ⎜⎟ ++ -+ ⎝⎠ ≥ +≥+≥ Do ñoù ( ) ( ) 2 222 2 222 ( ,, , ) ,, , , , , (1) f abcd f ab c cd d c cd d f ab a abb a abb ≥ -+ -+ = - + -+ Tieáp theo, ta seõ chöùng minh ( ) 2222 , , , 0 (2) f ab a abb a abb -+ -+ ≥ Thaät vaäy 2 2 22 (2) 2( ) 2( ) ab a abb ab ⇔ + - + ≥+ Tìm tài liệu Toán ? Chuyện nh ỏ - www.toanmath.com415 2 2 2 2 22 2 ( )( )() ( )0 (ñuùng) ab a abb ab abab ⇔ + - + ≥+ ⇔ -≥ Töø (1) vaø (2), ta suy ra ñieàu phaûi chöùng minh. Baøi toaùn 110. (Phaïm Kim Huøng) Cho caùc soá döông , ,. abc Chöùng minh raèng 2 22 2 2 2 333 12( ) b c c abc aba cab abc ⎛ ⎞⎛ ⎞⎛ ⎞ ++ + + + + +≥ ⎜ ⎟⎜ ⎟⎜ ⎟ ++ ⎝ ⎠⎝ ⎠⎝ ⎠ . Lôøi giaûi. Ta coù 2 22 2 2 2 3 33 2 4 3 33 2 2 24 22 2 3 33 2 2 22 12( ) 12( ) 2 2 22 12( ) 2 4() cyc cyc cyc cyc cyc cyc cyc b c c abc a ba cab abc ab a abc a c abc b aba ac ab b a c b abc ab a abc abc ⎛ ⎞⎛ ⎞⎛ ⎞ ++ + + + + +≥ ⎜ ⎟⎜ ⎟⎜ ⎟ ++ ⎝ ⎠⎝ ⎠⎝ ⎠ ++ ⇔ + +≥ ++ ⎛ ⎞⎛⎞ ⇔ + - + + -+ ⎜ ⎟⎜⎟ ⎝ ⎠⎝⎠ ⎛⎞ ++ + - ≥ - ++ ⎜⎟ ⎜⎟ ++ ⎝⎠ ⇔ ∑ ∑∑ ∑∑ ∑∑ 2 2 2 4 2() ( ) 40 cyc b a ab bc abcc c ⎛⎞ + - + + -≥ ⎜⎟ ++ ⎝⎠ ∑ Ñaët 2 2 2 2 2 2 4 2() 4 4 2() 4 4 2() 4 a b c b a ab S abcc c c b bc S abca a a c ca S abcb b + =+ +- ++ + =+ +- ++ + =+ +- ++ Baát ñaúng thöùc caàn chöùng minh töông ñöông vôùi 222 ( )( )( ) 0 a bc S b c Sc a Sa b - + -+ -≥ Tìm tài liệu Toán ? Chuyện nh ỏ - www.toanmath.com416 Coù 2 tröôøng hôïp xaûy ra + Tröôøng hôïp 1. 0. c ba ≥ ≥> Khi ñoù, ta coù 0. b S ≥ Ta coù 22 22 4( ) 2( ) 2( ) 80 ab b c ab a b bc SS abc c a ca + ++ + = + + + + -≥ ++ Vì 22 22 2 22 2, 4, 2 b c a cb c aa ca + ≥+ ≥≥ 22 22 4( ) 2( ) 2( ) 80 cb a c b c ac bc SS abc b a ba + ++ + = + + + + -≥ ++ Vì 22 22 2 22 2, 4, 2 a c a bc b aa ba + ≥ + ≥≥ Do ñoù 2 2 2 22 ( ) ( ) ( )( )( ) ( )( )0 a b c ab cb S b c S c a S ab S S bc S S ab - + -+ -≥ + - ++ -≥ + Tröôøng hôïp 2. 0. abc ≥≥> Khi ñoù, ta coù 1, 1. ac SS ≥ ≥- Ta coù 22 22 2 8 4 2( ) 4() 2 120 ab b c b a ab bc SS abc c a ca + ++ + = + + + + -≥ ++ Vì 4 8 2 2 22 4, 4,4 a b a b ca abc c a ac + ≥ + ≥ +≥ ++ 22 22 22 22 4 16 4 2( ) 8( ) 4 20 4 8 4 2( ) 8( ) 16 () ab b c b a a b bc SS abc ca ca b c b a ab bc fb abc c a ca + ++ + =+ + + +- ++ + ++ ≥ + + + + -= ++ Deã daøng kieåm tra () fb laø haøm ñoàng bieán. Do ñoù 2 2 4 162 () () 9 2 32 91 c ca f b fc ac a ≥ = + + - ≥ -> + Khaû naêng 2.1. 2 2( ) 0 0. a c b bc ac bc ab +≤ ⇔ - ≥ - ≥ ∧ - ≥ -≥ Neáu 0 b S ≥ thì ta coù ngay ñpcm. Neáu 0 b S ≤ , thì Tìm tài liệu Toán ? Chuyện nh ỏ - www.toanmath.com417 2 2 22 () ( ) ( ) ( 4 1)() 0 a b c ab S b c S c a S ab S S bc - + - + - ≥ +-- ≥ + Khaû naêng 2.2. 2. acb +≥ . Khi ñoù, ta seõ chöùng minh 2 0. cb SS +≥ Thaät vaäy, ta coù 22 22 2 8 4 2( ) 4( ) 2 12 () cb a c b c a c bc S S gc abc ba ba + ++ + = + + + + -= ++ + Khaû naêng 2.2.1. 2 ab ≥ . Khi ñoù, do () gc laø haøm taêng neân 2 2 8 42 ( ) (0) 12 0 a b ba gcg ab ab b ≥ =+ ++ - ≥ + Vì 2 2 941 5, 4, 6, 3 a b ab aab b ab b a b ab b - + ≥ + ≥ + ≥ ≥- ++ + Khaû naêng 2.2.2. 2 ab ≤ . Khi ñoù, do () gc laø haøm taêng neân 22 22 8 4 4 14 () (2 ) 02 33 (do ) a b ba gc g b a bab ab ba ≥ - = + + - - ≥ ≥≥ Vaäy 2 2 2 22 () ( ) ( ) ( 2)() (2)( ) 0 a b c ab cb S b c S c a S ab S S bc S S ab - + -+ -≥ + - ++ -≥ Toùm laïi, trong moïi tröôøng hôïp, ta luoân coù 222 ( ) ( ) ( )0 (ñpcm) a bc S b c Sc a Sa b - + - + -≥ Ñaúng thöùc xaûy ra khi vaø chæ khi . abc == Baøi toaùn 111. Chöùng minh raèng vôùi moïi soá döông , ,, abcd ta coù baát ñaúng thöùc () ( ) () ( ) 4 ( ) ( ) ( ) () ba c cb d d c a ad b ca b d b c ac d bd a + +++ + + +≥ + + ++ Lôøi giaûi. Ta coù baát ñaúng thöùc caàn chöùng minh töông ñöông vôùi Tìm tài liệu Toán ? Chuyện nh ỏ - www.toanmath.com418 () ()4 ( ) ( ) ( ) () ( )4 ( )( ) ( )() 1 1 11 1 1 11 4 1 1 1 1 1 1 11 b d ca ac bd ca b ac d d b c bd a a c bd abc abd acd bcd acabc d bdbcd a ac bd abcd a b cd bc d a ⎛ ⎞⎛⎞ + + ++ +≥ ⎜ ⎟⎜⎟ + + ++ ⎝ ⎠⎝⎠ ⎛⎞ ++ ⇔+++ +≥ ⎜⎟ + + ++ ⎝⎠ ⎛⎞ ++ ⎜⎟ ⎛⎞ ⎜⎟ ⇔+++ +≥ ⎜⎟ ⎛ ⎞⎛ ⎞ ⎛ ⎞⎛ ⎞ ⎜⎟ ⎝⎠ + + ++ ⎜ ⎟⎜ ⎟ ⎜ ⎟⎜ ⎟ ⎜⎟ ⎝ ⎠⎝ ⎠ ⎝ ⎠⎝ ⎠ ⎝⎠ AÙp duïng baát ñaúng thöùc AM-GM, ta coù 22 1 1 11 1 1 11 1 1 1 1 1 1 11 1 1 11 1 1 11 4 11 11 11 11 4 ac bd a b cd ab c d bc d a ac bd a b cd abc d ab c d ⎛⎞ ++ ⎜⎟ ⎛⎞ ⎜⎟ +++ +≥ ⎜⎟ ⎛ ⎞⎛ ⎞ ⎛ ⎞⎛ ⎞ ⎜⎟ ⎝⎠ + + ++ ⎜ ⎟⎜ ⎟ ⎜ ⎟⎜ ⎟ ⎜⎟ ⎝ ⎠⎝ ⎠ ⎝ ⎠⎝ ⎠ ⎝⎠ ⎛⎞ ⎜⎟ ++ ⎛⎞ ⎜⎟ ≥++++ ⎜⎟ ⎜⎟ ⎝⎠ ⎛⎞⎛⎞ + + + + ++ ⎜⎟ ⎜⎟⎜⎟ ⎝⎠⎝⎠ ⎝⎠ = ⇒ ñpcm. Baøi toaùn 112. (Voõ Quoác Baù Caån) Chöùng minh raèng vôùi moïi soá thöïc döông ,, abc ta coù baát ñaúng thöùc 111 222 333 333 2 2 2 2 22 a b c a b b c c a ab bc c a +++ ++ ≥ ++ + ++ + ++ Lôøi giaûi. Tröôùc heát, ta chöùng minh Boå ñeà sau Boå ñeà. Vôùi moïi soá thöïc döông ,, xyz ta coù baát ñaúng thöùc 3 3 3 2 22 2 22 2( )3() x y z xy yz zx x y y z z x + + + + + ≥ ++ Chöùng minh. Ta coù Tìm tài liệu Toán ? Chuyện nh ỏ - www.toanmath.com419 3 3 3 2 22 2 22 3 3 3 2 22 2 22 3 3 2 22 2 23 2 2( )3() 3( ) 6( ) 9( ) (23)60 ( ) (2 ) 6( )( )( ) 0 ()(2)2 ( ) 0 ( )( cyc cyc cyc cyc cyc cyc x y z xy yz zx x y yz zx x y z xy yz zx x y y z z x x y x y xy xy xy x y xy yz zx xy x y xy xy + + + + + ≥ ++ ⇔ + + + + + ≥ ++ ⎛⎞ ⇔ +- + -≥ ⎜⎟ ⎜⎟ ⎝⎠ ⇔ - + + - - -≥ ⇔ - + + -≥ ⇔- ∑ ∑∑ ∑ ∑∑ 4 )0 cyc xy -≥ ∑ Ñaët 4 , 4 ,4 x yz S y zS z xS xy = - = - =- Baát ñaúng thöùc caàn chöùng minh trôû thaønh 222 ( ) ( ) ( )0 x yz S yz S zx S xy - + - + -≥ Coù 2 tröôøng hôïp xaûy ra * Tröôøng hôïp 1. . x yz ≤≤ Khi ñoù, ta coù 0 y S ≥ . Ta laïi coù 4 30 340 yz yx S S z yx S S z yx +=- +≥ + = + -≥ Chuù yù raèng 2 22 ( ) ( ) () z x x y yz - ≥ - +- neân ta coù 2 2 2 22 ( ) ( ) ( )( )( ) ( )( )0 x y z x y yz S yz S zx S xy S S yz S S xy - + - + - ≥ + - ++ -≥ * Tröôøng hôïp 2. , 0. xz x y z SS ≥≥⇒≥ Neáu 0 y S ≥ thì ta coù ngay ñpcm, do ñoù ta chæ caàn xeùt tröôøng hôïp 0 y S ≤ laø ñuû. + Tröôøng hôïp 2.1.2 2. yx z yx ≥ +⇒≥ Khi ñoù, ta coù 2 4 2 70 2 2 80 xy zy S S yxz S S x yz + = -+≥ + = - +≥ Maët khaùc theo baát ñaúng thöùc Bunhiacopxki thì 2 22 ( ) 2( ) 2( ) z x x y yz - ≤ - +- . Do ñoù 2 2 2 22 ( ) ( ) ( )( 2 )( ) (2 )( )0 x y z x y yz S yz S zx S xy S S yz S S xy - + - + -≥ + - + + -≥ + Tröôøng hôïp 2.2. 2 2( ) 0. x z y x y xz + ≥ ⇔ - ≥ -≥ Tìm tài liệu Toán ? Chuyện nh ỏ - www.toanmath.com420 + Tröôøng hôïp 2.2.1. ( ) 3 1 3 3( ) 0. xz y x y xz - +≥ ⇔ - ≥ -≥ Khi ñoù, ta coù 3 120 zy S S xyz + =-+ ≥ Do ñoù 2 2 22 ( ) ( ) ( ) (3 )( ) 0 x y z yz S yz S zx S xy S S xy - + - + -≥ + -≥ + Tröôøng hôïp 2.2.2. ( ) ( ) 31 3 31( ) 0. x z y y z xy - +≤ ⇔ -≥ - - ≥ Khi ñoù, ta coù ( ) ( ) ( ) 2 31 4 1583 26 30 x yz S SS yz - + += - + +≥ Do ñoù ( ) 2 2222 ( ) ( ) ( ) 3 1 4 ( )0 x y z x yz S y z S z x S xy S S S xy ⎛⎞ - + - + -≥ - + + -≥ ⎜⎟ ⎝⎠ Toùm laïi, trong moïi tröôøng hôïp, ta luoân coù 222 ( ) ( ) ( )0 x yz S yz S zx S xy - + - + -≥ Boå ñeà ñöôïc chöùng minh hoaøn toaøn. Ñaúng thöùc xaûy ra khi vaø chæ khi . xyz == Trôû laïi baøi toaùn cuûa ta AÙp duïng Boå ñeà treân vôùi , , ( 0) a bc x t yt zt t = = => , ta ñöôïc 3 3 3 22 2 22 2 3 1 31 3 1 2 1 21 2 1 2 12 12 1 1 31 3 1 3 1 21 2 1 21 0 2 12 2( )3( )0 2() 3( )0 ( 2( )) 3( dt a b c ab bc c a ab bc ca a b c abbcca a b bc ca a b c a b b c ca abb t t t t tt t tt t t t t t tt t t tt t t t ttt tt + ++ + ++ - - - +- +- + - + - + - +- - - - + - + - +- +-+ + + + ++ ≥ ++ ∀> ⇒ + + + ++ ≥ + + ∀> ⇒ + + + ++ ≥+ ∫ 1 1 21 0 )dt c ca t - +- + ∫ Tìm tài liệu Toán ? Chuyện nh ỏ - www.toanmath.com421 111 222 333 3332 2 2 2 22 ñpcm. a b c a b b c c a ab bc c a ⇔+++ ++ ≥ ++ + ++ + ++ ⇒ Ñaúng thöùc xaûy ra khi vaø chæ khi . abc == Baøi toaùn 113. Chöùng minh raèng vôùi moïi soá thöïc döông , ,, abcd ta coù baát ñaúng thöùc 0 2 2 22 ab bc c d d a a bc b c d c d a d ab - - -- + + +≥ + + + + + + ++ Lôøi giaûi. Ta coù baát ñaúng thöùc caàn chöùng minh töông ñöông vôùi 2 22 222 2 2 0 2 2 22 2 2 22 11 22 2 2 22 1 14 22 3 3 33 4 2 2 22 2 2 22 ab bc c d d a abc bcd c d a d ab ab bc abc bcd c d da c d a d ab a c bd ca d b a b c b c d c d a d ab a bc a b c b c dc - - -- + + +≥ + + + + + + ++ -- ⎛ ⎞⎛⎞ ⇔ ++ ++ ⎜ ⎟⎜⎟ + + ++ ⎝ ⎠⎝⎠ -- ⎛ ⎞⎛⎞ + + + +≥ ⎜ ⎟⎜⎟ + + ++ ⎝ ⎠⎝⎠ + + ++ ⇔++ + ≥ + + + + + + ++ ⇔ ++ ++ +++ 2 4 2 2 22 d d a d ab a c bd ca d b a b c b c d c d a d ab ⎛⎞ ++ ⎜⎟ + ++ ⎝⎠ + + ++ ⎛⎞ + + + +≥ ⎜⎟ + + + + + + ++ ⎝⎠ AÙp duïng baát ñaúng thöùc Bunhiacopxki, ta coù 2 2 22 2 2 2 2 2 22 ( 2 ) ( 2 )( 2 ) (2 ) () ( 2 ) ( 2 )(2 ) ( 2 ) () () 1 a bcd abc bcd c d a d ab a b cd aa bc bb c d cc d a dd ab abcd aabc bbcd cc d a dd ab abcd abcd + + += + + + + + + ++ = + ++ + + + + + + ++ + ++ ≥ + + + +++ + + + + + + ++ = + ++ = Tìm tài liệu Toán ? Chuyện nh ỏ - www.toanmath.com422 Do ñoù 2 2 (1) 2 2 22 a bcd a bc b c d c d a d ab ⎛⎞ + + +≥ ⎜⎟ + + + + + + ++ ⎝⎠ Maët khaùc, aùp duïng baát ñaúng thöùc AM-GM, ta laïi coù 2 2 22 1 1 11 ( ) () 2 2 22 a c bd ca d b a bc b c d c d a d ab a c bd a bc c d a b c d d ab + + ++ + + += + + + + + + ++ ⎛ ⎞⎛⎞ =+ + +++ ⎜ ⎟⎜⎟ + + + + + + ++ ⎝ ⎠⎝⎠ 44 ( ). ( ). ( 2 )( 2 ) ( 2 )( 2) 2 (2) ac bd a b c c d a b c d d ab ≥ + ++ + + + + + +++ + + = Töø (1) vaø (2), ta suy ra ñieàu phaûi chöùng minh. Ñaúng thöùc xaûy ra khi vaø chæ khi ,. acbd == Baøi toaùn 114. Cho caùc soá döông , ,. abc Chöùng minh raèng 2 22 2 22 222 1 abc a ab b b bc c c ca a + +≥ + + + + ++ Lôøi giaûi. Do caû 2 veá cuûa baát ñaúng thöùc ñaõ cho ñoàng baäc neân khoâng maát tính toång quaùt, ta coù theå giaû söû 1 abc = . Ñaët ,, y zx abc x yz = == ( , , 0) xyz > . Khi ñoù, baát ñaúng thöùc caàn chöùng minh trôû thaønh 4 4 2 22 1 cyc x x x yz yz ≥ ++ ∑ AÙp duïng baát ñaúng thöùc Bunhiacopxki, ta coù 4 2 2 22 4 2 22 4 44 22 2222 () () cyc x x yz x xyz yz x y z x y yz zx xyzx y z ++ ≥ + + + + + + + + ++ ∑ Do ñoù, ñeå chöùng minh baát ñaúng thöùc ñaõ cho, ta chæ caàn chöùng minh Tìm tài liệu Toán ? Chuyện nh ỏ - www.toanmath.com423 22 22 44 4 2222 22 22 ( ) () 1 . ( )0 2 cyc xyz x y zxy yz zx xyzx y z z xy + + ≥++ + + + + + + ⇔ -≥ ∑ (ñuùng) ⇒ ñpcm. Ñaúng thöùc xaûy ra khi vaø chæ khi . abc == Baøi toaùn 115. Chöùng minh raèng vôùi moïi soá thöïc döông ,, abc ta coù baát ñaúng thöùc 222 2 2 2 2 22 22 2 0 7 2 2 7 2 2 7 22 a bc b ca c ab ab c b c a c ab --- + +≥ + + + + ++ Lôøi giaûi. Ta coù baát ñaúng thöùc caàn chöùng minh töông ñöông vôùi 2 222 2 22 22 0 7 22 ( )( ) ( )( ) 0 7 22 cyc cyc a bc abc a ba c ca ab abc - ≥ ++ - +--+ ⇔≥ ++ ∑ ∑ 2 22 2 22 2 22 22 2 2 ( )( ) ( )( ) 0 7 2 2 7 22 ()0 7 22 72 2 ( )0 cyc cyc cyc c cyc a ba c ca ab abc abc a c bc ab a bc bc a S ab - + -+ ⇔ -≥ ++ ++ ⎛⎞ ++ ⇔- -≥ ⎜⎟ + + ++ ⎝⎠ ⇔ -≥ ∑∑ ∑ ∑ Trong ñoù 22 3 22 2 2 2 2 22 2 2 22 2 2 3 2 2 2 2 22 2 2 2 2 22 22 (2( )( ) 4 4 ( ) 3 ( ) 10 ) 2 2 7 ( )7 2 2 ( )7 2 2 (2( )( ) 4 4 ( ) 3 ( ) 10 ) 2 2 7 ( )72 2 ( )7 22 (2( )( ) a b c bcb c aab c abc abc bc a S ab cab ac bca c a c a b b c a b c a abc c a b S bc a bc ab cab abab S + ++ + +- + - + + = + + + + + ++ + + + + + - + - ++ = + + ++ + ++ + ++ = 3 2 2 2 2 22 22 2 2 22 4 4( )3( ) 10 ) 2 2 7 ( )72 2 ()7 22 c ca b cab abc abc ac bc a bc abc + + - + - ++ + + + + + ++ Tìm tài liệu Toán ? Chuyện nh ỏ - www.toanmath.com424 AÙp duïng baát ñaúng thöùc AM-GM vaø baát ñaúng thöùc Bunhiacopxki, ta coù 22 3 22 2 3 22 32 2 2 2( )( ) 4 4 ( ) 3 ( ) 10 8 4 8 6 10 2 2 22 ( 2)4 52 22 0 aba b c ca b c ab abc ab ab ab ab cc cc a b ab abc cc + ++ + +- + - ≥ + + ++ ⎛⎞ ⎛ ⎞ ⎛⎞⎛⎞ ≥ ++-- ⎜⎟ ⎜ ⎟ ⎜⎟⎜⎟ ⎝⎠ ⎝ ⎠ ⎝⎠⎝⎠ ⎛⎞ ++ ⎛⎞⎛⎞ =++ -+ ⎜⎟ ⎜⎟⎜⎟ ⎜⎟ ⎝⎠⎝⎠ ⎝⎠ ≥ Do ñoù 0 c S ≥ . Töông töï, ta coù , 0. ab SS ≥ Vaäy 222 ( ) ( ) ( )0 (ñpcm) a bc S b c Sc a Sa b - + - + -≥ Baøi toaùn 116. (Voõ Quoác Baù Caån) Chöùng minh raèng vôùi moïi soá thöïc döông ,, abc ta coù baát ñaúng thöùc 2 2 22 ( ) 9() abc abc abc bca ⎛⎞ + + + + ≥ ++ ⎜⎟ ⎝⎠ Lôøi giaûi. Ta coù baát ñaúng thöùc caàn chöùng minh töông ñöông vôùi 322 22 3 22 2 3 2 29 2 73 cyc cyc cyc cyc cyc cyc cyc cyc cyc cyc cyc a a b ab abaa bcc a a b ab a ab b cc + + + +≥ ⇔ + + ≥- ∑ ∑ ∑ ∑ ∑∑ ∑ ∑ ∑ ∑∑ 32 2 2 2 2 22 2 2 22 2 2 55 ( ) ( ) ( )5 2 . () 2 ( )0 cyc cyc cyc cyc cyc cyc cyc cyc cyc a cyc a ab ab a bc ab bc ab ac ab a ab c aab bc a abc ab b cc S bc ⎛ ⎞⎛⎞ ⇔ + - + + -+ ⎜ ⎟⎜⎟ ⎝ ⎠⎝⎠ ⎛⎞ + +- ≥- ⎜⎟ ⎝⎠ - -- ⇔ + + ≥- ⇔ -≥ ∑∑ ∑ ∑∑ ∑ ∑ ∑∑ ∑ Tìm tài liệu Toán ? Chuyện nh ỏ - www.toanmath.com425 Trong ñoù 25 2 25 2 25 2 a b c b aa S c bc c bb S a ca a cc S b ab =++ - = + +- = + +- Coù 2 tröôøng hôïp xaûy ra + Tröôøng hôïp 1. 0. abc ≥≥> Khi ñoù, ta coù 0. a S ≥ + Tröôøng hôïp 1.1. 0 b S ≥ . Khi ñoù, ta seõ chöùng minh 0 (1) bc SS +≥ Thaät vaäy 2 22 (*)5 222 5 22 ab bc c b a c ba a b b c bc b a c b ca ⎛ ⎞⎛⎞ ⇔ + + + +≥ ⎜ ⎟⎜⎟ ⎝ ⎠⎝⎠ ⎛ ⎞⎛⎞⎛⎞ ⇔ ++ + + +≥ ⎜ ⎟⎜⎟⎜⎟ ⎝ ⎠⎝⎠⎝⎠ AÙp duïng baát ñaúng thöùc AM-GM, ta coù 3 2 2 2 2 22 2 2 2 22 3 2 2 bc cb a bb c a bb ba c a ba a a b a bb b a b aa +≥ ⎛ ⎞⎛⎞ ++ +≥+ + ⎜ ⎟⎜⎟ ⎝ ⎠⎝⎠ ⎛⎞ ⎛⎞ = +++ + ⎜⎟ ⎜⎟ ⎝⎠ ⎝⎠ ≥+ Do ñoù 3 2 2 23 45 22 2 a b bc bc b a c b ca ⎛ ⎞⎛ ⎞⎛ ⎞ + ++++ ≥+ > ⎜ ⎟⎜ ⎟⎜ ⎟ ⎝ ⎠⎝ ⎠⎝ ⎠ Vaäy (1) ñuùng. Do ñoù Tìm tài liệu Toán ? Chuyện nh ỏ - www.toanmath.com426 2 2 22 ( ) ( ) ( )( )( )0 a b c bc S b c S c a S ab S S ab - + - + -≥ + -≥ + Tröôøng hôïp 1.2. 0. b S ≤ Khi ñoù, ta seõ chöùng minh 2 0 (2) 2 0 (3) ab cb SS SS +≥ +≥ Thaät vaäy, ta coù 4 3 15 22 2 15 4 43 2 0 ab a b acb SS b a cac ⎛ ⎞ ⎛⎞ + = + + + +- ⎜ ⎟ ⎜⎟ ⎝ ⎠ ⎝⎠ ≥ + +- > ⇒ (2) ñuùng. 4 3 15 22 2 15 4 40 2 0 cb a b bcc SS b a cba ⎛ ⎞ ⎛⎞ + = + + + +- ⎜ ⎟ ⎜⎟ ⎝ ⎠ ⎝⎠ ≥ ++- > ⇒ (3) ñuùng. Chuù yù raèng 2 22 ( ) 2( ) 2( ) a c ab bc - ≤ - +- Do ñoù 2 2 2 22 ( ) ( ) ( )(2 )( ) ( 2 )( )0 a b c b c ab S bc S c a S a b S S ab S S bc - + - + - ≥ + -+ + -≥ + Tröôøng hôïp 2. 0 c ba ≥ ≥> . Khi ñoù, ta coù 0 b S > . Theo (1), ta coù 0 bc SS +≥ Ta seõ chöùng minh 0 (4) ab SS +≥ Thaät vaäy 2 22 5 32 2 05 0 ab ba c ab SS ab a cc ⎛ ⎞⎛⎞ + = ++ + +- ⎜ ⎟⎜⎟ ⎝ ⎠⎝⎠ ≥+ +- > Tìm tài liệu Toán ? Chuyện nh ỏ - www.toanmath.com427 ⇒ (4) ñuùng. Do ñoù 2 2 2 22 () ( ) ( ) ( )( ) ( )() 0 a b c b c ab S b c S c a S ab S S ab S S bc - + - + - ≥ + - + + -≥ Toùm laïi, trong moïi tröôøng hôïp, ta luoân coù 222 ( )( )( ) 0 a bc S b c Sc a Sa b - + -+ -≥ ⇒ ñpcm. Ñaúng thöùc xaûy ra khi vaø chæ khi . abc == Baøi toaùn 117. Cho caùc soá thöïc döông ,, abc thoûa 1 abc = . Chöùng minh raèng 222 1 1 1 16 41 (1 )(1 )(1 ) (1 ) (1 ) ( 1 ) ab bc ca a bc b ca c ab ⎛⎞ + + ≤+ ⎜⎟ + ++ +++ ⎝⎠ Lôøi giaûi. Ta coù baát ñaúng thöùc caàn chöùng minh töông ñöông vôùi 2 2 2 2 1 16 41 (1 )(1 )( 1 ) (1) 16 41 ( )( )( ) () 16 41 ( 1)( 1)( 1) ( 1) cyc cyc cyc ab bc ca a bc a abc a abc b abc c abc a a abc a abc a ≤+ + ++ + ⇔ ≤+ + ++ + ⇔ ≤+ + ++ + ∑ ∑ ∑ Ñaët 2 22 1, 1, 1 1 11 x yz a bc = - = - =- + ++ thì ta coù ( 1 )(1 )( 1 ) (1 )(1 )(1 ) 0 x y z x y z x yzxyz - - - = + + + ⇒+++= Baát ñaúng thöùc caàn chöùng minh trôû thaønh 2 22 2 (1 )(1 ) (1 )(1 ) (1 )(1 ) 1 2(1 )(1 )(1 ) 2( ) 2( ) 0 ( )0 (ñuùng) xx y y z z x yz x y z xy yz zx x y z xyz xyz -+ +- + +- + ≤+ ++ + ⇔++ + + + + + + + ≥ ⇔ ++≥ ⇒ ñpcm. Tìm tài liệu Toán ? Chuyện nh ỏ - www.toanmath.com Baøi toaùn 118. (Phaïm Vaên Thuaän) Cho caùc soá khoâng aâm thoûa ,, abc 22 2 1. abc + += Chöùng minh raèng 33 3 222 22 2 2 ab c bbc c c ca a a ab b ++ −+ − + − + ≥ Lôøi giaûi. Ta coù baát ñaúng thöùc caàn chöùng minh töông ñöông vôùi 3 22 2 cyc a bbc c ≥ −+ ∑ 3 33 3 33 33 3 33 33 3 22 .( ) 2 .( ) 2( ) 2 .( ) 2( ) 2 1 (). 2( ) cyc cyc cyc cyc a bc bc a bc bc abc bc abc bc abc bc abc a b c aabb ⇔+≥ + ⎛⎞ ⇔+++≥++ ⎜⎟ + ⎝⎠ ++ ⇔+≥+++ + ⇔+ + ≥ ++ + −+ ∑ ∑ ∑ ∑ 2 + AÙp duïng baát ñaúng thöùc AM-GM, ta coù 22 222 19 2( ) cyc a abb abc abbcca ≥ −+ + + −− − ∑ Do ñoù, ñeå chöùng minh baát ñaúng thöùc ñaõ cho, ta chæ caàn chöùng minh 33 3 22 2 9( ) 22( ) (*) 2( ) abc abc abc abbcca ++ ≥+ + + ++ − − − Ñaët ,, 001, pa b cqab bc car abc q p q =+ + = + + = ≥ ⇒ ≤ ≤ = +12. Khi ñoù ( ) (*) 9( (1 ) 3 ) 2 2 (2 ) 99 27 4 2 2 22 5 7 2 27 2 2 (**) pq r p q ppq r p pq q ppq q r ⇔−+ ≥ + − ⇔− + ≥ − − + ⇔− + + ≥ 427 Tìm tài liệu Toán ? Chuyện nh ỏ - www.toanmath.com . Coù 2 tröôøng hôïp xaûy ra + Tröôøng hôïp 1. 21 q ≤ Khi ñoù (**) ( ) 5 2 1 7 2 1 2 27 2 2 fq q q q q r ⇔ = +− ++ + ≥ Ta coù / 57 () 7 2 1 2 21 21 q fq q qq =− +− + ++ 2(2 1) (21 2) 21 qq q +− + = + 2(1 1) 21 2 21 21 21 0 q q q q +− − ≤ + =− + ≤ () 1 () 22 27 22 2 (**) laø haøm nghòch bieán. ñuùng. fq fq f r ⇒ ⎛⎞ ⇒≥ = + ≥ ⎜⎟ ⎝⎠ ⇒ + Tröôøng hôïp 2. Khi ñoù, theo baát ñaúng thöùc Schur, ta coù 21 q ≥. 32 4(4)(2 0 999 pq p p q p p q r −− − ≥= = 1) ≥ Do ñoù, ñeå chöùng minh (**), ta chæ caàn chöùng minh 57 2 3(2 1) 22 2222 () 22 1 2 1 2 22 ppq q pq ppq q gq q q q q −+ + − ≥ ⇔− + ≥ ⇔ = +− ++ ≥ Ta coù / 2 () 2 1 2 21 21 q gq q qq =− +− + ++ 428 Tìm tài liệu Toán ? Chuyện nh ỏ - www.toanmath.com 2(2 1) 3 1 21 2(1 1) 3 1 21 3(1 ) 21 0( 1 do ) qq q q q q q q +− + = + +− + ≥ + − = + ≥≤ () 1 () 2 2 2 laø haøm ñoàng bieán. gq gq g ⇒ ⎛⎞ ⇒≥ = ⎜⎟ ⎝⎠ (**) ñuùng. ⇒ Toùm laïi, trong moïi tröôøng hôïp, ta luoân coù 33 3 222 22 2 2 ab c bbc c c ca a a abb ++ −+ − + − + ≥ (ñpcm) Ñaúng thöùc xaûy ra khi vaø chæ khi 11 (, , ) , ,0 22 abc ⎛⎞ = ⎜⎟ ⎝⎠ . Baøi toaùn 119. (Belarus 1998) Chöùng minh raèng vôùi moïi soá döông ta coù baát ñaúng thöùc ,, abc 1 ab c a b b c bca b c a b + + ++ ≥ + + ++ Lôøi giaûi. + Caùch 1. Ta coù baát ñaúng thöùc caàn chöùng minh töông ñöông vôùi 22 2 3 2 ab bc ca abc a b b c abc bc ab ++ − + + ≥+ − ++ 2 2 11 1 () . . . 66 2 () ()( cyc ab a b c ca abc abbc ⎛⎞ −−+ ⎜⎟ − ⎝⎠ ⇔≥ ) + + ∑ 429 Tìm tài liệu Toán ? Chuyện nh ỏ - www.toanmath.com 22 2 33 () ( ) 36 () ()( ) ab c bc a ab bc abc abc ca b ca abc abbc −+ − + ⎛⎞ ⎛⎞ ⇔− + − + ⎜⎟ ⎜⎟ ⎝⎠ ⎝⎠ ⎛⎞ −+ +− − ≥ ⎜⎟ ++ ⎝⎠ 0 Ñaët 3 36 ()( 3 a b c bc a S abc ca b S abc a b b c ab c S abc −+ = −+ =− ++ −+ = ) 2 0 Baát ñaúng thöùc caàn chöùng minh trôû thaønh 22 () ( ) ( ) ab c Sb c S c a S a b − +− + − ≥ Coù 2 tröôøng hôïp xaûy ra + Tröôøng hôïp 1. 0. ac ≥> + Tröôøng hôïp 1.1. Khi ñoù, ta coù 0. abc ≥≥ > ,0 ac SS ≥. + Tröôøng hôïp 1.1.1. bc a + ≥ . Khi ñoù, ta coù 36 ()( ) ()2 6 ()( 26 ()( ) 2( ( ) 2 ) 0 ()( ) b ca b S abc a b b c bc a b abc a b b c ac abbc ab b b c ac ac a b b c −+ =− ++ +− + =− ) + + ≥− ++ ++ − =≥ ++ Do ñoù 22 () ( ) ( ) ab c Sb c S c a S a b − + −+ −≥ 2 0 + Tröôøng hôïp 1.1.2. . ab c ≥+ Khi ñoù, xeùt haøm soá () 1 ab c a b b c fa bca b c a b + + =++−− ++ − vôùi ab c ≥+ Ta coù 430 Tìm tài liệu Toán ? Chuyện nh ỏ - www.toanmath.com / 22 22 11 () () () () 0 () 2 () ( ) 1 2 2 2 laø haøm ñoàng bieán. cbc fa bbc aab cc bc bb c aab fa bc b c bc bc fa f b c b c bc bc bc bc b b c c b bc bc + =− − + + + + =−+ + + > ⇒ ++ ⇒≥ + = ++ − − ++ + + =+ − − ++ + − *) Ta laïi coù ()0 ( fb c +> Thaät vaäy 22 2 2 22 3 (*) ( )( )(2 ) 2 (2 ) ( ) 0 ()(2 ) ()2 0 0 (ñuùng do ) b c bc bc bc bc bcbc bc b c bcbc bc b c ⇔+ + + − + − + > ⇔− + + − + > ≥ > ⇒ (*) ñuùng. ⇒ () 0 fa > ⇒ 1 ab c a b b c bca b c a b ++ ++ > + + ++ + Tröôøng hôïp 1.2. Khi ñoù, ta coù 0. ac b ≥≥ > ,0 ac SS ≥. Ta coù 2 24 6 ()( 46 ()( ) 2(2( )( ) 3 ) ()( ) 2(2 2 2 ) ()( ) 0 bc bc SS abc a b b c c abc a b b c abbc ab ababbc bbc acab ababbc + += − ++ ≥− ++ ++ − = ++ ++ − = ++ ≥ ) 431 Tìm tài liệu Toán ? Chuyện nh ỏ - www.toanmath.com ) 0 Do neân . Do ñoù 0 ac b ≥≥ > 22 () ( ab a c −≥ − 22 2 2 ( ) () ( ) ()( ) ab c bc Sb c S c a S a b c a S S − + −+ −≥ − + ≥ + Tröôøng hôïp 1.3. Khi ñoù, ta coù 0. bac ≥≥ > 2 3 0 36 ()( 26 ()( ) 2( 2 ) ()( ) 0 42 0 a b ac bc a S abc ca b S abc a b b c ac abbc babbc ac acabbc ac SS abc −+ =≥ −+ =− ++ ≥− ++ ++ − = ++ ≥ + += ≥ ) . 0 Do neân Do ñoù 0 bac ≥≥ > 22 () ( ) bc ab −≥ − 22 2 2 () ( ) ( ) ( )( ) ab c ac Sb c S c a S a b a b S S − + −+ −≥ − + ≥ + Tröôøng hôïp 2. 0. ca ≥> + Tröôøng hôïp 2.1. Khi ñoù, ta coù 0. cb a ≥≥ > 2 3 0 36 ()( 26 ()( ) 36 ()( ) 3( ) ()( ) 0 c b ab c S abc ca b S abc a b b c cb abc a b b c ac abbc babbcac acabbc −+ =≥ −+ =− ++ + ≥− ++ ≥− ++ ++ − = ++ ≥ ) 432 Tìm tài liệu Toán ? Chuyện nh ỏ - www.toanmath.com 2 24 6 ()( 36 ()( ) 3( ) 0 ()( ) ab ab SS abc a b b c ac a b b c babbcac ac a b b c + += − ++ ≥− ++ ++ − =≥ ++ ) 2 . 0 Do neân Do ñoù 0 cb a ≥≥ > 2 () ( ) ca b c −≥ − 22 2 2 () ( ) ( ) ()( ) ab c ab Sb c S c a S a b b c S S − + −+ −≥ − + ≥ + Tröôøng hôïp 2.2. Khi ñoù, ta coù . 0. ca b ≥≥ > 0 c S ≥ + Tröôøng hôïp 2.2.1. 0. ca b ≥+ > Khi ñoù, xeùt haøm soá () 1 ab c a b b c gc bca b ca b + + =++−− ++ − vôùi ca b ≥+ Ta coù / 22 22 11 () () () () 0 () 2 () ( ) 1 2 23 2 laø haøm ñoàng bieán. bab gc aab cbc bb ab aa b cbc gc a b ab ab a b gc ga b babaa b ab ab a b ba a b + =− − + + + + =−+ + + > ⇒ +++ ⇒≥ + =+ + − − − ++ + + =+ − + Ta laïi coù ( ) 0 (**) ga b +> Thaät vaäy 22 32 3 (**) ( )( 2 ) (2 3 ) 0 22 0 (ñuùng) ab a b aba b bbaa ⇔+ + − + > ⇔− + > ⇒ (**) ñuùng. 433 Tìm tài liệu Toán ? Chuyện nh ỏ - www.toanmath.com ⇒ 1 ab c a b b c bca b c a b ++ ++ > + + ++ + Tröôøng hôïp 2.2.2. . ab c + ≥ Khi ñoù, ta coù 0. a S ≥ Ta coù 2 24 6 ()( ) 26 ()( ) 2(( )( ) 3 ) ()( ) 2( 2 ) 0 ()( ) ab ab SS abc a b b c c abc a b b c abbc ab ab a b b c ac bc b ab ab a b b c + += − ++ ≥− ++ ++ − = ++ ++ − =≥ ++ Do neân Do ñoù 0 ca b ≥≥ > 22 () ( ) bc c a −≥ − . 0 22 2 2 ( ) () ( ) ()( ) ab c ab Sb c S c a S a b c a S S − + −+ −≥ − + ≥ + Tröôøng hôïp 2.3. Khi ñoù, ta coù 0. bc a ≥≥ > 3 0 36 ()( 36 ()( ) 3 2 0 a b bc a S abc ca b S abc a b b c ac a a c c ac −+ =≥ −+ =− ++ ≥− ++ = > ) + Tröôøng hôïp 2.3.1. 0. ca b + ≥> Khi ñoù, ta coù Do ñoù 0. b S ≥ 22 () ( ) ( ) ab c Sb c S c a S a b − + −+ −≥ 2 0 + Tröôøng hôïp 2.3.2. . bc a ≥+ Khi ñoù, xeùt haøm soá () 1 ab c a b b c hb bca b c a b + + =++−− ++ − vôùi . bc a ≥+ Ta coù 434 Tìm tài liệu Toán ? Chuyện nh ỏ - www.toanmath.com 2 / 22 11 () ( ) 0 () ( ) bac hb c a bc b a b c ⎛⎞ − =+− − ⎜⎟ ++ ⎝⎠ 2 ≥ ⇒ laø haøm ñoàng bieán. () hb 22 2 22 () ( ) 1 22 22 22 22 22 22 () () (2)(2 ) 11 () (2 )( 2 ) aacc aca c hb ha c ac c a a c ac aac aca c ac c a a c ac aac aca c ac c a a c ac aca ca ac ca a c ac a ca ac ca ac a c 0 + ++ ⇒ ≥ += + +−−− +++ ++ = ++− − ++ + ++ ⎛⎞⎛ =+ +− − + − ⎜⎟⎜ ++ ⎝⎠⎝ −− =+ − +++ ⎛⎞ =+− − ≥ ⎜⎟ ++ ⎝⎠ ⎞ ⎟ + ⎠ + Toùm laïi, trong moïi tröôøng hôïp, ta luoân coù 1 ab c a b b c bca b c a b ++ ++ ≥ + + ++ Ñaúng thöùc xaûy ra khi vaø chæ khi . abc = = + Caùch 2. Ta coù baát ñaúng thöùc caàn chöùng minh töông ñöông vôùi 22 22 222 22 2 22 2 ( )() ( )() ( )() () ( ) ()( ) () ( ) 33 3 () ( ) 22 aabbc babbc cabbc bc a ab bc abbc ac b a b bcb c aabac b abc bc bc aacc b ab b ac b a b bcb c ab bc b bc a ++ ++ ++ ++ ≥ ≥+ + + + + + ++ ⇔ + ++ + + ++ + + ≥ ≥+ + + + + ++ ⇔+ + ≥ + + a c AÙp duïng baát ñaúng thöùc AM-GM, ta coù 23 2 2 3 2 2 11 1 .. . 22 2 ac b ac bc b bc a c b bc b a c a c a ⎛⎞ ⎛ ⎞ ⎛ ⎞ ⎛⎞ + + ++ ++ + ⎜⎟ ⎜ ⎟ ⎜ ⎟ ⎜⎟ ⎝⎠ ⎝⎠ ⎝ ⎠ ⎝ ⎠ ≥ 435 Tìm tài liệu Toán ? Chuyện nh ỏ - www.toanmath.com 4 32 2 22 2 2 22 () ( ) 22 ñpcm. bc ab ac b a ab bc b ac b a b bcb c ab bc b bc a ⎛⎞ ⎜⎟ ≥+ + + ⎜⎟ ⎝⎠ ≥+ + ++ ⇒+ + ≥ + + ⇒ Ñaúng thöùc xaûy ra khi vaø chæ khi . abc = = Baøi toaùn 120. Chöùng minh raèng vôùi moïi soá thöïc khoâng aâm ta coù baát ñaúng thöùc ,, abc 222 22 2 11 1 3 ab bc ca aabb b bcc c ca a ++ ≥ + + −+ − + − + Lôøi giaûi. Khoâng maát tính toång quaùt, ta coù theå giaû söû min{ , , }. cabc = Khi ñoù, ta coù 222 22 222 22 11 0 11 0 bbc c b bbc c b cca a a cca a a ⎧ ≥ ⎪ ⎧ ≤− + ≤ ⎪⎪ −+ ⇒ ⎨⎨ ≤−+ ≤ ⎪ ⎪ ⎩ ≥ ⎪ −+ ⎩ 2 2 Do ñoù 222 22 2 2222 11 1 3 111 3 ab bc ca aabb b bcc c ca a ab bc ca aabb b c ++ − ++ −+ − + − + ≥++− ≥ + + −+ 2222 4 22 2 2 111 () () 0 ab aabb b a ab ab a ab b ≥++ −+ − = −+ ≥ 3 − ñpcm. ⇒ Ñaúng thöùc xaûy ra khi vaø chæ khi (, , ) (,,0)( 0). abc tt t = > 436 Tìm tài liệu Toán ? Chuyện nh ỏ - www.toanmath.com Baøi toaùn 121. Tìm lôùn nhaát sao cho vôùi moïi soá khoâng aâm k , , (( )( )( ) 0) abc abbcc a + ++ > ta coù baát ñaúng thöùc 22 2 2 2 2 11 1 ab c k ab bc c a bc c a ab ⎛⎞ ++ ≥ + + ⎜⎟ ++ + ++ + ⎝⎠ Lôøi giaûi. Cho ta suy ra ñöôïc 1, 0 ab c == = 4 . 5 k ≤ Ta chöùng minh ñaây laø giaù trò caàn tìm, töùc laø chöùng minh 22 2 2 2 2 41 1 1 . 5 ab c ab bc c a bc c a ab ⎛⎞ ++ ≥ + + ⎜⎟ ++ + ++ + ⎝⎠ + Caùch 1. Do 2 veá cuûa baát ñaúng thöùc treân ñoàng baäc neân khoâng maát tính toång quaùt, ta coù theå giaû söû Ñaët 1. abc ++ = 11 ,0, 327 qab bc car abc q r =+ + = ⇒ ≥ > ≥ ≥0. Khi ñoù, ta coù 22 2 2 22 2 2 2 2 2 2 22 2 2 22 22 2 2222 22 222 33 3 2 2 2 22 22 ()( ) ()( )( ) (( ) ) ()( ) ()( )( (1 2 )( 2 ) (3 1 3 )(1 2 ) 2(12) (12) cyc cyc cyc aa b a c a bc ab bc c a aa a b c bc a b c ab bc ca abc a b c a b c abc ab bc ca qq r r rq qqr rr q q q ++ = ++ + + ++ + = ++ + + − ++ + + + + + = −− − +− − + = −− − + − ∑ ∑ ∑ ) 22 (3 5 ) (1 2 )(1 3 ) 2(12) (12) qr q q rr q q q −+− − = −− − + − ()( ) (( ) ) 11 ()( )( )( )( ) cyc cyc cyc ab a c aa bc bc q ab abbcc a abc ab bc ca abc q r ++ ++ + + == ++ + + ++ ++ − ∑∑ ∑ = − 437 Tìm tài liệu Toán ? Chuyện nh ỏ - www.toanmath.com Do ñoù, baát ñaúng thöùc caàn chöùng minh trôû thaønh 22 22 (3 5 ) (1 2 )(1 3 ) 4 1 . 5 2(12) (12) ( ) (29 11) (3 32 71 ) (1 2 )(5 )(1 4 ) 0 qr q q q rq rr q q q fr q r q q r q q q q −+− − + ≥ −+ −− − + − ⇔= − ++ − + − + − ≥ Ta coù /2 2 2 ( ) 2(29 11) 3 32 71 1 2(29 11). 3 32 71 27 59 922 1 .71 0 0 27 27 3 (do fr q r q q qqq qq q =− ++ − ≥− ++ − =+ − ≥ ≤ ≤) () laø haøm ñoàng bieán. fr ⇒ + Neáu 1 thì ta coù 4q ≥ ( ) (0) (1 2 )(5 )(1 4 ) 0 fr f q q q q ≥= − + − ≥ + Neáu thì theo baát ñaúng thöùc Schur, ta coù 4q ≥1 41 0. 9 q r − ≥≥ Do ñoù 32 4 1 2(4 1)(81 103 95 19) () 0 981 qq q qq fr f −− + −+ ⎛⎞ ≥= ⎜⎟ ⎝⎠ ≥ Toùm laïi, trong moïi tröôøng hôïp, ta luoân coù () 0 . ñpcm fr ≥⇒ Vaäy max 4 . 5 k = + Caùch 2. Ta coù baát ñaúng thöùc caàn chöùng minh töông ñöông vôùi 22 54 cyc cyc a bc bc ≥ + + ∑∑ 22 5( ) 4( ) cyc cyc aa bc abc bc bc ++ ++ ⇔≥ + + ∑∑ 2 22 10 10 ( ) 8 24 cyc cyc aabc a bc bc ++ ⇔≥+ + + ∑∑ 438 Tìm tài liệu Toán ? Chuyện nh ỏ - www.toanmath.com 22 22 22 22 22 29 8 2 () 3 () 92 2 cyc cyc cyc cyc cyc aa abc bc bc bc ab c bc ab c bc bc ⎛⎞⎛ ⎞ + ⇔− + −+ ⎜⎟⎜ ⎟ ⎜⎟⎜ ⎟ + ++ ⎝⎠⎝ ⎠ ⎛⎞⎛ +− + +−+ ⎜⎟⎜ ⎜⎟⎜ ++ ⎝⎠⎝ ∑∑ ∑ ∑∑ 0 ⎞ −≥ ⎟ ⎟ ⎠ Do ñoù, ñeå chöùng minh baát ñaúng thöùc ñaõ cho, ta chæ caàn chöùng minh 2 22 2 22 22 22 (1) 29 (2) 2 () 3 (3) 2 () 2( cyc cyc cyc cyc cyc aa bc bc abc bc ab c bc bc ab c bc ≥ + + + ≥ + +− ≥ + + ≥ + ∑∑ ∑ ∑ ∑ 4) * Chöùng minh (1). Ta coù 2 22 2 22 22 22 22 22 22 22 2 0 () ( ) 0 ()( ) () ( ) 0 ()( ) ()( ) () () 0 ()( ) ( )( ) (). cyc cyc cyc cyc cyc cyc cyc cyc aa bc bc aa bc bc ab a b ca c a bc bc ab a b ca c a bc bc bc bc ab a b ab a b bc bc a c a c abc abbc ca ≥ + + ⎛⎞ ⇔− ≥ ⎜⎟ + + ⎝⎠ −− − ⇔≥ ++ −− ⇔− ≥ ++ ++ −− ⇔− ≥ ++ + + ⇔+ + + + + ∑∑ ∑ ∑ ∑∑ ∑∑ 2 22 22 () 0 ()( )( )( ) (ñuùng) cyc ab a b ac b c acbc − ≥ ++ + + ∑ * Chöùng minh (2). Ta coù 439 Tìm tài liệu Toán ? Chuyện nh ỏ - www.toanmath.com 2 22 2 22 29 2 42 30 cyc cyc abc bc qbc bc + ≥ + ⎛⎞ + ⇔− ⎜⎟ + ⎝⎠ ∑ ∑ ≥ 22 2 22 22 22 22 22 22 22 2 2 43( )2 0 (2 3 )( ) (2 3 )( ) 0 (2 3 )( ) (2 3 )( ) 0 (2 3 )() (3 2 )() 0 ()(2 2 ( )3) ( cyc cyc cyc cyc cyc cyc ab c bc bc abcab abcca bc abcab abcca bc bc abcab a bcab bc a c ab a b c ca b ab −+ + ⇔≥ + +− − − − + − ⇔≥ + +− − − + − ⇔− ≥ ++ +− − + − − ⇔− ≥ ++ −+ −−++ ⇔ ∑ ∑ ∑∑ ∑∑ 22 22 22 2 2 2 2 22 2 2 2 2 2 22 2 2 2 2 22 2 22 0 )( ) ()(2 2 ( )3)( )0 ( ) (2 2 2 2 ( ( ) ))( ) 0 2( )( )( ) 2 ( )( ) ()( )( ).()( ) cyc cyc cyc cyc cyc cyc bc a c ab a b c ca b ab a b ab a b c ab c ca b ab a b ab a b c a b aba b a b abbc c a ab a b ≥ ++ ⇔− + −− ++ + ≥ ⇔− + − + + − ++ + ≥ ⇔− +− + + − + − 0 − −− − − + ≥ ∑ ∑ ∑ ∑∑ ∑ 22 2 2 2 2 22 2 22 2 2( )( )( ) 2 ( )( ) ()( )( ) 0 cyc cyc ab a b c a b aba b a b ab bc c a ⇔− +− + + − + − −− − − ≥ ∑∑ Khoâng maát tính toång quaùt, ta coù theå giaû söû Khi ñoù, ta coù 0. abc ≥≥ ≥ 22 2 2 2 2 22 2 2 2 2 2 2 2 2 2 2 22 2 2 2 2 2 2 2 2 2 2 222 2 ()( )( ) ()()( )()()( ()( )( )()( )( 2( )( ) 0 cyc ab a b c a b bc b c a b c c a c a b c a bc b c a b c bc c a b b c cb c b c −+− + ≥ ≥− + − + + − + − + ≥− + − + + − + − + =− + ≥ ∑ ) ) 440 Tìm tài liệu Toán ? Chuyện nh ỏ - www.toanmath.com 2 0 22 2 2 2 2 2 22 22 2( )( )2( )( )4( ) 4( )( )( ) cyc abab a b abab a b ab ab ab bc c a −+ ≥ − + ≥ − ≥− − − ∑ 22 2 2 2 2 22 2 22 2 2( )( )( ) 2 ( )( ) ()( )( ) cyc cyc ab a b c a b abab a b ab bc c a ⇒− +− + + − + − −−− − ≥ ∑∑ * Chöùng minh (3). Ta coù 22 22 22 22 22 22 22 22 22 () 3 2 2( ) 2 10 ()2( )2 0 ()( )()( ) 0 ()( ) ()( ) 0 ()( ) ( )( ) cyc cyc cyc cyc cyc cyc cyc cyc ab c bc bc ab c bc bc bc abc bc bc bc a b bcc a bc bc a b bcc a bc bc b c ab a c ab bc a c +− ≥ + +− ⎛⎞ ⇔−≥ ⎜⎟ + ⎝⎠ −+ + + − ⇔≥ + + −−+ − ⇔≥ + +− + − ⇔− ≥ ++ +− +− ⇔−≥ ++ ∑ ∑ ∑ ∑ ∑∑ ∑∑ 22 22 2 2 22 2 2 22 2 2 22 2 2 2 22 2 0 ()( ( )) 0 ()( ) ()( ( ))( )0 ()(( ( ) )2)( )0 2 ( ) ( ) ( )( )( ). ( )( ) 0 2( )( )( cyc cyc cyc cyc cyc cyc ab c ca b ab bc a c ab c ca b ab a b ab c ca b ab ab a b abab a b abbcc a ab a b ab a b a b a −− + + + ⇔≥ ++ ⇔− −+ ++ + ≥ ⇔− −+ +− + + ≥ ⇔− ++−−− − + ⇔− ++− ∑ ∑ ∑ ∑∑ ∑ 22 2 )( )( ) 0 (ñuùng) bb c c a −− ≥ ≥ * Chöùng minh (4). Ta coù 441 Tìm tài liệu Toán ? Chuyện nh ỏ - www.toanmath.com 22 2 2 222 22 2 2 22 2 2 22 ()( 2)8 () 20 ()( )( ) () 2 cyc cyc cyc ab a b a b c a b c ab c bc ab bc c a ab c bc −++ + + −=≥ ++++ + ⇒≥ + ∑ ∑ ∑ Vaäy (1), (2), (3) vaø (4) ñuùng. Töø ñaây, ta suy ra ñpcm. Vaäy max 4 . 5 k = * Caùch 3. AÙp duïng baát ñaúng Bunhiacopxki, ta coù 2 22 () () cyc cyc a abc ab a b bc ++ ≥ + + ∑ ∑ Do ñoù, ñeå chöùng minh baát ñaúng thöùc ñaõ cho, ta chæ caàn chöùng minh 2 2222 ()41 1 1 . ()5 ()4 3( . ()5 ()2 5( 2 ) 4( 3 ) 2 10 20 2 cyc cyc cyc abc abab ab bc c a abc a b c ab bc ca ab a b ab a b abc SP S P QQabc SQ abcS abcP PQ ++⎛⎞ ≥+ + ⎜⎟ ++ + + ⎝⎠ ++ + + + + + ⇔≥ +++ ++ ⇔≥ + ⇔+ + ≥ ∑ ∑∑ ) (). Trong ñoù 22 2 ,, cyc S abcP abbccaQ abab =+ + = + + = + ∑ Deã thaáy 22 22 22 ()2 ( ) ()( )2 ( cyc cyc cyc PQ a b a b abc S P SQ ab a b a b a b a b =++ + ≥+ +≥ ∑ ∑∑ )+ Töø ñaây, ta coù ngay ñpcm. 442 Tìm tài liệu Toán ? Chuyện nh ỏ - www.toanmath.com Vaäy max 4 . 5 k = Baøi toaùn 122. (Vasile Cirtoaje) Chöùng minh raèng vôùi moïi soá khoâng ta coù baát ñaúng thöùc ,, abc 33 3 2 2 22 22 2 2 2 2 2 2 1 (2 )(2 ) (2 )(2 ) (2 )(2 ) abc abc ab a c bc b a c a c b ++≤ ++ ++ + + + + Lôøi giaûi. AÙp duïng baát ñaúng thöùc Bunhiacopxki, ta coù 2 2 2 2 22 2 2 22 22 22 (2 )(2 ) ( )( ) () () ab a c a a b a c a aac ab a abc ++=++ ++ ≥+ + =++ 3 22 2 2 (2 )(2 ) ( ) aa ab a c abc ⇒≤ ++ ++ 2 Töông töï, ta coù 3 22 2 2 3 22 2 2 (2 )(2 ) ( ) (2 )(2 ) ( ) bb b c b a abc cc ca c b abc ≤ ++ ++ ≤ ++ ++ 2 2 Do ñoù 33 3 2 2 22 22 2 2 2 2 2 2 1 (2 )(2 ) (2 )(2 ) (2 )(2 ) abc abc ab a c bc b a c a c b ++≤ ++ ++ + + + + ñpcm. ⇒ Ñaúng thöùc xaûy ra khi vaø chæ khi . abc = = 443 Tìm tài liệu Toán ? Chuyện nh ỏ - www.toanmath.com Baøi toaùn 123. (Phaïm Kim Huøng) Chöùng minh raèng vôùi moïi daõy soá döông ta coù baát ñaúng thöùc 12 , ,..., n aa a 11 2 1 2 1 2 11 1 1 1 1 ... 2 ... ... nn aa a a a a aa a ⎛⎞ +++ ≤ +++ ⎜⎟ ++++ ⎝⎠ Lôøi giaûi. Neáu thì baát ñaúng thöùc treân hieån nhieân ñuùng. 1 n = Xeùt . 2 n ≥ AÙp duïng baát ñaúng thöùc Bunhiacopxki, ta coù vôùi moïi soá döông 12 , ,..., n xx x thì 2 22 2 12 12 1 2 12 2 22 12 2 12 1 2 12 ( ... ) ... ( ... ) 11 . ... ... ( ... ) k kk k k kk k x xx aa a x x x aa a x xx aa a a a a xx x ⎛⎞ ++ + + + + ≥ + + + ⎜⎟ ⎝⎠ ⎛⎞ ⇒≤ ++ ⎜⎟ ++ + ++ + ⎝⎠ + Cho chaïy töø 1 ñeán roài coäng caùc baát ñaúng thöùc treân laïi, ta ñöôïc k n 12 11 2 1 2 1 2 11 1 ... ... ... n nn c cc aa a a a a aa a +++ ≤+++ ++++ Trong ñoù 22 2 22 2 12 12 1 12 ... 1, ( ... ) ( ... ) ( ... ) kk k k kk n xx x ck xx x xx x xx x + =+ ++ ++ + + + + ++ + n∀= Ta coù theå choïn 1, . k x kk n =∀ = Khi ñoù 2 k ∀ ≥ , ta coù 2 22 11 1 ... (1 2 ... ) (1 2 ... ( 1)) (1 2 ... ) k ck kk ⎛⎞ =+ ++ ⎜⎟ + + + + ++ + + ++ ⎝⎠ 2 n 2 22 2 2 2 2 22 2 2 2 2 22 2 44 4 ... (1) ( 1)( 2) ( 1) 44 4 ... ( 1) ( 2) ( 1) 11 1 4... ( 1) ( 2) ( 1) k kk k k n n k kk kk kn kk n ⎛⎞ =+ ++ ⎜⎟ ++ + + ⎝⎠ ⎛⎞ ≤+ ++ ⎜⎟ ++ + ⎝⎠ ⎛⎞ ≤+ ++ ⎜⎟ ++ + ⎝⎠ 2 444 Tìm tài liệu Toán ? Chuyện nh ỏ - www.toanmath.com 11 1 4 ... (1) (1)( 2) ( 1) kk k k nn ⎛⎞ ≤+ ++ ⎜⎟ ++ + + ⎝⎠ 11 1 1 1 1 4.. 11 2 11 4 42 1 kk k k n n kn k ⎛⎞ ⎛⎞⎛ ⎞ ⎛ =− + − ++ − ⎜⎟⎜ ⎟ ⎜ ⎜⎟ ++ + + ⎝⎠⎝ ⎠ ⎝ ⎝⎠ ⎛⎞ =− ≤≤ ⎜⎟ + ⎝⎠ . 1 ⎞ ⎟ ⎠ Ngoaøi ra 1 22 22 2 2 22 2 2 22 11 1... (1 2) (1 2 ... ) 44 1 ... 2.3 .( 1) 44 1 ... 2.3 2.( 1) 11 1 ... 3(1) 11 1... 2.3 ( 1) 11 1 1 1... 23 1 11 1 21 2 c n nn n n nn nn n =+ + + +++ =+ + + + ≤+ + + + =+ + + + ≤+ + + + ⎛⎞ ⎛ ⎞ =+ − + + − ⎜⎟ ⎜ ⎟ + ⎝⎠ ⎝ ⎠ =+ − + < + Do ñoù 21 k ck ≤∀ =,n Töø ñaây, ta suy ra ñieàu phaûi chöùng minh. Baøi toaùn 124. (Phaïm Vaên Thuaän, Voõ Quoác Baù Caån) Cho caùc soá khoâng aâm thoûa ,, abc 22 2 1. abc + += Tìm giaù trò lôùn nhaát cuûa bieåu thöùc ()( )( )( Pabbccaabc =− − − + +) Lôøi giaûi. 445 Tìm tài liệu Toán ? Chuyện nh ỏ - www.toanmath.com Tröôùc heát, ta xeùt tröôøng hôïp 0 a = . Khi ñoù, baøi toaùn chuyeån veà “Caùc soá khoâng aâm thoûa , bc 22 1. bc + = Tìm giaù trò lôùn nhaát cuûa bieåu thöùc 22 ()( ) ( Q bcc b c b bcc b =− + = −) .” Khoâng maát tính, toång quaùt ta chæ caàn xeùt c laø ñuû b ≥ 2 1 2 c ⇒≥ . Ta coù 2222 22 2 2 2 2 2 () (1 )(2 1) (1)(21) ( Qbcc b c c c m m m fm =− = − −= − −= ) Trong ñoù 2 1 . 2 mc =≥ Ta coù /2 / () (1 2)(8 8 1) 22 1 () 0 ( ) 42 do fm m m m fm m m =− − + + =⇔ = ≥ Qua 2 4 +2 thì / () f m ñoåi daáu töø döông sang aâm neân 22 1 () 416 fm f m ⎛⎞ + ≤=∀ ⎜⎟ ⎝⎠ 1 2 ≥ 1 . 4 Q ⇒≤ Ñaúng thöùc xaûy ra khi vaø chæ khi 22 2 2 ,. 22 bc −+ == Vaäy 1 max . 4 Q = Trôû laïi baøi toaùn cuûa ta Khoâng maát tính toång quaùt, ta coù theå giaû söû 0. abc ≤ ≤≤ Ta seõ chöùng minh 1 max , 4 P = töùc laø chöùng minh 446 Tìm tài liệu Toán ? Chuyện nh ỏ - www.toanmath.com 22 22 () (, , ) 4 ()( )( )( ) abc Fabc abbcc a a b c ++ =≥ −− − ++ Khi ñoù, vôùi moïi 0m , ñaët in{,, tab ≤≤ }c , ,, x aty b tz ct = −= − = − ta coù 22 22 22222 22 22 22 2 2 2 2 2 2 2 (( )( )( )) (, , ) ()( )( )( 3) (3 2( ) ) ()( )( )( 3) (2( ) ) ()( )( )( 3) 4( )4()( )() ( xt y t z t Fabc xy y z z t x y z t txyztx y z xy y z z t x y z t xy ztx y z xy y z z t x y z t xy z t x y z x y zt x y z ++ ++ + = −− − +++ ++ + + + + = −− − +++ ++ + + + ≥ −− − +++ ++ + + + ++ + + + = 22 2 22 22 22 22 22 2 22 22 )( )( )( 3 ) 4( )( ) ( ) ()( )( )( 3) 4( ) ()( )( )( ) (, , ) 4( )( ) ( ) () ()( )( )( 3) (vì haøm soá xy y z z t x y z t xy z x y zt xy z xy y z z t x y z t xy z xy y z z t x y z Fxyz xy z x y zt xy z gt xy y z z t x y z t −− − +++ ++ + + + ++ ≥ −− − +++ ++ ≥ −− − ++ = ++ + + + ++ = −− − +++ laø haøm ñoàng bieán) AÙp duïng keát quaû naøy vôùi ta ñöôïc , ta = 222 22 () (, , ) (0, , ) (0, , ) () mn Fabc F b ac a F mn mn n m + ≥− −= = − Vôùi 0 nc a m b a =− ≥ = − ≥ Do ñoù, ñeå chöùng minh , ta chæ caàn chöùng minh (, , ) 4 Fabc ≥ 222 2 2 (0, , ) 4 ()4( ) Fmn mn mnn m ≥ ⇔+ ≥ − (*) Khoâng maát tính toång quaùt, ta coù theå giaû söû 22 1 mn + = . Khi ñoù 22 1 (*) ( ) 4 nm n m ⇔− ≤ Theo chöùng minh treân thì baát ñaúng thöùc naøy ñuùng, töø ñaây, ta suy ra ñpcm, töùc laø 447 Tìm tài liệu Toán ? Chuyện nh ỏ - www.toanmath.com 1 . 4 P ≤ Ñaúng thöùc xaûy ra chaúng haïn khi 22 2 2 0, , 22 ab c −+ == = . Vaäy 1 max . 4 P = 448 Tìm tài liệu Toán ? Chuyện nh ỏ - www.toanmath.comTÀI LI ỆU THAM KH ẢO dreescu, V.Cirtoaje, G.Dospinescu, M.Lascu, Old and New Inequalities [1] T.An Hojoo Lee, Topics In Inequalities [2] Pierre Bornsztein, Inégalités [3] K.S.Kedlaya, A < B [4] Thomas J.Mildorf, Olympiad Inequalities [5] Kin – Yin Li, Using Tangent to Prove Inequalities, Mathematical Excalibur, [6] Vol.10, No. 05, Dec.05 − Jan.06 Lau Chi Hin, Muirhead’ s Inequality, Mathematical Excalibur, Vol.11, No.01, [7] Feb.05 − Mar.06 Crux Mathematicorum [8] Phan Huy Kh ải, 10000 Bài Toán S ơ C ấp − B ất Đẳng Th ức Kinh Đi ển, [9] NXB Hà N ội 2001 T ạp chí Toán H ọc Và Tu ổi Tr ẻ [10] Ph ạm Kim Hùng, Sáng t ạo b ất đẳng th ức, NXB Tri Th ức 2006 [11] Các trang web toán h ọc: [12] www.mathlinks.ro diendantoanhoc.net mathnfriend.net 449 Tìm tài liệu Toán ? Chuyện nh ỏ - www.toanmath.com